Đột phá toán đại số edited

0 124 0
Đột phá toán đại số   edited

Đang tải... (xem toàn văn)

Thông tin tài liệu

CHƯƠNG 1: MỆNH ĐỀ, TẬP HỢP, HÀM SỐ CHUYÊN ĐỀ 1: MỆNH ĐỀ, TẬP HỢP PHẦN 1: LÝ THUYẾT TRỌNG TÂM Mệnh đề Định nghĩa: • Mệnh đề câu khẳng định câu khẳng định sai • Một mệnh đề khơng thể vừa đúng, vừa sai Mệnh đề phủ định Cho mệnh đề P, mệnh đề “không phải P” gọi mệnh đề phủ định P kí hiệu P Nếu P P sai, P sai P Mệnh đề kéo theo Cho mệnh đề P Q Mệnh đề “Nếu P Q” gọi mệnh đề kéo theo kí hiệu P  Q , (P suy Q) Mệnh đề P  Q sai P Q sai Chú ý: Các định lí tốn học thường có dạng P  Q Khi đó: P giả thiết, Q kết luận, P điều kiện đủ để có Q, Q điều kiện cần để có P Mệnh đề đảo • Cho mệnh đề kéo theo P  Q Mệnh đề Q  P gọi mệnh đề đảo mệnh đề P  Q • Cho mệnh đề P Q Mệnh đề “P Q” gọi mệnh đề tương đương kí hiệu P  Q Mệnh đề P  Q hai mệnh đề P  Q Q  P Chú ý: Nếu mệnh đề P  Q định lí ta nói P điều kiện cần đủ để có Q Kí hiệu   : Cho mệnh đề chứa biến P (x) Khi đó: “Với x thuộc X để P (x) đúng” ký hiệu là: “ x  X, P  x  ” “ x  X : P  x  ” “Tồn x thuộc X để P (x) đúng” ký hiệu “  x  X, P  x  ” “  x  X : P  x  ” • Mệnh đề phủ định mệnh đề “ x  X, P  x  ” “  x  X, P  x  ” • Mệnh đề phủ định mệnh đề “  x  X, P  x  ” “ x  X, P  x  ” Tập hợp Tập hợp khái niệm toán học, không định nghĩa Các xác định tập hợp Liệt kê phân từ: Viết phần tử tập hợp hai dấu móc { ; ; } Chỉ tính chất đặc trưng cho phần tử tập hợp Tập rỗng: tập hợp không chứa phần tử nào, kí hiệu  Tập hợp con: A  B    x  A  x  B  Trang A  A,  A   A, A A  B, B  C  A  C A  B Tập hợp nhau: A  B   B  A Chú ý: Nếu tập hợp có n phần tử có 2n tập Một số tập hợp tập hợp số thực  *         * : tập hợp số tự nhiên khơng có số  : tập hợp số nguyên  : tập hợp số tự nhiên  : tập hợp số hữu tỉ    ;   : tập hợp số thực Khoảng  a; b   x   | a  x  b :  a;    x   | a  x :  ; b   x   | x  b : Đoạn:  a; b   x   | a  x  b : Nửa khoảng: a; b   x   | a  x  b :  a; b  x   | a  x  b : a;    x   | a  x :  ; b  x   | x  b : Các phép toán tập hợp Giao hai tập hợp A  B  { x|x  A x  B } Hợp hai tập hợp A  B  { x | x  A x  B } Hiệu hai tập hợp: A \ B  { x | x  A x  B } Phần bù: Cho B  A CA B  A \ B Số gần Sai số tuyệt đối Nếu a số gần số a  a  a  a gọi sai số tuyệt đối số gần a Độ xác số gần Nếu  a  a  a  d a  d  a  a  d Ta nói a số gần a với độ xác d qui ước viết gọn a  a  d Trang Sai số tương đối Sai số tương đối số gần a tỉ số sai số tuyệt đối a , kí hiệu a  a a a nhỏ độ xác phép đo đạc tính tốn lớn Ta thường viết a dạng phần trăm Quy tròn số gần Nếu chữ số sau hàng qui tròn nhỏ ta việc thay chữ số chữ số bên phải số Nếu chữ số sau hàng qui tròn lớn hay ta thay chữ số chữ số bên phải số cộng thêm đơn vị vào chữ số hàng qui tròn Chữ số Cho số gần a số a với độ xác d Trong số a, chữ số gọi chữ số (hay đáng tin) d không vượt nửa đơn vị hàng có chữ số Nhận xét: Tất chữ số đứng bên trái chữ số chữ số Tất chữ số đứng bên phải chữ số không chữ số không PHẦN 2: CÁC DẠNG BÀI TẬP Dạng 1: Mệnh đề Ví dụ minh họa Ví dụ 1: Các câu sau đây, có câu mệnh đề đúng? (1) Chạy đi! (2) Phương trình x  3x   vô nghiệm (3) 16 không số nguyên tố (4) Hai phương trình x  4x   x  x    có nghiệm chung (5) Ba sáng anh chưa ngủ, tương tư em biết cho đủ? (6) U23 Việt Nam đoạt giải chơi đẹp U23 Châu Á (7) Hai tam giác chúng có diện tích (8) Một tứ giác hình thoi có hai đường chéo vng góc với A B C D Hướng dẫn Câu (1) (5) khơng mệnh đề (vì câu đầu khiến, câu nghi vấn) Các câu (3), (4), (6), (8) mệnh đề Câu (2) (7) mệnh đề sai  Chọn A Ví dụ 2: Mệnh đề P  x  :"  x  , x  x   0" Phủ định mệnh đề P A  x  , x  x   B  x  , x  x   C  x  , x  x   D  x  , x  x   Trang Hướng dẫn Phủ định mệnh đề P P  x  : "  x  , x  x   0"  Chọn D Ví dụ 3: Mệnh đề sau phủ định mệnh đề: “Mọi động vật di chuyển”? A Mọi động vật không di chuyển B Mọi động vật đứng yên C Có động vật khơng di chuyển D Có động vật di chuyển Hướng dẫn Phủ định mệnh đề "  x  K, P  x  " mệnh đề "  x  K, P  x  " Do đó, phủ định mệnh đề: “Mọi động vật di chuyển” mệnh đề: “Có động vật khơng di chuyển”  Chọn C Bài tập tự luyện Câu Trong mệnh đề sau, mệnh đề mệnh đề đúng? A Tổng hai số tự nhiên số chẵn hai số số chẵn B Tích hai số tự nhiên số chẵn hai số số chẵn C Tổng hai số tự nhiên số lẻ hai số số lẻ D Tích hai số tự nhiên số lẻ hai số số lẻ Câu Trong mệnh đề sau, mệnh đề mệnh đề sai? A “ABC tam giác tam giác ABC cân” B “ABC tam giác tam giác ABC cân có góc 60 ” C “ABC tam giác ABC tam giác có ba cạnh nhau” D “ABC tam giác tam giác ABC có hai góc 60 ” Câu Cho mệnh đề P  x  :"  x  , x  x   0" Mệnh đề phủ định mệnh đề P (x) A "  x  , x  x   0" B "  x  , x  x   0" C "  x  , x  x   0" D " x  , x  x   0" Câu Lập mệnh đề phủ định mệnh đề: “Số chia hết cho 3” A Số chia hết cho B Số không chia hết cho C Số không chia hết cho D Số không chia hết cho 2, chia hết cho Đáp án: 1–D 2–A 3–C 4–C Dạng 2: Tập hợp phép toán tập hợp Ví dụ minh họa Trang Ví dụ 1: Hãy liệt kê phần tử tập X  x   | 2x  5x   0 A X  0 3 C X    2 B X  1  3 D X  1;   2 Hướng dẫn  x  1  Ta có 2x  5x     x     2 Vậy X  1  Chọn B Ví dụ 2: Cho X  0;1; 2;3; 4;8;9;7 Tập X có tập hợp con? A B 128 C 256 D 64 Hướng dẫn Nếu tập hợp có n phần tử có 2n tập hợp Tập X có phần tử nên có 28  256 tập hợp  Chọn C Ví dụ 3: Cho tập hợp X  1; 2;3; 4 Câu sau đúng? A Số tập X 16 B Số tập X gồm có phần tử C Số tập X chứa số D Số tập X gồm có phần tử Hướng dẫn Số tập tập hợp X là: 24  16 Số tập có phần tử tập hợp X là: C24  Số tập tập hợp X chứa số là: 8, bao gồm: 1 , 1; 2 , 1;3 , 1; 4 , 1; 2;3 , 1; 2; 4 , 1;3; 4 , 1; 2;3; 4 Số tập có phần tử tập hợp X là: C34   Chọn A Ví dụ 4: Cho A  0;1; 2;3; 4 ; B  2;3; 4;5;6 Tập hợp  A \ B    B \ A  A {0;1;5;6} B {1;2} C {5} D  Hướng dẫn A \ B  0;1 Ta có    A \ B   B \ A    B \ A  5;6  Chọn D Ví dụ 5: Lớp 12A có học sinh giỏi Toán, học sinh giỏi Lý học sinh giỏi Hóa, học sinh giỏi Tốn Lý, học sinh giỏi Tốn Hóa, học sinh giỏi Lý Hóa, học sinh giỏi mơn Tốn, Lý, Hóa Số học sinh giỏi mơn (Tốn, Lý, Hóa) lớp 12A Trang A B 10 C 18 D 28 Hướng dẫn Có học sinh giỏi mơn học Ta có: học sinh giỏi Tốn Hóa, số học sinh giỏi Tốn, Hóa, không giỏi Lý   (học sinh) học sinh giỏi Lý Hóa, số học sinh giỏi Lý Hóa, khơng giỏi Toán   (học sinh) học sinh giỏi Lý Tốn, số học sinh giỏi Lý Tốn, khơng giỏi Hóa   (học sinh) Số học sinh giỏi Tốn, khơng giỏi Lý, Hóa     (học sinh) Số học sinh giỏi Hóa, khơng giỏi Lý, Toán     (học sinh) Số học sinh giỏi Lý, không giỏi Tốn, Hóa     (học sinh) Từ lập biểu đồ Ven ta được: Theo biểu đồ, số học sinh giỏi môn là:        10 (học sinh)  Chọn B Ví dụ 6: Cho A   ; 2 ; B  3;   ; C   0;  Khi  A  B   C A 3; 4 B 3;  C  ; 2   3;   D  ; 2   3;   Hướng dẫn Ta có A  B   ; 2  3;     A  B   C  3;   Chọn B Ví dụ 7: Cho hai tập hợp A   4;7  B   ; 2    3;   Khi A  B A  ; 2   3;   B  4; 2    3;7  C  4; 2    3;7  D  ; 2   3;   Hướng dẫn Ta có A  B  1;7    ; 2    3;     4; 2    3;7  Trang  Chọn B Bài tập tự luyện Câu Trong tập hợp sau, tập hợp rỗng? A A  x   | x   0 B B  x   | x  2x   0 C C  x   | x   0 D D  x   | x  x  12  0 Câu Cho tập hợp: X  1;3;5;8 ; Y 3;5;7;9 Tập hợp X  Y tập hợp sau đây? A 3;5 B 1;3;5;7;8;9 C 1;7;9 D 1;3;5 Câu Cho A  0;1; 2;3; 4 ; B  2;3; 4;5;6 Tập hợp A \ B A 0 B 0;1 C 1; 2 D 1;5 Câu Cho A  1; 4 ; B  2;6  ; C  1;  Khi đó, A  B  C A 1;6  B  2; 4 C 1; 2 D  Câu Cho A  0; 2; 4;6 Tập A có tập có phần tử? A B C D Đáp án: 1–B 2–B 3–B 4–D 5–B Dạng 3: Số gần sai số Ví dụ minh họa Ví dụ 1: Cho giá trị gần A 0,0025 0,56 Sai số tuyệt đối số 0,56 16 B 0,002 C 0,003 D 0,0075 Hướng dẫn Ta có   0,5625 nên sai số tuyệt đối 0,56 là: 16  0,56  0,5625  0,56  0, 0025 16  Chọn A Ví dụ 2: Độ dài cạnh mảnh vườn hình chữ nhật là: x  7,1m  7cm y  25, 6m  4cm Số đo chu vi mảnh vườn dạng chuẩn A 66m  12cm B 67m  11cm C 66m  11cm D 65m  22cm Hướng dẫn Trang Ta có x  7,1m  7cm  7, 03m  x  7,17 m y  25, 6m  4cm  25,56m  y  25, 64m Do chu vi hình chữ nhật P   x  y    65,18;65, 62  P  65, 4m  22cm Vì d  22cm  0, 22m  0,5  nên chữ số Do dạng chuẩn chu vi 65m  22cm  Chọn D Bài tập tự luyện Câu Cho số gần a  23748023 với độ xác d  101 Hãy viết số quy tròn số a A 23749000 B 23748000 Câu Cho giá trị gần A 0,001 C 23746000 D 23747000 17 0,42 Sai số tuyệt đối số 0,42 40 B 0,002 C 0,004 D 0,005 Câu Một miếng đất hình chữ nhật có chiều rộng x  43m  0,5m chiều dài y  63m  0,5m Tính chu vi P miếng đất cho A P  212m  4m B P  212m  2m C P  212m  0,5m D P  212m  1m C a   a; b  D a   a; b  Đáp án: 1–B 2–D 3–B PHẦN 3: CÁC DẠNG BÀI TẬP Câu Cách viết sau A a   a; b  B a   a; b  Câu Cho giá trị gần  a  3,141592653589 với độ xác 1010 Hãy viết số quy tròn số a A 3,141592654 B 3,1415926536 C 3,141592653 D 3,1415926535 Câu Trong khẳng định sau khẳng định A  \    B *     C *     D *    * Câu Cho X  7; 2;8; 4;9;12 ; Y  1;3;7; 4 Tập sau tập X  Y ? A 1; 2;3; 4;12 B 2;8;9;12 C 4;7 D 1;3 Câu Cho hai tập hợp A  2; 4;6;9 B  1; 2;3; 4 Tập hợp A\ B tập sau đây? A A  1; 2;3;5 B 1;3;6;9 C 6;9 D  Câu Cho A  0;1; 2;3; 4 , B  2;3; 4;5;6 Tập hợp  A \ B    B \ A  ? A 0;1;5;6 B 1; 2 C 2;3; 4 D 5;6 Câu Một ruộng hình chữ nhật có chiều dài x  23m  0, 01m chiều rộng y  15m  0, 01m Tính diện tích S ruộng cho Trang A S  345m  0, 001m B S  345m  0,38m C S  345m  0, 01m D S  345m  0,3801m   3; 11 Tập C C  5; 11  Câu Cho tập hợp C A   3; C B   5;     A 3; B  R  A  B D  3;     3; Câu Số tập phần tử B  a; b;c;d;e;f  A 15 B 16 C 22 D 25 Câu 10 Cho A  x   | x   0 , B  x   |  x  0 Khi A  B A  2;5 B  2;6 C  5; 2 D  2;   Đáp án: 1–B 2–A 3–D 4–C 5–C 6–A 7–B 8–C 9–A 10 – A Trang CHƯƠNG 1: MỆNH ĐỀ, TẬP HỢP, HÀM SỐ CHUYÊN ĐỀ 3: HÀM SỐ BẬC NHẤT, HÀM SỐ BẬC HAI PHẦN 1: LÝ THUYẾT TRỌNG TÂM Hàm số bậc y  ax  b  a  0 Tập xác định: D   Chiều biến thiên: Với a  hàm số đồng biến  Với a  hàm số nghịch biến  Bảng biến thiên: a0 X  a0  x  Y    y   Đồ thị: Đường thẳng song song với đường thẳng y  ax (nếu b  ) qua hai điểm A  0; b  ,  b  B   ;0   a  a0 a0 Chú ý: • Hàm số y  b : Đồ thị hàm số y  b đường thẳng song song trùng với trục hoành cắt trục tung điểm  0; b  Đường thẳng gọi đường thẳng y  b b  x   ax  b a • Đối với hàm số y  ax  b ,  a   ta có: y  ax  b    a  0   ax  b  x   b  a Trường hợp a  ta làm tương tự Trang Do để vẽ hàm số y  ax  b , ta vẽ hai đường thẳng y  ax  b y  ax  b , xóa phần đường thẳng nằm phía trục hồnh Ox • Cho hai đường thẳng d: y  ax  b d : y  a x  b Khi đó: d // d  a  a  b  b d  d  a.a   1 d  d  a  a  b  b d  d  a  a  • Phương trình đường thẳng d qua A  x A ; y A  có hệ số góc k có dạng: y  k  x  x A   y A Hàm số bậc hai y  ax  bx  c  a  0 Tập xác định: D   Bảng biến thiên: a0 X Y   a0 b 2a      4a x  y   b 2a   4a   b   • Nếu a  hàm số y  ax  bx  c nghịch biến khoảng  ;   , đồng biến khoảng 2a    b    ;    2a  b   • Nếu a  hàm số y  ax  bx  c đồng biến khoảng  ;   , nghịch biến khoảng 2a    b    ;    2a  Đồ thị hàm số bậc hai:   b Đồ thị hàm số y  ax  bx  c  a   đường parabol có đỉnh điểm I   ;   , có trục  2a 4a  b đối xứng đường thẳng x   Parabol quay bề lõm lên a  0, xuống a  2a Trang Chú ý: Đồ thị hàm y  f  x   ax  bx  c ,  a   Đồ thị hàm y  f  x   ax  b x  c,  a   • Bước 1: Vẽ  P  : y  ax  bx  c • Bước 1: Vẽ (P): y  ax  bx  c • Bước 2: Do y  f  x  hàm chẵn nên đồ thị đối f  x  f  x   • Bước 2:Do y  f  x    nên xứng qua trục Oy, đồ thị hàm số vẽ f  x  f  x   sau: đồ thị hàm số y  f  x  vẽ sau Giữ nguyên phần (P) bên phải Oy Giữ nguyên phần (P) phía Ox Lấy đối xứng phần qua Oy Lấy đối xứng phần (P) Ox qua Ox Đồ thị y  f  x  hợp hai phần Đồ thị y  f  x  hợp hai phần PHẦN 2: CÁC DẠNG BÀI TẬP Dạng 1: Khảo sát hàm số bậc nhất, bậc hai Ví dụ minh họa Ví dụ 1: Tìm m để hàm số y  m  x    x  2m  1 nghịch biến  A m  2 B m   C m  1 D m   Hướng dẫn y  m  x    x  2m  1  mx  2m  x  2m  1   1  m  x  2m Hàm số bậc y  ax  b nghịch biến  a   1  m   m  1  Chọn C Ví dụ 2: Có giá trị nguyên tham số m thuộc đoạn  2019; 2019 để hàm số y   m   x  2m đồng biến  ? A 2022 B 2019 C Vô số D 2017 Hướng dẫn Hàm số bậc y  ax  b đồng biến a   m    m  Mà m  , thuộc đoạn  2019; 2019 nên m  3; 4;5; ; 2019 Vậy có 2019    2017 giá trị nguyên m cần tìm Trang  Chọn D Ví dụ 3: Cho hàm số y  2x  4x  Chọn đáp án A Hàm số đồng biến khoảng  ; 2  nghịch biến khoảng  2;   B Hàm số nghịch biến khoảng  ; 2  đồng biến khoảng  2;   C Hàm số đồng biến khoảng  ; 1 nghịch biến khoảng  1;   D Hàm số nghịch biến khoảng  ; 1 đồng biến khoảng  1;   Hướng dẫn  b  Áp dụng: Hàm số y  ax  bx  c với a  đồng biến khoảng   ;   , nghịch biến khoảng  2a  b    ;   2a   Ta có  b  1 Do hàm số nghịch biến khoảng  ; 1 đồng biến khoảng  1;   2a  Chọn D Ví dụ 4: Tìm tất giá trị thực tham số m để hàm số y   x   m  1 x  nghịch biến khoảng 1;  A m  B m  C m  D m  Hướng dẫn Hàm số có a  1  0;  b m 1  m 1  ;     hàm số nghịch biến khoảng  2a   m 1  m 1  ;      m  Vậy để hàm số nghịch biến khoảng (1;2) 1;       Chọn C Ví dụ 5: Tìm giá trị lớn M giá trị nhỏ m hàm số y  f  x   x  3x đoạn  0; 2 A M  0; m   B M  ; m  C M  2; m   D M  2; m   Hướng dẫn Cách 1: Hàm số y  x  3x có a   nên bề lõm hướng lên Hoành độ đỉnh x   b    0; 2 2a 3 Ta có: f     ;f    0;f    2 2 3 Vậy: m  y  f     ; M  max y  f    2 Cách 2: Sử dụng máy tính Fx 570 VN PLUS Trang Bước 1: Sử dụng Mode Nhập hàm số F  x   X  3X Start   End   Step 0.2 Bước 2: Quan sát giá trị cột F(x), giá trị nhỏ lớn cột F(x) xấp xỉ giá trị M m cần tìm  Chọn A Bài tập tự luyện Câu Cho hàm số f  x    3x Khẳng định sau đúng? 4  A Hàm số đồng biến khoảng  ;   3  4  B Hàm số nghịch biến khoảng  ;   3  C Hàm số đồng biến  3  D Hàm số đồng biến khoảng  ;   4  Câu Xét tính đồng biến, nghịch biến hàm số f  x   x  4x  khoảng  ;  khoảng  2;   Khẳng định sau đúng? A Hàm số nghịch biến  ;  , đồng biến  2;   B Hàm số đồng biến  ;  , nghịch biến  2;   C Hàm số nghịch biến khoảng  ;   2;   D Hàm số đồng biến khoảng  ;   2;   Câu Tìm giá trị nhỏ y hàm số y  x  4x  A y  B y  2 C y  D y  Đáp án: 1–B 2–A 3–D Dạng 2: Xác định hàm số Ví dụ minh họa Ví dụ 1: Tìm tất giá trị thực tham số m để đường thẳng y   m  3 x  2m  song song với đường thẳng y  x  A m  B m  2 C m  2 D m  Hướng dẫn a  a Để đường thẳng y   m  3 x  2m  song song với đường thẳng y  x   b1  b m   m  2    m  2 m  2m     Trang  Chọn C Ví dụ 2: Tìm a b để đồ thị hàm số y  ax  b qua điểm A  2;1 , B 1; 2  A a  2, b  1 B a  2, b  C a  1, b  D a  1, b  1 Hướng dẫn Đồ thị hàm số qua điểm A  2;1 , B 1; 2  nên ta có hệ phương trình: 1  a  2   b a  1   2  a.1  b b  1  Chọn D Ví dụ 3: Biết đồ thị hàm số y  ax  b qua điểm N  4; 1 vng góc với đường thẳng 4x  y   Tính tích P  ab A P  B P   C P  D P   Hướng dẫn Đồ thị hàm số qua điểm N  4; 1 nên 1  a.4  b 1 Mặt khác, đồ thị hàm số vng góc với đường y  4x  nên 4.a  1  2  1  a.4  b a   Từ (1) (2), ta có hệ    P  ab  4.a  1 b   Chọn A Ví dụ 4: Biết  P  : y  ax  bx   a  1 qua điểm M  1;6  có tung độ đỉnh  Tính tích T  ab A T  3 B T  2 C T  192 D T  28 Hướng dẫn Vì (P) qua điểm M  1;6  có tung độ đỉnh  nên ta có hệ phương trình:  a  16  a  b   a   b a  b  a   b  b  12        a  b  4.2a  a b  9b  36  b    b    b  4a     b  3 a  16 Do a  nên  Suy T  ab  16.12  192 b  12  Chọn C Ví dụ 5: Xác định phương trình parabol (P): y  ax  bx  c, biết (P) qua ba điểm A 1;1 , B  1; 3 O  0;0  Trang A y  x  2x B y   x  2x C y   x  2x D y  x  2x Hướng dẫn Cách 1: Vì (P) qua ba điểm A 1;1 , B  1; 3 , O  0;0  nên ta có hệ phương trình: a  b  c  a  1   a  b  c  3  b  c  c    Vậy phương trình (P): y   x  2x Cách 2: Thay tọa độ ba điểm vào đáp án xem đáp án chứa điểm A, B O  Chọn C Ví dụ 6: Xác định phương trình parabol (P): y  ax  bx  c, biết (P) có đỉnh I  2; 1 cắt trục tung điểm có tung độ 3 A y  x  2x  B y   x  2x  C y  x  2x  D y   x  2x  Hướng dẫn  b  2a  2 b  4a  Vì (P) có đỉnh I  2; 1 nên ta có  b  4ac  4a    1  4a 1 Gọi A giao điểm (P) với Oy điểm có tung độ 3 Suy A  0; 3 Theo giả thiết, A  0; 3 thuộc (P) nên a.0  b.0  c  3  c  3  2  a    a   loai  b  4a    Từ (1) (2), ta có hệ 16a  8a   b  b  2 c  3 c  3 c  3     Vậy phương trình (P): y   x  2x   Chọn B Bài tập tự luyện Câu Biết đồ thị hàm số y  ax  b qua điểm M 1;  song song với đường thẳng y  2x  Tính tổng S  a  b A S  B S  C S  D S  4 Câu Biết đồ thị hàm số y  ax  b qua hai điểm M  1;3 N 1;  Tính tổng S  a  b A S   B S  C S  D S  Trang Câu Xác định phương trình parabol (P): y  ax  bx  c, biết (P) cắt trục Ox điểm có hồnh độ 1 2, cắt trục Oy điểm có tung độ 2 A y  2x  x  B y   x  x  C y  x  x  2 D y  x  x  Đáp án: 1–A 2–C 3–D Dạng 3: Sự tương giao hàm số Ví dụ minh họa Ví dụ 1: Cho hàm số y  2x  m  Tìm giá trị thực m để đồ thị hàm số cắt trục hồnh điểm có hồnh độ A m  B m  C m  7 D m  7 Hướng dẫn Đồ thị hàm số cắt trục hoành điểm có hồnh độ  A  3;0  thuộc đồ thị hàm số Thay x  3, y  vào hàm số ta  2.3  m   m  7  Chọn C Ví dụ 2: Cho hàm số bậc y  ax  b Tìm a b, biết đồ thị hàm số cắt đường thẳng 1 : y  2x  điểm có hồnh độ 2 cắt đường thẳng  : y  3x  điểm có tung độ 2 A a  ; b  B a   ; b  C a   ; b   D a  ; b   Hướng dẫn Với x  2 thay vào y  2x  , ta y  Đồ thị hàm số cắt đường thẳng 1 điểm có hoành độ 2 nên qua điểm A  2;1 Do ta có  a  2   b 1 Với y  2 thay vào y  3x  4, ta x  Đồ thị hàm số cắt đường thẳng y  3x  điểm có tung độ 2 nên qua điểm B  2; 2  Do ta có 2  a.2  b  2  a  1  a  2   b  a  b      Từ (1) (2), ta có hệ phương trình:  2  a.2  b 2a  b  2 b     Chọn C Ví dụ 3: Tìm giá trị thực tham số m để ba đường thẳng y  2x, y   x  y  mx  phân biệt đồng quy Trang A m  7 B m  C m  5 D m  Hướng dẫn Tọa độ giao điểm A hai đường thẳng y  2x y   x  nghiệm hệ:  y  2x x  1   A  1; 2    y   x   y  2 Để ba đường thẳng đồng quy y  mx  qua A  2  1.m   m   Chọn D Ví dụ 4: Tìm phương trình đường thẳng d: y  ax  b Biết đường thẳng d qua điểm I 1;  tạo với hai tia Ox, Oy tam giác có diện tích A y  2x  B y  2x  C y  2x  D y  2x  Hướng dẫn Đường thẳng d: y  ax  b qua điểm I 1;    a  b 1  b  Ta có d  Ox  A   ;0  ; d  Oy  B  0; b   a  Suy OA   b b   OB  b  b (do A, B thuộc hai tia Ox, Oy) a a Tam giác OAB vuông O 1  b Do đó, ta có SABC  OA.OB      b   b  8a   2  a Từ (1) suy b   a Thay vào (2), ta được: 2  a  8a  a  4a   8a  a  4a    a  2 Với a  2  b  Vậy đường thẳng cần tìm d: y  2x   Chọn B Ví dụ 5: Cho parabol (P): y  x  2x  m  Tìm tất giá trị thực m để parabol cắt Ox hai điểm phân biệt có hồnh độ dương A  m  B m  C m  D m  Hướng dẫn Phương trình hồnh độ giao điểm (P) trục Ox x  2x  m   1 Để parabol cắt Ox hai điểm phân biệt có hồnh độ dương (1) có hai nghiệm dương     m  m   đó: S      m  m   P  m     Chọn A Ví dụ 6: Cho parabol (P): y  x  4x  đường thẳng d: y  mx  Tìm giá trị thực tham số m để d cắt (P) hai điểm phân biệt A, B có hồnh độ x1 , x thỏa mãn x13  x 32  Trang A m  B m  2 C m  D m  Hướng dẫn Phương trình hồnh độ giao điểm (P) d là: x  4x   mx  x   x  x   m       x  m  Để d cắt (P) hai điểm phân biệt A, B  m   m  4 Khi đó, ta có x13  x 32      m     m   m  2  Chọn B Bài tập tự luyện Câu Tọa độ giao điểm hai đường thẳng y  A  0; 1  3x x  y     1 3   1 C  0;   4 B  2; 3 D  3; 2  Câu Tìm giá trị thực m để hai đường thẳng d: y  mx   : y  x  m cắt điểm nằm trục tung A m  3 B m  C m  3 D m  Câu Tìm giá trị thực tham số m để ba đường thẳng y  5  x  1 , y  mx  y  3x  m phân biệt đồng quy A m  B m  13 C m  13 D m  Câu Parabol (P): y  x  4x  có số điểm chung với trục hồnh A B C D Câu Cho parabol (P): y  x  2x  m  Tìm tất giá trị thực m để parabol không cắt Ox A m  B m  C m  D m  Đáp án: 1–D 2–A 3–C 4–B 5–B Dạng 4: Đồ thị hàm số Ví dụ minh họa Ví dụ 1: Hình sau đồ thị hàm số nào? A y  x  B y   x  C y  2x  D y   x  Hướng dẫn Trang 10 Đồ thị xuống từ trái sang phải nên hệ số góc a  Loại đáp án A C Đồ thị hàm số cắt trục tung điểm  0;1 Thay x  0; y  vào ta thấy hàm số y   x  thỏa mãn  Chọn D Ví dụ 2: Hình bên đồ thị hàm số nào? A y  x B y  x  C y   x D y  x  Hướng dẫn Giao điểm đồ thị hàm số với trục tung  0;1 Loại đáp án A D Giao điểm đồ thị hàm số với trục hoành  1;0  1;0  Thay vào hai đáp án lại ta thấy y   x thỏa mãn  Chọn C Ví dụ 3: Hình sau đồ thị hàm số nào? A y  x  4x  B y  2x  4x  C y  2x  4x  D y  2x  4x  Hướng dẫn • Parabol có bề lõm hướng lên góc bên phải ngồi hướng lên trên, nên a  Loại C • Đồ thị cắt Oy điểm có tung độ 1 nên c  1 Loại D • Đỉnh parabol điểm 1; 3 Thay vào A B, ta thấy B thỏa mãn Do hàm số y  2x  4x   Chọn B Ví dụ 4: Cho hàm số y  ax  bx  c có đồ thị hình Khẳng định sau đúng? A a  0, b  0, c  B a  0, b  0, c  C a  0, b  0, c  D a  0, b  0, c  Hướng dẫn Bề lõm hướng lên góc ngồi bên phải hướng lên nên a  Trang 11 Hoành độ đỉnh parabol x   b  0, mà a  nên b  2a Parabol cắt trục tung điểm có tung độ dương nên c   Chọn B Ví dụ 5: Cho hàm số f  x   ax  bx  c có đồ thị hình Với giá trị tham số thực m phương trình f  x   m có nghiệm phân biệt A  m  B m  C m  1, m  D 1  m  Hướng dẫn Cách vẽ đồ thị hàm số (C) từ đồ thị hàm số y  f  x  sau: Giữ nguyên đồ thị y  f  x  phía trục hồnh Lấy đối xứng phần đồ thị y  f  x  phía trục hoành qua trục hoành (bỏ phần dưới) Kết hợp hai phần ta đồ thị hàm số y  f  x  hình vẽ Phương trình f  x   m phương trình hồnh độ giao điểm đồ thị hàm số y  f  x  đường thẳng y  m (song song trùng với trục hoành) Dựa vào đồ thị, với  m  phương trình f  x   m có bốn nghiệm phân biệt  Chọn A Ví dụ 6: Cho hàm số f  x   ax  bx  c có đồ thị hình Với giá trị tham số thực m phương trình f  x    m có nghiệm phân biệt A m  B m  1 C m  D m  2 Hướng dẫn Cách vẽ đồ thị hàm số (C) từ đồ thị hàm số y  f  x  sau: • Giữ nguyên đồ thị y  f  x  phía bên phải trục tung • Lấy đối xứng phần đồ thị y  f  x  phía bên phải trục tung qua trục tung Kết hợp hai phần ta đồ thị hàm số y  f  x  hình vẽ Trang 12 Phương trình f  x    m  f  x   m  phương trình hồnh độ giao điểm đồ thị hàm số y  f  x  đường thẳng y  m  (song song trùng với trục hồnh) Dựa vào đồ thị, theo u cầu tốn m    m   Chọn C Bài tập tự luyện Câu Cho hàm số y  ax  b có đồ thị hình bên Tìm a b A a  2 b  B a   C a  3 b  D a  b  2 b  Câu Hình sau đồ thị hàm số nào? A y   x  3x  B y  2x  3x  C y  2x  3x  D y  x  3x  Câu Cho hàm số y  ax  bx  c có đồ thị hình bên Khẳng định sau đúng? A a  0, b  0, c  B a  0, b  0, c  C a  0, b  0, c  D a  0, b  0, c  Đáp án: 1–D 2–C 3–C PHẦN 3: BÀI TẬP TỔNG HỢP Câu Tìm giá trị thực tham số m để parabol (P): y  mx  2mx  3m   m  0 có đỉnh thuộc đường thẳng y  3x  A m  B m  1 C m  6 D m  Câu Cho hàm số y  ax  bx  c có đồ thị hình bên Khẳng định sau đúng? A a  0, b  0, c  B a  0, b  0, c  C a  0, b  0, c  D a  0, b  0, c  Trang 13 Câu Biết (P): y  ax  bx  c , qua điểm A  2;3 có đỉnh I 1;  Tính tổng S  a  b  c A S  B S  C S  D S  14 Câu Xác định phương trình parabol (P): y  ax  bx  c , biết (P) có đỉnh thuộc trục hoành qua hai điểm M  0;1 , N  2;1 A y  x  2x  B y  x  3x  C y  x  2x  D y  x  3x  Câu Hình bên đồ thị hàm số nào? A y  2x  B y  2x   C y  x  D y  3x   x y   1,  a  0; b   qua điểm M  1;6  tạo với tia Ox, Oy tam a b giác có diện tích Tính S  a  2b Câu Đường thẳng d: A S   38 B S  5  7 C S  10 D S  Câu Hình sau đồ thị hàm số nào? A y   x  2x  B y   x  x  2 C y  x  2x D y   x  x  3  a  0 Câu Cho parabol (P): y  ax  bx  c Xét dấu hệ số a biệt thức  phương trình parabol (P) cắt trục hồnh hai điểm phân biệt có đỉnh nằm phía trục hồnh A a  0,   B a  0,   C a  0,   D a  0,   Đáp án: 1–B 2–A 3–D 4–A 5–B 6–C 7–D 8–D Trang 14 CHƯƠNG 2: PHƯƠNG TRÌNH, HỆ PHƯƠNG TRÌNH VÀ BẤT PHƯƠNG TRÌNH CHUYÊN ĐỀ : PHƯƠNG TRÌNH BẬC NHẤT VÀ BẬC HAI MỘT ẨN PHẦN 1: LÝ THUYẾT TRỌNG TÂM Đại cương phương trình Phương trình ẩn Phương trình ẩn x mệnh đề chứa biến có dạng f  x   g  x  (1) Phương trình tương đương Hai phương trình gọi tương đương chúng có tập nghiệm Nếu phương trình f  x   g  x  tương đương Trong đó: f(x) g(x) biểu thức x x gọi ẩn Nếu có số thực x0 cho f  x   g  x  mệnh đề x0 gọi nghiệm phương trình (1) Ta nói: f(x) vế trái phương trình (1), g(x) vế phải phương trình (1) Ta có: f(x) g(x) xác định Df Dg Khi D  Df  Dg gọi tập xác định với phương trình f1  x   g1  x  ta viết f  x   g  x   f1  x   g1  x  Cho phương trình f  x   g  x  xác định D h(x) xác định D Khi đó, ta có: f  x   g  x   f  x   h  x   g  x   h  x  f  x   g  x   f  x  h  x   g  x  h  x  ;  h  x    Phương trình hệ phương trình Nếu nghiệm phương trình Tập hợp chứa tất nghiệm phương trình (1) gọi tập nghiệm phương trình (1) f1  x   g1  x  phương trình f1  x   g1  x  Phương trình nhiều ẩn: gọi phương trình hệ phương trình f  x   g  x  nghiệm phương trình Ví dụ: f x  gx 3x  4y   9x  : phương trình hai ẩn x y Ta viết f  x   g  x   f1  x   g1  x  4x  y  z  : phương trình ba ẩn x; y; z Ta có f  x   g  x   f  x    g  x   Phương trình chứa tham số Ví dụ: 5x  m   : phương trình ẩn x, tham số m 2 Chú ý: Nếu hai vế phương trình ln dấu bình phương hai vế nó, ta 4x  y3   m : phương trình hai ẩn x phương trình tương đương y, tham số m Nếu phép biến đổi tương đương dẫn đến Chú ý: Than sốm phương trình đóng vai trò phương trình hệ quả, ta phải thử lại nghiệm tìm số vào phương trình ban đầu kết luận nghiệm Phương trình bậc nhất, bậc hai ẩn Phương trình bậc ẩn có dạng: ax  b   a   Phương trình bậc hai ẩn có dạng: ax  bx  c   a   Trang b Ta có:   b  4ac  '   b '  ac, b '  b  S  x1  x     a Định lí Vi-ét: Phương trình ax  bx  c   a   có hai nghiệm x1; x2 thì:  P  x x  c  a u  v  S Nếu u v có  u v nghiệm phương trình x  Sx  P  uv  P Một số phương trình quy phương trình bậc nhất, bậc hai Phương trình chứa dấu giá trị tuyệt đối Phương trình chứa ẩn mẫu Phương trình chứa dấu Phương trình bậc ba Phương trình bậc bốn trùng phương PHẦN 2: CÁC DẠNG BÀI TẬP Dạng 1: Tìm điều kiện xác định phương trình Phương pháp giải Một số cách xác định điều kiện: Chú ý: Đa thức xác định với giá trị thuộc  Ta cần phân biệt điều kiện xác định tập xác định f x xác định g  x   Phân thức gx Căn thức f  x  xác định f  x   Phân thức Phân thức f x f x gx Điều kiện xác định điều kiện ẩn  Tập xác định tập hợp Ví dụ: phương trình xác định g  x   gx  x  có điều kiện xác định x  , có tập xác định D   0;   xác định g  x   Ví dụ minh họa Ví dụ 1: Tập xác định phương trình A  \ 2;0 x2   x  x x  x  2 B  \  2;   C  \  ; 2 D  \ 2;0; 2 Hướng dẫn x    x  2   Cách 1: Phương trình có điều kiện xác định :  x   x  x   x    Vậy tập xác định phương trình  \ 2;0; 2 Cách 2: Sử dụng máy tính CASIO fx 570VN PLUS Trang Thử đáp án: Thay x  vào phương trình , ta thấy máy tính MATH ERROR, x  khơng thuộc tập xác định Nên loại đáp án B Thay x  2 vào phương trình, ta thấy máy tính MATH ERROR, x  2 khơng thuộc tập xác định Nên loại đáp án A C  Chọn D Ví dụ 2: Tập xác định phương trình 4 A  \   3 3x    3x  2 4 B  ;  3 3 2 4 C  ;  3 3 2 4 D  \  ;  3 3 Hướng dẫn   x  3x    Cách 1: Phương trình có điều kiện xác định :  4  3x  x   2 4  x   ;  3 3 Cách 2: Sử dụng máy tính CASIO fx 570VN PLUS Thử đáp án: 4 vào phương trình , ta thấy máy tính 1  , x  thuộc tập xác định Nên loại 3 đáp án A, B D Thay x   Chọn C Ví dụ 3: Tìm tập xác định phương trình A D   3x 1 4 x 2 3x  B D   \ 2 C D   \ 3 D D   \ 5 Hướng dẫn  x   Phương trình có điều kiện xác định:  (ln đúng) 3x   Vậy tập xác định phương trình D    Chọn A Ví dụ 4: Cho phương trình 2x  7x 6x    , tập xác định phương trình x  5x  x  6x  x  7x  12 A  4;   B  \ 2;3; 4 C  D  \ 4 Hướng dẫn  x  5x   x    Phương trình có điều kiện xác định :  x  6x     x   x  7x  12  x    Vậy tập xác định phương trình  \ 2;3; 4 Trang  Chọn B Bài tập tự luyện Câu (ID:31923) Tập xác định phương trình A  2;   C  2;   B  \ 2; 2 Câu (ID: 31924) Tập xác định phương trình 2x  5x    x 2x  3x  Câu (ID: 31925) Điều kiện xác định phương trình A x  D  1 2 C  \  ;3;  2 3 B 3;   A  3;     x2 x2 x 4  x   x C x  0; x   B x  1 3 D  \  ;3;  2 2 D x  0; x   Đáp án: 1-B 2-C 3-C Dạng 2: Phương trình tương đương, phương trình hệ Ví dụ minh họa Ví dụ 1: Chọn khẳng định khẳng định sau A 3x  x   x  x   3x  x B 10x   3x  10x   9x C 3x  x   x  3x  x  x  D Cả A, B, C sai Hướng dẫn Đáp án A B: Ta thấy hai phương trình khơng có tập nghiệm Từ suy hai phương trình khơng tương đương với Đáp án C: chuyển vế hạng tử phương trình ta phương trình tương đương  Chọn C Ví dụ 2: Trong khẳng định sau, khẳng định sai? B x   x  A x    x   C x  x  3 x 3   x  D x 2  2x  x 2  Hướng dẫn Đáp án A: Ta bình phương hai vế phương trình hệ Đáp án B: Vì x   x  2 Nên đáp án B sai  Chọn B Ví dụ 3: Phương trình  x    x  phương trình hệ phương trình nào? A x   x  B x   x  C x   x  D x   x  Hướng dẫn Trang Đáp án A: Ta có x   x   x    x  4  Chọn A   Ví dụ 4: Cho phương trình x   x  1 x  1  Phương trình cho tương đương với phương trình B x   A x   C x   D  x  1 x  1  Hướng dẫn Phương trình cho có nghiệm x  x  1 Đáp án A: phương trình có nghiệm x  1 Loại đáp án A Đáp án B: phương trình vơ nghiệm Loại đáp án B Đáp án C: phương trình có nghiệm x  Loại đáp án C Đáp án D: phương trình có nghiệm x  x  1 Chọn đáp án D  Chọn D Ví dụ 5: Cho hai phương trình x  x   1    Chọn khẳng định x2 2x  khẳng định sau A Phương trình (1) (2) tương đương B Phương trình (2) phương trình hệ phương trình (1) C Phương trình (1) phương trình hệ phương trình (2) D Cả A, B, C Hướng dẫn Giải phương trình (1), ta thấy phương trình (1) vô nghiệm 2  x  Giải phương trình (2), ta có điều kiện   x  nên phương trình (2) vơ nghiệm x   Nên đáp án A, B, C  Chọn D Ví dụ 6: Tập nghiệm phương trình A T  0 x  5x  5x  x B T  0;5 C T   \ 0;5 D T  5 Hướng dẫn x   x  5x  Phương trình có điều kiện xác định:   x  5x    x  5x  x  Thay x  x  vào phương trình, ta thấy thỏa mãn Vậy tập nghiệm phương trình T  0;5  Chọn B Bài tập tự luyện Câu 1.(ID:32)Trong phương trình sau, phương trình tương đương với phương trình 3x   ? A 5x   2x  B 4x   x  C 6x   3x  D 7x   x  Trang Câu ID: 38) Phương trình 2x   3x  nhận phương trình sau phương trình hệ quả? A x  2x   B x   C x  5x   D x   Câu (ID: 31) Hai phương trình 2x    2m   x  2m   tương đương A m số nguyên tố C m  1 B m  D m  Đáp án: 1-D 2-C 3-C Dạng 3: Giải biện luận phương trình bậc nhất, bậc hai ẩn Phương pháp giải Giải biện luận phương trình dạng ax  b  1 Trường hợp 1: a  0; b  suy phương Trường hợp 1: a  Ta có:    bx  c  (Đưa dạng trên) Trường hợp : a  Ta có : (2) phương trình (1) nghiệm với x Trường hợp 2: a  0; b  suy phương trình bậc hai ẩn có   b  4ac   , phương trình (2) vơ nghiệm trình (1) vơ nghiệm   , phương trình (2) có nghiệm kép b x 2a a  suy phương trình (1) Là phương trình bậc ẩn   b (1) Có nghiệm x    a Chú ý:   0, phương trình (2) có hai nghiệm phân biệt x1,2  a  Phương trình (1) vơ nghiệm  b  b   2a Chú ý: Phương trình (2) vơ nghiệm Phương trình (1) có vơ số nghiệm a  b   c  a   b  a     Phương trình (2) có nghiệm a  a    b    Phương trình (1) có nghiệm a  Khi tìm điều kiện để phương trình (1) có nghiệm (hoặc vơ nghiệm), ta tìm điều kiện Phương trình (2) có hai nghiệm phân biệt để phương trình (1) vơ nghiệm (hoặc có nghiệm), a  sau lấy kết ngược lại  0  Giải biện luận phương trình dạng ax  bx  c    Ví dụ minh họa Ví dụ 1: Phương trình  a  3 x  b  vô nghiệm giá trị a, b A a  ; b tùy ý B a tùy ý; b  C a  ; b  D a  ; b  Hướng dẫn Ta có :  a  3 x  b    a  3 x   b Trang a   a  Phương trình vơ nghiệm   2  b  b   Chọn C   Ví dụ 2: Với m  a phương trình m  x  3m  m  3 có nghiệm Giá trị a  A B 11 C -11 D -2 Hướng dẫn   Để phương trình m  x  3m  m  3 có nghiệm m    m  3 Vậy a   a    Chọn A   Ví dụ 3: Phương trình m  4m  x  m  3m  có vơ số nghiệm A m  B m  C m  m  D m  Hướng dẫn m   m  4m    m  Phương trình có vơ số nghiệm    m  m  3m   m    m    Chọn D Ví dụ 4: Với điều kiện a phương trình  a   x   4x  a có nghiệm nghiệm âm ? A  a  B a  C  a D a  0;a  Hướng dẫn   Ta có  a   x   4x  a  a  4a x   a a  Phương trình có nghiệm a  4a    a  Khi phương trình có nghiệm x  4a  a  4a a Phương trình có nghiệm âm x     a  a Kết hợp điều kiện trên, ta có  a   Chọn A Ví dụ 5: Cho phương trình x   m   x  2m   1 Giá trị m phương trình (1) có nghiệm ? A m  5 m  1 B m  5 m  1 C 5  m  1 D m  m  Trang Hướng dẫn Phương trình có nghiệm khi:      m     4.1  2m  1    m     2m  1  2  m  1   m    2m    m  6m      m  5  Chọn B Ví dụ 6: Với giá trị tham số m phương trình mx   m   x  m   có hai nghiệm phân biệt? A m  C m  B m  D m  m  Hướng dẫn Phương trình có hai nghiệm phân biệt : m  m  m  m  m           m   m   m    m  m  3     m     4m  m  3   Chọn D Bài tập tự luyện Câu (ID: 20) Để phương trình mx   3x  2m có nghiệm m  m phải khác số nào? A B C 12 D Câu 2.(ID: 24) Tìm tập hợp m để phương trình mx  m  vô nghiệm A  B 0 D  C   Câu (ID: 31947) Phương trình x  m  có nghiệm : A m  B m  C m  D m  Câu (ID: 31948) Cho phương trình mx   m   x  m   Trong khẳng định sau, khẳng định sai ? A Nếu m  phương trình vơ nghiệm B Nếu  m  phương trình có nghiệm x  C Nếu m  phương trình có nghiệm x  D Nếu m  phương trình có nghiệm kép x   m2 4m m2 4m ,x  m m  Câu (ID: 31949) Cho phương trình  x  1 x  4mx   Phương trình có ba nghiệm phân biệt A m   B m   C m  D m  Câu (ID: 311950) Cho phương trình  m  1 x   m  1 x  2m   (1) Với giá trị sau m phương trình (1) có nghiệm kép ? Trang A m  B m  C m   D m  1 Đáp án: 1-C 2-A 3-C 4-D 5-B 6-C Dạng 4: Ứng dụng định lí Vi -ét Phương pháp giải Cho phương trình bậc hai ax  bx  c   a   Phương trình (1) có hai nghiệm phân biệt dương 1    S  Phương trình (1) có hai nghiệm x1; x2 Khi ta có: P   b  S  x1  x   a Phương trình (1) có hai nghiệm phân biệt âm    P  x x  c  a  S  P  Chú ý:  Phương trình (1) có hai nghiệm trái dấu ac  Ví dụ minh họa Ví dụ 1: Cho phương trình      x   x    Chọn khẳng định A Phương trình vơ nghiệm B Phương trình có hai nghiệm dương C Phương trình có hai nghiệm âm D Phương trình có hai nghiệm trái dấu Hướng dẫn Vì ac    1    nên phương trình có hai nghiệm trái dấu  Chọn D Ví dụ 2: Hai số   nghiệm phương trình A x  2x   B x  2x   C x  2x   D x  2x   Hướng dẫn S  x1  x  Đặt x1   2; x   Ta có  P  x1.x  1 Suy phương trình nhận x1; x2 nghiệm x  Sx  P   x  2x    Chọn B Ví dụ 3: Cho phương trình x   m  1 x  m   Giá trị m để phương trình có ba nghiệm phân biệt A m  B m  C m  D m không dương Hướng dẫn Đặt t  x  0, phương trình cho trở thành t   m  1 t  m   1 Trang Để phương trình cho có ba nghiệm phân biệt phương trình (1) có nghiệm t  nghiệm dương Thay t  vào (1) ta được: 02   m  1  m    m    m  2 t  x  x  Với m  , phương trình (1) trở thành t  t      t   x  1  x  Vậy với m  phương trình ban đầu có ba nghiệm  Chọn A Ví dụ 4: Phương trình x  mx  m   (với m tham số) có nghiệm với m Gọi x1, x2 hai nghiệm phương trình Tìm hệ thức liên hệ x1, x2 không chứa m A x1.x  2x1  x  B x1.x  x1  x  C x1.x  x1  x  2 D x1.x  x1  x  Hướng dẫn x  x  m  m  x1  x Áp dụng định lí Vi-ét có    x1 x  m   x1 x  x1  x  Vậy hệ thức liên hệ x1; x2 không chứa m x1x  x1  x   Chọn B Ví dụ 5: Cho phương trình x  7x  260  có nghiệm x1  13 Tìm x2 A -20 B -27 C 20 D Hướng dẫn Ta có x1  x  7  x  7  x1  20  Chọn A Ví dụ 6: Có giá trị m để phương trình 2x  3x  m  có hai nghiệm thỏa mãn hệ thức x12  x 22  1? A vô số B C.0 D Hướng dẫn Phương trình có hai nghiệm     3  4.2.m    8m   m    x1  x  Áp dụng hệ thức Vi-ét ta có  x x  m  2 m 3 Ta có x12  x 22    x1  x   2x1x        m  (loại) 2 Vậy khơng có giá trị m thỏa mãn  Chọn C Trang 10 Ví dụ 7: Cho phương trình x   m   x  m   Tổng bình phương giá trị m để phương trình có hai nghiệm phân biệt nghiệm gấp hai lần nghiệm kia? A B C.2 D Hướng dẫn Ta có: x   m   x  m   Phương trình có hai nghiệm phân biệt    m    4m   m   m  m2  x   x1  x  m   m   m2   Theo định lí Vi-ét giả thiết ta có  x1x  m   2    m 1   m    x  2x     x1  2x    1  Vậy tổng bình phương giá trị m thỏa mãn        Chọn B Bài tập tự luyện Câu (ID: 229) Biết phương trình x  2mx  m   có hai nghiệm phân biệt x1; x2 với m Tìm m để x1  x  2x1x   A m  m  2 B m  C m  D m  Câu (ID: 231) Xét phương trình ax  bx  c  Điều kiện để phương trình có hai nghiệm phân biệt không dương a     A  ab  ac     B ab  ac   a     C  ab  ac  a     D  ab  ac  Câu (ID: 288) Cho phương trình x   m  1 x  m   Tìm m để phương trình có hai nghiệm phân biệt cho A  x12  x 22  3x1x đạt giá trị lớn A m  4 B m  2 C m  8 D m  Câu (ID: 291) Cho phương trình 3x   m  1 x  m  4m   Có giá trị m để phương trình có hai nghiệm x1; x2 thỏa mãn A B 1    x1  x  ? x1 x 2 C D Đáp án: 1-A 2-D 3-A 4-B Trang 11 Dạng 5: Một số phương trình quy phương trình ẩn Ví dụ minh họa Ví dụ 1: Tập nghiệm phương trình x   3x  tập hợp sau đây?  3 A  ;    2  7 B  ;   4 3 7 C  ;  2 4  3 D  ;   2 Hướng dẫn  x   x   3x   2x    Ta có x   3x     x    3x  4x  x    Chọn C Ví dụ 2: Phương trình 2x   x   có nghiệm? A B C.0 D Vô số Hướng dẫn Điều kiện xác định : x   x  10  2x   x    x  10    Ta có 2x   x     (loại) x   2x   x    3x    Vậy phương trình vơ nghiệm  Chọn C Ví dụ 3: Tập nghiệm phương trình A S  2 x  5x   2x  2x  B S  1 C S  0;1 D S  7 Hướng dẫn Điều kiện xác định : x   x  (loại x  5x  ) Ta có  2x   x  5x   2x   x  7x    2x   x  (thỏa mãn) Vậy S  7  Chọn D Ví dụ 4: Số nghiệm phương trình A   x  x  3x   B C.1 D Hướng dẫn Điều kiện xác định: x  1 Trang 12 Ta có  x  1  x 1  x  x  3x       x  (thỏa mãn)  x  3x    x    Vậy phương trình có ba nghiệm  Chọn D Ví dụ 5: Phương trình sau có bao nhiều nghiệm âm x  2019x  2018  0? A B C.2 D.6 Hướng dẫn Phương trình x  2019x  2018  (1) Đặt x  t, phương trình (1) trở thành t  2019t  2018  (2) Ta thấy:  2018   suy phương trình (2) có nghiệm t trái dấu Với nghiệm t âm ta có nghiệm x âm Vậy phương trình (1) có nghiệm âm  Chọn B Ví dụ 6: Phương trình  x  A      x   2  có nghiệm? B C D Hướng dẫn Phương trình  x      1 x2   2  (1) Đặt t  x  t   , phương trình (1) trở thành  t       1 t   2  (2)  Phương trình (2) có a.c   1  2  Suy phương trình (2) có hai nghiệm trái dấu Suy phương trình (1) có hai nghiệm phân biệt  Chọn A Bài tập tự luyện Câu (ID: 730) Tổng bình phương tất nghiệm x   A 41 B Câu (ID :745) Phương trình A m  x  x 1 B m  Câu (ID: 755) Cho phương trình phương trình A B -1 C 25 x2  x   x2 D 81 m có nghiệm x 1 C m  D m  x  x   x  x   2x  2x  Tổng nghiệm C D Đáp án: Trang 13 1-C 2-A 3-B PHẦN 3: BÀI TẬP TỔNG HỢP Câu (ID: 22) Với a  0; b  phương trình ax  b  A Có nghiệm B Có vơ số nghiệm C Vơ nghiệm D Có hai nghiệm phân biệt   Câu (ID: 27) Cho phương trình m  3m  x  m  4m   Có giá trị m để phương trình có tập nghiệm  ? A B C D Vô số Câu (ID: 37) Trong phương trình sau, phương trình có nghiệm nhất? A 3x   x     2x  B  x  1  x  C x   3x   2x  1 D x  3x  Câu (ID: 66) Tổng giá trị m để phương trình A B C Câu (ID: 682) Tập xác định phương trình A  2;5 2x  3m x    vô nghiệm x2 x 1 D 11 x  x2 x 1 5 x B  2;5 \ 1 C  2;5 \ 1 D  2;5  \ 1 Câu (ID: 690) Phương trình  x    x  x  16 tương đương với phương trình đây? x   A    x  x  1  x   B   x       x  x     x  x   C  x    1  x   x   D    x  x    Câu (ID: 711) Tổng bình phương giá trị m để phương trình m  x   vô nghiệm là: A B C    D 29 Câu (ID: 726) Phương trình 9x  3x   5x   có nghiệm ? A B C  D  Câu (ID: 759) Phương trình x   x  có nghiệm x1; x2 Tính x1  x A B C  37 37  D  37  Câu 10 (ID: 761) Số nghiệm phương trình x  3x   x  A B C D Trang 14 1-C 2-A 3-B 4-D 5-D 6-B 7-B 8-A 9–B 10 - B Trang 15 CHƯƠNG 2: PHƯƠNG TRÌNH, HỆ PHƯƠNG TRÌNH VÀ BẤT PHƯƠNG TRÌNH CHUYÊN ĐỀ 2: HỆ PHƯƠNG TRÌNH BẬC NHẤT VÀ BẬC HAI HAI ẨN PHẦN 1: LÝ THUYẾT TRỌNG TÂM Các hệ phương trình bản: Các hệ phương trình đặc biệt: Hệ hai phương trình bậc hai ẩn Hệ phương trình đối xứng loại Hệ phương trình đối xứng loại a1x  b1 y  c1  a x  b y  c Hệ phương trình đẳng cấp bậc Hệ ba phương trình bậc ba ẩn a1x  b1 y  c1z  d1  a x  b y  c z  d a x  b y  c z  d 3  PHẦN 2: CÁC DẠNG BÀI TẬP Dạng 1: Hệ phương trình bậc hai, ba ẩn Phương pháp giải  Một số phương pháp giải hệ phương trình Hệ có nghiệm a1b  a b1  Phương pháp Khi hệ có nghiệm Phương pháp đại số  c1b  c b1 a c a c  ;y  2  x  a1b  a b1 a1b  a b1   Đặt ẩn phụ Chú ý: Ta sử dụng máy tính để tìm nghiệm  số hệ phương trình đơn giản Sử dụng máy tính CASIO fx 570VN PLUS: MODE  Nếu máy tính No – Solution hệ phương trình vơ nghiệm  Nếu máy tính Infinite – Sol hệ phương  trình có vơ số nghiệm  Nếu hệ phương trình có nghiệm, máy tính cho kết x, y Hệ vô nghiệm a1b  a b1   c1b  c b1  a1b  a b1   a1c  a c1  a1b  a b1   Hệ có vô số nghiệm c1b  c b1  a c  a c   2 Biện luận hệ phương trình bậc hai ẩn a1x  b1 y  c1  a x  b y  c 2 Ví dụ minh họa  3x  y  Ví dụ 1: Nghiệm hệ phương trình  4x  3y      A 2  3;    B 2  3;  Trang    C  3;  D 2  3;   Hướng dẫn Cách 1: Từ phương trình đầu ta y   3x Thế vào phương trình thứ hai ta được:    2     4x   3x  2  4x   3x  2  x  2  Suy y   Cách 2: Sử dụng máy tính fx 570VN PLUS Nhập MODE 1, sau nhập hệ số hệ phương trình 3 4 1 1 Ta nhận  2  nghiệm đáp án B → Chọn B Ví dụ 2: Hệ phương trình sau có nghiệm? 5x  y  A  10x  2y  1  x  y  B  2x  2y  6 x  y  C   x  2y  3x  y  D  6x  2y  Hướng dẫn Bấm nghiệm hệ phương trình này, ta thấy hệ phương trình đáp án C có nghiệm → Chọn C 2x  y   Ví dụ 3: Hệ phương trình  x  2z   2 có nghiệm  a; b;c  Giá trị a  b  c  y  z   A 3  B 3  C  D  Hướng dẫn Cách 1: Từ phương trình đầu ta có y   2x , vào phương trình thứ ba ta được:  2x  z    2x  z  2  Kết hợp với phương trình thứ hai, ta có hệ phương trình: 2x  z  2  2x  z  2  2x  z  2        x  2z   2 2x  4z   5z  2x   2   x     y  z  z    Suy nghiệm hệ phương trình 1; 2; Tức a  1; b  2;c  Vậy a  b  c      Cách 2: Sử dụng máy tính fx 570VN PLUS Nhập MODE 2, sau nhập hệ số hệ phương trình: Trang 2  1  4     2  Ta nhận nghiệm hệ phương trình 1; 2;  1 1 2    Vậy a  b  c      → Chọn C ax  2y  a  Ví dụ 4: Cho hệ phương trình  Với giá trị tham số a hệ có nghiệm nhất? 2x  ay  2a  A a  2 B a  C a  3 D a  2 Hướng dẫn Hệ có nghiệm a1b  a b1  0, tức a.a  2.2   a    a  2 → Chọn D  x  4y  Ví dụ 5: Hệ phương trình  có nghiệm  x; y  Giá trị 3x  2y  x  2y  A -1 B.4 C.3 D.2 Hướng dẫn  x  4y   x   2y  x   2y x  Ta có:      3x  2y  2 y   x  2y    2y   4y   y   Chọn B 2x  y  Ví dụ 6: Cho hệ phương trình  Tìm a để hệ có nghiệm  x; y  cho biểu thức 2y  x  10a  x  y đạt giá trị nhỏ A a   B a   D a  C a  1 Hướng dẫn 2x  y  2x  y  5y  20a  15  y  4a  Ta có     2y  x  10a  2x  4y  20a  10 2x  y   x   2a x  y   4a  3  1  2a   16a  24a    4a  4a 2 2 1 1 1 1     20a  20a  10  20  a  a    20  a  2a     20  a     5, a   2 4 2    Dấu xảy a   → Chọn A Bài tập tự luyện Câu (ID: 352) Hệ phương trình sau có nghiệm 1;1 Trang x  y  A   x  2y  2x  y  B  4x  2 x  y  C   x  2y  4x  y  D  y  ax  y  a Câu (ID: 382) Với a  m hệ phương trình  có nghiệm Tổng lập phương giá trị x  ay   m A B C D -1 x  y   Câu (ID: 380) Hệ phương trình  y  z  1 có nghiệm  a; b;c  Khi a  c  b z  x   A B C D Đáp án: 1-C 2–D 3-B Dạng 2: Hệ phương trình đối xứng loại 1, loại Phương pháp giải Hệ đối xứng loại Hệ đối xứng loại Dấu hiệu: Khi thay đổi vị trí x y cho Dấu hiện: Khi thay đổi vị trí x y cho hệ phương trình khơng thay đổi trật tự hệ khơng thay đổi trật tự phương phương trình khơng thay đổi trình thay đổi (phương trình thành phương trình kia) Phương pháp: Phương pháp: Biến đổi dạng tổng tích hai biến Đặt Lấy vế trừ vế phân tích thành nhân tử, ý ta S  x  y, P  xy Giải hệ với ẩn S P với điều kiện có nghiệm ln nhận x  y  x; y  S2  4P Ta có x; y nghiệm phương trình Chú ý: Ta thử đáp án với tập hỏi nghiệm t  St  P  Ví dụ minh họa  x.y  x  y  11 Ví dụ 1: Cho hệ phương trình  Khẳng định đúng? x y  xy  30  A Hệ có nghiệm  5;6  B Hệ có hai nghiệm  2;1  3;5  C Hệ có hai nghiệm  2;3 1;5  D Hệ có bốn nghiệm  2;3 ,  3;  , 1;5  ,  5;1 Hướng dẫn Đây hệ phương trình đối xứng loại  x.y  x  y  11  x.y  x  y  11 Ta có:   Đặt S  x  y, P  xy S2  4P  xy x  y  30   x y  xy  30     Khi đó, hệ phương trình tương đương với hệ sau: Trang S  11  P S  11  P S  P  11 S  11  P S  6; P      P     S  5; P   P  11P  30  11  P  P  30 SP  30   P   t  Trường hợp 1: S  P  , x; y nghiệm phương trình t  6t     t  x  x  Suy   Nên hệ phương trình có nghiệm 1;5  ,  5;1 y  y  t  Trường hợp 2: S  P  , x; y nghiệm phương trình t  5t     t  x  x  Suy   Nên hệ phương trình có nghiệm  2;3 ,  3;  y  y  Vậy hệ phương trình có nghiệm  2;3 ,  3;  , 1;5  ,  5;1 → Chọn D  x  xy  y  Ví dụ 2: Hệ phương trình  có cặp nghiệm  x; y  ? x  xy  y   A B C D Hướng dẫn Đây hệ phương trình đối xứng loại  x  y 2  xy   x  xy  y   x  2xy  y  xy  Ta có     x  xy  y   x  y  xy   x  y  xy    Đặt S  x  y, P  xy S2  4P  Khi ta thu được: S2  P  S2  P  S2  S   S    S      S  P  P   S P   S P   S  S  S  2; P     S  3   S  3; P  P   S  t  Trường hợp 1: S  P  , ta có x; y nghiệm phương trình : t  2t    t  x  x   Nên hệ phương trình có hai nghiệm  y  y  Trường hợp 2: S  3 P  , ta có x; y nghiệm phương trình : t  3t   (vơ nghiệm) Nên hệ phương trình có nghiệm Vậy hệ phương trình có hai cặp nghiệm  x; y  → Chọn B Trang  x  2x  3y Ví dụ 3: Hệ phương trình  có số cặp nghiệm  y  2y  3x A B C D Hướng dẫn Đây hệ phương trình đối xứng loại Trừ hai vế hai phương trình ta được: x  y3  2x  2y  3y  3x  x  y3  x  y      x  y  x  xy  y   x  y   2  x  xy  y   (vơ nghiệm) Vì x  xy  y   0, x, y Nên ta có x  y x  Thay x  y vào phương trình thứ ta có x  2x  3x  x  5x    x    Vậy nghiệm hệ phương trình  0;0  ,    5; ,  5;   Chọn D Bài tập tự luyện  x  y  xy  Câu (ID: 392) Hệ phương trình  có số cặp nghiệm  x  y  xy  A B C D  x y  xy  2m Câu (ID: 381) Có giá trị m để hệ phương trình  có nghiệm ? x  y  A B C D  x  y  xy  Câu 3.(ID: 31997) Hệ phương trình  có cặp nghiệm  x  y  xy  A 1;   2;1 B  2; 3  3; 2  ; C  2;3  3;  ; D  1; 2   2; 1 2x  9x  5y Câu (ID: 31998) Cặp nghiệm  x; y  , x  0, y  hệ phương trình  2y  9y  5x A  3;3 B 1;1 ,  2;  C  2;  D  2;  ,  3;3  x  7y  15 Câu (ID: 31999) Hệ phương trình  có nghiệm ?  y  7x  15 A B C D Đáp án: Trang 1-B 2-B 3-A 4-C -D Dạng 3: Hệ phương trình đẳng cấp Phương pháp giải Lấy (1) - (2) ta thu phương trình đẳng cấp bậc 2, từ ta tìm mối liên hệ x y a1x  b1xy  c1 y  d1 Dạng tổng quát  2 a x  b xy  c y  d Phương pháp: Chú ý: a1x  b1xy  c1 y  d1 Ta có:  2 a x  b xy  c y  d   Ta thử đáp án với tập hỏi nghiệm   d a1x  b1xy  c1 y  d1.d   2 d1 a x  b xy  c y  d1.d 1  2 Ví dụ minh họa 3x  4xy  2y  17 Ví dụ 1: Hệ phương trình  có nghiệm?  y  x  16 A B C D Hướng dẫn 2 3x  4xy  2y  17 Ta có   16 3x  4xy  2y  17 y  x 2  y  x  16      65x  64xy  15y   13x  5y  5x  3y   x y hay x  y 13 5 144 169 5  Trường hợp 1: Với x  y , ta có y   y   16  y  16  y  13 169  13  13  y   x    y   13  x    3 16 3  Trường hợp 2: Với x  y , ta có y   y   16  y  16  y  25 5 25   y   x    y  5  x  3 Vậy hệ phương trình có bốn cặp nghiệm → Chọn A 2 3x  5xy  4y  3 Ví dụ 2: Nghiệm hệ phương trình  9x  11xy  8y  Trang A  3;  ,  3;  C  3; ,  3;    B      1   D  ; ; ,   2  2  2;  ,  2;  Hướng dẫn 3x  5xy  4y  3 Ta có:   3x  5xy  4y  3 9x  11xy  8y 2 9x  11xy  8y       45x  3xy  48y    x  y 15x  16y    x  y hay x   16 y 15 Trường hợp 1: Với x  y, ta có: 9x  11x  8x   12x   x  1   x  y=  1   x   y    Trường hợp : Với x   16 y, ta có 15 256 176  16   16    y   11  y  y  8y   y  y  8y  25 15  15   15   712 y  (phương trình vơ nghiệm) 75   1   Vậy nghiệm hệ phương trình  ; ; ,   2  2   Chọn D PHẦN 3: BÀI TẬP TỔNG HỢP Câu (ID: 357) Hệ phương trình sau vơ nghiệm ? x  y  A   x  2y   x  y  B  2x  2y  6 4x  3y  C   x  2y  x  y  D   x  y  3  x  my  Câu (ID: 365) Hệ phương trình  có nghiệm mx  y  m  A m  B m  1 C m  D m  1 x  y  Câu (ID: 32005) Hệ phương trình  có nghiệm  x.y  90 A 15;6  ,  6;15  B  15; 6  ,  6; 15  C 15;6  ,  6; 15  D 15;6  ,  6;15  ,  15; 6  ,  6; 15  Trang Câu (ID: 384) Cho tam giác vng Khi ta tăng cạnh góc vng lên 2cm diện tích tam giác tăng thêm 17cm2 Nếu ta giảm cạnh góc vng 3cm 1cm diện tích tam giác giảm 11cm2 Tính diện tích tam giác ban đầu A 50cm2 C 50 5cm B 25 cm2 D 50 2cm 2x  y  Câu (ID: 32002) Hệ phương trình  có nghiệm y  x  m A m   B m  C m   D m tùy ý Câu (ID: 385) Một công ty Taxi có 85 xe chở khách gồm hai loại, xe bốn chỗ xe chỗ Dùng tất xe đó, tối đa lần cơng ty chở 445 khách Hỏi cơng ty có xe loại? A 50 xe chỗ 35 xe chỗ B 35 xe chỗ 50 xe chỗ C 45 xe chỗ 40 xe chỗ D 40 xe chỗ 45 xe chỗ 2x  y   Câu (ID: 32003) Hệ phương trình  x  2z   2 có nghiệm  y  z     A 1; 2; 2   B 1; 2;   C 1;6;   D 2;0; Câu (ID: 387) Một ruộng hình chữ nhật có chu vi 250m Tìm chiều dài chiều rộng ruộng biết ta giảm chiều dài lần chiều rộng tăng lần chu vi ruộng không đổi A 32m 25m B 50m 45m C 75m 50m D 60m 40m  x   y  Câu (ID: 32004) Hệ phương trình  có cặp nghiệm? 2x  y  A B C D  x  y  3x  2y  Câu 10 (ID: 32006) Cho hệ phương trình  Từ hệ phương trình ta thu x  y  phương trình sau đây? A 10x  24  B 16x   C 3x   D Một kết khác 2  x  y  13  Câu 11.(ID: 32007) Hệ phương trình  có nghiệm    12  x y 1 A x  ; y   1 B x  ; y  1 C x   ; y  D Hệ vô nghiệm 2x  y  3xy  12 Câu 12 (ID: 32008) Hệ phương trình  có cặp nghiệm  x; y  với giá trị x 2 x  y  y  14    y thỏa mãn x  0; y  Trang A 1;  ,   2; B  2;1 ,   3;  1   D  ;1 ,  ;  2     2   C  ;3  ,  3;  3 3    x  y  10 Câu 13.(ID: 32009) Hệ phương trình  có nghiệm x  y  58  x  A  y  x  B  y  x  C  , y  x   y  D Một đáp số khác  x  3x  y3  3y Câu 14 (ID :32010) Hệ phương trình  có nghiệm?  x  y  27 A B C D 3  x  y    x  y   17 Câu 15 (ID: 320011) Hệ phương trình  có nghiệm  x  y    x  y   5 1 7 A  ;  2 2  7 B   ;    2 1 7 C  ;   2 2  7 D   ;   2 Câu 16 (ID: 359) Hệ phương trình sau có vơ số nghiệm ? x  y  A   x  2y  2x  y  B  4x  2y  2 3x  y  C   x  2y  4x  y  D   x  2y  mx  2y  m  Câu 17 (ID: 368) Cho hệ phương trình  mệnh đề sau 2x  my  2m  Hệ phương trình có nghiệm m  Hệ phương trình có vơ số nghiệm m  2 Hệ vô nghiệm m  Các mệnh đề A Chỉ B Chỉ C Chỉ D Cả 1, 2, Đáp án: 1-B 2-D 3-C 4-B -A 6–B 7–B 11 – B 12 – A 13 – C 14 – A 15 – D 16 – B 17 - C 8–C 9–B 10 - D Trang 10 CHƯƠNG 2: PHƯƠNG TRÌNH, HỆ PHƯƠNG TRÌNH VÀ BẤT PHƯƠNG TRÌNH CHUYÊN ĐỀ 3: BẤT ĐẲNG THỨC PHẦN 1: LÝ THUYẾT TRỌNG TÂM Khái niệm Các mệnh đề dạng "a  b" "a  b" gọi bất đẳng thức Nếu mệnh đề "a  b  c  d" bất đẳng thức c < d bất đẳng thức hệ bất đẳng thức a 0, ta có a   2 Nếu x, y dương có tổng khơng đổi tích xy lớn x = y (Trong hình chữ nhật có chu vi, hình vng có diện tích lớn nhất) Nếu x, y dương có tích khơng đổi tổng x + y nhỏ x = y (Trong tất hình chữ nhật có diện tích, hình vng có chu vi nhỏ nhất) Bất đẳng thức giá trị tuyệt đối x  0, x  x, x   x Với a > 0: x  a  a  x  a a  b  ab  a  b x  a  x  a x  a PHẦN 2: CÁC DẠNG BÀI TẬP Phương pháp giải Áp dụng công thức phần lý thuyết Ví dụ minh họa Ví dụ 1: Mệnh đề sau đúng? A x  , x  2  x  B x  , x   x  C x  , x   x  C x  , x   x  2 Trang Hướng dẫn x  Ta có mệnh đề tương đương x    , nên có mệnh đề đáp án C  x  2 →Chọn C Ví dụ 2: Cho số x > 6, số số sau nhỏ nhất? A x B  x C  x D x Hướng dẫn Ta có 6 6 x     Để tìm số nhỏ nhất, ta so sánh  x x x x 6 6  x   x     Vì x    x 1  Suy x 1  x Vậy số nhỏ số  x →Chọn C Ví dụ 3: Giá trị nhỏ biểu thức P  x  8x  12 với x   A – B – C – D – Hướng dẫn Ta có P  x  8x  12  x  2x.4  16   (x  4)  (x  4)   (x  4)   4  P  4 Vậy P đạt giá trị nhỏ – x    x  →Chọn B Ví dụ 4: Cho biểu thức f (x)   x Kết luận sau đúng? A Hàm số f (x) có giá trị lớn nhất, khơng có giá trị nhỏ B Hàm số f (x) có giá trị nhỏ nhất, khơng có giá trị lớn C Hàm số f (x) có giá trị nhỏ giá trị lớn D Hàm số f (x) khơng có giá trị nhỏ khơng có giá trị lớn Hướng dẫn Ta có f (x)  f(1) = Vì x    x    x  Mà  x  2 Suy   x    x   f (x)  f(0) = Vậy hàm số f(x) có giá trị nhỏ giá trị lớn →Chọn C Trang Ví dụ 5: Cho biểu thức T  x  3x  với x  , giá trị nhỏ biểu thức A B C D Hướng dẫn  3 Ta có T  x  3x   x  2x     x    4  2 2 2 3  3 5 25   25 Vì x   x       x        x       2  2 2 2 4   T  Vậy T đạt giá trị nhỏ x = →Chọn A Ví dụ 6: Với hai số x, y dương thỏa mãn xy  36, bất đẳng thức sau đúng? A 4xy  x  y B x  y  2xy  72 C x  y  xy  12 xy D    xy  36   Hướng dẫn Áp dụng bất đẳng thức Cô-si cho hai số không âm x, y ta có: x  y  xy  36  12 →Chọn C PHẦN 3: BÀI TẬP TỔNG HỢP Câu Nếu a  2c  b  2c bất đẳng thức sau đúng? A 3a  3b B a  b C 2a  2b D 1  a b Câu Nếu  a  bất đẳng thức sau đúng? A  a a B a  a Câu Tìm giá trị nhỏ m hàm số f (x)  x  A m   2 D a  a C a  a B m   2 với x > x 1 C m   D m   Câu Trong khẳng định sau, khẳng định đúng? 0  a  b a b A    c d 0  c  d a  b  a b B    c d c  d  a  b a b C    c d c  d a  b  a d D    b c c  d  Câu Tìm giá trị nhỏ m hàm số f (x)  A m = B m = x  2x  với x > - x 1 D m  C m = Đáp án: 1-C 2-A 3-B 4-D 5-C Trang Trang CHƯƠNG 2: PHƯƠNG TRÌNH, HỆ PHƯƠNG TRÌNH VÀ BẤT PHƯƠNG TRÌNH CHUYÊN ĐỀ 4: BẤT PHƯƠNG TRÌNH VÀ HỆ BẤT PHƯƠNG TRÌNH PHẦN 1: LÝ THUYẾT TRỌNG TÂM Định nghĩa bất phương trình ẩn Cho hai hàm số y  f (x) y  g(x) có tập xác định Df Dg Đặt D  Df  Dg Mệnh đề chứa biến có dạng f (x)  g(x), f(x)  g(x), f(x)  (x), f (x)  g(x) gọi bất phương trình ẩn; x gọi ẩn số (hay ẩn) D gọi tập xác định bất phương trình x  D nghiệm bất phương trình f (x)  g(x) f (x )  g(x ) mệnh đề Bất phương trình tương đương, biến đổi tương đương bất phương trình Định nghĩa: Hai bất phương trình (cùng ẩn) gọi tương đương chúng có tập nghiệm Ký hiệu: Nếu f1 (x)  g1 (x) tương đương với f (x)  g (x) ta viết f1 (x)  g1 (x)  f (x)  g (x) Phép biến đổi không làm thay đổi tập nghiệm bất phương trình gọi phép biến đổi tương đương Định lý: Cho bất phương trình f (x)  g(x) có tập xác định D; y  h(x) hàm số xác định D Khi D, bất phương trình cho tương đương với bất phương trình sau: 1) f (x)  h(x)  g(x)  h(x) 2) f(x).h(x)  g(x).h(x) h(x)  với x  D 3) f(x).h(x)  g(x).h(x) h(x)  với x  D Hệ quả: Cho bất phương trình f (x)  g(x) có tập xác định D Khi đó: 1) f (x)  g(x)  f (x)  g (x) 2) f (x)  g(x)  f (x)  g (x) với f (x)  0, g(x)  0, x  D PHẦN 2: CÁC DẠNG BÀI TẬP Dạng 1: Bất phương trình hệ bất phương trình bậc Phương pháp giải Giải bất phương trình dạng ax  b  (1) Nếu a = bất phương trình có dạng 0.x + b (1)  x   b b  suy tập nghiệm S   ;   a a  Nếu a < (1)  x   b  b  suy tập nghiệm S    ;   a  a  Các bất phương trình dạng ax  b  0, ax  b  0, ax  b  giải tương tự Trang Chú ý: Để giải hệ bất phương trình bậc ẩn ta giải bất phương trình hệ bất phương trình Khi tập nghiệm hệ bất phương trình giao tập nghiệm bất phương trình Ví dụ minh họa Ví dụ 1: Chi nhị thức bậc f (x)  5x  20 Khẳng định sau đúng? A f (x)  với x   B f (x)  với x   ; 4  C f (x)  với x   ;  D f (x)  với x   4;   Hướng dẫn 5x  20   5x  20  x  Vậy f (x)  với x   ;  →Chọn C Ví dụ 2: Tập nghiệm bất phương trình A  1;   2x B (0; ) C (; 4) D (; 1)  (2; ) Hướng dẫn Cách 1: Ta có 3 3 2 x 1 x 1 1   0  2x 2x 2x 2x Khi ta có bảng xét dấu sau:  x -1  -2 1+x - + + 2–x + | + - f(x) - + || - Nhìn vào bảng xét dấu ta thấy tập nghiệm bất phương trình (; 1)  (2; ) Cách 2: Sử dụng máy tính CASIO Fx570 VNPLUS Bước 1: Nhập hàm số 1 2X Bước 2: Sử dụng phím CALC Chọn giá trị x thuộc đáp án không thuộc đáp án để loại bỏ đáp án làm biểu thức không âm Chọn x = 0, nhập CALC = ta kết  0, loại đáp án chứa x = 0, loại A C Chọn x = 1, nhập CALC 1= ta kết > 0, loại đáp án chứa x = 1, loại B → Chọn D Ví dụ 3: Tìm giá trị m để phương trình (m  2)x  2(m  1) x  vô nghiệm A  m  B m = C 5 m D Không tồn m Hướng dẫn (m  2)x  2(m  1) x   (m  2m  4)x   Trang m  2m   Để bất phương trỡnh vụ nghim thỡ (luôn Đúng) Hệ phương trình vơ nghiệm nên khơng tồn m thỏa mãn → Chọn D Ví dụ 4: Tìm m để bất phương trình m x  m(x  1)  2(x  1)  nghiệm với x   2;1 A  m  C m  B m  m  D  m   Hướng dẫn Đặt f(x)  (m  m  2)x  m  (m  m  2)(2)  m   f(2)  Bất phương trình nghiệm với x   2;1    (m  m  2)(1)  m   f(1)   2  m   2m  m      0m m   2 m  2m      m  → Chọn A 2x   3x  Ví dụ 5: Giải hệ bất phương trình  5x   8x  A x  13 B 3  x  13 C x  3 D x  3 Hướng dẫn x  3 2x   3x  x  3     13  x  3 5x   8x  3x  13 x   → Chọn C x  2m  2x  m Ví dụ 6: Với giá trị m hệ sau có nghiệm:  3x   x  A m  B m > C m = D m < -1 Hướng dẫn x  2m  2x  m x  m   3x   x  x  Để phương trình có nghiệm m > → Chọn B Bài tập tự luyện Câu Tìm m để bất phương trình x  m  có tập nghiệm S   3;   Trang A m = -3 B m = Câu Tập nghiệm bất phương trình 1  A (;0)   ;   2   11  A S    ;     D m = 1 C (; ) D (0; )  x B (0; ) Câu Giải bất phương trình: C m = - x 1 x  x  x    1  11  B S    ;5      C S    ;       D S    ;     Câu Tìm m để bất phương trình m x   mx  có nghiệm A m = B m = C m = D x   Đáp án: 1-B 2-D 3-A 4-D Dạng 2: Bất phương trình bậc hai hệ bất phương trình bậc hai ẩn Phương pháp giải Tam thức bậc hai (đối với x) biểu thức dạng ax  bx  c Trong a, b, c số cho trước với a  Nghiệm phương trình ax  bx  c  gọi nghiệm tam thức bậc hai f(x)  ax  bx  c;   b  4ac  '  b '  ac theo thứ tự gọi biệt thức biệt thức thu gọn 2 tam thức bậc hai f(x)  ax  bx  c Dấu tam thức bậc hai thể bảng sau: f(x)  ax  bx  c,(a  0) 0 a.f(x)  0, x   0  b a.f(x)  0, x   \    2a  a.f(x)  0, x  (;x1 )  (x ; ) 0 a.f(x)  0, x  (x1 ;x ) Bất phương trình bậc hai (ẩn x) bất phương trình có dạng ax  bx  c  , ax  bx  c  0, ax  bx  c  0, ax  bx  c  0, ax  bx  c làm tam thức bậc hai Chú ý: Cho tam thức bậc hai ax  bx  c   a  ax  bx  c  0, x       a  ax  bx  c, x         a  ax  bx  c  0, x       a  ax  bx  c  0, x       Cách giải: Để giải bất phương trình bậc hai, ta áp dụng định lý dấu tam thức bậc hai Ví dụ minh họa Trang Ví dụ 1: Tìm tập nghiệm bất phương trình sau: 5x  4x  12    A   ;2    2  B  ;    (2; ) 5  6  C  ;     8;   5  6  D  ;     2;   5  Hướng dẫn Tam thức bậc hai: f(x)  5x  4x  12 có hai nghiệm x   x = Bảng xét dấu: x  f(x)  -  + - 6  Vậy tập nghiệm bất phương trình là:  ;     2;   5  → Chọn D Ví dụ 2: Tìm m để f(x)  x  2(2m  3)x  4m   0, x   A m  B m  C 3 m D  m  Hướng dẫn 4m  16m  12    f(x)  x  2(2m  3)x  4m   0, x       m  a  1  lu«n §óng → Chọn D Ví dụ 3: Tìm nghiệm hệ bất phương trình sau A 4  x   4  x  B     x  2  x 8  2x   x2  x  3x    0  x  C     x  2  D   x  2 Hướng dẫn    x  2 x 8 (2x  4)(x  2)(x  8) 2x  7x  0 0  Ta có: 2x    x2 x2 x2 x  x  3x    (x  1)(x  4)   4  x  0  x  Kết hợp nghiệm ta được, hệ bất phương trình có nghiệm     x  2  → Chọn C Trang x   Ví dụ 4: Hệ bất phương trình  có nghiệm khi: x  m  A m  1 B m  C m  D m  1 Hướng dẫn x   1  x  Ta có   x  m x  m  Để bất phương trình có nghiệm m  → Chọn C Bài tập tự luyện Câu Tìm giá trị m để f(x)  (m  4)x  (m  1)x  2m  âm A m  B m   C m  D m  Câu Tìm giá trị m để bất phương trình sau (m  2)x  2(m  1)x  vô nghiệm A  m  B m  C   m  D Không tồn m C x  12  20  D (x  2)2 10  x  Câu Bất phương trình có tập nghiệm (2;10) A x  12x  20  B x  3x   (x  3)(4  x)  Câu Hệ bất phương trình  vơ nghiệm khi: x  m  A m  4 B m  D m  2 C m  Đáp án: 1-A 2-D 3-C 4-D Dạng 3: Bất phương trình hệ bất phương trình bậc hai ẩn Phương pháp giải Bất phương trình bậc hai ẩn x, y bất phương trình có dạng: ax  by  c  0,ax  by  c  0,ax  by  c  0,ax  by  c  a, b, c số thực cho, a b không đồng thời 0; x y ẩn số Mỗi cặp số (x ;y0 ) cho ax  by0  c  gọi nghiệm bất phương trình ax  by  c  Cách xác định miền nghiệm bất phương trình bậc hai ẩn ax  by  c  : Bước 1: Vẽ đường thẳng d: ax  by  c  Bước 2: Xét điểm M(x ;y0 ) không nằm d  Nếu ax  by0  c  nửa mặt phẳng (khơng kể bờ d) chứa điểm M miền nghiệm bất phương trình ax  by  c   Nếu ax  by0  c  nửa mặt phẳng (không kể bờ d) chứa điểm M miền nghiệm bất phương trình ax  by  c  Chú ý: Trang Đối với bất phương trình dạng ax  by  c  ax  by  c  miền nghiệm nửa mặt phẳng kể bờ Ví dụ minh họa Ví dụ 1: Miền nghiệm bất phương trình 4(x  1)  5(y  3)  2x  nửa mặt phẳng chứa điểm: A (0;0) B (1;1) C (- 1; 1) D (2;5) Hướng dẫn Ta có: 4(x  1)  5(y  3)  x   x   5y  15  2x   2x  5y  10  Thay tọa độ điểm (2;5) vào bất phương trình ta có: 2.2 + 5.5 -10 > (đúng) → Chọn D Ví dụ 2: Phần khơng gạch chéo hình sau biểu diễn miền nghiệm hệ bất phương trình nào? y  A  3x  2y  y  B  3x  2y  6 x  C  3x  2y  x  D  3x  2y  6 Hướng dẫn Dựa vào hình vẽ ta thấy đồ thị gồm hai đường thẳng (d1 ) : y  đường thẳng (d ) : 3x  y  Miền nghiệm gồm phần y nhận giá trị dương Lại có (0;1) thỏa mãn bất phương trình 3x  2y  → Chọn A 0  y  x   Ví dụ 3: Giá trị lớn biểu thức F(x;y)  x  2y với điều kiện  x  y   x  2y  10  A B C 10 D 12 Hướng dẫn Vẽ đường thẳng d1 : x  y   0, đường thẳng d1 qua hai điểm (0; -1) (1;0) Vẽ đường thẳng d : x  2y  10  0, đường thẳng d2 qua hai điểm (0;5) (2;4) Vẽ đường thẳng d : y  Trang Miền nghiệm ngũ giác ABCOD với A(4;3), B(2;4), C(0,4), D(1,0) Ta có: F(4;3) = 10, F(2;4) = 10, F(0;4) = 8, F(1;0) = 1, F(0;0) = Vậy giá trị lớn biểu thức F(x;y) = x+ 2y 10 → Chọn C Bài tập tự luyện 2x  3y  (1)  Câu CHo hệ  Gọi S1 tập nghiệm bất phương trình (1), S1 tập nghiệm x  y  (2)  bất phương trình (2) S tập nghiệm hệ A S1  S B S  S1 C S  S D S  S Câu Trong thi pha chế, đội chơi sử dụng tối đa 24 gam hương liệu, lít nước 210 gam đường để pha chế nước cam nước táo Để pha chế lít nước cam cần 30 gam đường, lít nước gam hương liệu; pha chế lít nước táo cần 10 gam đường, lít nước gam hương liệu Mỗi lít nước cam nhận 60 điểm thưởng, lít nước táo nhận 80 điểm thưởng Hỏi cần pha chế lít nước trái loại để đạt số điểm thưởng cao nhất? A lít nước cam lít nước táo B lít nước cam lít nước táo C lít nước cam lít nước táo D lít nước cam lít nước táo Đáp án: 1-B 2-D PHẦN 3: BÀI TẬP TỔNG HỢP 3  x  Câu Giải hệ bất phương trình  x   A 2  x  B 1  x  C 2  x  3 D 2  x  Câu Hàm số có bảng xét dấu sau hàm số nào?  x f(x) - +  - + A f(x)  (x  3)(x  3x  2) B f(x)  (1  x)(x  5x  6) C f(x)  (x  2)(x  4x  3) D f(x)  (1  x)(2  x)(3  x) Câu Bất phương trình x 1  có nghiệm x x3 Trang A x > 1 C x  B x < D x > Câu Tìm m để f(x)  mx  2(m  1)x  4m dương với x    1 A  1;   3 1  B  ; 1   ;   3  1  D  ;   3  C (0; ) x  9x  14 0 Câu Giải bất phương trình x  5x  A (;1)  (2;6)  (8; ) B (;1)  (2;4)  (7; ) C (;1)  (3;4)  (7; ) D (;1)  (2;4)  (6; ) Câu Giải bất phương trình A x < -1 x  x  x  x  11    89 86 83 80 B x < -9 C x < -91 D x < -90 2 (1  x)   3x  x Câu Giải hệ bất phương trình:  2 (x  2)  2x  7x  13 A x  B x   C x  D x   x  2m  2x  m Câu Với giá trị m hệ sau có nghiệm:  3x   x  A m  B m  C m  D m  1 Đáp án: 1-C 2-A 3-A 4-D 5-B 6- C 7-B 8-B Trang CHƯƠNG 2: PHƯƠNG TRÌNH, HỆ PHƯƠNG TRÌNH VÀ BẤT PHƯƠNG TRÌNH CHUYÊN ĐỀ 5: MỘT SỐ BẤT PHƯƠNG TRÌNH THƯỜNG GẶP PHẦN 1: LÝ THUYẾT TRỌNG TÂM Một số bất phương trình quy bất phương trình bậc hai: Bất phương trình chứa dấu giá trị tuyệt đối Bất phương trình chứa Chú ý: Ta sử dụng máy tính để tìm đáp án PHẦN 2: CÁC DẠNG BÀI TẬP Dạng 1: Bất phương trình chứa dấu giá trị tuyệt đối Phương pháp giải Bước 1: Đặt điều kiện cho x để bỏ dấu giá trị tuyệt đối Bước 2: Giải bất phương trình với điều kiện x Bước 3: Kết hợp kết giải với điều kiện ban đầu Bước 4: Kết luận tập nghiệm bất phương trình Ví dụ minh họa Ví dụ 1: Tập nghiệm bất phương trình x  8x  x   15 A S   ;3 B S   ;3   4;   C S   6;   D S   ;3   6;   Hướng dẫn Cách 1: Bất phương trình x  8x  x   15 (1) Trường hợp 1: Nếu x    x  3, bất phương trình (1) trở thành: x  x  8x  x   15  x  7x  12    x  Kết hợp với điều kiện x  3, ta x > Trường hợp 2: Nếu x    x  3, bất phương trình (1) trở thành: x  x  8x  (x  3)  15  x  9x  18    x  Kết hợp với điều kiện x < 3, ta x < Vậy tập nghiệp bất phương trình S   ;3   4;   Cách 2: Sử dụng máy tính CASIO fx 570VN PLUS Thay x = vào bất phương trình (1), ta thấy thỏa mãn bất phương trình (1) Nên loại đáp án A, C D →Chọn B Ví dụ 2: Điều kiện x thõa mãn bất phương trình 2x   x  5x  6? Trang A  x   57 B  57  x  2 C  57  x  D  57  x  Hướng dẫn Cách 1: Bất phương trình 2x   x  5x  (1) Trường hợp 1: Nếu 2x    x  2, bất phương trình (1) trở thành: 2x   x  5x   x  3x  10   2  x  Kết hợp với điều kiện x  , ta  x  Trường hợp 2: Nếu 2x    x  2, bất phương trình (1) trở thành:   2x    x  5x   x  7x    Kết hợp với điều kiện x < 2, ta  57  57 x 2  57  x  2 Kết hợp hai trường hợp ta điều kiện x  57  x  Cách 2: Sử dụng máy tính CASIO fx 570VN PLUS Thay x = vào bất phương trình (1), ta thấy thỏa mãn bất phương trình (1) Nên loại đáp án B Thay x   57 vào bất phương trình (1), ta thấy thỏa mãn bất phương trình (1) Nên loại đáp án A D →Chọn B Ví dụ 3: Tìm m để 4x  2m  A m  1   x  2x   m với x 2 B m  C m > D -2 < m < Hướng dẫn Để 4x  2m  1   x  2x   m với x  x  2x   m  0, x   2 1  Ta có:  x  2x   m  0, x   22  4(1)   m      4m   m  2 2  →Chọn A Dạng 2: Bất phương trình chứa dấu Ví dụ minh họa Ví dụ 1: Tập nghiệm bất phương trình 5  A  ;  2  5  B  ;  2  2x   2x  5  C  ;   2  5  D  ;   2  Hướng dẫn Trang Cách 1: Ta có  x  2x      2x   2x   2x    x  2x   (2x  3)   2x   4x  12x     x  3    x  x      5 2 4x  14x  10  4x  14x  10   x   x      x  Cách 2: Sử dụng máy tính CASIO fx 570VN PLUS Thay x  vào bất phương trình ban đầu, ta thấy thỏa mãn bất phương trình Nên loại đáp án B C Thay x = vào bất phương trình ban đầu, ta thấy thỏa mãn bất phương trình Nên loại đáp án A →Chọn D Ví dụ 2: Tập nghiệm bất phương trình:  x  x   2x   là:   13  A 1;    2;     9  B 4; 5;   2   2   C  2;  ;1       17  D  ; 5  5;   3  5 Hướng dẫn Cách 1: Ta có  x  x     x     2x   2x        x  x    x   2  x   2    x   2; ;1      Cách 2: Sử dụng máy tính CASIO fx 570VN PLUS Thay x = vào bất phương trình ban đầu, ta thấy khơng thỏa mãn bất phương trình Nên loại đáp án A D Thay x = -4 vào bất phương trình ban đầu, ta thấy khơng thỏa mãn bất phương trình Nên loại đáp án B →Chọn C Trang Ví dụ 3: Bất phương trình: x  2x   x  có nghiệm nguyên? A B C D Nhiều Hướng dẫn Bất phương trình x  2x   x  (1) Đặt t  x (t  0), (1) trở thành t  2t   t  t  Nếu t  2t     , ta suy t  t    33 t  2 Nếu t  2t    1  t  ta có  t  t       33 t   Kết hợp với điều kiện 1  t  , ta suy t  Vậy bất phương trình (1) vơ nghiệm →Chọn A PHẦN 3: BÀI TẬP TỔNG HỢP Câu Tập nghiệm bất phương trình x  1(x  2)  A S  1   2;   B S  1   ; 2 C S   D S   Câu Cho bất phương trình: x  x  m  2mx  3m  3m   Để bất phương trình có nghiệm, giá trị thích hợp tham số m là: A 1  m   B 1  m  Câu Bất phương trình   A   ;  2    A  x  D  m  2x    x có tập nghiệm B (3;  2) Câu Bất phương trình C   m  C (4  2;3) D (4  2; )  x  6x    2x có nghiệm B  x  C 5  x  3 D 3  x  2 Đáp án: 1-A 2-D 3-A 4-A Trang CHƯƠNG 3: LƯỢNG GIÁC CHUN ĐỀ CUNG VÀ GĨC, CƠNG THỨC LƯỢNG GIÁC PHẦN 1: LÝ THUYẾT TRỌNG TÂM Đơn vị đo góc cung tròn, độ dài cung tròn Cung tròn bán kính R có số đo      2  , có số đo a   a  360  Độ dài I cung tròn I = Rα = R πa 180 Quan hệ số đo độ số đo rađian  a   180   180  Đặc biệt: rad   rad  ,1  180    Đường tròn lượng giác * Đường tròn lượng giác đường tròn đơn vị, định hướng (quy ước chiều dương chiều ngược chiều kim đồng hồ) chọn điểm A làm gốc * Điểm M  x; y  đường tròn lượng giác cho  OA, OM    gọi điểm đường tròn lượng giác biểu diễn cung (góc) lượng giác có số đo  Trục Ox gọi trục giá trị cos Trục Oy gọi trục giá trị sin Trục At gốc A hướng với trục Oy gọi trục giá trị tang Trục Bs gốc B hướng với trục Ox gọi trục giá trị cotang * Giá trị lượng giác sin, côsin, tang cotang: cos   OK  x sin   OH  y, tan   AT  sin         k  cos    cot   BS  cos    k  sin  Dấu giá trị lượng giác Phần tư Giá trị I II III IV cos  + - - + sin  + + - - tan  + - + - cot  + - + - lượng giác Trang Cung liên kết Góc đối Góc bù Góc phụ Góc  (cos đối) (sin bù) (phụ chéo) (khác pi tan) cos     cos  sin      sin    sin      cos  2  sin       sin  sin      sin  cos       cos    cos      sin  2  cos       cos  tan      tan  tan       tan    tan      cot  2  tan      tan  cot      cot  cot       cot    cot      tan  2  cot      cot  Công thức lượng giác sin x cos x sin x  cos x  tan x  tan x.cot x  1  tan x  Công thức cộng cos  a  b   cos a.cos b  sin a.sin b cos  a  b   cos a.cos b  sin a.sin b sin  a  b   sin a.cos b  cos a.sin b sin  a  b   sin a.cos b  cos a.sin b tan a  tan b  tan a.tan b tan a  tan b tan  a  b    tan a.tan b tan  a  b   Cơng thức biến đổi tổng thành tích ab a b cos 2 ab a b cos a  cos b  2sin sin 2 ab a b sin a  sin b  2sin cos 2 ab a b sin a  sin b  cos sin 2 cos a  cos b  cos cot x  cos x cos x sin x  cot x  sin x Công thức nhân đôi, hạ bậc sin 2a  2sin a.cos a , tan 2a  tan a  tan a cos 2a  cos a    2sin a  cos a  sin a  cos 2a  cos 2a sin a  , cos a  2 3cos a  cos 3a cos 3a  cos3 a  3cos a  cos3 a  3sin a  sin 3a sin 3a  3sin a  4sin a  sin a  Cơng thức biến đổi tích thành tổng cos  a  b   cos  a  b   2 sin a sin b  cos  a  b   cos  a  b   sin a cos b  sin  a  b   sin  a  b   cos a cos b  Trang PHẦN 2: CÁC DẠNG BÀI TẬP Dạng 1: Góc cung lượng giác Ví dụ minh họa Ví dụ 1: Góc có số đo 108o đổi radian A 3 B  10 C 5 D 10 Hướng dẫn Áp dụng công thức đổi từ độ sang rad ta có:   a 108 3   180 180 → Chọn A Ví dụ 2: Góc có số đo A 240 2 đổi sang độ B 135 C 72 D 270 Hướng dẫn Áp dụng công thức đổi từ độ sang rad ta có: a   180 2 180   72  5 → Chọn C Ví dụ 3: 20 giây bánh xe xe gắn máy quay 60 vòng Tính độ dài qng đường xe gắn máy vòng phút, biết bán kính bánh xe gắn máy 6,5 cm (lấy   3,1416 ) A 22054 cm B 22063 cm C 22054 mm D 22044 cm Hướng dẫn Độ dài quãng đường bánh xe lăn vòng I  R  6,5.2  13 Trong 20s, bánh xe quay 60 vòng Trong phút = 3.60 = 180s, bánh xe quay 60.180  540 vòng 20 Vậy độ dài quãng đường xe gắn máy S  540.13  22054 (cm) → Chọn A Ví dụ 4: Góc x    A 12 k , k   biểu diễn điểm đường tròn lượng giác? B C D Hướng dẫn Áp dụng góc x    x   k 2 , k   biểu diễn n điểm đường tròn lượng giác, góc n k  k 2   biểu diễn điểm đường tròn lượng giác 3 → Chọn D Bài tập tự luyện Trang Câu Góc có số đo  đổi sang độ A 15 B 18 C 20 D 25 Câu Góc có số đo 120 đổi sang rađian góc A  10 B 3 C  D 2 Câu Một đường tròn có bán kính R = 10cm Độ dài cung 40 đường tròn gần A 7cm B 9cm C 11cm D 13cm Câu Một bánh xe có 72 Số đo góc mà bánh xe quay di chuyển 10 A 30 B 40 C 50 D 60 Đáp án: 1-C 2-D 3-A 4-C Dạng 2: Tính giá trị biểu thức lượng giác Ví dụ minh họa Ví dụ 1: tính giá trị A  cos10.cos 30.cos 50.cos 70 A 16 B C 16 D Hướng dẫn Cách 1: Áp dụng công thức: cos a.cos b  A  cos10.cos 30 Vì cos 30  A  cos  a  b   cos  a  b   2 1  cos120  cos 20   cos 30  cos10.cos120  cos10.cos 20  2 1 , cos120  nên ta có: 2  cos10  cos10 cos 30  cos10  cos 30    cos10.cos 20      2 2 2    3  4 16 Cách 2: Sử dụng máy tính Casio fx 570 VN Plus Bước 1: Thiết lập mơi trường tính theo độ: SHIFT MODE Bước 2: nhập cos 10  x cos  30  x cos  50  x cos  70  ta có kết 16 → Chọn C Ví dụ 2: Cho sin   A      Giá trị cos  B  C  D 16 25 Trang Hướng dẫn Cách 1: Vì       cos    cos    16   Ta có: sin   cos    cos    sin    25 25 cos     Kết hợp điều kiện ta có: cos    Cách 2: Sử dụng máy tính Casio fx 570 VN PLUS Bước 1: thiết lập mơi trường tính theo rađian: SHIFT MODE Bước 2: xác định dấu cos  : chọn giá trị α thỏa mãn      3 Bước 3: Sử dụng SHIFT sin để tìm góc  : sin 1   , ta kết 0,6435011 5 Bước 4: Nhập cos  Ans  ta kết 4 , theo bước 2, ta thấy cos   , cos    5 Chú ý: Phải xác định trước dấu giá trị lượng giác cần tính, không dẫn tới kết luận kết → Chọn B Ví dụ 3: Cho tan   Giá trị A  A B 3sin   cos  sin   cos  C D Hướng dẫn Cách 1: Chia tử mẫu A cho cosx ta được: 3sin  cos   tan   3.2  A  cos  cos    7 sin  cos  tan     cos  cos  Cách 2: Sử dụng máy tính Casio fx 570 VN PLUS Bước 1: thiết lập môi trường tính theo rađian: SHIFT MODE Bước 2: sử dụng SHIFT tan để tìm góc  : tan 1   , ta kết 1,1071487 Bước 3: nhập 3sin  Ans   cos  Ans  ta kết A = sin  Ans   cos  Ans  → Chọn C Ví dụ 4: Cho sin   cot   tan  90    180 Giá trị biểu thức E  tan   3cot  Trang A 57 B  57 C 57 D  57 Hướng dẫn Cách 1: Vì 90    180  cos   , ta có:  cos    16 sin   cos    cos    sin      25 25 cos     sin    cot    Kết hợp điều kiện  cos    , tan   cos  tan  3     cot   tan  4   E  tan   3cot  57  4       3 Cách 2: Sử dụng máy tính Casio fx 570 VN PLUS Bước 1: Thiết lập mơi trường tính theo độ: SHIFT MODE Bước 2: Xác định dấu tan  : chọn giá trị α thỏa mãn 90    180 Ta chọn   100 , nhập tan 100  ta kết -5,671 < tan   3 Bước 3: Sử dụng SHIFT sin để tìm góc  : sin 1   , ta kết 36.869897 5 Bước 4: nhập tan(Ans) ta kết cot   3 , theo bước ta thấy tan   nên tan    4   , thay vào E ta giá trị cần tính tan  → chọn B Bài tập tự luyện Câu Giá trị biểu thức cos A B  Câu Giá trị biểu thức A  A 3  2 4 6  cos  cos 7 11 10 B  3 B D  cos 750  sin 420 sin  330   cos  390  Câu Cho tan   Giá trị biểu thức A  A C 10 11 C 1 D 1 sin  sin   3cos3  C  10 11 D  11 10 Câu Biết tan   180    270 Giá trị cos   sin  Trang A  B  C D 1 Đáp án: 1-B 2-A 3-B 4-A Dạng 3: Rút gọn biểu thức lượng giác Ví dụ minh họa Ví dụ 1: Đơn giản biểu thức A  1  sin x  cot x  1  cot x  Ta có A A  sin x B A  cos x C A   sin x D A   cos x Hướng dẫn Cách 1: A  1  sin x  cot x  1  cot x   cot x  sin x.cot x  1  cot x   cot x  sin x cos x   cot x  cot x  cos x   cot x   cos x  sin x sin x Cách 2: Sử dụng máy tính Casio fx 570 VN PLUS Bước 1: thiết lập mơi trường tính theo rađian: SHIFT MODE Bước 2: Chọn giá trị x thay vào biểu thức A Chú ý cot x    Ta chọn x  , thay vào ta A   sin 1 tan x        0, 7080734 2 tan 1  tan 1  Bước 3: thay x  vào bốn đáp án, đáp án kết bước chọn Đáp án A, ta có sin 1  0, 7080734 đáp án A đáp án → Chọn A Ví dụ 2: Tính giá trị biểu thức A  sin x  cos x  3sin x cos x A A  1 C A  B A  D A  4 Hướng dẫn Cách 1: Áp dụng đẳng thức a  b3   a  b   3ab  a  b  ta được: A  sin x  cos x  3sin x cos x   sin x    cos x   3sin x cos x 3   sin x  cos x   3sin x.cos x  sin x  cos x   3sin x cos x   3sin x.cos x  3sin x.cos x  Cách 2: Sử dụng máy tính Casio fx 570 VN PLUS Bước 1: Thiết lập mơi trường tính theo rađian: SHIFT MODE Bước 2: Chọn giá trị x thay vào Biểu thức A Ta chọn x = 1, thay vào ta A  sin 1  cos 1  3sin 1 x cos 1  6 2 → Chọn B Trang Ví dụ 3: Nếu 5sin   3sin     giá trị tan     A tan  B tan  C tan  D tan  Hướng dẫn 5sin   3sin      5sin         3sin         5sin     cos   5cos     sin   3sin     cos   3cos     sin   2sin     cos   8cos     sin   sin     sin  4  tan      tan  cos     cos  → Chọn C Bài tập tự luyện     Câu Biểu thức A  cos x  cos   x   cos   x  không phụ thuộc x có giá trị 3  3  A B C D Câu Biểu thức D  cos x.cot x  3cos x  cot x  2sin x có giá trị A B -2 C D -3 Câu Rút gọn biểu thức cos 120  x   cos 120  x   cos x A B –cos x C -2cosx D sinx – cosx C sin   D sin    Đáp án: 1-C 2-A 3-C PHẦN 3: BÀI TẬP TỔNG HỢP Câu Cho cos   A sin    với    Tính sin  B sin    Câu Một đường tròn có bán kính R  A 10 cm B cm 10  cm Tìm độ dài cung C 20  cm  đường tròn D 2 20 cm Câu Một đồng hồ treo tường có kim dài 10,57cm kim phút dài 13,34cm Trong 30 phút, mũi kim vạch lên cung tròn có độ dài A 2,77 cm B 2,9 cm C 2,76 cm D 2,8 cm 1 Câu Cho hai góc nhọn a b Biết cos a  , cos b  Giá trị cos  a  b  cos  a  b  A  113 144 B  115 144 C  117 144 D  119 144 Trang Câu Giá trị biểu thức A  sin A  3 5 7  sin  sin 8 B -2 C Câu Gía trị biểu thức A  A  sin B D tan 30  tan 40  tan 50  tan 60 bao nhiêu? cos 20 C D Câu Cho tam giác ABC mệnh đề (I) cos BC A  sin 2 (II) tan A B C tan  2 (III) cos  A  B  C   cos 2C  Mệnh đề là: A Chỉ (I) B (II) (III) Câu Cho cot   3 với  C (I) (II)     Khi giá trị tan B 2 19 A 19   cot D Chỉ (III)  C  19 D 19 Câu Rút gọn biểu thức: cos 120  x   cos 120  x   cos x B  cos x A C 2 cos x D sin x  cos x 1 Câu 10 Cho A, B, C góc nhọn tan A  ; tan B  , tan C  Tổng A + B + C A  B  C  D  Câu 11 Cho A, B, C ba góc tam giác Hãy hệ thức sai A cos A B C  sin 2 B cos  A  B  2C    cos C B sin  A  C    sin B D cos  A  B    cos C Câu 12 Cho tan a  cot a  m Khi cot a  tan a có giá trị A m3  3m B m3  3m C 3m3  m D 3m3  m Đáp án: 1–C 2–B 3–A 10 – C 11 – C 12 – B 4–D 5–A 6–D 7–C 8–A 9–C Trang CHƯƠNG CHUYÊN ĐỀ 2: HÀM SỐ LƯỢNG GIÁC PHẦN 1: LÝ THUYẾT TRỌNG TÂM Hàm số y = sinx Hàm số y = cosx * Tập xác định:  * Tập xác định:  * Hàm số tuần hoàn với chu kì T  2 * Hàm số tuần hồn với chu kì T  2 * Tập giá trị:  1;1 * Tập giá trị:  1;1  * Đồng biến  k 2 ;   k 2  nghịch biến    * Đồng biến    k 2 ;  k 2  nghịch      k 2 ; k 2  , k   3    k 2  , k   biến   k 2 ; * Hàm số chẵn nên đồ thị hàm số nhận trục Oy 2  tâm đối xứng * Hàm số lẻ nên đồ thị hàm số nhận gốc tọa độ O tâm đối xứng Hàm số lượng giác Hàm số y = tanx * Tập giá trị:  Hàm số y = cotx * Tập giá trị:    * Tập xác định: D   \   k , k    2  * Hàm số tuần hồn với chu kì T   * Tập xác định: D   \ k , k   * Hàm số tuần hồn với chu kì T   * Hàm số lẻ, đồ thị hàm số nhận gốc tọa độ O * Hàm số lẻ, đồ thị hàm số nhận gốc tọa độ O tâm tâm đối xứng đối xứng * Hàm nghịch biến  k ;   k  , k       * Hàm đồng biến    k ;  k  , k   * Đồ thị nhận đường thẳng x  k , k   làm   đường tiệm cận  * Đồ thị nhận đường thẳng x   k , k   làm đường tiệm cận PHẦN 2: CÁC DẠNG BÀI TẬP Dạng 1: Tập xác định hàm số lượng giác Phương pháp giải y f  x xác định g  x   0, g  x y  n f  x  , n  * xác định f  x   Trang y  sin u  x   xác định u  x  xác định, y  cos u  x   xác định u  x  xác định  y  tan u  x   xác định u  x  xác định cos u  x     u  x    k , k   y  cot u  x   xác định u  x  xác định sin u  x     u  x   k , k   Ví dụ minh họa Ví dụ 1: tìm tập xác định D hàm số y  2019 sin x   A D   \   k , k    2    B D   \   k 2  3  C D   \ k , k   D D   Hướng dẫn Cách 1: Hàm số xác định sin x   x  k , k   Vậy tập xác định D   \ k , k   Cách 2: Sử dụng máy tính CASIO Fx 570 VN PLUS Bước 1: Thiết lập chế độ rađian SHIFT MODE Bước 2: Nhập hàm số 2019 sin(X) Bước 3: Sử dụng phím gán giá trị CALC, thử giá trị không thuộc đáp án, đáp án cho giá trị báo lỗi Math ERROR đáp án Đáp án A: Ấn CALC, nhập X  Đáp án B: Ấn CALC, nhập X    , ta kết 2019, loại A , ta kết 2331,34, loại B Đáp án C: Ấn CALC, nhập X  , ta Math ERROR, chọn C → Chọn C 1 Ví dụ 2: Tìm tập xác định hàm số y  cos    x x A D   2; 2 C D   B D   1;1 \ 0 D D   \ 0 Hướng dẫn Hàm số cho xác định cos xác định x  x → Chọn D Ví dụ 3: Điều kiện xác định hàm số y  tan x A x     k , k   B x    k , k   C x    k , k   D x    k , k   Trang Hướng dẫn Điều kiện xác định hàm số y  tan x  cos x   x    k  x    sin x là: cos x k ,k  → Chọn C Bài tập tự luyện Câu Tập xác định hàm số y  cot x A x    k , k   B x    k , k   C x   k  , k   D x  k , k     Câu Tập xác định hàm số y  tan  x   3     A D   \   k , k    3     B D   \   k , k    4     C D   \   k , k    12     D D   \   k , k    8  Câu Tập xác định hàm số y  cos x là: A D   0; 2  B D   0;   C D   D D   \ 0 Đáp án: 1–D 2–C 3-B Dạng 2: Tính đơn điệu hàm số lượng giác Phương pháp giải     Hàm số y  sin x đồng biến khoảng    k 2 ;  k 2   k    , nghịch biến khoảng   3    k 2   k      k 2 ; 2  Hàm số y  cos x nghịch biến khoảng  k 2 ;   k 2  k    , đồng biến khoảng    k 2 ; k 2  k        Hàm số y  tan x đồng biến khoảng    k ;  k   k      Hàm số y  cot x nghịch biến khoảng  k ;   k  k    Ví dụ minh họa Ví dụ 1: Xét hàm số y  sin x đoạn   ;0 Khẳng định sau đúng? Trang      A Hàm số đồng biến khoảng   ;     ;0  2      B Hàm số cho đồng biến khoảng   ;   ; nghịch biến khoảng 2       ;0      C Hàm số cho nghịch biến khoảng   ;   ; đồng biến khoảng 2       ;0         D Hàm số nghịch biến khoảng   ;     ;0  2    Hướng dẫn        Cách 1: Hàm số y  sin x đồng biến khoảng   ;0      k 2 ;  k 2   k    nghịch        3    k 2   k    biến khoảng   ;      k 2 ; 2 2   Cách 2: Sử dụng máy tính CASIO Fx 570 VN PLUS: sử dụng phím d   x  dx Bước 1: Thiết lập chế độ rađian SHIFT MODE  d  2  sin  X   x   Bước 2: Chọn giá trị thuộc khoảng   ;   , ta chọn x   , nhập  2 dx 3     kết 0,5 > 0, hàm số đồng biến khoảng   ;0    → Chọn C Ví dụ 2: Hàm số y  cos x nghịch biến khoảng sau đây?    A  k ;  k   k          B    k 2 ;  k 2   k        C   k ;   k   k    2  3    k 2   k    D   k 2 ; 2  Hướng dẫn Hàm số y  cos x nghịch biến k 2  x    k 2  k  x    k , k      Hay hàm số y  cos x nghịch biến khoảng  k ;  k   k      → chọn A Ví dụ 3: Xét mệnh đề sau:  3 (I): x    ;   nghịch biến  : hàm số y  sin x   3  (II): x    ;  : hàm số y  nghịch biến  cos x  Trang Hãy chọn mệnh đề đúng: A Chỉ (I) B Chỉ (II) C Cả hai D Cả hai sai Hướng dẫn  3 Cách 1: x    ;   3 x    ;   đồng biến, (I) sai  : hàm y  sin x nghịch biến, suy y  sin x   nghịch biến, (II)  : hàm y  cos x đồng biến suy hàm y  cos x  Cách 2: Sử dụng máy tính CASIO Fx 570 VN PLUS Sử dụng phím d   x  dx Bước 1: Thiết lập chế độ rađian SHIFT MODE  3 Bước 2: Chọn giá trị thuộc khoảng   ;    , ta chọn  x  1, 2  6 , nhập d   6 , kết 2,3416 > 0, hàm số y  đồng biến biến khoảng   x  sin x dx  sin  X    3  ;     Bước 3: Nhập d   6 , kết -0,898 < 0, hàm số y  nghịch biến   x  cos x dx  cos  X    3 biến khoảng   ;     → Chọn B Bài tập tự luyện Câu Hàm số y  sin x đồng biến khoảng khoảng sau?   A  0;   4   B  ;   2   3 C   ;      3  D  ; 2    Câu Xét hàm số y  cos x đoạn   ;   Khẳng định sau đúng? A Hàm số nghịch biến khoảng   ;0   0;   B Hàm số đồng biến khoảng   ;0  nghịch biến khoảng  0;   C Hàm số nghịch biến khoảng   ;0  đồng biến khoảng  0;   D Hàm số đồng biến khoảng   ;0   0;    31 33  ; Câu Với x    , mệnh đề sau đúng?   A Hàm số y  cot x nghịch biến B Hàm số y  tan x nghịch biến C Hàm số y  sin x đồng biến D Hàm số y  cos x nghịch biến Trang Đáp án: 1–A 2–B 3-C Dạng 3: Giá trị lớn nhất, giá trị nhỏ hàm số Phương pháp giải Áp dụng bất đẳng thức sau: 1  sin x  1  cos x  1  sin  ax  b   1  sin  ax  b    sin x   cos x   sin  ax  b    cos  ax  b    sin x   cos x   sin  ax  b    cos  ax  b   Ví dụ minh họa   Ví dụ 1: Tìm giá trị nhỏ nhất, giá trị lớn hàm số sau y   3sin  x   4  A max y  2, y  B max y  2, y  C max y  2, y  D max y  4, y  2 Hướng dẫn       Cách 1: Vì 1  sin  x     3  3sin  x        3sin  x     4 4 4       2   3sin  x    hay 2  y  4  Vậy max y  4, y  2 Cách 2: Sử dụng máy tính CASIO Fx 570 VN PLUS Bước 1: Thiết lập chế độ rađian SHIFT MODE   Bước 2: Sử dụng MODE 7, nhập hàm số f  x    3sin  X   ấn = 4  Start ?  End ? 2  Step ? 2 End  Start (ta thường chọn Step  ) 15 15 Bước 3: Quan sát giá trị cột F  x  , ta tìm xấp xỉ giá trị lớn 3,963  xấp xỉ giá trị nhỏ 1,995  2 → Chọn D Ví dụ 2: Tìm giá trị lớn hàm số y  cos x  4sin x A -5 B C D Hướng dẫn Cách 1: y  cos x  4sin x   2sin x  4sin x  2  sin x  2sin x  1     sin x  1 Ta có 1  sin x   2  sin x      sin x  1   8   sin x  1  2 Trang      sin x  1    5  y  Vậy giá trị lớn hàm số sin x   x    k 2 , k   Cách 2: Sử dụng máy tính CASIO Fx 570 VN PLUS Bước 1: Thiết lập chế độ rađian SHIFT MODE Bước 2: Sử dụng MODE 7, nhập hàm số f  x   cos  X   4sin  X  , ấn = Start ?  End ? 2  Step ? 2 End  Start (ta thường chọn Step  ) 15 15 Bước 3: Quan sát giá trị cột F  x  , ta tìm xấp xỉ giá trị lớn 2,999  → Chọn B Ví dụ 3: Hàm số y   cos x đạt giá trị nhỏ x bao nhiêu? A x    k 2 , k   B x  C x  k 2 , k     k , k   D x  k , k   Hướng dẫn Ta có 1  cos x    cos x     cos x    y  Vậy giá trị nhỏ hàm số cos x   x    k , k   → chọn B Bài tập tự luyện   Câu Tìm tổng giá trị lớn giá trị nhỏ hàm số sau y  2sin  x   3  A B C D -2 Câu Tìm giá trị lớn nhất, giá trị nhỏ hàm số y   cos 2 x A y  2; max y  B y  3; max y  C y  5; max y  D y  3; max y  Câu Hàm số y  sin x  cos x đạt giá trị nhỏ x bao nhiêu? A x    k B x    k 3 C x    k D x    k Đáp án: 1–C 2–D 3-D Dạng 4: Tính chẵn lẻ hàm số Phương pháp giải x  D thi  x  D Hàm số y  f  x  với tập xác định D gọi hàm số chẵn nếu:   f   x   f  x  Trang Đồ thị hàm số chẵn nhận trục tung làm trục đối xứng x  D thi  x  D Hàm số y  f  x  với tập xác định D gọi hàm số lẻ nếu:   f   x    f  x  Đồ thị hàm số lẻ nhận gốc tọa độ O làm tâm đối xứng Ví dụ minh họa Ví dụ 1: Hàm số sau hàm số chẵn? A y  2 cos x C y  2sin   x  B y  2sin x D y  sin x  cos x Hướng dẫn Cách 1: xét đáp án y  2 cos x Do tập xác định D   nên x     x   Ta có f   x   2 cos   x   2 cos x  f  x  Vậy hàm số y  2 cos x làm hàm số chẵn Cách 2: Sử dụng máy tính CASIO Fx 570 VN PLUS Bước 1: Thiết lập chế độ rađian SHIFT MODE Bước 2: Sử dụng CALC để thử trường hợp x -x Đáp án A: Nhập vào hình hàm số 2 cos  X  sử dụng CALC với trường hợp x = trường hợp x  1 đưa kết giống Vì f  x    f  x   hàm số chẵn, chọn A → Chọn A Ví dụ 2: Trong hàm số sau, hàm số hàm số lẻ? A y  cos x B y  sin x  16 C y  sin 2 x D y   sin 3 x Hướng dẫn Đáp án A: y  cos x hàm số chẵn, có tập xác định D   , với x  D  x  D f   x   cos  2 x   cos x  f  x  Đáp án B: y  sin x  16 hàm số không chẵn khơng lẻ, có tập xác định 16;   , tập đối xứng Đáp án C: y  sin 2 x hàm số lẻ, có tập xác định D   , với x  D  x  D f   x    sin  3 x   sin x   f  x  → Chọn D Bài tập tự luyện Câu Trong hàm số sau hàm số hàm số không chẵn không lẻ? A y  sin x.cos x B y  sin x  cos x C y   cos x D y  cos x  sin x C y  cos x.cot x D y  Câu Hàm số sau hàm số chẵn? A y  sin x B y  x.cos x tan x sin x Đáp án: Trang 1–B 2–D Dạng 5: Tính tuần hồn hàm số lượng giác Phương pháp giải Định nghĩa tính tuần hoàn hàm số Hàm số y  f  x  xác định tập D gọi hàm số tuần hồn có số T  , cho x  D Khi đó: x  T  D f  x  T   f  x  Nếu có số T dương nhỏ thỏa mãn điều kiện hàm số gọi hàm số tuần hồn với chu kì T Chú ý: Các hàm số y  sin  ax  b  , y  cos  ax  b  tuần hoàn với chu kỳ T  2 a Các hàm số y  tan  ax  b  , y  cot  ax  b  tuần hoàn với chu kỳ T   a Ví dụ minh họa   Ví dụ 1: tìm chu kì T hàm số y  sin  x   4  A T  2 B T  5 C T   D T   Hướng dẫn Hàm số y  sin  ax  b  tuần hoàn với chu kì T  2 a  2  Do hàm số y  sin  x   có a  tuần hồn với chu kì T  4  → Chọn A Ví dụ 2: tìm chu kì T hàm số y  cot A T  4 x  sin x C T  3 B T   D T   Hướng dẫn Hàm số y  cot  x có a1  tuần hồn với chu kì T1   3 a1 3 Hàm số y  sin x có a2  tuần hồn với chu kì T2  Suy hàm số y  cot 2  a2 x  sin x tuần hồn với chu kì T2  3 → Chọn C Trang x  Ví dụ 3: Nếu chu kì T hàm số y  sin    a nhận giá trị đây?  a  A 2 B 4 D 8 C Hướng dẫn 2 x    T    a   a   a  4 Chu kì hàm số y  sin    a  a → Chọn B Bài tập tự luyện Câu Chu kỳ hàm số y  sin A  x là: B C  D Câu Mệnh đề sau sai? A Hàm số y  sin x tuần hồn với chu kì 2 B Hàm số y  cos x tuần hồn với chu kì 2 C Hàm số y  tan x tuần hồn với chu kì 2 D Hàm số y  cot x tuần hoàn với chu kì  Câu Trong hàm số sau đây, hàm số hàm số tuần hoàn? A y  sin x x B y  tan x  x C y  x  D y  cot x Đáp án: 1–D 2–C 3-D Dạng 6: Đồ thị hàm số lượng giác Phương pháp giải Đồ thị hàm số y  m sin  ax  b  , y  m cos  ax  b  có chu kỳ T  2 , biên độ: m a Cho hàm số y  f  x  có đồ thị (C), với p > 0, ta có: * Tịnh tiến (C) lên p đơn vị đồ thị hàm số y  f  x   p * Tịnh tiến (C) xuống p đơn vị đồ thị hàm số y  f  x   p * Tịnh tiến (C) sang trái p đơn vị đồ thị hàm số y  f  x  p  * Tịnh tiến (C) sang phải p đơn vị đồ thị hàm số y  f  x  p  Ví dụ minh họa x Ví dụ 1: Khẳng định sau đồ thị hàm số y  3cos ? Trang 10 A Biên độ 3, chu kì 4 B Biên độ -3, chu kì 180 C Biên độ 3, chu kì 2 D Biên độ 3, chu kì  Hướng dẫn Hàm số y  3cos 2 2 x   4 có m = -3 có biên độ m  , chu kì T  a 2 → Chọn A Ví dụ 2: Đồ thị hàm số đồ thị hàm số y  cos x dịch theo phương thẳng đứng lên 2 ? A y  cos x  2 B y  cos  x  2  C y  cos  x  2  D y  cos x  2 Hướng dẫn Đồ thị hàm số y  f  x  dịch theo phương thẳng đứng lên a đơn vị trở thành đồ thị hàm số y  f  x   a Do đó, đồ thị hàm số y  cos x dịch theo phương thẳng đứng lên 2 trở thành đồ thị hàm số y  cos x  2 → Chọn D Ví dụ 3: Đường cong hình đồ thị hàm số nào? A sin x B cos x C  cos x  x D sin     2 Hướng dẫn Tại x = y = loại B C cos   x   Tại x   y  1 Thay x   vào hai đáp án lại có sin     sin     1 thỏa mãn  2  2 → Chọn D Ví dụ 4: Hình vẽ thuộc đồ thị hàm số nào? A y  3cos x B y  cos  x C y  2sin  x D y  3sin  x Hướng dẫn Từ đồ thị hàm số ta thấy hàm số có biên độ nên ta loại đáp án A D Đồ thị hàm số qua gốc tọa độ O, thay x = vào hai đáp án lại Trang 11 y  cos  x  cos    ta loại B y  2sin  x  đồ thị hàm số y  2sin  x qua gốc tọa độ O Vậy hình vẽ đồ thị hàm số y  2sin  x → Chọn C Bài tập tự luyện Câu Đường cong hình đồ thị hàm số nào?   A sin  x   4  3   B cos  x     C   sin  x   4    D cos  x   4  Câu Trong hàm số đây, hàm số có biên độ chu kỳ 4 ? A y  3cos x x B y   cos x C y  cos D y  3cos x Câu Đồ thị hàm số y  sin x suy từ đồ thị y  cos x  1 C  cách: A Tịnh tiến (C) qua trái đoạn có độ dài   B Tịnh tiến (C) qua phải đoạn có độ dài C Tịnh tiến (C) qua đoạn có độ dài D Tịnh tiến (C) qua trái đoạn có độ dài lên đơn vị 2   lên đơn vị xuống đơn vị xuống đơn vị Đáp án: 1–A 2–A 3-D PHẦN BÀI TẬP TỔNG HỢP Câu Tập xác định hàm số y  A x    k 2  sin x sin x  B x  k 2 C x  3  k 2 D x    k 2   Câu Trong khoảng  0;  , hàm số y  sin x  cos x hàm số:  2 A Đồng biến B Nghịch biến C Không đổi D Vừa đồng biến vừa nghịch biến Trang 12   Câu Tập xác định hàm số y  tan  x   3  A x    k B x  5  k 12  D x  5  k 12 C y  cos x.cot x D y  tan x sin x C  2;8 D 5;8 C x   k Câu Trong hàm số sau, hàm số hàm số chẵn? B y  x cos x A y  sin x Câu Tìm tập giá trị hàm số y  3cos x  A  1;1 B  1;11 Câu Trong hàm số sau, hàm số có đồ thị đối xứng qua trục tung? A y  sin x.cos x C y  B y  cos x.sin x   D y  sin x.cos  x   2  tan x tan x  Câu Hai hàm số sau có chu kì khác nhau? A y  cos x y  cot C y  sin x B y  sin x y  tan x x x y  cos 2 D y  tan x y  cot x x  Câu Tìm chu kì T hàm số y  cos   2019  2  A T  4 B T  2 D T   C T  2 Đáp án: 1–C 2–A 3–D 4–D 5–C 6–D 7–B 8–A Trang 13 CHƯƠNG LƯỢNG GIÁC CHUYÊN ĐỀ PHƯƠNG TRÌNH LƯỢNG GIÁC PHẦN 1: LÝ THUYẾT TRỌNG TÂM Phương trình lượng giác  x    k2 sin x  sin    ,k   x      k2 cos x  cos   x     k2, k   tan x  tan   x    k, k   cot x  cot   x    k, k   Các trường hợp đặc biệt cos x   x  sin x   x  k, k     k, k   cos x   x  k2, k   sin x   x  cos x  1  x    k2, k   sin x  1  x    k2, k     k2, k   Một vài phép biến đổi đặc biệt hay gặp  sin 2x   sin x  cos x   sin 2x   sin x  cos x  x x   sin x   sin  cos  2  x x   sin x   sin  cos  2  sin x  cos x   sin 2x sin x  cos x   sin 2x PHẦN 2: CÁC DẠNG BÀI TẬP Dạng 1: Phương trình lượng giác Phương pháp giải Nếu u, v hàm theo biến x  u  v  k2 sin u  sin v   ,k   u    v  k2 tan u  tan v  u  v  k, k   cos u  cos v  u   v  k2, k   cot u  cot v  u  v  k, k   Ví dụ minh họa  2x   Ví dụ 1: Giải phương trình sin   0  3 A x  k  k    C x    k  k    B x  2 k3  k   D x   k3  k   2 Trang Hướng dẫn 2x  2x   k3  2x    0   k    k  x   Cách 1: sin  k   3 3 2  3 Cách 2: Sử dụng máy tính CASIO fx 570 VN PLUS Bước 1: Thiết lập môi trường radian: SHIFT MODE  2X     vào máy tính Bước 2: Nhập biểu thức sin   3 Bước 3: Sử dụng phím CALC (phím gán giá trị) để kiểm tra bốn đáp án  , loại A Đối với đáp án A, ta thay x = π: Nhập CALC π ta kết Đối với đáp án B, ta thay x  2 2 : Nhập CALC ta kết 0,342  0, loại B 3 Đối với đáp án B, ta thay x    : Nhập CALC ta kết -0,342  0, loại C 3 Đối với đáp án D, ta thay x    : Nhập CALC ta kết 2 → Chọn D khoảng (0; 3π) là: Ví dụ 2: Số nghiệm phương trình sin 2x  A B C D Hướng dẫn   2x    sin 2x  sin   Cách 1: sin x   2x      k2 x   2 x   k2    k ,k    k   17 Với x    k ta có:   k  3    k   k  0;1; 2 6 6 Với x     k ta có:   k  3    k   k  0;1; 2 3 3 Mỗi họ nghiệm có nghiệm thuộc (0;3π) nên phương trình có nghiệm thuộc (0;3π) Cách 2: Sử dụng máy tính CASIO fx 570VN PLUS Bước 1: Thiết lập mơi trường tính theo radian: SHIFT MODE Bước 2: Sử dụng công cụ TABLE: MODE Nhập hàm số f  x   sin  2X   Start? = End? 3π = Step? 3 End  Start = (Ta thường lấy Step 15 15 Ta bảng giá trị gồm hai cột x f (x) Trang Bước 3: Ta tìm số nghiệm phương trình dựa vào tính chất đổi dấu, quan sát cột f (x) xem có lần đổi dấu từ âm sang dương từ dương sang âm, ta thấy có lần đổi dấu, phương trình có nghiệm khoảng (0; 3π) → Chọn C   Ví dụ 3: Gọi S tập hợp giá trị nguyên tham số m để phương trình cos  2x    m  có 3  nghiệm Tính tổng T phần tử S A T = B T = C T = -2 D T = -6 Hướng dẫn     cos  2x    m   cos  2x    m  3 3     Vì 1  cos  2x    nên để phương trình có nghiệm thì: 1  m    3  m  1 3  Vậy tập số nguyên m thỏa mãn S  3; 2; 1 Tổng T   3   2    1  6 → Chọn D  a  Ví dụ 4: Nghiệm dương nhỏ phương trình cos  3x    cos x  có dạng , a; b hai 3 b  số nguyên tố Tính a + b A B C D Hướng dẫn       Cách 1: cos  3x    cos x   cos  3x     cos x  cos  3x    cos  x    3 3 3     2   3x   x    k2  x   k   k   3x      x     k2  x     k   Với k = 0, phương trình có nghiệm dương là: x  2 5 2  x   Với k = 1, phương trình có hai nghiệm dương là:   x        2  3 Vậy nghiệm dương bé phương trình là: x   Do a; b hai số nguyên tố nên a = 1; b = → a + b = Cách 2: Sử dụng máy tính CASIO fx 570VN PLUS Bước 1: Thiết lập mơi trường tính theo độ: SHIFT Mode Bước 2: Sử dụng công cụ TABLE: MODE Trang   Nhập hàm f  x   cos  3X    cos  X  3  Start? = → End? 180 = → Step? 10 = Ta bảng giá trị gồm hai cột x f (x) Bước 3: Nhìn vào giá trị cột f (x), xem giá trị f (x) = đầu tiên, ứng với f (x) = 0, ta thấy x = 60 Do nghiệm dương nhỏ phương trình x = 600, ứng với x   → Chọn D  tan x  Ví dụ 5: Nghiệm phương trình A x    k  k     C x    k  k    B x    k2  k    D x    k  k    Hướng dẫn Cách 1:  tan x   tan x      tan x    x    k  k    6 Cách 2: Sử dụng máy tính CASIO fx 570VN PLUS Bước 1: Thiết lập môi trường radian: SHIFT MODE Bước 2: Nhập biểu thức  tan  X  vào máy tính Bước 3: Sử dụng phím CALC (phím gán giá trị) để kiểm tra bốn đáp án Đối với đáp án A, ta thay x    Nhập CALC ta kết  , loại A 3 Đối với đáp án B, ta thay x    Nhập CALC ta kết Math error, loại B D 2   Đối với đáp án C, ta thay x   Nhập CALC  ta kết 6 → Chọn C Bài tập tự luyện Câu Nghiệm phương trình đặc biệt sau sai?  A sin x  1  x    k2 B sin x   x  k C sin x   x  k2 D sin x   x    k2 Câu Nghiệm phương trình sinx cosx = là: A x    k2 B x  k C x  k2 D x    k2 Câu Phương trình 2sin  2x  40o   có số nghiệm thuộc  180o ;180o  là: A B C D Trang Câu Phương trình cosx = m + có nghiệm m là: A 1  m  C m  2 B m  D 2  m  Câu Nghiệm âm lớn nghiệm dương nhỏ phương trình sin 4x  cos 5x  theo thứ tự là: A x     ;x  18 B x    2 ;x  18 C x     ;x  18 D x     ;x  18 Đáp án: 1–C 2–B 3–B 4–D 5–A Dạng 2: Phương trình bậc hai hàm số lượng giác Phương pháp giải Dạng phương trình: Ví dụ: Tìm họ nghiệm phương trình a sin x  b sin x  c  sin x  3sin x   a cos x  bcosx  c  A x  a tan x  b tan x  c    k2, k   B x    k2, k   a cot x  b cot x  c  Ta đặt ẩn phụ t giải phương trình bậc hai đổi với C x  k, k   t là: at  bt  c  D x  Cụ thể: a sin x  b sin x  c  Đặt t  sin x  1  t  1 acos x  b cos x  c  Đặt t  cos x  1  t  1 a tan x  b tan x  c  Điều kiện xác định cosx  Đặt t = tanx a cot x  b cot x  c    k, k   Hướng dẫn Đặt t  s inx  1  t  1 , phương trình trở thành: t  t  3t     t  4  I  Với t = 1, ta có: sin x   x    k2  k    → Chọn A Điều kiện xác định sinx  Đặt t = cotx Ví dụ minh họa Ví dụ 1: Phương trình cos x  3cos x   có nghiệm là: A x  k2, x     k2, x    k2 3  C x    k2, x  k, x  k2 B x  k, x     k, x    k 3 D x  k2, x     k2, x   k Hướng dẫn Cách 1: Đặt t  cos x  1  t  1 Phương trình trở thành: Trang t  2t  3t     (thỏa mãn điều kiện) t   2 Với t =  cos x   x  k2  k      x   k2  1  Với t   cos x   cos x  cos   k   2  x     k2  Vậy họ nghiệm phương trình: x  k2, x     k2, x    k2  k    3 Cách 2: Sử dụng máy tính CASIO fx 570VN PLUS Bước 1: Thiết lập môi trường radian: SHIFT Mode Bước 2: Nhập biểu thức cos  X   3cos  X   Bước 3: Sử dụng phím CALC (phím gán giá trị) để kiểm tra bốn đáp án: Ta thay x = 2π thuộc họ nghiệm x = k2π 0, nghiệm x = 2π thỏa mãn Ta thay x = π thuộc họ nghiệm x = kπ 6, nghiệm x = π khơng phải nghiệm phương trình nên loại đáp án chứa x = kπ đáp án B C    thuộc họ nghiệm x   k2 , 1, nghiệm x   k2 nghiệm 2 phương trình nên loại đáp án D Ta thay x  → Chọn A Ví dụ 2: Nghiệm dương bé phương trình  5sin x  cos x  là: A  B   C  D  Hướng dẫn Cách 1:  5sin x  cos x    5sin x  1  sin x   sin x   2sin x  5sin x     sin x    2 Với sin x    1 , phương trình vơ nghiệm Với sin x   x     k2, k   , nghiệm dương bé phương trình 2 Cách 2: Sử dụng máy tính CASIO fx 570VN PLUS Bước 1: Thiết lập môi trường radian: SHIFT Mode Bước 2: Nhập biểu thức  5sin  X   cos  X  Bước 3: Sử dụng phím CALC (Phím gán giá trị) để kiểm tra bốn đáp án, xem đáp án làm biểu thức có giá trị nhỏ Trang → Chọn C     Ví dụ 3: Tìm m để phương trình cos 2x   2m  1 cos x  m   * có hai nghiệm x   ;  :  2 A 1  m  B  m  D 1  m  C  m  Hướng dẫn  cos x    Cách 1: cos 2x   2m  1 cos x  m    cos x   2m  1 cos x  m    cos x  m      Vì x   ;  nên  cos x  Do cos x   (loại)  2     Vậy để phương trình có hai nghiệm x   ;   cos x    m   2 Cách 2: Sử dụng máy tính CASIO fx 570VN PLUS Bước 1: Thiết lập mơi trường tính theo radian: SHIFT Mode Bước 2: Sử dụng công cụ TABLE: MODE Thay m = -1 thuộc đáp án A vào (*) Phương trình (*) trở thành: cos 2x  3cos x   Nhập giá trị hàm f  x   cos  2X   3cos  X   vào ô f (x) = Start ?      End ?   Step ?  2 15 Ta bảng giá trị gồm hai cột x f (x) Ta tìm số nghiệm phương trình dựa vào tính chất đổi dấu, quan sát cột f(x) xem có lần đổi dấu từ âm sang dương từ dương sang âm, ta thấy lần đổi dấu nào, phương trình khơng có nghiệm với m = -1 Ta loại đáp án A D Thay m = thuộc đáp án C Phương trình trở thành: cos x  cosx  Nhập giá trị hàm f  x   cos 2x  3cos x  vào ô f (x) = Start ?      End ?   Step ?  2 15 Ta bảng giá trị gồm cột x f (x) Ta tìm số nghiệm phương trình dựa vào tính chất đổi dấu, quan sát cột f (x) xem có lần đổi dấu từ âm sang dương từ dương sang âm, ta thấy khơng có lần đổi dấu nào, phương trình khơng có nghiệm với m = -1 Ta loại đáp án C → Chọn B Bài tập tự luyện Câu Nghiệm dương bé phương trình: 2sin x  5sin x   là: A x   B x   C x  3 D x  5 Câu Nghiệm phương trình sin x  sin x  thỏa mãn điều kiện  x   là: Trang A x   B x   C x  D x    D x     k2 Câu Nghiệm phương trình cos x  sin x   là:  A x    k2 B x    k2  C x    k Câu Họ nghiệm phương trình sin 2x  2sin 2x   là:  A   k B   k C   k2  D   k2    Câu Tìm m để phương trình 2sin x   2m  1 sin x  m  có nghiệm x    ;0    A   m  B < m < C -1 < m < D < m < Đáp án 1–A 2–A 3–A 4–B 5–C Dạng 3: Phương trình bậc sinx cosx Phương pháp giải * Dạng phương trình: asinx + bcosx = c Ví dụ: Tìm họ nghiệm phương trình * Điều kiện để phương trình có nghiệm sin x  cos x  a b c 2 Bước 1: Kiểm tra điều kiện để phương trình có nghiệm A x    5  k2; x   k2 12 12  3  k2 B x    k2; x  Nếu a  b  c , ta kết luận phương trình vơ 4 nghiệm  2 x   k2  ; x   k2 C 3 Nếu a  b  c , ta thực bước 2 2 Bước 2: Chia hai vế phương trình cho D x     k2; x   5  k2 4 a  b ta được: Hướng dẫn a b c sin x  cos x  Phương trình có a = 1; b = ; c = a  b2 a  b2 a  b2 a  b2   c2  a   cos   Chia hai vế phương trình cho  a  b2 Đặt  , phương trình trở b  a  b  12   ta được:  sin   a  b thành: sin x  cos x  2 c cos  sin x  sin  cos x   1 a  b2   cos c Đặt  , phương trình trở thành:  sin  x      2  a b  sin    Trang   cos sin x  sin cosx  3    x    k2      sin  x    sin   3   x    3  k2     x   12  k2  k    x  5  k2  12 → Chọn A Chú ý: Ta có kết sau:  a  b  a sin x  b cos x  a  b , kết ứng dụng ta gặp tồn tìm giá trị lớn nhỏ hàm số a sin x  b cos x f  x   a sin x  b cos x, f  x   c sin x  d cos x Một vài công thức hay dùng:     sin x  cos x  sin  x    cos  x   4 4       sin x  cos x  sin  x    cos  x   4 4       sin x  cos x  cos  x    2sin  x   6 3       sin x  cos x  cos  x    2sin  x   6 3       sin x  cos x  2sin  x    cos  x   6 3       sin x  cos x  2sin  x    cos  x   6 3   Ví dụ minh họa Ví dụ 1: Phương trình sau vơ nghiệm? A 2sin x  cos x  C B 3sin x  cos x  sin 2x  cos 2x  D 3sin x  cos x  Hướng dẫn Điều kiện để phương trình asinx + bcosx = c vơ nghiệm a  b  c Đáp án A có a = 2; b = -1; c = 3, ta có: a  b  22   1   32   c 2 Do phương trình 2sinx – cosx = vơ nghiệm → Chọn A Ví dụ 2: Với giá trị m phương trình  m  1 sin x  cos x  có nghiệm? A 3  m  B  m  m  C   m  3 D   m  Hướng dẫn Cách 1: Điều kiện để phương trình có nghiệm là: a  b  c Trang Phương trình:  m  1 sin x  cos x  có a  m  1; b  1;c  Để phương trình có nghiệm thì: m   m  2 a  b  c   m  1     m  1      m   2  m  3 Cách 2: Sử dụng máy tính CASIO fx 570VN PLUS Sử dụng phím SHIFT SOLVE thay giá trị m thuộc khoảng để kiểm tra tính có nghiệm phương trình Đáp án kết Can’t Solve, tức giá trị m làm phương trình vơ nghiệm Từ đáp án A, ta thay m = vào phương trình ta sin x  cos x  Nhập sin  X   cos  X   Ấn SHIFT SOLVE, ta kết Can’t Solve, tức với m = 0, phương trình vơ nghiệm, ta loại đáp án chứa m = đáp án A, B, D → Chọn C Ví dụ 3: Tìm m để phương trình m sin x   m  1 cos x  vô nghiệm A m  1;  B m   ; 1   0;   C m   1;0  D m   ; 1   0;   Hướng dẫn Cách 1: Điều kiện để phương trình vơ nghiệm a  b  c Ta có a = m, b = m + 1, c = Để phương trình vơ nghiệm thì: a  b  c m   m  1   m  m   1  m  Vậy với m   1;0  phương trình ban đầu vơ nghiệm Cách 2: Sử dụng máy tính CASIO fx 570VN PLUS Sử dụng phím SHIFT SOLVE thay giá trị m thuộc khoảng để kiểm tra tính có nghiệm phương trình Từ đáp án A, ta thay m  3 3  vào phương trình, nhập sin  X     1 cos  X   2 2  Ấn SHIFT SOLVE, ta kết số, tức với m  chứa m  , phương trình có nghiệm, ta loại đáp án đáp án A, B, D → Chọn C Ví dụ 4: Hàm số y  3sin x  cos x  đạt giá trị lớn a giá trị nhỏ b Giá trị a – 6b là: A B 10 C 12 D 20 Hướng dẫn Áp dụng kết  a  b  a sin x  b cos x  a  b ta có: Trang 10 Ta có  32   4   3sin x  cos x  32   4   5  3sin x  cos x  2  5   3sin x  cos x      y  12 Vậy a = max y = 12, b = y = Do a – 6b = 12 – 6.2 = → Chọn A Bài tập tự luyện Câu Điều kiện để phương trình asin5x + bcos5x = c có nghiệm là: A a  b  c B a  b  c C a  b  c D a  b  c Câu Nghiệm phương trình sin x  cos x  là: A x    k2 B x    k2 C x    k D x    k Câu Số nghiệm phương trình sinx + cosx = khoảng (0; π) là: A B C D Câu Điều kiện để phương trình msinx + 8cosx = 10 vô nghiệm là:  m  6 B  m  A m > C m < -6 D -6 < m < Câu Điều kiện để phương trình 12sinx + mcosx = 13 có nghiệm là:  m  5 B  m  A m > C m < -5 D -5 < m < Đáp án 1–C 2–D 3–B 4–D 5–B Dạng 4: Phương trình bậc sinx cosx Phương pháp giải Dạng phương trình: Ví dụ: a.sin x  b.sin x.cos x  c.cos x  d 6sin x  14 sin x.cos x  8cos x  là:  x  A  x   Họ   k   k    x   k C   x    k  12 nghiệm  x  B  x   phương trình   k   k 3   x   k D   x    k  Trường hợp 1: Với cosx = Thế vào phương trình  Trường hợp 1: với cosx =  x   k phương thử nghiệm trình trở thành: 6sin x   sin x   cos x  Trang 11  cos x   x  Trường hợp 2: Với cos x   x    k2 Trường hợp 2: Với cos x   x  Chia hai vế phương trình cho cos2x ta được: a   k sin x sin x d  b c 0 cos x cos x cos x   k Chia hai vế phương trình cho cos2x ta được: tan x  14 tan x   cos x  a.tan x  b.tan x  c  d 1  tan x    tan x  14 tan x   1  tan x    a  d  tan x  b.tan x  c  d   14 30 tan x  14  tan x  Đặt t = tanx, đưa phương trình phương trình bậc hai ẩn t: x  a  d  t  bt  c  d    k Giải phương trình theo ẩn t, sau suy nghiệm phương trình lượng giác Bước Kết luận họ nghiệm phương trình Vậy phương trình có nghiệm là: Chú ý: Công thức x     tan x  1 x   k  cos x      k, x   k → Chọn A Ví dụ minh họa Ví dụ 1: Phương trình 3cos 4x  5sin 4x   sin x cos x có nghiệm là:  A x    k, k   C x      k ,k  18 B x      k ,k  12 D x      k ,k  24 Hướng dẫn Cách 1: Trường hợp 1: Với cos 4x   4x  5sin 4x   sin 4x  (mâu thuẫn cos x   sin x   cos x  ) Trường hợp 2: Với cos 4x   x   tan 4x    k  k  x   , thay vào phương trình ta có:  k  chia hai vế cho cos 4x ta được:  tan 4x   tan 4x  1  tan 4x   tan 4x cos 4x  tan 4x  tan 4x    tan 4x     k  4x    k  x    24 Cách 2: Sử dụng máy tính CASIO fx 570VN PLUS Bước 1: Thiết lập môi trường radian: SHIFT Mode Trang 12 Bước 2: Nhập biểu thức 3cos  4X   5sin  4X    sin  4X  cos  4X  2 Bước 3: Sử dụng phím CALC (phím gán giá trị) để kiểm tra bốn đáp án: Đáp án A: Ta thay x    vào kết khác 0, loại A Đáp án B: Ta thay x    vào kết khác 0, loại B 12 Đáp án C: Ta thay x    vào kết khác 0, loại C 18 → Chọn D Ví dụ 2: Cho phương trình sin x  cos3 x  sin x  cos x Tính tổng nghiệm dương nhỏ 2π phương trình A 5 B π C 2π D  Hướng dẫn Cách 1: Trường hợp 1: cos x   x  sin x  1 (thỏa mãn) Do x    k , phương trình trở thành:   k nghiệm phương trình Trường hợp 2: cos x  Chia hai vế phương trình cho cos2x ta được: tan x   tan x 1  tan x    tan x  tan x  tan x   (phương trình vơ nghiệm) Theo đề ta có:  x  Với k = ta có: x   1  k  2   k  nên k  0;1 2   3 Với k = ta có: x     2 Do phương trình có nghiệm dương nhỏ 2π Vậy tổng nghiệm phương trình  3 ; 2  3   2 Cách 2: Sử dụng máy tính CASIO fx 570VN PLUS Bước 1: Thiết lập mơi trường tính theo độ: SHIFT Mode Bước 2: Sử dụng công cụ TABLE: MODE Vì khoảng xét lớn nên ta chia nhỏ thành bốn khoảng xét Chuyển vế phải sang vế trái Nhập hàm số f  x   sin  X   cos  X   sin  X   cos  X  Start? = → End? 180 = → Step? 10 = Ta bảng giá trị gồm hai cột x f (x) Kiểm tra cột f (x), xét giá trị f (x) = 0, ta giá trị x = 90 Bấm AC Giữ nguyên hàm f (x) Start? 180 = → End? 360 = → Step? 10 = Trang 13 Ta bảng giá trị gồm hai cột x f (x) Kiểm tra cột f (x), xét giá trị f (x) = 0, ta giá trị x = 270 Bước 3: Do tổng nghiệm 90 + 270 = 360 Đổi sang radian ta 360  2 180 → Chọn C Ví dụ 3: Tìm m để phương trình sin x   2m   sin x cos x  1  m  cos x  m 1 có nghiệm B m   A m  2  m  C 2  m  D 2  m  Hướng dẫn Cách 1: Nếu cosx = nghiệm (1), từ (1) suy ra: m  sin x  cos x       sin x  m sin x  m  x   k, k   Nếu m  cosx = khơng nghiệm (1), chia hai vế cho cos2x được: tan x   2m   tan x   m  1  m 1  tan x    m  1 tan x   m  1 tan x  2m   Đặt t = tanx, phương trình (1) có dạng:  m  1 t   m  1 t  2m      2  m  m   Phương trình (2) có nghiệm    2  m  m  m  Vậy với 2  m  phương trình (1) có nghiệm Cách 2: Sử dụng máy tính CASIO fx 570 VN PLUS Sử dụng phím SHIFT SOLVE thay giá trị m thuộc khoảng để kiểm tra tính có nghiệm phương trình Từ đáp án A, ta thay m = vào phương trình, nhập phương trình: sin X   2.4   sin X cos X  1   cos X  Ấn SHIFT SOLVE, ta kết Can’t solve, tức với m = 4, phương trình vơ nghiệm, ta loại đáp án A B Từ đáp án C D, ta thay m = vào phương trình, nhập phương trình: sin X   2.2   sin X cos X  1   cos X  Ấn SHIFT SOLVE, ta kết Can’t solve, tức với m = 2, phương trình vơ nghiệm, ta loại đáp án có chứa m = 2, tức đáp án C → Chọn D Ví dụ 4: Cho phương trình 6sin x  cos3 x  5sin 2x cos x Tìm số nghiệm dương nhỏ 3π phương trình A B C D Hướng dẫn 6sin x  cos3 x  5sin 2x cos x  6sin x  cos3 x  10sin x cos x 1 Trang 14 Trường hợp 1: Với cos x   sin x   1: 1  6  (vô lý) Trường hợp 2: Với cos x  Chia hai vế (1) cho cos3x được: sin x cos3 x sin x cos x   10  tan x 1  tan x    10 tan x cos3 x cos3 x cos3 x   tan x  tan x    tan x  1  x    k, k    13 Ta có:    k  3   k  4 Do k  1; 2;3 nên phương trình có ba nghiệm dương nhỏ 3π → Chọn C Bài tập tự luyện Câu Một họ nghiệm phương trình 3sin x  4sin x cos x  5sos x  là:  A   k2 B   k  C   k D 3  k2 Câu Phương trình cos x  sin x   sin x có nghiệm là:  x  k2 A   x    k2   x  k  B  x    k   2  x  k  C  x    k   3  x  k D   x    k  Đáp án: 1–B 1–D Dạng 5: Phương trình đối xứng sinx cosx Phương pháp giải Dạng phương trình: Ví dụ: Tìm họ nghiệm phương trình a  sin x  cos x   b sin x.cos x  c  sin x  cos x  sin x cos x    Đặt ẩn phụ t  sin x  cos x, t    x   k2  A k     x  k   x   k  B k     x  k   x   k2  C k     x  k2   x   k2  D k     x  k2 Hướng dẫn   Đặt t  sin x  cos x, t   t 1 t   2sin x.cos x  sin x.cos x   vào t2 1 t   2sin x.cos x  sin x.cos x  phương trình ta phương trình bậc hai Thay vào phương trình ta được: t, giải t, sau tìm nghiệm phương trình Trang 15 t Chú ý:     sin x  cos x  sin  x    cos  x   4 4       sin x  cos x  sin  x     cos  x   4 4   t2 1    t  2t    t  Với t = thỏa mãn điều kiện, ta có:    x    k2    4 cos  x      4   x       k2  4   x   k2   ,k    x  k2 → Chọn C Ví dụ minh họa Ví dụ 1: Họ nghiệm phương trình sin x   sin x  cos x    k2  x   B  k    x    k2    x   k  A k     x    k  k  x   C  k   k  x       x   k2 D  k     x    k2 Hướng dẫn Đặt t  sin x  cos x, t    2;  , ta có t   2sin x.cos x  2sin x.cos x   t Thay vào phương trình ta được:  t  4t   t  4t    t  (thỏa mãn)   x   k2    Với t = ta có sin x  cos x   sin  x     k    4   x    k2 → Chọn D Ví dụ 2: Số nghiệm dương nhỏ 7π phương trình 2  sin x  cos x    sin 2x A B C D Hướng dẫn   Đặt t  sin x  cos x t   t   sin x  cos x   sin 2x   t 2 Phương trình trở thành: 2   1  t   t  2t    t  (thỏa mãn) Với t  ta có: x       sin  x     sin  x     x    k2 4 4   3  k2  k    Trang 16 Nghiệm phương trình: x  3  k2,  k    3 3 25  k2    k Do k nguyên nên k  0;1; 2;3 8 Vì < x < 7π nên  Do có bốn nghiệm dương phương trình nhỏ 7π → Chọn C Bài tập tự luyện Câu Phương trình sin2x – 12 (sinx – cosx) + 12 = có họ nghiệm là: A x    k, x    k2  k    B x   k2  k2, x    k   C x   k k2  , x    k   3 D x    k2, x    k2  k    2  sin x  cos x   tan x  cot x có họ nghiệm là: Câu Phương trình A x    k,  k    B x   k  k   C x   k2  k   D x    k2  k    Đáp án: 1–D 2–D Phần 3: Bài tập tổng hợp Câu Cho biết x   A cos x     k2 họ nghiệm phương trình sau đây? B cos x   C 2sinx + = D 2sin x   Câu Phương trình sin2x – 3cosx – = có nghiệm?  A x    k2 B x    k2 C x    k D Vô nghiệm Câu Với giá trị m phương trình 2sinx – m = vô nghiệm? A 2  m  B m < - C m > D m < -2 m > Câu Có giá trị nguyên m để phương trình (sinx + 2cosx + 3) m = + cosx có nghiệm? A B C D Câu Một họ nghiệm phương trình cos x  6sin x cos x   A 3  k2  k    B   k  k     C   k  k     D   k2  k    Câu Phương trình cos2x + 2cosx – = có nghiệm A x  k2  k    B x = C x    k2  k    D Vô nghiệm Câu Tìm điều kiện để phương trình msinx + 12cosx = -13 vô nghiệm A m >  m  5 B  m  C m < -5 D -5 < m < Trang 17     Câu Phương trình tan  4x    tan   2x   có nghiệm dương nhỏ π? 3  6  A B C D Câu Cho phương trình (m2 + 2) cos2x – 2msin2x + = Để phương trình có nghiệm giá trị thích hợp tham số m là: 1 B   m  2 A 1  m  1 C   m  4 D m  Đáp án: 1–B 2–D 3–D 4–C 5–B 6–A 7–D 8–A 9–D Trang 18 CHƯƠNG 4: TỔ HỢP, XÁC SUẤT CHUYÊN ĐỀ 1: HAI QUY TẮC ĐẾM, HOÁN VỊ, TỔ HỢP, CHỈNH HỢP PHẦN 1: LÝ THUYẾT TRỌNG TÂM Quy tắc cộng Quy tắc nhân Một cơng việc thực theo hai phương án A B Nếu phương án A có m cách thực hiện, phương án B có n cách thực khơng trùng với cách phương án A cơng việc có m  n cách thực Một cơng việc bao gồm hai công đoạn A B Nếu công đoạn A có m cách thực ứng với cách có n cách thực cơng đoạn B cơng việc có m.n cách thực hiện Mở rộng: Một cơng việc hồn thành Mở rộng: Một cơng việc hồn thành m m phương án, đó: cơng đoạn liên tiếp, đó: • Phương án có n1 cách thực • Cơng đoạn có n1 cách thực • Phương án có n2 cách thực • Cơng đoạn có n2 cách thực •… •… • Phương án m có nm cách thực • Cơng đoạn m có nm cách thực Khi cơng việc có n1  n   n m cách thực Khi cơng việc có n1n2 … nm cách thực hiện Chú ý: • Nếu A B hai tập hợp hữu hạn n  A  B   n  A   n  B   n  A  B  • Nếu A B hai tập hợp hữu hạn khơng giao n  A  B   n  A   n  B  • Nếu A1 , A , , A m tập hợp hữu hạn tùy ý, đôi không giao thì: n  A1  A   A m   n  A1   n  A    n  A m  Hoán vị Định nghĩa: Chỉnh hợp Định nghĩa: Tổ hợp Định nghĩa: Một tập hợp gồm n phần tử Cho tập hợp A gồm n phần tử  n  1 Mỗi cách xếp n phần Mỗi cách xếp k phần tử tập hợp A theo thứ tự tử theo thứ tự gọi chỉnh hợp chập k gọi hoán vị n n phần tử tập A phần tử Cho tập hợp A gồm n phần tử Số hoán vị: Số tổ hợp: Số chỉnh hợp: Mỗi tập gồm k 1  k  n  phần tử A gọi tổ hợp chập k n phần tử Trang Số hoán vị n phần tử Số chỉnh hợp chập k n Số tổ hợp chập k n phần kí hiệu Pn : phần tử 1  k  n  kí hiệu tử 1  k  n  kí hiệu Ckn , Pn  n!  n  n  1 2.1 ta có: A kn , ta có: A kn  n!  n  k ! Ckn  n! k! n  k  ! Tính chất: Quy ước: 0!  A 0n   n   C0n  Cnn   n   A nn  Pn  n! Ckn  Cnn  k   k  n  Ckn 11  Ckn 1  Ckn 1  k  n  Dấu hiệu phân biệt: Dấu hiệu phân biệt: Dấu hiệu phân biệt: • Lấy n phần tử n phần • Lấy k phần tử n phần • Lấy k phần tử n phần tử tử tử • Có xếp theo thứ tự • Có xếp theo thứ tự • Khơng có xếp theo thứ tự PHẦN 2: CÁC DẠNG BÀI TẬP Dạng 1: Hai quy tắc đếm Ví dụ minh họa Ví dụ 1: Phương Anh có postcard SNSD, postcard TVXQ 10 postcard EXO Phương Anh cần chọn postcard để tặng bạn Hỏi có cách chọn? A 20 cách B 240 cách C 30 cách D 42 cách Hướng dẫn Trường hợp 1: Phương Anh chọn postcard SNSD  Có cách chọn Trường hợp 2: Phương Anh chọn postcard SNSD  Có cách chọn Trường hợp 3: Phương Anh chọn 10 postcard SNSD  Có 10 cách chọn Theo quy tắc cộng, có tổng cộng   10  20 cách chọn  Chọn A Ví dụ 2: Từ tỉnh A tới tỉnh B ô tô, tàu hỏa, tàu thủy máy bay Từ tỉnh B tới tỉnh C ô tô tàu hỏa Muốn từ tỉnh A đến tỉnh C bắt buộc phải qua tỉnh B Số cách từ tỉnh A đến tỉnh C là: A cách B cách C cách D cách Hướng dẫn Giai đoạn 1: Để từ tỉnh A đến tỉnh B có cách chọn phương tiện di chuyển Giai đoạn 2: Để từ tỉnh B đến tỉnh C có cách chọn phương tiện di chuyển Theo quy tắc nhân có 4.2  cách di chuyển từ A đến C  Chọn D Ví dụ 3: Trong tuần, bạn A dự định ngày thăm người bạn 12 người bạn Trang Hỏi bạn A lập kế hoạch thăm bạn (biết A thăm bạn nhiều lần)? A 7! B 35831808 C 12! D 3991680 Hướng dẫn Vào thứ 2: A có 12 cách chọn 12 người bạn để thăm Vào thứ 3: A có 12 cách chọn 12 người bạn để thăm Vào thứ 4: A có 12 cách chọn 12 người bạn để thăm Vào thứ 5: A có 12 cách chọn 12 người bạn để thăm Vào thứ 6: A có 12 cách chọn 12 người bạn để thăm Vào thứ 7: A có 12 cách chọn 12 người bạn để thăm Vào chủ nhật: A có 12 cách chọn 12 người bạn để thăm Theo quy tắc nhân có 127  35831808 kế hoạch thăm bạn  Chọn B Bài tập tự luyện Câu Một lớp học có 18 học sinh nam 20 học sinh nữ Nếu muốn chọn học sinh nam học sinh nữ dự thi số cách chọn là: A 38 B 18 C 20 D 360 Câu Có cách xếp người A, B, C, D lên toa tàu? Biết toa chứa người A 81 B 68 C 42 D 98 Đáp án: 1–D 2–A Dạng 2: Bài toán đếm số Phương pháp giải Khi lập số tự nhiên x  a1 a n , a i  0,1, 2, ,9 a1  0, ta cần lưu ý: • x số chẵn  a n số chẵn • x số lẻ  a n số lẻ • x chia hết cho  a1  a   a n chia hết cho • x chia hết cho  hai số tận x chia hết cho • x chia hết cho  a n  0;5 • x chia hết cho  x số chẵn chia hết cho • x chia hết cho  ba số tận x chia hết cho • x chia hết cho   a1  a   a n  chia hết cho • x chia hết cho 11  tổng chữ số hàng lẻ trừ tổng chữ số hàng chẵn số chia hết cho 11 • x chia hết cho 25  hai chữ số tận 00, 25, 50, 75 Trang Ví dụ minh họa Ví dụ 1: Cho tập hợp M có 10 phần tử Số tập gồm phần tử M là: A A10 B A10 C C10 D 102 Hướng dẫn 2 Lấy 10 phần tử, có C10 cách Vậy số tập gồm phần tử M C10  Chọn C Ví dụ 2: Từ chữ số 1,2, 3, 4, 5, 6, lập số có bốn chữ số khác nhau? A 804 B 408 C 480 D 840 Hướng dẫn Ta chọn số chữ số xếp để lập thành số cỏ chữ số, nên số có chữ số lập là: A 74  840 số  Chọn D Ví dụ 3: Có số tự nhiên có bốn chữ số? A 5040 B 9000 C 720 D 1440 Hướng dẫn Gọi số tự nhiên có bốn chữ số abcd , a  0; a, b, c, d  0;1; 2; ;9 a có cách chọn tập 1; 2; ;9 b, c, d cso 10 cách chọn tập 0;1; 2; ;9 Vậy số tự nhiên có bốn chữ số là: 9.10.10.10  9.103  9000 số  Chọn B Ví dụ 4: Có số tự nhiên chẵn gồm hai chữ số khác lập từ tập hợp A  0;1; 2;3; 4;5 ? A B 15 C 13 D 22 Hướng dẫn Số tự nhiên thỏa mãn có dạng ab , đó: a  1; 2; ;9 , b  0; 2; 4 số cần lập số chẵn Trường hợp 1: Với b   a  1; 2;3; 4;5 , lập số Trường hợp 2: Với b   b có cách chọn 2, a có cách chọn   Do có 2.4  số Theo quy tắc cộng, có:   13 số  Chọn C Ví dụ 5: Có số tự nhiên có sáu chữ số cho chữ số đứng sau lớn chữ số đứng trước? A 84 B 60480 C 84600 D 75600 Hướng dẫn Gọi a1a a 3a a 5a số có chữ số a1  a  a  a  a  a Trang Ta có a i  nên a i  E  1; 2;3; 4;5;6;7;8;9 * Lấy chữ số thuộc E có C96 cách * Mỗi chữ số lập số thỏa yêu cầu toán Vậy số số lập C96  84 số  Chọn A Ví dụ 6: Tìm số ước số dương số 490000? A 260 B 32 C 25 D 75 Hướng dẫn B  490000  2.104  24.54.7 Vì ước số dương B có dạng U  2m.5n.7 p  m, n, p  ;  m  4,  n  4,  p   Ta có: m có cách chọn (từ tới 4); n có cách chọn (từ tới 4); p có cách chọn (từ tới 2) Vậy có 5.5.3  75 ước số dương B  Chọn D Chú ý: Công thức tổng quát tìm ước số dương số X: Phân tích X thừa số nguyên tố: Giả sử X  A a Bb Cc Dd E e (A, B, C, D, E số nguyên tố, a, b, c, d, e  0;1; 2; ;9 ) Tổng tất ước số X là:  a  1 b  1 c  1 d  1 e  1 Ví dụ 7: Có số tự nhiên gồm sáu chữ số khác đôi có chữ số khơng có chữ số 1? A 12000 B 23300 C 33600 D 6720 Hướng dẫn Gọi a1a a 3a a 5a số có sáu chữ số khác cần lập, a i  a j , i  j, a i  0;1; 2; ;9 , a1  Xếp số vào năm vị trí từ a tới a có cách xếp Chọn số thuộc tập hợp 2;3; 4;5;6;7;8;9 xếp vào vị trí lại có A85 cách Vậy ta có 5.A85  33600 số  Chọn C Ví dụ 8: Có số tự nhiên gồm bảy chữ số chữ số xuất hai lần, chữ số xuất ba lần chữ số lại xuất khơng q lần? A 12900 B 23300 C 11280 D 13440 Hướng dẫn Gọi a1a a 3a a 5a a số có bảy chữ số cần lập  a i  0;1; 2; ;9 , a1   Ta tìm số số cần lập cách tìm số số lập (bao gồm trường hợp số đứng đầu) trừ số số có số đứng đầu Chọn hai vị trí để xếp hai số 2: có C72 cách; Trang Chọn ba vị trí để xếp ba số 3: có C35 cách; Chọn hai số (khác 3) xếp vào hai vị trí lại: có A82 cách;  Có C72 C35 A82  11760 số (tính số có số đứng đầu) * Khi số đứng vị trí a1 : có C62 cách xếp hai số 2; có C34 cách xếp ba số 3; có cách xếp số vào lại;  Có C62 C34  420 số mà chữ số đứng đầu Vậy số số lập 11760  420  11340  Chọn C Ví dụ 9: Tính tổng tất số tự nhiên gồm năm chữ số khác đôi thành lập từ số 1, 3, 4, 5, 7, A 38666280 B 18666480 C 3260400 D 3732960 Hướng dẫn Từ chữ số ta lập A  720 số có chữ số khác Ta có: Số có dạng abcd1: lấy bốn năm số lại xếp vào năm vị trí, có A 54 số Tương tự: Số có dạng abcd3 : có A 54 số; Số có dạng abcd4 : có A 54 số;  a  0; a, b, c, d  0;1; 2; ;9 Số có dạng abcd5 : A 54 số; Số có dạng abcd7 : A 54 số; Số có dạng abcd9 : A 54 số;  Tổng chữ số hàng đơn vị 720 số : 1       A 54  3480 Tương tự ta có: Tổng chữ số hàng chục 720 số là: 1       A 54  3480 Tổng chữ số hàng trăm 720 số là: 1       A 54  3480 Tổng chữ số hàng ngàn 720 số là: 1       A 54  3480 Tổng chữ số hàng chục ngàn 720 số là: 1       A 54  3480 Vậy tổng 720 số lập S  3480 1  10  102  103  104   38666280  Chọn A Bài tập tự luyện Câu Có số tự nhiên có ba chữ số? A 899 B 900 C 901 D 999 Câu Có số có bốn chữ số khác tạo thành từ chữ số 1, 2, 3, 4, 5? A P4 B P5 C A 54 D C54 Trang Câu Cho chữ số 0, 1, 4, 6, 8, Số số tự nhiên chẵn có bốn chữ số khác lập thành từ chữ số là: A 240 B 204 C 402 D 420 Câu Có số tự nhiên gồm năm chữ số khác đơi một, thiết phải có mặt hai chữ số 3? A 6216 B 2688 C 6598 D 8123 Đáp án: 1–B 2–C 3–B 4–A Dạng 3: Sắp xếp vị trí, phân cơng cơng việc Ví dụ minh họa Ví dụ 1: Trong tuần bạn A dự định ngày thăm người bạn 12 người bạn Hỏi bạn A lập kế hoạch thăm bạn (thăm bạn khơng q lần)? A 3991680 B 12! C 35831808 D 7! Hướng dẫn Chọn người 12 người xếp vào ngày để lên kế hoạch, có A12  3991680 cách  Chọn A Ví dụ 2: Một hộp có 14 đỏ, 12 vàng, xanh Số cách lấy cho lấy có đủ ba màu là: A 24912 B 24192 C 29412 D 29124 Hướng dẫn Trường hợp 1: Lấy đỏ, vàng xanh có: C114 C112 C92  6048 cách Trường hợp 2: Lấy đỏ, vàng xanh có: C114 C12 C19  8316 cách Trường hợp 3: Lấy đỏ, vàng xanh có: C14 C112 C19  9828 cách Vậy số cách lấy đủ ba màu là: 6048  8316  9828  24192 cách  Chọn B Ví dụ 3: Một đội niên tình nguyện có 15 người gồm 12 nam nữ Hỏi có cách phân cơng đội niên tình nguyện ba tỉnh miền núi cho tỉnh có nam nữ? A 2037131 B 3912363 C 207900 D 213930 Hướng dẫn Chọn nam 12 nam nữ nữ phân cơng tỉnh 1, có C12 C13 cách Chọn nam nam nữ nữ lại phân cơng tỉnh 2, có C84 C12 cách Chọn nam nam nữ nữ lại phân cơng tỉnh 3, có C44 C11 cách Vậy số cách phân cơng thỏa mãn u cầu tốn là: C12 C13 C84 C12 C44 C11  207900  Chọn C Trang Ví dụ 4: Có hộp đựng viên bi xanh, viên bi đỏ viên bi vàng, có cách lấy viên bi có đủ ba màu? A 4560 B 1240 C 4939 D 5005 Hướng dẫn 15 Lấy viên bi 15 viên bi bất kỳ, có C cách Ta tìm số cách lấy viên bi khơng có đủ màu: Trường hợp 1: lấy viên bi có màu xanh đỏ, có C11 cách Trường hợp 2: lấy viên bi có màu xanh vàng, có C99 cách Trường hợp 3: lấy viên bi có màu đỏ vàng, có C10 cách 9   C11  C99  C10 Vậy có : C15   4939 cách  Chọn C Bài tập tự luyện Câu Một liên đồn bóng đá có 10 đội, đội phải đá trận với đội khác, trận sân nhà trận sân khách, số trận đấu xếp là: A 180 B 160 C 90 D 45 Câu Có hai hộp đựng bóng Hộp thứ chứa màu đỏ màu xanh Hộp thứ hai chứa màu đỏ màu xanh Hỏi có cách lấy bóng mà có hai màu? A 364 B 349 C 934 D 943 Câu Có 12 học sinh giỏi gồm học sinh khối 12, học sinh khối 11 học sinh khối 10 Hỏi có cách chọn học sinh số học sinh giỏi cho khối có học sinh? A 85 B 58 C 508 D 805 Câu Đội học sinh giỏi cấp trường mơn Tốn trường THPT Thanh Oai B theo khối sau: khối 10 có học sinh, khối 11 có học sinh khối 12 có học sinh Nhà trường cần chọn đội tuyển gồm 10 học sinh tham gia thi học sinh giỏi Tính số cách lập đội tuyển cho có học sinh ba khối có nhiều học sinh khối 10 A 50 B 500 C 502 D 501 Đáp án: 1–A 2–A 3–D 4–B Dạng 4: Bài toán xếp vị trí theo hàng Ví dụ minh họa Ví dụ 1: Có cách xếp khác cho người ngồi vào bàn dài? A 120 B C 20 D 25 Hướng dẫn Trang Số cách xếp khác cho người ngồi vào bàn dài hoán vị phần tử nên có 5!  120 cách  Chọn A Ví dụ 2: Một nhóm học sinh có bạn nam bạn nữ Hỏi có cách xếp 10 bạn hàng ngang biết hai vị trí đầu cuối hàng bạn nam khơng có bạn nữ ngồi cạnh nhau? A 344000 B 100800 C 604800 D 120120 Hướng dẫn Bước 1: Xếp bạn nam thành hàng ngang, có 7! cách xếp Bước 2: Xem bạn nam vách ngăn, bạn nam có sáu vị trí để xếp bạn nữ Chọn vị trí sáu vị trí để xếp bạn nữ có A 36 cách Theo quy tắc nhân có: 7!.A 36  604800 cách  Chọn C Ví dụ 3: Một bàn dài có hai dãy ghế đối diện nhau, dãy có ghế Người ta muốn xếp chỗ ngồi cho học sinh trường A học sinh trường B vào bàn nói Hỏi có cách xếp biết học sinh ngồi cạnh đối diện khác trường với nhau? A 1036800 B 1202540 C 136000 D 518400 Hướng dẫn Giai đoạn 1: Xếp chỗ ngồi cho hai nhóm học sinh, có cách xếp: A B A B A B B A B A B A B A B A B A A B A B A B Giai đoạn 2: Trong nhóm học sinh trường A, có 6! cách xếp học sinh vào chỗ Tương tự, có 6! cách xếp học sinh trường B vào chỗ Vậy có 2.6!.6!  1036800 cách  Chọn A Ví dụ 4: Có 10 học sinh Hỏi có cách để xếp 10 học sinh ngồi vào bàn tròn 10 ghế? A 10! B 9! C 2.10! D 2.9! Hướng dẫn Với bàn tròn, người ta khơng phân biệt vị trí chỗ ngồi, tức kết có đổi chỗ vòng tròn khơng coi khác Do để làm tốn “bàn tròn”, ta thường cố định người ngồi vị trí Lấy cố định người vào bàn tròn, người để xếp vào vị trí lại Do ta có 9! cách xếp 10 người vào bàn tròn  Chọn B Ví dụ 5: Có bạn nữ bạn nam ngồi quanh bàn tròn có chỗ ngồi Số cách xếp bạn nam nữ ngồi xen kẽ là: A 142 B 143 C 144 D 145 Hướng dẫn Trang Cố định bạn nam vào vị trí đầu tiên, xếp bạn nam ngồi vào bàn tròn có 3! cách Giữa bạn nam tạo khoảng trống, xếp bạn nữ vào chỗ trống có 4! cách Do có 3!.4!  144 cách xếp  Chọn C Bài tập tự luyện Câu Số cách xếp nam sinh nữ sinh vào dãy ghế hàng ngang có 10 chỗ ngồi là: A 6!4! B 10! C 6! 4! D 6!  4! Câu Sắp xếp năm bạn học sinh An, Bình, Chi, Dũng, Lệ vào ghế dài có chỗ ngồi, số cách xếp cho bạn Chi ln ngồi là: A 24 B 120 C 60 D 16 Câu Hỏi có cách xếp cặp vợ chồng ngồi xung quanh bàn tròn, cho nam nữ ngồi xen kẽ nhau? A 3600 B 720 C 68400 D 86400 Câu Có nam, nữ xếp thành hàng ngang Hỏi có cách xếp cho hai vị trí đầu cuối nam khơng có nữ đứng cạnh nhau? A 118540800 B 152409600 C 12700800 D 3628800 Đáp án: 1–B 2–A 3–D 4–D Dạng 5: Các tốn liên quan đến hình học Ví dụ minh họa Ví dụ 1: Cho hai đường thẳng song song d1,d2 Trên đường thẳng d1 lấy 10 điểm phân biệt, d2 lấy 15 điểm phân biệt Hỏi có tam giác mà ba đỉnh chọn từ 25 điểm vừa nói trên? A 675 B 1050 C 1725 D 708750 Hướng dẫn Trường hợp 1: Tam giác gồm hai đỉnh thuộc d1 đỉnh thuộc d2 Số cách chọn hai điểm 10 điểm thuộc d1 là: C10 Số cách chọn điểm 15 điểm thuộc d2 là: C115 Theo quy tắc nhân, có: C10 C115 tam giác Trường hợp 2: Tam giác gồm đỉnh thuộc d1 hai đỉnh thuộc d2 Số cách chọn điểm 10 điểm thuộc d1 là: C110 Số cách chọn hai điểm 15 điểm thuộc d2 là: C15 Theo quy tắc nhân, có C110 C15 tam giác 2 Vậy có C10 C115  C110 C15  1725 tam giác thỏa yêu cầu tốn  Chọn C Trang 10 Ví dụ 2: Trong mặt phẳng, cho điểm phân biệt cho khơng có ba điểm thẳng hàng Hỏi lập tam giác mà đỉnh thuộc tập hợp điểm cho? A 15 B 20 C 60 D 120 Hướng dẫn Cứ điểm phân biệt không thẳng hàng tạo thành tam giác Lấy điểm điểm có C36  20 tam giác tạo thành  Chọn B Ví dụ 3: Cho đa giác 12 cạnh Hỏi đa giác có đường chéo? A 121 B 66 C 132 D 54 Hướng dẫn Cứ đỉnh đa giác tạo thành đoạn thẳng Khi có C12  66 đoạn thẳng (bao gồm cạnh đa giác đường chéo) Vậy số đường chéo là: 66  12  54  Chọn D Ví dụ 4: Một đa giác có số đường chéo gấp đơi số cạnh Hỏi đa giác có cạnh? A B C D Hướng dẫn Đa giác có n cạnh  n  , n  3 Lấy cạnh tạo thành đoạn thẳng, có C2n đoạn thẳng (bao gồm cạnh đa giác đường chéo) Số đường chéo đa giác là: C2n  n Ta có: C2n  n  2n  n  n!  3n  n  n  1  6n    n   n  !.2! n   Chọn C Bài tập tự luyện Câu 12 đường thẳng có nhiều giao điểm? A 12 B 66 C 132 D 144 Câu Trong mặt phẳng cho 2010 điểm phân biệt cho ba điểm khơng thẳng hàng Hỏi có véctơ (khác véctơ khơng) có điểm đầu điểm cuối thuộc 2010 điểm cho? A 4039137 B 4038090 C 4167114 D 167541284 Câu Số tam giác xác định đỉnh đa giác 10 cạnh là: A 35 B 120 C 240 D 720 Câu Cho hai đường thẳng d1 d2 song song với Trên d1 có 10 điểm phân biệt, d2 có n điểm phân biệt  n   Biết có 2800 tam giác có đỉnh điểm nói Tìm n A 20 B 21 C 30 D 32 Đáp án: Trang 11 1–B 2–B 3–B 4–A Dạng 6: Phương trình, bất phương trình hốn vị, chỉnh hợp, tổ hợp Ví dụ minh họa Ví dụ 1: Có số tự nhiên x thỏa mãn: 3A 2x  A 22x  42  A B C D Hướng dẫn Điều kiện: x  x   Ta có 3A 2x  A 22x  42    2x !  42  x!   x  !  2x  !  x  7  lo¹i    x  1 x   2x  1 2x  42   x  x  42     x   tháa m·n  Do có số tự nhiên x thỏa mãn yêu cầu đề  Chọn B Ví dụ 2: Tìm tổng giá trị n   thỏa mãn C1n 1  3C2n   C3n 1 A 12 B 10 D 10 C 16 Hướng dẫn Điều kiện: n  n   Ta có C1n 1  3C2n   C3n 1   n    n  1!   n  !   n  1! 1!.n! 2!.n! 3!  n   !  n  1  n     n  1 n  n  1    n     n  1 n 6  n  2  lo¹i    9n  18  n  n  n  10n  24     n  12  tháa m·n  Do tổng giá trị n thỏa mãn đẳng thức 12  Chọn A Ví dụ 3: Giải bất phương trình A n  Cnn 13  A n 1 14P3 B  n  C n  11 D  n  Hướng dẫn Điều kiện: n  3, n   n 3 n 1 n 1 C A  n  1!  n  3!2!    n  1!    1   14P3  n  1! 14.3!  n  1! 84 2n  n  1 84  n  3 ! Trang 12 n   n  n  1  42  n  n  42     n  7 Kết hợp với điều kiện ta n   Chọn A Bài tập tự luyện Câu Tính tổng S tất giá trị x thỏa mãn P2 x  P3 x  A S  4 B S  1 C S  D S  Câu Cho đẳng thức A10 x  A x  9A x Mệnh đề sau đúng? A x số phương B x số nguyên tố C x số chẵn D x số chia hết cho Câu Có số tự nhiên n thỏa mãn A  5A   n  15  ? n A B n C x 2 14 Câu Tính tích P tất giá trị x thỏa mãn C  C x 14 A P  B P  32 D x 1 14  2C C P  32 D P  12 Đáp án: 1–D 2–B 3–B 4–B PHẦN 3: BÀI TẬP TỔNG HỢP Câu Một đội văn nghệ gồm 20 người, có 10 nam 10 nữ Hỏi có cách chọn người, cho có nam người đó? A 1203 B 3600 C 5400 D 4768 Câu Một thi trắc nghiệm khách quan gồm 10 câu, câu có phương án trả lời Hỏi thi có phương án trả lời? A 400 B 410 C 100 D 104 Câu Có hai dãy ghế, mỗl dãy ghế xếp nam, nữ vào hai dãy ghế Có cách nam nữ xếp tùy ý? A 340980 B 3628800 C 120 D 210 Câu Có số tự nhiên có chữ số đơi khác nhau? A 27216 B 72216 C 22716 D 62721 Câu Một đội văn nghệ gồm 20 người, có 10 nam, 10 nữ Hỏi có cách chọn người, cho có hai nam, nữ? A 10800 B 7500 C 12900 D 47010 Câu Xếp học sinh A, B, C, D, E, F vào ghế dài Có cách xếp A F ngồi hai đầu ghế? A 36 B 24 C 96 D 48 Trang 13 Câu Một lớp học có 40 học sinh gồm 25 nam 15 nữ Giáo viên chủ nhiệm muốn chọn học sinh lập thành đoàn đại biểu để tham gia tổ chức lễ khai giảng Hỏi có cách chọn học sinh, có khơng q nữ? A 378000 B 567750 C 620880 D 567750 Câu Cho số 1,2,4,5,7 Có cách tạo số chẵn gồm chữ số khác từ năm chữ số cho? A 120 B 256 C 24 D 36 Câu Từ chữ số 0, 2, 3, 4, 5, 6, lập số có bốn chữ số khác chia hết cho 25? A 36 B 60 C 52 D 38 Câu 10 Có thể lập số tự nhiên có chữ số khác từ tập hợp A  1; 2;3; 4;5;6 , tổng chữ số đầu nhỏ tổng chữ số sau đơn vị? A 104 B 106 C 108 D 112 Câu 11 Nếu đa giác có 44 đường chéo, số cạnh đa giác là: A 11 B 10 C Câu 12 Tính tổng S tất giá trị n thỏa mãn A S  B S  11 D 1   Cn Cn 1 6Cn  C S  12 D S  15 x  C  y  x  D  y  2A xy  5C xy  90 Câu 13 Giải hệ phương trình  y y 5A x  2C x  80 x  A  y   x  20 B   y  10 Đáp án: 1–C 2–B 3–B 11 – A 12 – B 13 – A 4–A 5–C 6–D 7–C 8–C 9–C 10 – C Trang 14 CHƯƠNG 4: TỔ HỢP XÁC SUẤT CHUYÊN ĐỀ 2: NHỊ THỨC NIU-TƠN PHẦN 1: LÝ THUYẾT TRỌNG TÂM Công thức nhị thức Niu-tơn a  b n n   Ckn a n  k b k  C0n a n  C1n a n 1b   Ckn a n  k b k   Cnn b n k 0 Tính chất Số số hạng khai triển n  Tổng số mũ a b số hạng n Số mũ a giảm dần từ n đến 0, số mũ b tăng dần từ đến n Số hạng tổng quát thứ k  có dạng: Tk 1  Ckn a n  k b k , 1  k  n  Các hệ số cặp số hạng cách số hạng đầu cuối nhau: Ckn  Cnn  k Một số công thức khai triển hay sử dụng n 2n  1  1   Ckn  Cnn  Cnn 1   C0n n k 0 n  1  1    1 Ckn  C0n  C1n    1 Cnn n k n k 0 1  x  n n   Ckn x n  k  C0n x n  C1n x n 1   Cnn x k 0 1  x  n n    1 Ckn x k  C0n x  C1n x1    1 Cnn x n n n k 0  x  1 n n    1 Ckn x n  k  C0n x n  C1n x n 1    1 Cnn x k n k 0 Tam giác Pascal Trong công thức nhị thức Niu-tơn, cho n  0;1; xếp hệ số thành dòng, ta nhận tam giác Pa-xcan n 1 C10 C11 n2 C02 C12 n 3 C30 C13 n4 C04 n 5 n6 C22 C32 C33 C14  C24 C34 C44 C50 C15 C52 C35 C54 C55 C06 C16 C62 C36 C64 C56 + C66 Trang PHẦN 2: CÁC DẠNG BÀI TẬP Dạng 1: Khai triển nhị thức Niu-tơn Phương pháp giải n Khai triển nhị thức Niu-tơn:  a  b    Ckn a n  k b k  C0n a n  C1n a n 1b   Ckn a n  k b k   Cnn b n n k 0 • Số số hạng khai triển n  • Số hạng tổng quát thứ k  có dạng: Tk 1  Ckn a n  k b k • Nếu n chẵn, số hạng khai triển  a  b  Ckn a n  k b k với k  n • Nếu n lẻ, hai số hạng khai triển k1  a  b n n Ckn1 a n  k1 b k1 , Ckn a n  k b k với n 1 n 1 , k2  2 Ví dụ minh họa Ví dụ 1: Biểu thức khai triển biểu thức  x  2y  ? A x  10x y  40x y  80x y3  80xy  32y5 B x  10x y  40x y  90x y3  80xy  12y5  C x  10x y  40x y  90x y3  80xy  10y5 D x  10x y  40x y  90x y3  80xy  10y5 Hướng dẫn Cách 1:  x  2y  5   C5k x 5 k  2y   C50 x  C15 x  2y   C52 x  2y   C35 x  2y   C54 x  2y   C55  2y  k k 0  x  10x y  40x y  80x y3  80xy  32y5 Cách 2: Sử dụng máy tính CASIO Fx 570 VNPLUS Bước 1: Khai triển  x  2y    C x k 0 k 5 k  2y  k   C5k 2k.x 5 k y k k 0 Bước 2: Sử dụng MODE Nhập f  X   5CX  25 X Start?  End?  Step? Bước 3: Nhìn vào cột F(X), cột F(X) hệ số khai triển nhị thức Niu-tơn x F(X) 1 10 Trang 2 40 80 80 32  Chọn A Ví dụ 2: Trong khai triển nhị thức  x  3 A 17 n 6 ,  n    có tất 18 số hạng Tìm n B 11 C 10 D 12 Hướng dẫn Khai triển  x  3 n 6 ,  n    có tất n    n  số hạng Do n   18  n  11  Chọn B Ví dụ 3: Trong khai triển  0,  0,8  , số hạng thứ ba là: A 0,0064 B 0,4096 C 0,0512 D 0,2048 Hướng dẫn Khai triển  0,  0,8    C5k  0,  5 k  0,8  k k 0 Số hạng tổng quát khai triển C5k  0,  5 k  0,8  k Vậy số hạng thứ ba ứng với k  C52  0,   0,8   0, 0512  Chọn C Ví dụ 4: Trong khai triển,  3x  y  , hệ số số hạng bao nhiêu? 10 B 34.C10 A 34.C10 D 35.C10 C 35.C10 Hướng dẫn 10 k Khai triển  3x  y    C10  3x  10 10  k k 0  y k 10 k   C10 310 k  1 x 303k y k k k 0 k 310 k  1 Hệ số số hạng tổng quát khai triển C10 Vì n  10 chẵn nên số hạng ứng với k  k n 10   2 5 35  1  35.C10 Vậy hệ số số hạng khai triển là: C10  Chọn D Ví dụ 5: Tìm số hạng số nguyên khai triển A 1203 B 3600   3 ? C 4768 D 4544 Hướng dẫn Trang Cách 1: Khai triển  3   C  3   9 9 k k k 0 k  1   C  32  k 0   9 k k k 9 k k  13  k 2  C 23    k 0     k   Để có số hạng chứa số nguyên k  0  k   k  Vì  nên k  0;3;6;9 0  k  k 9–k Loại Loại Thỏa mãn   k    k  Thỏa mãn Vậy số hạng nguyên khai triển C39 33  C99 23  4544 Cách 2: Sử dụng máy tính CASIO Fx 570 VNPLUS Bước 1: Khai triển  3    C9k  3    2  Start?  End?  Step? k 0 9 k k Bước 2: Sử dụng MODE Nhập f  X   9CX   3 9 X  X Bước 3: Nhìn vào cột F(X) hệ số khai triển nhị thức Niu-tơn, nhìn xem có hệ số hệ số nguyên Tại x  ta thấy f  X   4536 Tại x  ta thấy f  X   Vậy số hạng nguyên khai triển 4536   4544  Chọn D Bài tập tự luyện Câu Số hạng tử khai triển  2x  1 là: 15 A 14 B 15 C 16 D 17 1  Câu Trong khai triển  a   , số hạng thứ năm là: b  A 35.a b 4 B 35.a b 4 C 35.a b 5 D 35.a b Câu Trong khai triển 1  30  với số mũ tăng dần, hệ số số hạng đứng là: 20 A 39 C920 B 312 C12 20 Câu Tìm số hạng số nguyên khai triển C 311 C11 20   D 310 C10 20  15 Trang A 1020 B 7500 C 15552 D 4700 Đáp án: 1–C 2–A 3–D 4–C Dạng 2: Xác định hệ số khai triển nhị thức Niu-tơn Phương pháp giải n  ax p  bx q    Ckn  ax p  n n k k 0 n  bx q    Ckn a n k bk x nppk qk k k 0 Số hạng chứa x m ứng với giá trị k thỏa mãn: np  pk  qk  m Từ k  m  np  p  q pq Vậy hệ số số hạng chứa x m là: Ckn a n  k b k với giá trị k tìm Nếu k khơng ngun k  n khai triển khơng chứa x m , hệ số phải tìm Chú ý: Để xác định hệ số lớn khai triển nhị thức Niu-tơn Ta làm sau: * Tính hệ số a k theo k n; * Giải bất phương trình a k 1  a k với ẩn số k; * Hệ số lớn phải tìm ứng với số tự nhiên k lớn thỏa mãn bất phương trình Các cơng thức mũ thường sử dụng: a a  a m n a  m n mn a m.n m am  a mn  a   an  a n m am a   b  0   bm b  ab  m  n am  a n n  a n  a  0 n a  a  0 m  a m bm a n b  n ab  a, b   Ví dụ minh họa Ví dụ 1: Tìm hệ số x101 y99 khai triển  2x  3y  101 A C98  3 200 99 101 B C99  3 200 200 101 C C100  3 200 99 99 101 D C102  3 200 99 Hướng dẫn Ta có  2x  3y  200   2x   3y   200 200   Ck200  2x  k 0 200  k 200  3y    Ck200 2200k  3 x 200k yk k k k 0 200  k  101 Để có hệ số x101 y99   k  99 (thỏa mãn) k  99 101 Vậy hệ số x101 y99 C99  3 200 99  Chọn B Trang 15 1  Ví dụ 2: Tìm số hạng khơng chứa x khai triển  x   x  A C15 B C115 C C15 D C11 15 Hướng dẫn 15 k 15 15 15 1  k 15  k   k 30  2k k k 30 3k Ta có  x     C15 x  C x x  C15 x       15 x  x k 0 k 0 k 0 Để có số hạng khơng chứa x 30  3k   k  10 Vậy hệ số số hạng không chứa x khai triển C10 15  C15  Chọn C   Ví dụ 3: Trong khai triển  x   x  A 60  x   , hệ số x3 là: B 80 C 160 D 240 Hướng dẫn k k 1 3k 6 6   6 k    k k k 6 k  k k 2 Khai triển  x   C x  C x x  C x      6     x  k 0   x k 0 k    Số hạng tổng quát khai triển C6k 2k.x Hệ số x nên  6 3k 3k   k  Khi hệ số x là: C64 24  240  Chọn D 1  Ví dụ 4: Trong khai triển  x   x  hạng không chứa x A 252 n  x  0 , hệ số số hạng thứ ba lớn hệ số số hạng hai 35 Tính số B 720 C 124 D 210 Hướng dẫn n k n n 1  1 Khai triển  x     Ckn x n  k     Ckn x n  2k x  x k 0 k 0 Số hạng tổng quát khai triển Ckn x n  2k Hệ số số hạng thứ hai C1n Hệ số số hạng thứ ba C2n  n  10 (chän) Từ giả thiết suy C2n  C1n  35  n  3n  70     n  7 (lo¹i) k 10  2k Với n  10, số hạng C10 không phụ thuộc x 10  2k   k  x Vậy số hạng C10  252  Chọn A Trang Ví dụ 5: Cho đa thức P  x   1  x   1  x   1  x    1  x   a10 x10  a x   a Tính hệ số 10 a8 A 60 B 16 C 42 D 55 Hướng dẫn n Khai triển 1  x    Ckn x k n k 0 Vì a hệ số số hạng chứa x8 , hệ số a khai triển 1  x  C8n n Hệ số x8 1  x  C88 Hệ số x8 1  x  C89 Hệ số x8 1  x  C10 10 Vậy a  C88  C89  C10  55  Chọn D Ví dụ 6: Tìm hệ số x khai triển: P  x 1  3x   x 1  2x  A 1200 B 1365 10 C 1480 D 405 Hướng dẫn 10 m Ta có P  x 1  3x   x 1  2x   x  C5k  3x   x  C10  2x  10 k k 0 10 m m 0 m   C5k  3 x k 1   C10  2  x m k k 0 m m 0 k   k  Để có hệ số x   m   m  3 Vậy hệ số x a  C54 34  C10  2   1365  Chọn B Ví dụ 7: Cho đa thức P  x   1  2x  Tìm hệ số lớn khai triển nhị thức Niu-tơn 12 A 126720 B 421785 C 112640 D 101376 Hướng dẫn 12 12 k Cách 1: P  x   1  2x    C12  2x    C12k 2k x k  a  a1x  a x   a12 x12 12 k k 0 k 0 k Hệ số số hạng tổng quát a k  C12 k k k 1 k 1 Ta có a k  a k 1  C12 2k  C12  12! 12! 23    k k!12  k  !  k  1 !11  k  ! 12  k k  Tức với k  , ta có a k  a k 1 hay a  a  a10  a11  a12 Trang k k 1 k 1 Tương tự, ta có a k  a k 1  C12 2k  C12  12! 12! 23    k k!12  k  !  k  1 !11  k  ! 12  k k  Tức với k  7, ta có a k  a k 1 hay a  a1  a  a  a  a  a  a  a Vậy hệ số lớn khai triển là: a  C12 28  126720 Cách 2: Sử dụng máy tính CASIO Fx 570 VNPLUS 12 12 k Bước 1: Khai triển 1  2x    C12  2x    C12k 2k x k 12 k k 0 k 0 Bước 2: Sử dụng MODE Nhập f  X   12CX  2X  Start?  End? 12  Step? Bước 3: Nhìn vào cột F(X), cột F(X) hệ số khai triển nhị thức Niu-tơn, nhìn xem có hệ số hệ số lớn Tại x  ta thấy f  X   126720 hệ số lớn khai triển  Chọn A n 2  Ví dụ 8: Tìm hệ số khơng chứa x khai triển  x   , biết Cnn 1  Cnn   78 với x  x  A 112643 C 112640 B 112640 D 112643 Hướng dẫn Ta có: Cnn 1  Cnn   78  n n! n!   78  n  1!1!  n  !2! n  n  1  78  n  n  156   n  12 12 12 2 k  k Khi đó:  x     C12  2  x 364k x  k 0 Số hạng không chứa x ứng với giá trị k thỏa mãn 36  4k   k  9  112640 Vậy số hạng không chứa x là:  2  C12  Chọn C Ví dụ 9: Tìm hệ số x8 khai triển 1  x  x  A 190 B 230 C 238 D 70 Hướng dẫn 8 k k 0 m 0 Ta có 1  x  x    C8k  x  x    C8k  Cmk  x  k 0 k k k m  x  m    C8k Ckm  1 x 2k  m m k 0 m 0 Trang 2k  m  m  m   Để có hệ số x 0  m  k     k  k    m, k    Vậy hệ số x8 a  C84 C04  C83C32  238  Chọn C Bài tập tự luyện Câu Trong khai triển  2x  5y  , hệ số số hạng chứa x y3 là: A 224000 B 40000 C 8960 D 4000   Câu Trong khai triển  x   , số hạng không chứa x là: x   A 4308 B 86016 C 84 D 43008 2  Câu Tìm hệ số số hạng chứa x khai triển nhị thức  2x   ; x  x  B 4480 A 1090 C 8960 D 4480 Câu Xét khai triển  3x    a  a1x  a x x   a x Tìm max  a1 , a , , a  A 314928 B 489888 C 326592 D 1134008 Đáp án: 1–A 2–D 3–D 4–B Dạng 3: Sử dụng nhị thức Niu-tơn chứng minh đẳng thức tổ hợp Phương pháp giải Ta thường sử dụng kết sau với giá trị thích hợp x: 1  x  n  C0n  C1n x  C2n x  Cnn x n 1  x  n  C0n x  C1n x1    1 Cnn x n  x  1 n  C0n x n  C1n x n 1    1 Cnn x n n Ví dụ minh họa Ví dụ 1: Tính giá trị biểu thức S1  C1n  C2n   Cnn C 2n  B 2n A D 2n  Hướng dẫn Ta có 1  x   C0n  C1n x  C2n x   Cnn x n n  * Chọn x  thay vào (*) ta được: 1  1  C0n  C1n  C2n   Cnn n Trang hay 2n  C0n  C1n  C2n   Cnn Vậy S1  C1n  C2n   Cnn  2n  C0n  2n   Chọn C Ví dụ 2: Tính giá trị biểu thức S  C02019  2C12019  22 C22019   22019 C2019 2019 B 32019  A 32019 C 32020 D Hướng dẫn Ta có 1  x  2019  * 2019  C02019  C12019 x  C22019 x   C2019 2019 x Chọn x  2, thay vào (*) ta được: 1   2019  C02019  2C12019  22 C22019   22019 C2019 2019 hay 32019  C02019  2C12019  22 C2019   22019 C2019 2019  S Vậy S  32019  Chọn A Ví dụ 3: Tìm số nguyên dương n cho: C0n  2C1n  4C2n   2n Cnn  243 A B C D Hướng dẫn Ta có: 1  x   C0n  C1n x  Cn2 x   Cnn x n n  * Thay x  vào hai vế (*) ta được: C0n  2C1n  4C2n   2n Cnn  3n Theo đề có 3n  243  n   Chọn B Ví dụ 4: Tính tổng S2  C02011  22 C22011   22010 C2010 2011 A 32011  B 3211  C 32011  12 D 32011  Hướng dẫn Ta có 1  x  2011 2011 2011  C02011  xC12011  x C22011   x 2010 C2010 C2011 2011  x  * Thay x  vào hai vế (*) ta được: 2011 2011 32011  C02011  2.C12011  22 C22011   22010 C2010 C2011 2011  (1) Thay x  2 vào hai vế (*) ta được: 2011 2011 1  C02011  2.C12011  22 C22011   22010 C2010 C2011 2011  (2) 2010   22010 C2011 Lấy (1) + (2) ta có:  C02011  22 C2011   32011  2010  Suy ra: S2  C02011  22 C22011   22010 C2011 32011   Chọn D Trang 10 n Ví dụ 5: Tìm hệ số số hạng chứa x 26   khai triển nhị thức Niu tơn   x  , biết x  C12n 1  C22n 1   Cn2n 1  220  A 612 B 230 C 210 D 310 Hướng dẫn Từ giả thiết suy ra: C 2n 1 C 2n 1 C 2n 1   C n 2n 1  220  1 k Vì Ck2n 1  C2n 2n 1 , k,  k  2n  nên: n n 1 n 2 2n 1 C02n 1  C12n 1  C2n 1   C 2n 1  C 2n 1  C 2n 1   C 2n 1 Do đó: C02n 1  C12n 1  C22n 1   Cn2n 1  Hay: 220  1 C2n 1  C12n 1   C2n  2n 1  1 2n 1 21 C2n 1  C12n 1   C2n  2n 1   C 2n 1  C 2n 1   C 2n 1  2 Ta có: 1  x  2n 1 1 2n 1  C02n 1  C12n 1x  C22n 1x   C2n 2n 1 x 1  * 1 Thay x  vào hai vế (*) ta được: C02n 1  C12n 1  C22n 1   C2n 2n 1  1  1 2n 1  22n 1  2 Từ (1), (2) suy ra: 22n 1  221  2n   21  n  10 10 10 10   k 4 10  k k k 11k  40 • Ta có:   x    C10 x x  C10 x      x  k 0 k 0 Số hạng chứa x 26 ứng với giá trị k thỏa mãn 11k  40  26  k  6 Vậy hệ số x 26 C10  210  Chọn C Ví dụ 6: Tính tổng S  C1n  2C2n   nCnn A 2n.2n 1 B n.2n 1 C 2n.2n 1 D n.2n 1 Hướng dẫn Ta có: 1  x   C0n  C1n x  C2n x   Cnn x n n (*) Lấy đạo hàm theo x hai vế (*) ta được: n 1  x  n 1 **  C1n  2C2n x  3C3n x   nCnn x n 1 Thay x  vào hai vế (**) ta được: n 1  1 n 1  C1n  2C2n   nCnn Hay: n.2n 1  C1n  2C2n   nCnn  S  Chọn D Bài tập tự luyện Câu Giá trị tổng S  C06  C16   C66 A 100 B 48 C 72 D 64 Trang 11 Câu Tính giá trị biểu thức S  C0n  10C1n  102 C2n   10n Cnn C 11n 1 B 11n  A 11n D Câu Tính giá trị biểu thức S  2n C0n  2n 1 C1n  2n  C2n   Cnn A 3n  B 3n C 2n D 2n  Đáp án: 1–D 2–A 3–B PHẦN 3: BÀI TẬP TỔNG HỢP Câu Trong khai triển  3x  y  , số hạng chứa x y3 A 2835x y3 B 2835x y3 C 945x y3 D 945x y3 C D 12 C 41184x y D 41184x8 y5 10 C C19 11 D C10 19 Câu Tính tổng S  C50  C15   C55 A 64 B 32 Câu Tìm số hạng thứ khai triển  x  2y  13 A 2x y B 4100x8 y Câu Tìm hệ số x khai triển   x  19 11 A C10 19 B C19 29  Câu Trong khai triển x  y A 16x y15  y8  16 , tổng hai số hạng cuối B 16x y15  y C 16xy15  y D 16xy15  y8   Câu Trong khai triển  x   , số hạng không chứa x x   A 4308 B 86016 C 84 D 43008 n 2  Câu Trong khai triển nhị thức  x   , cho biết tổng hệ số hạng khai x  triển 97 Tìm hệ số số hạng có chứa x A 1120 B 600 C 1220 D 70 Câu Tìm hệ số x khai triển biểu thức sau thành đa thức f  x    2x  1   2x  1   2x  1   2x  1 A 1020 B 280 C 896 D 964 Câu Tìm n   cho: C04n   C24n   C44n    C2n 4n   256 A B C 12 D Trang 12 Câu 10 Khai triển 1  3x  30 thành đa thức: a  a1x  a x   a 30 x 30 Tìm hệ số lớn hệ số a ;a1 ;a ; ;a 30 23 23 A C30 23 24 B C30 22 22 C C30 20 29 D C30 Câu 11 Số hạng khai triển  3x  2y  B  3x   2y  A C24 x y C 6C24 x y D 36C24 x y 12   Câu 12 Tìm số hạng không chứa x triển khai  x   x   A C12 12 B C12 C C12 D C12 n   Câu 13 Tìm hệ số số hạng chứa x khai triển nhị thức Niu-tơn   x  , biết rằng: x  Cnn 14  Cnn 3   n  3 (n nguyên dương, x  ) A 424 B 280 C 495 D 322   Câu 14 Tìm hệ số số hạng chứa x khai triển biểu thức   x  x  n  x   Biết số nguyên dương n thỏa mãn: C1n  C2n   Cnn  4095 A 7920 B 1400 C 6590 D 8120 Đáp án: 1–A 2–B 3–D 4–C 11 – B 12 – D 13 – C 14 – A 5–A 6–D 7–A 8–C 9–B 10 – A Trang 13 CHƯƠNG 4: TỔ HỢP XÁC SUẤT CHUYÊN ĐỀ 3: XÁC SUẤT PHẦN 1: LÝ THUYẾT TRỌNG TÂM Biến cố Phép thử không gian mẫu Phép thử ngẫu nhiên (gọi tắt phép thử) thí nghiệm hay hành động mà: • Kết khơng đốn trước • Có thể xác định tập hợp tất kết xảy phép thử Tập hợp kết phép thử T gọi không gian mẫu T kí hiệu  Số phần tử khơng gian mẫu kí hiệu n    hay  Biến cố • Biến cố A liên quan đến phép thử T biến cố mà việc xảy hay không xảy A tùy thuộc vào kết T • Mỗi kết phép thử T làm cho A xảy gọi kết thuận lợi cho A • Tập hợp kết thuận lợi cho A kí hiệu n(A) hay  A Xác suất Giả sử A biến cố liên quan đến phép thử  không gian mẫu phép thử Xác suất biến cố A, kí hiệu P(A), xác định cơng thức: P A  n  A  A  n   Trong đó: •  A hay n(A) số phần tử biến cố A •  hay n    số phần tử khơng gian mẫu Tính chất • P     0, P     • Với biến cố A,  P  A   Quy tắc cộng xác suất Biến cố hợp Cho hai biến cố A B Biến cố “A B xảy ra”, kí hiệu A  B gọi hợp hai biến cố A B Khi đó:  A   B   Biến cố xung khắc Cho hai biến cố A B Hai biến cố A B gọi xung khắc biến cố xảy biến cố khơng xảy Khi đó:  A   B   Quy tắc cộng xác suất hai biến cố xung khắc: Nếu A B hai biến cố xung khắc xác suất biến cố A  B P  A  B   P  A   P  B  Trang Cho n biến cố A1, A2, , An đôi xung khắc với Khi đó: P  A1  A   A n   P  A1   P  A    P  A n  Biến cố đối Cho A biến cố Khi biến cố “khơng A”, kí hiệu A , gọi biến cố đối A Ta nói A A hai biến cố đối   Khi đó:  A   \  A  P A   P  A  Quy tắc nhân xác suất Giao hai biến cố A B Biến cố “A B xảy ra”, kí hiệu A  B (hay AB), gọi giao hai biến cố A B Hai biến cố độc lập Hai biến cố gọi độc lập với việc xảy hay không xảy biến cố không làm ảnh hưởng xác suất xảy biến cố Nếu hai biến cố A B độc lập với A B , A B, A B độc lập Quy tắc nhân xác suất hai biến cố độc lập Nếu A B hai biến cố độc lập với ta ln có P  AB   P  A  P  B  Cho n biến cố A1, A2, ……, An độc lập với đơi Khi đó: P  A1 , A , , A n   P  A1  P  A  P  A n  PHẦN 2: CÁC DẠNG BÀI TẬP Dạng 1: Biến cố xác suất biến cố Phương pháp giải Bước 1: Tìm số phần tử khơng gian mẫu n    hay  Ví dụ: Gieo hai đồng xu cân đối cách độc lập Tính xác suất để hai đồng xu sấp A B C D Hướng dẫn Không gian mẫu: Gieo hai đồng xu cách cân Bước 2: Gọi tên biến cố A (người ta thường sử đối, độc lập, đồng xu khả sấp (S) dụng chữ in hoa để gọi tên biến cố) ngửa (N), phần tử không gian mẫu Tìm kết thuận lợi biến cố A n(A)hay    S;S ;  S; N  ;  N;S ;  N; N      A dựa vào quy tắc đếm công thức Gọi A biến cố “cả hai đồng xu sấp” hoán vị, tổ hợp, chỉnh hợp, sử dụng phương Các phần tử biến cố A pháp liệt kê A   S;S   A  Xác suất biến cố A là: Trang Bước 3: Tính xác suất biến cố A P A  P A  n  A  A  n   A    Chọn A Ví dụ minh họa Ví dụ 1: Một tổ học sinh có nam nữ Chọn ngẫu nhiên người Tính xác suất cho người chọn nữ A 15 B 15 C 15 D Hướng dẫn 2 Không gian mẫu: Chọn ngẫu nhiên người 10 người, có C10 cách    C10 Gọi A biến cố “2 người chọn nữ” Kết thuận lợi biến cố A: Chọn học sinh nữ có C32 cách   A  C32 Vậy xác suất biến cố A là: P  A   A C2  23   C10 15  Chọn A Ví dụ 2: Một hộp chứa 11 cầu gồm cầu màu xanh cầu màu đỏ Chọn ngẫu nhiên đồng thời cầu từ hộp Xác suất để cầu chọn màu bằng: A 22 B 11 C 11 D 11 Hướng dẫn 2 Không gian mẫu: Chọn ngẫu nhiên từ 11 nên có C11 cách    C11  55 cách Gọi A biến cố “Chọn hai cầu màu” Kết thuận lợi biến cố A: Trường hợp 1: Chọn cầu cầu xanh, có C52 cách Trường hợp 2: Chọn cầu cầu đỏ, có C62 cách Suy  A  C52  C62  25 Vậy xác suất biến cố A P  A    A 25    55 11  Chọn A Ví dụ 3: Từ hộp có 13 bóng đèn, đỏ có bóng hỏng Lấy ngẫu nhiên bóng khỏi hộp Tính xác suất cho có nhiều bóng hỏng A 427 429 B 61 68 C 63 68 D 84 143 Hướng dẫn Trang 5 Khơng gian mẫu: Chọn bòng đèn 13 bóng có C13 cách  n     C13 Gọi A biến cố “Chọn bóng nhiều bóng hỏng” Kết thuận lợi biến cố A là: Trường hợp 1: Chọn bóng hỏng bỏng tốt có C62 C37 cách Trường hợp 2: Chọn bóng hỏng bóng tốt có C16 C74 cách Trường hợp 3: Chọn bóng tốt có C57 cách Số cách thuận lợi cho A là: n  A   C62 C37  C16 C74  C57  756 cách Xác suất biến cố A là: P  A   n   A  756 84   n  C13 143  Chọn D Ví dụ 4: Cho hai đường thẳng song song d1, d2 Trên d1 có điểm phân biệt tơ màu đỏ, d2 có điểm phân biệt tô màu xanh Xét tất tam giác tạo thành nối điểm với Chọn ngẫu nhiên tam giác, xác suất để thu tam giác có hai đỉnh màu đỏ là: A B C D Hướng dẫn Không gian mẫu: Trường hợp 1: Lấy điểm d1, điểm d2 có C62 C14 cách Trường hợp 2: Lấy điểm d1, điểm d2 có C16 C42 cách    C62 C14  C16 C24  96 Gọi A biến cố “tam giác có hai đỉnh màu đỏ” Số phần tử thuận lợi biến cố A lấy điểm d1; điểm d2 có C62 C14 cách   A  C62 C14  60 Xác suất biến cố A là: P  A   A    Chọn D Ví dụ 5: Xếp học sinh nam học sinh nữ vào bàn tròn 10 ghế Tính xác suất khơng có hai học sinh nữ ngồi cạnh A 37 42 B 42 C 1008 D Hướng dẫn Không gian mẫu: Cố định vị trí cho học sinh nam (hoặc nữ), đánh dấu ghế lại từ đến Khơng gian mẫu hốn vị học sinh (còn lại khơng cố định) ghế đánh dấu Suy số phần tử không gian mẫu   9! Gọi A biến cố “khơng có hai học sinh nữ ngồi cạnh nhau” Ta mô tả khả thuận lợi biến cố A sau: Trang Đầu tiên, ta cố định học sinh nam, học sinh nam lại có 5! cách xếp Ta xem học sinh nam vách ngăn vòng tròn, tạo ô trống để ta xếp học sinh nữ vào (mỗi ô trống xếp học sinh nữ) Do có A 64 cách Suy số phần tử biến cố A  A  5!.A 64 Vậy xác suất cần tính P  A    A 5!.A 64    9! 42  Chọn B Ví dụ 6: Một hộp đựng bút màu xanh, bút màu đen, bút màu tím bút màu đỏ Lấy ngẫu nhiên bút Tính xác suất để lấy bút màu A 200 323 B 287 323 C D Hướng dẫn Không gian mẫu: Lấy bút từ 20 bút cho có C420  4845 cách    C420  4845 Gọi A biến cố “lấy hai bút màu” A biến cố “lấy bút khơng có hai màu” Số cách lấy bút khơng có hai màu là: A  C16 C16 C15 C13 Vậy xác suất lấy bút có hai bút màu là:   P A  1 P A  1 C16 C16 C15 C13 287  C420 323  Chọn B Ví dụ 7: Xếp ngẫu nhiên 10 học sinh gồm học sinh lớp 12A, học sinh lớp 12B học sinh lớp 12C thành hàng ngang Xác suất để 10 học sinh khơng có học sinh lớp đứng cạnh bằng: A 11 630 B 126 C 105 D 42 Hướng dẫn Kí hiệu học sinh lớp 12A, 12B, 12C A, B, C Không gian mẫu: Số cách xếp 10 học sinh thành hàng ngang 10! (cách)    10! Gọi X biến cố “trong 10 học sinh khơng có học sinh lớp đứng cạnh nhau” Ta xếp học sinh lớp 12C trước, số cách xếp chia thành trường hợp sau: Trường hợp 1: C  C  C  C  C (quy ước vị trí – vị trí trống), đổi chỗ học sinh cho ta có 5! cách xếp Xếp học sinh lại vào vị trí trống ta có 5! cách xếp Vậy trường hợp có 5!.5! cách xếp Trường hợp 2: C  C  C  C  C , tương tự trường hợp ta có 5!.5! cách Trường hợp 3: C  C  C  C  C , đổi chỗ học sinh cho ta có 5! cách xếp Trang Ta có vị trí trống liền nhau, chọn học sinh lớp 12A học sinh lớp 12B để xếp vào vị trí trống đó, học sinh đổi chỗ cho nên có: C12 C13 2!  2.3.2  12 cách Xếp học sinh lại vào chỗ trống có 3! cách Vậy trường hợp có 5!.12.3! cách Trường hợp 4: C  C  C  C  C ; Trường hợp 5: C  C  C  C  C ; Trường hợp 6: C  C  C  C  C ; Ba trường hợp 4, 5, có số cách xếp giống trường hợp Vậy có tất cả: 5!.5!.2  4.5!.12.3!  63360 cách xếp   A  63360 Vậy xác suất biến cố X P  X    A 63360 11    10! 630  Chọn A Bài tập tự luyện Câu Gieo hai đồng xu cân đối cách độc lập Tính xác suất để có đồng xu ngửa A B C D Câu Một bình đựng viên bi khác màu, có viên màu xanh, viên màu vàng, viên màu đỏ Lấy ngẫu nhiên viên bi Tính xác suất để viên bi xanh A B C 15 D Câu Một lô hàng có 100 sản phẩm, biết có sản phẩm hỏng Người kiểm định lấy ngẫu nhiên từ sản phẩm Tính xác suất biến cố A: “ Người lấy sản phẩm hỏng” A 25 B 229 6402 C 50 D 2688840 Câu Cho hộp đựng 12 viên bi, có viên bi màu đỏ, viên bi màu xanh Lấy ngẫu nhiên lần viên bi Tính xác suất lấy viên bi màu đỏ A 19 220 B 11 C 44 D 21 44 Đáp án: 1–C 2–C 3–B 4–B Dạng 2: Ví dụ minh họa Ví dụ 1: Hai người độc lập ném bóng vào rổ Mỗi người ném vào rổ bóng Biết xác suất ném bóng trúng vào rổ người tương ứng Gọi A biến cố: “Cả hai Trang ném bóng trúng vào rổ” Khi đó, xác suất biến cố A bao nhiêu? A 12 35 B 25 49 C D 35 Hướng dẫn Gọi A biến cố: “Cả hai ném bóng trúng vào rổ.” Gọi X biên cố: “Người thứ ném trúng rổ.” P  X   Goi Y biến cố: “Người thứ hai ném trúng rổ.” P  Y   5 Ta thấy biến cố X, Y biến cố độc lập A  X.Y , theo công thức nhân xác suất: 2 P  A   P  X  P  Y    35  Chọn D Ví dụ 2: Ba người bắn vào bia Xác suất để người thứ nhất, thứ hai, thứ ba bắn trúng đích 0,8; 0,6; 0,5 Tính xác suất để có hai người bắn trúng đích A 0,24 B 0,96 C 0,46 D 0,92 Hướng dẫn Gọi A1 biến cố người thứ bắn trúng đích, ta có: P  A1   0,8   A1 biến cố người thứ bắn trượt đích, ta có: P A1   P  A1    0,8  0, Gọi A2 biến cố người thứ hai bắn trúng đích, ta có: P  A   0,   A biến cố người thứ hai bắn trượt đích, ta có: P A   P  A    0,  0, Gọi A3 biến cố người thứ ba bắn trúng đích, ta có: P  A   0,5   A biến cố người thứ ba bắn trượt đích, ta có: P A   P  A    0,5  0,5       Gọi B biến cố: “Có hai người bắn trúng đích” B  A1A A  A1 A A  A1A A Xác suất để có hai người bắn trúng đích là: P  B   P  A1  P  A  P  A   P  A1  P  A P  A   P  A1 P  A  P  A   0, 46  Chọn C Ví dụ 3: Bài kiểm tra mơn tốn có 20 câu trắc nghiệm khách quan Mỗi câu có lựa chọn có phương án Một học sinh không học nên làm cách lựa chọn ngẫu nhiên phương án trả lời Tính xác suất để học sinh trả lời sai 20 câu A  0, 25  20 B   0, 75  C   0, 25  20 20 D  0, 75  20 Hướng dẫn Gọi A biến cố: “Học sinh trả lời sai 20 câu.” Trong câu, xác suất học sinh trả lời sai là:  0, 75 Trang Vậy xác suất để học sinh trả lời sai 20 câu P  A   0, 7520  Chọn D Ví dụ 4: Hai xạ thủ A B bắn vào bia người phát Xác suất bắn trúng bia xạ thủ A 0,7 Tìm xác suất bắn trúng bia xạ thủ B biết xác suất có người bắn trúng bia 0,94 A 0,25 B 0,45 C 0,8 D 0,12 Hướng dẫn Gọi xác suất bắn trúng bia xạ thủ B P  B   b với  b  Gọi X xác suất hai xạ thủ bắn trật Có X  A  B A , B hai biến cố độc lập nên       P  X   P A  B  P A P B  1  0, 1  b  1 Gọi X biến cố có xạ thủ bắn trúng bia, dễ dàng thấy X X hai biến cố đối nên   P  X    P X   0,94  0, 06 (2) Từ (1) (2) 1  0, 1  b   0, 06  b  0,8  Chọn C Bài tập tự luyện Câu Xác suất bắn trúng mục tiêu vận động viên bắn viên đạn 0,6 Người bắn hai viên đạn cách độc lập Xác suất để viên trúng mục tiêu viên trượt mục tiêu là: A 0,4 B 0,6 C 0,48 D 0,24 Câu Hai xạ thủ bắn người viên đạn vào bia, biết xác suất bắn trúng vòng 10 xạ thủ thứ 0,75 xạ thủ thứ hai 0,85 Tính xác suất để có viên trúng vòng 10 A 0,9625 B 0,325 C 0,6375 D 0,0375 Đáp án: 1–C 2–A PHẦN 3: BÀI TẬP TỔNG HỢP Câu Trong thí nghiệm sau thí nghiệm khơng phải phép thử ngẫu nhiên? A Gieo đồng tiền xem mặt ngửa hay mặt sấp B Gieo đồng tiền xem có đồng tiền lật ngửa C Chọn học sinh lớp xem nam hay nữ D Bỏ viên bi xanh viên bi đỏ hộp, sau lấy viên để đếm xem có tất viên bi Câu Cho phép thử có khơng gian mẫu Q  1, 2,3, 4,5, 6 Các cặp biến cố không đối là: A A  1 B  2;3; 4;5;6 B C 1, 4,5 D  2;3;6 C E  1; 4;6 F  2;3 D   Trang Câu Gieo hai đồng xu cân đối cách độc lập Tính xác suất để có đồng xu sấp A B C D Câu Một hộp chứa 10 cầu đỏ, 20 cầu xanh Chọn ngẫu nhiên cầu hộp Tính xác suất cho cầu chọn màu đỏ A B C D 10 Câu Từ hộp chứa bi xanh, bi đỏ, bi vàng, lấy ngẫu nhiên bi Tính xác suất biến cố hai bi màu xanh A B C D 18 Câu Trong kì thi có 60% thí sinh đỗ Hai bạn A, B dự kì thi Xác suất để có bạn thi đỗ là: A 0,24 B 0,36 C 0,16 D 0,48 Câu Gieo hai đồng xu A B cách độc lập Đồng xu A chế tạo cân đối Đồng xu B chế tạo không cân đối nên xác suất xuất mặt sấp gấp lần xác suất xuất mặt ngửa Tính xác suất để gieo đồng xu lần hai ngửa A B C D Câu Một hộp chứa cầu kích thước khác gồm cầu đỏ, cầu xanh cầu vàng Chọn ngẫu nhiên cầu Tính xác suất để cầu chọn khác màu A 19 765 B 11 17 C 765 D 17 Câu Từ hộp chứa cầu trắng cầu đen, lấy ngẫu nhiên lúc Tính xác suất cho có màu trắng A 19 765 B 105 C 209 210 D 10 21 Câu 10 Có hộp: hộp chứa bi đỏ, bi trắng; hộp chứa bi đỏ, bi trắng Mỗi hộp chọn bi Tính xác suất biến cố bi màu đỏ A 27 B 13 27 C 14 27 D 72 Đáp án: 1–D 2–C 3–B 4–B 5–A 6–D 7–C 8–B 9–C 10 – A Trang CHƯƠNG 5: DÃY SỐ, GIỚI HẠN, ĐẠO HÀM CHUYÊN ĐỀ 1: DÃY SỐ GIỚI HẠN CỦA DÃY SỐ PHẦN 1: LÝ THUYẾT TRỌNG TÂM Phương pháp quy nạp toán học (Phương pháp quy nạp) Phương pháp thường để chứng minh mệnh đề liên quan đến số tự nhiên n  * với n  p mà thử trực tiếp Các bước giải: Bước 1: Kiểm tra mệnh đề với n = p Bước 2: Giả thiết mệnh đề với số tự nhiên n  k  p (giả thiết quy nạp) Chứng minh với n = k + Bước 3: Kết luận mệnh đề với n  p Dãy số Một dãy số thường kí hiệu u  n   u n  u n  u n số hạng tổng quát thứ n dãy số u  n   u1 số hạng đầu Dãy số tăng, dãy số giảm:  Dãy số u  n  tăng u n 1  u n , với n  *  u n 1  u n  , với n  *  u n 1  , u n  , với n  * un  Dãy số u  n  giảm u n 1  u n , với n  *  u n 1  u n  , với n  *  u n 1  , u n  , với n  * un Dãy số bị chặn:  Dãy số u  n  bị chặn M : u n  M , n  *  Dãy số u  n  bị chặn m : u n  m , n  *  Dãy số u  n  bị chặn vừa bị chặn vừa bị chặn dưới, tức là: m, M : m  u n  M ; n  * Chú ý:  Một dãy số có số hạng tổng qt khơng có số hạng tổng quát  Một dãy số tăng giảm không tăng, không giảm  Một dãy số bị chặn bị chặn bị chặn không bị chặn  Dãy số không bị chặn dãy số không bị chặn không bị chặn Giới hạn dãy số Trang Giới hạn hữu hạn lim u n  lim u n  a ; lum v n  lim v n  a Các loại giới hạn: n  n  Giới hạn vô cực lim u n  lim u n   ; lim u n  lim u n   n  n  PHẦN 2: CÁC CÔNG THỨC lim 1  ; lim k  n n lim q n   q  lim n   , n   * k lim c  c (c số) lim q n  q  lim  u n  v n   lim u n  lim v n lim  u n v n   lim u n lim v n lim lim u n  a ; lim v n   lim u n   ; lim v n  a  lim u n  a  ; lim v n  lim  u n v n   , a   lim  u n v n   , a  un  lim v  , a.v n   n  lim u n  , a.v  n   lim un  u n   n  ; lim u n  a u n lim u n  v n lim v n  lim   Định lí kẹp: u n  v n , n ; lim v n   lim u n   a  ; lim u n  a PHẦN 2: CÁC DẠNG BÀI TẬP Dạng 1: Số hạng, công thức tổng quát dãy số Ví dụ minh họa Ví dụ 1: Cho dãy số có số hạng đầu là: 1 A u n  n 1 B u n  1 1 ; ; ; ; ; Số hạng tổng quát dãy số là: 32 33 34 35 C u n  n 1 3n D u n  3n 1 Hướng dẫn Ta thấy năm số hạng đầu có dạng: 1 1 ; ; ; ; ; 32 33 34 35 Do số hạng tổng quát dãy số là: u n  3n  Chọn C u  Ví dụ 2: Tìm cơng thức số hạng tổng quát dãy số u n Biết  u n 1  3u n A u n  3n 1 B u n   3n C u n  3n D u n  3n Hướng dẫn u  Cách 1: Ta có  nên suy ra: u  3u1  3.3   32 u n 1  3u n Trang u  3u  3.9  27  33 u  3u  3.27  81  34 u  3u  3.81  243  35 Ta dự đốn số hạng tổng qt u n có dạng: u n  3n ; n  (1) Ta chứng minh (1) phương pháp quy nạp Với n = 1, ta có u1  31  (đúng) Vậy (1) với n = Giả sử (1) với n = k, tức ta có u k  3k Ta cần chứng minh (1) với n = k + 1, tức ta phải chứng minh u k 1  3k 1 Thật ta có u k 1  3.u k  3.3k  3k 1 Vậy (1) với n = k + Kết luận (1) với số nguyên dương n Cách 2: Sử dụng máy tính CASIO fx570VN PLUS Ta bấm máy tính, tìm vài số hạng đầu dãy số 3; 9; 27; 81; 243; … Thử đáp án: (Tìm vài số hạng đầu dãy số đáp án) Đáp án A: Các số hạng đầu dãy số u n  3n 1 9; 27; 81; 243; … Ta thấy không trùng với dãy số đề bài, nên loại đáp án A Đáp án B: Các số hạng đầu dãy số u n   3n 4; 10; 28; 82; 244; … Ta thấy không trùng với dãy số đề bài, nên loại đáp án B Đáp án C: Các số hạng đầu dãy số u n  3n 3; 6; 9; 12; 15; … Ta thấy không trùng với dãy số đề bài, nên loại đáp án C Đáp án D: Các số hạng đầu dãy số u n  3n 3; 9; 27; 81; 243; … Ta thấy trùng với dãy số đề  Chọn D u  Ví dụ 3: Cho dãy số  u n  với  Số hạng thứ n + dãy số u n là: u n 1  u n  n A u n     n   n  1 C u n    B u n     n   n  1 D u n     n   n  1  n   n  1 Hướng dẫn Ta có u1  u2  1 u3    u4  1  u5      u6  1     u n       n    … n  n  1 (Chứng minh quy nạp) Trang Do đó, số hạng tổng quát dãy số là: u n   Vậy số hạng thứ n + dãy số là: u n    n  n  1  n   n   1    n   n  1 2  Chọn A Bài tập tự luyện Câu Cho dãy số có số hạng đầu 8; 15; 22; 29; 36; … Số hạng tổng quát dãy số là: A u n  7n  B u n  7n C u n  7n  D u n không viết dạng công thức u  Câu Tìm cơng thức tính số hạng tổng quát u n theo n dãy số sau  u n 1  u n  A u n  2n  Đáp án 1–C B u n  n  C u n  n  D u n  3n 2–A Dạng 2: Dãy số tăng, giảm, bị chặn Phương pháp giải Cách 1: Sử dụng kiến thức phần lí thuyết trọng tâm Cách 2: Sử dụng máy tính CASIO fx570VN PLUS Dãy số tăng, giảm: Cho dãy số u n  f  n  Nhập MODE Nhập f  X    Start? = End? 10 =  Step = Ví dụ: Cho dãy số u n  2n  n 3 Nhập MODE Nhập f  X   2X    Start? = X3 End? 10 =  Step = Ta nhận bảng giá trị f  X  , tương ứng Ta nhận bảng giá trị f  X  , tương ứng với với số hạng dãy số số hạng dãy số là: Nhìn vào bảng giá trị này: 0,25; 0,6; 0,8333; 1; 1,125;  Các giá trị tăng dần dãy số u n tăng 1,2222; 1,3; 1,3636; 1,4166; 1,4615  Các giá trị giảm dần dãy số u n giảm Ta thấy giá trị tăng, nên dãy số u n dãy  Các trường hợp khác dãy số u n khơng tăng, tăng khơng giảm Dãy số bị chặn trên, bị chặn dưới, bị chặn: Cho dãy số u n  f  n  Nhập MODE Ví dụ: Cho dãy số u n  2n  n 3 Nhập MODE Trang Nhập f  X   Start? = End? 20 = Step = Nhập f  X   Ta nhận bảng giá trị f  X  , tương ứng với số hạng dãy số 2X   X3 Start? = End? 20 = Step = Ta nhận bảng giá trị f  X  tương ứng với số hạng dãy số là: Nhìn vào bảng giá trị này:  Các giá trị nhỏ số M  Dãy số u n bị chặn M  Các giá trị lớn số m  Dãy số u n bị chặn 0,25; 0,6; 0,8333; 1; 1,125; 1,2222; 1,3; 1,3636; 1,4166; 1,4615; 1,5; 1,5333; 1,5625; 1,5882; 1,6111; 1,6315; 1,65; 1,6666; 1,6818; 1,6956 Ta thấy giá trị tăng lớn  Các trường hợp khác  Dãy số u n không bị nhỏ 2, nên dãy số u bị chặn bị n chặn chặn 2 Ví dụ minh họa Ví dụ 1: Dãy số dãy số sau dãy số giảm? A u n   1  5n   n B u n  2n  n 3 C u n  6n n 1 D u n  5 n D u n  n2 n2 1 Hướng dẫn Xét dãy số u n  u n 1  u n   ta có: n 1 1 1      5     ; x  * n 1 n  n  n n  n  1 Vậy dãy số u n   dãy số giảm n  Chọn D Ví dụ 2: Trong dãy số sau, dãy số dãy số tăng? A u n   2n n 2 B u n  2n n2  n 1 C u n  3n  Hướng dẫn Xét dãy số u n  3n  2n ta có u n 1  u n  3n 1   n  1   3n  2n   3.3n  2n   3n  2n  2.3n   ; x  * Vậy dãy số u n  3n  2n dãy số tăng  Chọn A Ví dụ 3: Dãy số dãy số sau dãy số không bị chặn? 8n  A u n  5n  B u n  n  4n  n2 C u n  5n  2  D u n     n  n Trang Hướng dẫn Ta có: u n  n  4n     n     1 , n  Vậy dãy số bị chặn dưới, không bị chặn  Chọn B Ví dụ 4: Trong dãy số sau, dãy số bị chặn? A u n  7n  5n  B u n  n  n  C u n  2n  4n  D u n   1  32n   n Hướng dẫn Xét dãy số u n  Nhận xét   24 7n  ta có: u n   5  5n   5n  24 1   ; n      5n   5n  12 7 24 7       un  5  5n   5 Suy ra: u n  7n  dãy số bị chặn 5n   Chọn A Bài tập tự luyện Câu Dãy số dãy số sau dãy số tăng? B u n   n A u n   1   1 n n 2n  C u n  n 3  u1   D  2u n u n 1   u  n Câu Dãy số dãy số sau dãy số giảm? A u n   1  2n  1 n B u n  2n  4n  Câu Cho dãy số u n  sin A Dãy số âm Đáp án 1–C C u n  n 1 n 1 D u n  3n 2n 1  Chọn khẳng định khẳng định sau: n B Dãy số giảm 2–B C Dãy số tăng D Dãy số bị chặn 3–D Dạng 3: Giới hạn dãy số Phương pháp giải Cách 1: Sử dụng công thức phần Cách 2: Sử dụng máy tính CASIO fx570VN PLUS Tính giới hạn dãy số u n  f  n  Tính giới hạn dãy số u n  2n  n 3 Trang Nhập f  X  , CALC X  109 = Nhập 2X  , CALC X  109 = X3 Ta nhận kết 1,99999 Ta nhận kết gần với giá trị giới hạn Do giới hạn dãy số u n dãy số Ví dụ minh họa Ví dụ 1: Tìm giới hạn dãy số  u n  , biết u n  A B  3n  n  4n  2n  C D Hướng dẫn Cách 1: Ta thấy n lũy thừa cao tử mẫu, nên chia tử mẫu u n cho n được: 3n  n  2 3  2 3n  n  n n 3 lim u n   lim n  lim 4n  2n  4n  2n  4  n n n (Vì lim  ; lim  ; lim  ) n n n Cách 2: Sử dụng máy tính CASIO fx570VN PLUS 3X  X  Ta nhập vào máy tính Sau bấm nút CALC 109  0,750000 4X  2X  Ta thấy kết gần với đáp án A  Chọn A Ví dụ 2: Giới hạn dãy số u n  2n  n a a với phân số tối giản, b > Giá trị a  b3 là: n n 4 b b B  A C D Hướng dẫn n 2 2n  n 2n n  n   1 n n n n 7 1 7 7  lim n  lim n  lim   n  1  Ta có: lim u n  n n n n 4 7 4 4  n 1   n n 7 7 n (Vì n 2 4   lim    ;   lim    ) 7 7 7 Khi đó: a  1 ; b = nên a  b3   Chọn C Trang Ví dụ 3: Cho giới hạn lim A  16  4n  1  b Tính  n  n  n  a lim 4n a b B C D 65 16 Hướng dẫn Ta có: lim  lim   n  n  n  lim n   n    1 n    lim n nn 2 n n n  lim n  1 n 1    n  n 1  a 1 1 n 4n  n   b Vậy     65 Ta có: lim  lim 4  n3 a b   42 16    n3  2 4  Chọn D Ví dụ 4: Giá trị giới hạn lim 30  4n  5n bằng: A D  C  B Hướng dẫn Cách 1: Ta có: lim 30  4n  5n  lim Vì lim 30 4 n n 30     lim n   Nên lim 30  4n  5n   n n Cách 2: Sử dụng máy tính CASIO fx570VN PLUS Ta nhập vào máy tính 30  4X  5X Sau bấm nút CALC 109  1587401052 Ta thấy kết gần với đáp án C  Chọn C Ví dụ 5: Trong giới hạn sau, giới hạn có giá trị  ? A lim 2n  5n  7n B lim  5n  2n C lim  3n  2n   D lim  2n cos n  3n  Hướng dẫn    Ta có: L A  lim 2n  5n  7n  lim n      lim  n 2   n n  n n   Do lim    7  , lim  nên lim    lim n   n n n n Trang Suy L A       Ta có: L B  lim  5n  2n  lim n      lim  n 3    n n n n    Ta có lim   3  lim  , nên lim       lim n   3  n n  n n  Suy L B      Ta có: LC  lim  3n  2n    lim  n      n n    Do lim    lim  nên lim      lim n   n n  n n  Suy LC     cos n  Ta có L D  lim  2n cos n  3n   lim  n    n    cos n cos n 1 0 Mà   mà lim   lim n n n n n  cos n  Do lim     , lim n   Suy có L D   n    Chọn B Bài tập tự luyện Câu Giới hạn dãy số  u n  , với u n  A  3n  n là: 4n  B  Câu Tính lim A  10 n  n2 1 C D B 10 D  C Câu Trong giới hạn sau, giới hạn có giá trị 1? A lim 1 n B lim 2n  3n  2n  Câu Tìm giới hạn dãy  u n  , biết u n  A 1 Đáp án 2–C 3–C n2 n 1 D lim 3n  n  2n  3.2n  5n 5.4n  6.5n B  1–A C lim C D 4–A Trang PHẦN 4: BÀI TẬP TỔNG HỢP Câu Dãy số dãy số sau dãy số tăng? A u n  n2  n 1 2n  1 2 n B u n  C u n  3n  n D u n  n n 1 Câu Dãy số dãy số sau dãy số giảm? A u n   1   1 n Câu Cho dãy số  U n  với U n  n 1 C u n  n 1 n B u n  n 2n 3n  2n  D u n  n 1 n Khẳng định sau đúng? n 1 A Năm số hạng đầu dãy số là: 1 2 3 5 5 ; ; ; ; B Năm số hạng đầu dãy số là: 1 2 3 4 5 ; ; ; ; C Là dãy số tăng D Bị chặn số 2 Câu Cho giới hạn lim   8n  n   4n  n   a a với phân số tối giản, b > Giá trị b b a  b3 là: A 26 B 27 Câu Cho giới hạn lim A x   C 28 3n  2n  3n  A  a Giá trị a nghiệm phương trình đây? B x  4x   Câu Tìm giới hạn dãy số  u n  , biết lim 1 D 29 C x   D x  5x   3n  5.4n n  2n B D  C   Câu Biết giới hạn lim 3n   9n   m Mệnh đề sau đúng? A m   B m   C Giá trị m số dương D Giá trị m thỏa mãn bất phương trình 2x  6x   Câu Cho dãy số u n  A n Tìm giá trị nghịch đảo giới hạn dãy số u n 4n B Câu Tìm giới hạn dãy số  u n  , biết lim C  D Không xác định  n2  n  n Trang 10 A B 1 C D Đáp án: 1-C 2-B 3-B 4-A 5-A 6-C 7-D 8-D 9-B Trang 11 CHƯƠNG 5: DÃY SỐ, GIỚI HẠN, ĐẠO HÀM CHUYÊN ĐỀ 2: GIỚI HẠN CỦA HÀM SỐ, HÀM SỐ LIÊN TỤC PHẦN 1: LÝ THUYẾT TRỌNG TÂM Giới hạn hàm số Giới hạn hữu hạn lim f  x   L hay f  x   L x  x x x0  Giới hạn bên phải lim f  x   L x x0  Giới hạn bên trái lim f  x   L x x0 Định lí lim f  x   L  lim f  x   lim f  x   L x x0 x x0 x x0 Giới hạn vô cực lim f  x   L hay f  x   L x   ; x  lim f  x   L hay f  x   L x   x  Giới hạn vô cực lim f  x    hay f  x    x   ; x  lim f  x    hay f  x    x   ; x  lim f  x    hay f  x    x   ; x  lim f  x    hay f  x    x   x  Các dạng giới hạn vô định hàm số:  ; ;    ; 0. ; 1  Hàm số liên tục Hàm số liên tục điểm Cho hàm số y  f  x  xác định khoảng K x  K Hàm số y  f  x  liên tục x lim f  x   f  x  x x0 Hàm số liên tục khoảng, đoạn y  f  x  liên tục  a; b  y  f  x  liên tục điểm x   a; b  y  f  x  liên tục  a; b  y  f  x  liên tục  a; b  ; lim  f  a  x a  lim  f  b  x b Tính chất: Các hàm số sơ cấp (đa thức, lượng giác, mũ, lôgarit) liên tục khoảng tập xác định Nếu hàm số y  f  x  liên tục đoạn  a; b  f  a  f  b   tồn điểm c   a; b  cho f  c   Nói cách khác: Nếu hàm số y  f  x  liên trục đoạn  a; b  f  a  f  b   phương trình f  x   có nghiệm c   a; b  Trang Mở rộng: Nếu hàm số y  f  x  liên tục đoạn  a; b  Đặt m  f  x  , M  max f  x  Khi với a;b a;b T   m; M  ln tồn số c   a; b  cho f  c   T PHẦN 2: CÁC CÔNG THỨC Giới hạn đặc biệt lim x  x ; lim c  c x x0 lim c  c (c số) lim c  c x  x  (c số) (c số) x x0 lim x k   lim x k   ; k chẵn x  x  lim   ; x lim   x x 0 lim x   ; k lẻ k x  x 0 lim c 0 xk lim 1  lim   x x 0 x x  x 0 Định lí giới hạn lim f  x   L lim g  x   M x x0 lim f  x   g  x    L  M x x0 x x0 f x L  x x0 g  x  M lim f  x  g  x    L.M lim x x0 f  x   ; lim f  x   L  M  0 lim f  x   L  lim f  x   L x x0 x x0 x x0  L  lim f  x   L x x0 Quy tắc giới hạn vô cực lim f  x  lim g  x  x x0 L>0 L0 +    L nên m   Chọn A Bài tập tự luyện Câu Tìm giới hạn lim x    2x   x Trang A Câu Tìm giới hạn lim x 3 A C  B  D x 3 5x  15 B 1 C D 2x  2x, x  Câu Cho hàm số f  x    Nhận xét sau sai?  x  3x, x  A lim f  x   x 1 B lim f  x   x 1 C Hàm số cho khơng có giới hạn D Hàm số có tập xác định  x  2x  là: x 1 2x  Câu Kết lim A  Đáp án B 1–B 2–C C 3–A D  4–B Dạng 2: Hàm số liên tục Phương pháp giải Hàm số y  f  x  liên tục điểm x lim f  x   lim f  x   f  x  x x0 x x0 Chú ý: Ta tính giới hạn cách sử dụng máy tính Ví dụ minh họa   x   x  Ví dụ 1: Cho hàm số f  x    khẳng định: x  0 (I) Hàm số f  x  xác định x = (II) Hàm số f  x  không liên tục x = (IV) Hàm số f  x  liên tục x = (III) lim f  x   x 3 Chọn đáp án A Chỉ (I) B Chỉ (I) (IV) C Chỉ (II) (IV) sai D Tất Hướng dẫn Vì hàm số f  x  xác định x = Nên (I) đúng: Hàm số có khoảng xác định  3;3  3;   Vì x = thuộc khoảng xác định hàm số, hàm số liên tục x = Nên (II) sai Trang Mà lim f  x   lim  x  ; lim f  x   lim  ; f  3   32  x 3 x 3 x 3 x 3 Vậy lim f  x   hàm số liên tục x = Nên (III) sai, (IV) x 3 Vậy có (I) (IV)  Chọn B 3  x2  x  2  Ví dụ 2: Cho hàm số y  f  x    x  Nhận xét sau đúng?  x  2  A lim f  x   x 2 B lim f  x   f   x 2 C Hàm số liên tục x  2 D Hàm số xác định x  2 Hướng dẫn Ta thấy hàm số không xác định x  2 Nên đáp án D sai  x2   x2  1  lim  lim  x 2 x 2 x 4  x    x  x 2  x  Ta có lim f  x   lim x 2   Nên đáp án A sai Mà f      lim f  x   f   Nên đáp án B sai x 2 Tương tự ta tìm lim f  x   x  2  1  f  2  Do hàm số liên tục x  2  Chọn C  x  2x  x   Ví dụ 3: Giá trị a hàm số f  x    x  liên tục x = 3? 5a  x   A B C D Hướng dẫn Ta có lim f  x   lim x 3 x 3  x  1 x  3  lim x   ; x  2x   lim   x 3 x 3 x 3 x 3 lim  5a    5a  ; f  3  5a  x 3 Để hàm số liên tục x = 5a    a   Chọn A Trang  x  2x  x   Ví dụ 4: Cho hàm số y  f  x    x   Khẳng định là: 9x    x   A Hàm số liên tục điểm x  3 B Hàm số không liên tục điểm x = C Hàm số liên tục điểm x  4 D Hàm số liên tục điểm x = Hướng dẫn Hàm số có khoảng xác định  3;1 1;   Vì x  3 x  4 không thuộc khoảng xác định hàm số, hàm số khơng liên tục x  3 x  4 Nên đáp án A C sai Ta có lim f  x   lim x 1 x 1  lim  x  3 x 1    x  1 x  3 x   x  2x   lim x 3 x   x 1   x    16 Ta lại có lim f  x   lim  9x    16 ; f 1  9.1   16 x 1 x 1 Suy lim f  x   f 1 Do đó, hàm số liên tục điểm x = Nên đáp án D x 1  Chọn D Bài tập tự luyện  x  3x  x   Câu Cho hàm số f  x    x  Chọn khẳng định sai 1 x   A lim f  x   B Hàm số f  x  liên tục điểm x  C Hàm số liên tục x = D Hàm số có tập xác định  x 2 5x  6x  x x  Câu Cho hàm số f  x    Chọn khẳng định sai x   x  3x A lim f  x   B lim f  x   2 C Hàm số không liên tục x = D Hàm số có tập xác định  x 0 x 1  2x  7x   x   x2 Câu Cho hàm số y  f  x    Xác định a để hàm số f  x  liên tục x  a   x x    x A 3 B 1 C D Trang x  a   x  3x  x  liên tục x  Câu Tìm a để hàm số f  x    x  6x   x  b  A Đáp án B  1–A 2–C 3–A C D C D 4–B PHẦN 4: BÀI TẬP TỔNG HỢP x  2x  x 1 2x  Câu Tính giới hạn lim A 2 B  Câu Tìm giá trị lim x 3 A Không tồn x 3 x 3 B C D  x3  x 2 x  11x  18 Câu Tìm giới hạn lim A B 1 Câu Tìm giới hạn lim x  A 3x  2x  1  5x  1  x  2x  B  C 12 D C D 2x  5x  4x  1 Câu Tìm , với L giới hạn hàm số x  1 x3  x  x 1 L A B 1 C D 5x  6x  x x  Câu Cho hàm số f  x    Nhận xét sai? x   x  3x A lim f  x   B Hàm số khơng có giới hạn x = C lim f  x   2 D Hàm số có tập xác định  x 0 x 1  3x   4x  , x  3 Câu Tìm giơi hạn lim g  x  với g  x    x 3 62  x  , x  3  17 Trang A B Câu Biết lim x   A 11 C 11 17 D Không tồn giới hạn  ax  bx   x  với a, b   Khi a  ab  b bằng: B 101 C 111 D 110  x  a2  Câu Biết lim  x  1   Tìm giá trị a x 1 x 1   A a  2 B  Câu 10 Giá trị giới hạn lim C x 2017  2017x  2016  x  1 x 1 A Không tồn B 17 B  D là: C 2034000 Câu 11 Tìm giới hạn hàm số y  A  D 2033136 sin x  sin 2x x tiến đến  x x 1  2sin  2  16 C 1 D  x  3x  x   Câu 12 Tìm giá trị tham số m để hàm số f  x    x  2x liên tục  mx  m  x   A  B m  C Không tồn m D m    tan x x   x   k, k    Câu 13 Cho hàm số f  x    x Hàm số y  f  x  liên tục 0 x  khoảng sau đây? A  ;0    B  ;  4     C   ;   6   D  0;   2 x  1 a  b2  Câu 14 Cho hàm số f  x   ax  b  x  liên tục  Tính 7 x   A  B 73 C D 73 3   x  x   x  x  Câu 15 Cho hàm số f  x   m Tìm m để f  x  liên tục nửa khoảng 3  x   x 0;   Trang 10 A B C D   x   sin x     Câu 16 Cho hàm số f  x   a sin x  b   x  Tìm giá trị a để hàm số liên tục  2    2  cos x x   A B C D Đáp án: 1-A 2-A 3-C 4-C 5-D 6-B 11 - C 12 - A 13 - D 14 - D 15 - C 16 - A 7-C 8-C 9-A 10 - D Trang 11 CHƯƠNG DÃY SỐ, GIỚI HẠN, ĐẠO HÀM CHUYÊN ĐỀ 3: CẤP SỐ CỘNG, CẤP SỐ NHÂN PHẦN 1: LÝ THUYẾT TRỌNG TÂM Cấp số cộng un  cấp số cộng u n 1  u n  d, n  * , d số d: công sai; d  u n 1  u n Số hạng tổng quát u n  u1   n  1 d, n  2;d  Tính chất u k  u n  u1 n 1 u k 1  u k 1 hay u k 1  u k 1  2u k , k  Tổng n số hạng đầu Sn  u1  u  u   u n Sn  n  2u1   n  1 d  n  u1  u n  , n  * hay Sn   2 Cấp số nhân un  cấp số nhân u n 1  u n q, n  * , q số q: công bội; q  u n 1 un Số hạng tổng quát u n  u1q n 1 , n  Tính chất: u 2k  u k 1u k 1 hay u k  u k 1u k 1 , k  Tổng n số hạng đầu Sn  u1  u  u   u n • Nếu q  Sn  nu1 , n  * • Nếu q  Sn  u1 1  q n  1 q , n  * Cấp số nhân lùi vơ hạn cấp số nhân vơ hạn có công bội q thỏa mãn q  Tổng cấp số nhân lùi vô hạn Sn  u1  u  u   u n  u1 1 q Chú ý: Trong sách này, ta viết tắt cấp số cộng CSC; viết tắt cấp số nhân CSN PHẦN 2: CÁC DẠNG BÀI TẬP Dạng 1: Tìm cấp số cộng, cấp số nhân Ví dụ minh họa 3 3 Ví dụ 1: Cho dãy số 3;  ;  ;  ;  ; Khẳng định sau sai? 16 A Dãy số cấp số nhân u1  3;q  B Số hạng tổng quát u n  3 2n Trang C Số hạng tổng quát u n  3 2n 1 D Dãy số dãy số giảm Hướng dẫn Ta có:  3 3 3  3 ;    ;    ;    2 2 16 Vậy dãy số cấp số nhân giảm với u1  3; q  Số hạng tổng quát cấp số nhân là: u n  u1.q Nên đáp án A D n 1 1  3   2 n 1  3 2n 1  Chọn B Ví dụ 2: Trong dãy số sau, dãy cấp số cộng? A u n  3n  4n  B u n   1 n  8n n C u n  n  n  D u n  5n  Hướng dẫn Cách 1: Xét dãy số  u n  với u n  5n  Ta có: u n 1  u n   n  1    5n  1  5n    5n   Vậy  u n  cấp số cộng với công sai d  Cách 2: Sử dụng máy tính CASIO fx 570 VN PLUS Tìm vài số hạng dãy số đáp án kiểm tra xem dãy số cấp số cộng  Chọn D Ví dụ 3: Giữa số –2 –8192 ta đặt thêm ba số để tạo thành cấp số nhân Ba số cần điền thêm là: A 16; –128; 1024 B –16; 128; –1024 C 16; 128; 1024 D –16; –128;1024 Hướng dẫn Cách 1: Theo đề ta cấp số nhân có năm số hạng với số hạng đầu –2 số hạng cuối –8192 u1  2 u1  2 u  2    q  8 u  8192 u1q  8192 Với q  8  ba số cần điền thêm 16; –128; 1024 Với q   ba số cần điền thêm –16; –128; –1024 Cách 2: Sử dụng máy tính CASIO fx 570VN PLUS Thử đáp án kiểm tra xem dãy số cấp số nhân  Chọn A Ví dụ 4: Biết C1n ;C2n ;C3n lập thành cấp số cộng với n  Khi giá trị n là: A B C D 11 Hướng dẫn Trang Cách 1: Điều kiện n  * , n  Vì C1n ;Cn2 ;C3n lập thành cấp số cộng, nên ta có C1n  C3n  2Cn2 n n  n  1  n   n! n!  n  n  n  1 3!  n  3 ! 2!  n   !  n  (Loaïi) n    n  9n  14    6   n  1 n     n  1  n   Loại  Cách 2: Sử dụng máy tính CASIO fx 570VN PLUS Thử đáp án với giá trị n kiểm tra xem dãy số C1n ;C2n ;C3n cấp số cộng  Chọn B Bài tập tự luyện Câu Trong dãy số sau, dãy số cấp số cộng? B un   1  10n n A un  19n  C un  n  n  D un  2n3  Câu Viết ba số xen số 22 để cấp số cộng có năm số hạng A 7; 12; 17 B 6; 10; 14 C 8; 13; 18 D 6; 12; 18 Câu Trong dãy số sau, dãy số cấp số nhân? A un   1  10n n B un  3n   u1   C  u  n   un   u1  D  u n 1  u n Câu Hãy chọn cấp số nhân dãy số cho sau A u n  1 B u n  n n 2 C u n  n  D u n  n  Đáp án: 1–A 2–A 3–C 4–B Dạng 2: Tìm cơng bội, cơng sai, số hạng thứ n cấp số Phương pháp giải Sử dụng công thức: Cấp số cộng: d  u n 1  u n ;d  Cấp số nhân: q  u n  u1 ; u n  u1   n  1 d n 1 u n 1 ; u n  u1q n 1 un Ví dụ minh họa Ví dụ 1: Cho cấp số nhân có u  15; u  3645 Tìm số hạng đầu công bội cấp số nhân A u1  5;q  B u1  5;d  3 C u1  5;d  3 D u1  5;d  2 Hướng dẫn Trang u1.q  15 u  15 1 Ta có:      q  3  u1  243 u  3645 u1.q  3645 q Vậy u1  5;d  3  Chọn C u  u  u  9 Ví dụ 2: Tìm cơng sai cấp số cộng sau  u  u  26 A B C D Hướng dẫn u1  2d  u1  4d   u1  8d   9 u  u  u  9 u  2d  9 u  1 Ta có:     u1  d  u1  6d  26 d  2u1  7d  26 u  u  26 Vậy công sai d   Chọn A u1  3u  u  2 Ví dụ 3: Cho cấp số cộng  u n  thỏa mãn  Xác định số hạng tổng quát cấp số 2u  u  4 cộng A u n    n  1 B u n  2   n  1 C u n    n  1 D u n    n  1 Hướng dẫn u1  3u  u  2 u  d  2 u  u1   u1  2d   u1  5d  2 Ta có:     d  2 u1  4d  4 2u  u  4 2  u1  3d    u1  2d   4 Vậy số hạng tổng quát cấp số cộng u n    n  1 2  hay u n    n  1  Chọn D 35  u  u  u   Ví dụ 4: Cho cấp số nhân u1.u  25 có cơng bội ngun, số 320 số hạng thứ u  i  1, ,5     cấp số nhân? A Số hạng thứ B Số hạng thứ C Số hạng thứ D Số hạng thứ Hướng dẫn 35  35 35    u1  q  q  q   u  u  u  u1q  u1q  u1q  Ta có:    u1.u  25 u u q  25  u q 2  25  1   q  q  q3 35    u1  q  q  q    q2   u q  u q    1  2 Trang q   q  q2 2 Từ 1 q  ta có:   1  q  q   7q  2q  5q     q  q  Vì cấp số có cơng bội ngun, nên chọn q  nên u1  Suy số hạng tổng quát cấp số nhân u n  2n 1 Ta có 320  2n 1  n  Vậy số 320 số hạng thứ chín cấp số nhân  Chọn D u  u  14 Ví dụ 5: Cho cấp số cộng sau  Số hạng thứ mười hai cấp số S12  129 A 19 B 15 C 23 D 38 Hướng dẫn Ta có: Sn  n  u1  u n  12  u1  u12   S12   129 2   u1  u12   129   u1  u1  11d   129  12u1  66d  129  u1   u  u  14 u  2d  u1  4d  14 2u  6d  14     Mà:  12u1  66d  129 12u1  66d  129 S12  129 d   Suy số hạng tổng quát cấp số là: u n    n  1 2 Vậy số hạng thứ mười hai cấp số là: u12   12  1  19 2  Chọn A Ví dụ 6: Cho ba số x; 3; y theo thứ tự lập thành cấp số nhân x  y Tính giá trị biểu thức x  xy  2y A 88 B 77 C 66 D 99 Hướng dẫn Vì ba số x; 3; y theo thứ tự lập thành cấp số nhân nên ta có xy  32  1 x4 y Nếu x   y   x; 3; y không tạo thành cấp số nhân (Loại) Xét x  , từ 1 ta có y  x4  Thay vào x  y ta được: x  x5   x   y  3 x Trang Vậy x  xy  2y   3    3.3  3  66  Chọn C Bài tập tự luyện u  Câu Tìm công bội nguyên cấp số nhân sau:  S3  43 A B C D u  5u Câu Tìm cơng sai cấp số cộng sau:  u13  2u  A B C D u1  u  u  14 Câu Tìm số hạng đầu cấp số nhân công bội nguyên sau:  u1.u u  64 A –7 B –12 C D Câu Giữa số 160 5, ta chèn vào bốn số để tạo thành cấp số nhân Tìm số hạng thứ ba A 40 B 80 C 20 D 10 u1  u  Câu Có cấp số nhân thỏa mãn:  ? u  u  A B C D S12  34 Câu Tìm số hạng cấp số cộng sau:  S18  45 A 1 B 31 C D Đáp án: 1–C 2–B 3–C 4–A 5–C 6–B Dạng 3: Tính tổng cấp số, tìm số số hạng cấp số Phương pháp giải Sử dụng công thức: Cấp số cộng: Sn  n  2u1   n  1 d  n  u1  u n  ;Sn   2 Cấp số nhân: q   Sn  nu1 q   Sn  u1 1  q n  1 q Tổng cấp số nhân lùi vô hạn: S  u1 1 q Trang Ví dụ minh họa u1  u  51 Ví dụ 1: Tìm tổng 10 số hạng cấp số nhân sau:  u  u  102 A 3069 B 3096 C 3079 D 3097 Hướng dẫn u1 1  q   51  u  q  51  u1  u  51 q   u1  u1q  51  1     51  Ta có:  u   u q  q  102 u1q  u1q  102  u  u  102      q 102 Vậy tổng 10 số hạng là: S10  1  210  1  3069  Chọn A Ví dụ 2: Cho cấp số cộng có công sai âm, số hạng thứ tư 11 Hiệu số hạng thứ ba số hạng thứ sáu Hỏi 45 tổng số hạng đầu tiên? A 12 B 13 C 14 D 15 Hướng dẫn u1  3d  11 u  11 u  3d  11 u1  17 Theo đề ta có:     u1  2d   u1  5d   3d  d  2 u  u  Ta có: n  2u1   n  1 d  n  2.17   n  1 2   n  Sn    45    90  n  2n  36    2  n  15 Vậy 45 tổng 15 số hạng  Chọn D 21845 Ví dụ 3: Cho cấp số nhân  u n  , biết u  ; u  16 tổng Sn  Cấp số nhân có 16 số hạng? A B C D 10 Hướng dẫn 1   u1q  u1q  1       u q  u  4     u1q   u1  Ta có:      16 4 u  16 u q  16  q4   13  q  q   64 64  q  q Ta có: Sn  u1 1  q n  1 q 1  4n  21845 16     65535   4n  4n  65536  n  16 1  Chọn B Bài tập tự luyện Trang Câu Cho cấp số nhân có u1  3;q  A Tính tổng dãy số B C D u  150 Câu Cho dãy số  u n  xác định  với n  Khi số 300 tổng bao u n  u n 1  nhiêu số hạng đầu tiên? A 100 B 120 C 150 D 180 C a   D a  5 Đáp án: 1–D 2–A PHẦN 3: BÀI TẬP TỔNG HỢP Câu Cho cấp số nhân A a   1 1 ;a; Giá trị a là: 125 B a   25 Câu Cho cấp số nhân có u1  3;q  A u  27 16 B u  Tính u 16 27 C u  16 27 D u  27 16 u1  u  u  Câu Cấp số nhân  có cơng bội q1 q Tổng q1  q bằng: 2 u  u  u  21 A 19 B C Câu Tìm tích số dương a b cho  b  1 D a, a  2b, 2a  b lập thành cấp số cộng , ab  5,  a  1 lập thành cấp số nhân A 12 B Câu Cho cấp số nhân có u1  3;q  C 18 D 96 Số số hạng thứ cấp số này? 243 A Thứ năm B Thứ sáu C Thứ bảy D Không phải số hạng cấp số Câu Cho cấp số nhân  u n  , biết u1  5; u  405 tổng Sn  1820 Cấp số nhân có số hạng? A Câu B C D Cho cấp số cộng u1 ; u ; u ; có cơng sai d Biết u1  u  u  u10  u13  u16  147 Tính u1  u  u11  u16 ? A 34 B 29 C 98 D 71 Trang u1  u  51 Câu Cho cấp số nhân  u n  có số hạng thỏa mãn  Hỏi số 12288 số hạng thứ u  u  102 mấy? A 20 B 13 C D 12 Câu Cho cấp số cộng có u1  ; u  26 Tìm cơng sai d A d  11 3 11 B d  C d  10 D d  10 2 u  u  u  155 Câu 10 Cấp số nhân  có cơng bội q1 q Tính tổng q1  q S3  21 A B 17 C D S5  Câu 11 Tìm cơng sai cấp số cộng sau  , biết công sai số dương u1.u u u u  45 A B C D.5 u1  u  u  Câu 12 Cấp số cộng  có hai cơng sai d1 , d Tính tổng d1  d 2 u  u  u  35 A B C D Đáp án: 1–B 2–C 11–A 12–B 3–B 4–D 5–B 6–C 7–C 8–B 9–A 10–B Trang CHƯƠNG DÃY SỐ, GIỚI HẠN, ĐẠO HÀM CHUYÊN ĐỀ 4: ĐẠO HÀM PHẦN 1: LÝ THUYẾT TRỌNG TÂM Khái niệm đạo hàm Cho hàm số y  f  x  xác định khoảng (a;b) x   a; b  Giới hạn hữu hạn có tỉ số f  x   f  x0  x  x gọi đạo hàm hàm số cho x , kí hiệu f   x  hay y  x  x  x0 f  x   f  x0  y  lim x x0 x  x x  x0 Như ta có: f   x   lim  x  x  x , y  f  x  x   f  x   Chú ý: Nếu hàm số f  x  có đạo hàm điểm x f  x  liên tục x Các quy tắc tính đạo hàm Giả sử u  u  x  ; v  v  x  ; w  w  x  hàm số có đạo hàm, đó:  u  v   u  v  u  v   u  v  uv   uv  uv  u  u v  uv    v2 v  ku   ku, k   Bảng đạo hàm hàm số Đạo hàm hàm số sơ cấp  C   Đạo hàm hàm số hợp  u  u  x   (C số)  x   x    , x    u   u  x   x  x  0  u   2uu  u  0        x  0 x x u        u  0 u u    n x 1 n     n 1  x   x     n u 1 u     , u   n     n 1 u   u   u   s inx   cosx  s inu   cosu u  cosx    s inx  cosu    s inu.u  tan x     tan x cos x  tanu   u  1  tan u  u  cos u Trang     x   k, k      cot x        u   k, k       1  co t x  sin x  cotu    u   1  co t u  u  sin u  x  k, k    u  k, k    a   a ln a  a   a ln a.u  e   e  e   e u x x x u x  log a x    ln x   u ,  x  0 x.ln a u u  log a u   ,  x  0 x  lnu   u ,  u  0 u ln a u ,  u  0 u Một số cơng thức tính đạo hàm nhanh ad  bc  ax  b      cx  d   cx  d   ax  bx  c  adx  2aex  be  dc     dx  e   dx  e   ax  bx  c   ae  bd  x   af  dc  x  bf  ec     dx  ex  f   dx  ex  f  Vi phân Cho hàm số y  f  x  có đạo hàm x Ta gọi tích f   x  x vi phân hàm số f  x  điểm x ứng với số gia x (gọi tắt vi phân f điểm x) Kí hiệu df  x   f   x  x Nếu chọn hàm số y = x dy  dx  1.x  x Vì ta thường kí hiệu x  dx dy  f   x  dx Cơng thức tính gần nhờ vi phân là: f  x  x   f  x   f '  x  x Đạo hàm cấp cao Cho hàm số y  f  x  có đạo hàm f   x  Hàm số f   x  gọi đạo hàm cấp hàm số f  x  Nếu hàm số f   x  có đạo hàm đạo hàm gọi đạo hàm cấp hàm số f  x  , kí hiệu y" hay f "  x  Đạo hàm đạo hàm cấp gọi đạo hàm cấp hàm số f  x  , kí hiệu y hay f   x  n Tương tự, ta gọi đạo hàm đạo hàm cấp (n – 1) đạo hàm cấp n hàm số f  x  , kí hiệu y  hay f  n   x  , tức ta có:    y n   y n 1 ;  n  N, n  1 Chú ý: Vận tốc tức thời chuyển động đạo hàm độ dời chuyển động theo thời gian Gia tốc tức thời chuyển động đạo hàm vận tốc thức thời chuyển động theo thời gian Trang Đạo hàm cấp độ dời gia tốc tức thời chuyền động thời điểm t PHẦN 2: CÁC DẠNG BÀI TẬP Dạng 1: Các quy tắc tính đạo hàm Ví dụ minh họa 3   x x0  Ví dụ 1: Cho hàm số f  x    Khi f    kết sau đây?  x0  A B 16 C 32 D Không tồn Hướng dẫn f  x   f  0 Ta có: lim  lim x 0 x 0 x 0  lim 2  4x    4x 4x   x x 0  3 4 x  4  lim   x x 0 x 4x   lim x 0  x 4x   x   lim x 0  2 4x   16  Chọn B  x  x  Ví dụ 2: Tìm a, b để hàm số f  x    có đạo hàm  2x  ax  b x  A a  10, b  11 B a  0, b  1 C a  0, b  D a  20, b  Hướng dẫn Ta thấy với x  f  x  ln có đạo hàm Do hàm số có đạo hàm  hàm có đạo hàm x = Ta có lim f  x   lim  x  1  1, lim f  x   lim  2x +ax  b   b x 0 x 0 x 0 x 0 Để hàm số liên tục x = b = lim x 0 f  x   f  0 x2 11  lim  lim x  0; x 0 x 0 x x  lim x 0 f  x   f  0 2x +ax  lim  lim  2x  a   a x 0 x 0 x x Để hàm số có đạo hàm x = thì: lim x 0 f  x   f  0 f  x   f  0  lim  a  x 0 x x Vậy a = 0, b = giá trị cần tìm  Chọn C Ví dụ 3: Đạo hàm hàm số sau y   x  x  là: Trang A y   x  x  B y   7x  1 x  1 C y   7x  x  x  1 D y   x  x  7x  1 Hướng dẫn Cách 1: y   x  x  Sử dụng công thức  u    .u 1.u  (với u  x  x ) y   x  x   x  x    x  x  7x  1 Cách 2: Sử dụng máy tính CASIO fx 570 VN PLUS Bước 1: Sử dụng SHIFT    , nhập hàm số y   x  x  với x = (hoặc số chứa căn, nhập số lẻ kết có tính xác cao) d dx  X  X   x  ta kết 116740 Bước 2: Thay x = vào bốn đáp án, chọn đáp án có kết 116740 Với đáp án A, ta có y   x  x    27    260 Không thỏa mãn, loại A Với đáp án B, ta có y   7x  1 x  1   27   26  1  8450 Không thỏa mãn, loại B Với đáp án C, ta có y   7x  x  x  1   7.27   26  1  58370 Không thỏa mãn, loại C Với đáp án D, ta có y   x  x  7x  1   27   7.26  1  116740 Thỏa mãn, chọn D  Chọn D Ví dụ 4: Cho f  x   A x  2x  Tính f   1 x 1 B –3 C –5 D Hướng dẫn x  2x  4  x1  f  x  1  f   1  Cách 1: Ta có f  x   x 1 x 1 x    Cách 2: Sử dụng máy tính CASIO fx 570 VN PLUS Sử dụng SHIFT    x  2x  d  X  2X   , nhập hàm số f  x   với x  1:   x  1 x 1 dx  X 1  ta kết  Chọn D Ví dụ 5: Đạo hàm hàm số sau y  A y  2sin 2x cos2 2x B y  cos x  sin x 2cos2x cos2 2x C y  cos2x cos2 2x D y  sin 2x cos2 2x Hướng dẫn Trang y 1  Áp dụng 2 cos x  sin x cos2x y    cos2x   cos2x   sin 2x  2x  2sin 2x cos2 2x  cos 2x    u    ta được: u u  Chọn A Ví dụ 6: Đạo hàm hàm số y  A 2x  4x  ax  bx  c Tính tổng a + b + c ? x 3  x  3 B C D 10 Hướng dẫn Ta có  2x y   4x  1  x  3   x  3  2x  4x  1  x  3  4x   x  3   2x  4x  1 2x  12x  11   2  x  3  x  3 Do đó: a = 2; b = –12; c = 11 nên a + b + c = – 12 + 11 =  Chọn A Ví dụ 7: Cho hàm số y  x   x Tính  x y A B y C xy D Hướng dẫn Cách 1: Áp dụng công thức  y   x   x2    u   u u ta được:  1  x   x2     x   x2 x   x2    x    x2 x   x2   1  x2  x    2  x  x  1 x Do đó:  x y   x x   x2 1 x   x2    1 x  x2  x  x2       x   x2  y Cách 2: Sử dụng máy tính CASIO fx 570 VN PLUS Bước 1: Thay x  vào y  x   x ta tính y     1,931851653 Bước 2: Tính giá trị  x y   3.y  4y Sử dụng SHIFT    , nhập hàm số y  x   x với x  Trang d    X   X  x  ta kết 1,931851653 dx   Do đó:  x y  y  Chọn B x Ví dụ 8: Cho hàm số y  cot Hệ thức sau đúng? A y  2y  B y  2y   C y  2y   D y  2y   Hướng dẫn Cách 1: Ta có: y   1 x     cot  x 2 2 2sin 2 Do đó: y  2y  cot x 1 x x  x    cot   cot    cot   1 nên y  2y   2 2  2 Cách 2: Sử dụng máy tính CASIO fx 570 VN PLUS Bước 1: Thiết lập môi trường SHIFT MODE Thay x = vào y  cot x ta tính y  cot  2 Bước 2: Tính giá trị y  2y Sử dụng SHIFT    1 tan  1,830487722 , nhập hàm số y  cot x với x =   d      x  ta kết –1 X dx     tan   tan   2   Do đó: y  2y  1 hay y  2y    Chọn B Bài tập tự luyện Câu Đạo hàm hàm số sau y  xcosx là: A y  cosx  x s inx B y  xcosx  s inx C y  cosx  x s inx D y  xcosx  s inx Câu Tìm số gia hàm số y  f  x   x3  3x  ứng với số gia x  0,1 đối số x x  A 0,329 B 0,178 Câu Đạo hàm hàm số sau y  A y  6   x x x C 0,299 D 0,198  x  x x là: x B y    x x x Trang C y    x x x D y  6   x x3 x  Câu Cho hàm số y  cos3x.sin2x Giá trị y   bằng: 3 A –1 C  B D Đáp án: 1–A 2–C 3–A 4–B Dạng 2: Vi phân, đạo hàm cấp cao Ví dụ minh họa Ví dụ 1: Cho hàm số y  x3  2x  Tính vi phân hàm số điểm x  1, ứng với số gia x  0,02 A –0,02 B 0,12 C 0,03 D –0,12 Hướng dẫn Ta có : y  f   x   3x  4x Do vi phân hàm số điểm x  1, ứng với số gia x  0,02 là:   df 1  f  1 x  3.12  4.1 0,02  0,02  Chọn A Ví dụ 2: Cho hàm số y  x3  9x  12x  Vi phân hàm số là:    D dy   3x A dy  3x  18x  12 dx    18x  12  dx B dy  3x  18x  12 dx  C dy   3x  18x  12 dx Hướng dẫn      Ta có dy  x3  9x  12x  dx  3x  18x  12 dx  Chọn A Ví dụ 3: Tính đạo hàm cấp hàm số sau y  cos2 x A 2sin2x B 8sin2x C sin2x D 2cos2x Hướng dẫn Ta có: y  cos2 x  1  cos2x   y   sin 2x  y    sin 2x   2cos2x  y   2cos2x   4sin 2x  Chọn C Trang Ví dụ 4: Cho hàm số f  x    x  1 Giá trị f "   bằng: A B C 12 D 24 Hướng dẫn Cách 1: Vì: f   x    x  1 ,f   x    x  1  f     Cách 2: Sử dụng máy tính CASIO fx 570 VN PLUS Sử dụng SHIFT d dx    , nhập hàm số  X  1  x  0.0001  dxd  X  1  x  3 0,0001   60003  10000 Chú ý: Công thức bấm đạo hàm cấp hàm số y  f  x  x = a là: d d f  x  x  a  0.0001  f x x  a dx dx 0,0001      Chọn C   Ví dụ 5: Tìm đạo hàm cấp n hàm số y  sin x n  * ?   A y n  sin  x   2    B y n  cos  x  n  2    C y n  sin  x  n  2    D y n  cos  x   2  Hướng dẫn     Bước 1: Ta có: y  cosx  sin  x   ; y   s inx  sin  x   2 2     Dự đoán: y n  sin  x  n  , với n  * 1 2  Bước 2: Chứng minh (1) quy nạp: * n = 1: (1) hiển nhiên   * Giả sử (1) với n  k  nghĩa ta có: y k  sin  x  k  ta phải chứng minh (1) với 2  n  k  nghĩa ta phải chứng minh:   k 1 y   sin  x   k  1  2  2 Thật vậy: Vế trái    y k 1             y   sin  x  k    cos  x  k   sin  x   k  1   vế trái    2 2     k Trang Do   ln đúng, nghĩa 1 với n = k +1   Bước 3: theo nguyên lí quy nạp suy y n  sin  x  n  , n  * 2   Chọn C Bài tập tự luyện Câu Cho hàm số y  sin x Vi phân hàm số là: A dy   sin 2xdx B dy  sin 2xdx Câu Tìm đạo hàm cấp n hàm số y  n! A y n  C y n C dy  sin xdx n  * x3   , n  * n 1 B y n   1 , n  * D y n   1  x  3  n  1!   x  3 n 1 D dy  2cosxdx n! n  x  3 n 1 n  x  3 n 1 , n  * , n  * Câu Cho hàm số y  cos2 3x Tính giá trị biểu thức 18  2y  1  y" A B C D Đáp án: 1–B 2–B 3–A Dạng 2: Vi phân, đạo hàm cấp cao Ví dụ minh họa Ví dụ 1: Cho f  x   2x3  x  3,g  x   x3  x2  Tập nghiệm bất phương trình f   x   g  x  là: A  ;   1;   B  ;   1;   C  ;1   2;   D  0;1 Hướng dẫn  x2  f   x   2x  x   6x  2x,g  x    x     3x  x      f   x   g  x   6x  2x>3x  x  3x  3x   x   ;   1;    Chọn A Ví dụ 2: Cho f  x   3x  A 60 64   Tổng nghiệm phương trình f '  x   là: x x3 B C D Hướng dẫn Trang   60 64 60 192 Ta có : f   x    3x    5    x x x x   f  x    60 192   1 Đặt t   t   , ta có: x x x  t  1  192t  60t      thỏa mãn  t   16 Với t  1   x   x  2 4 x Với t  1   x  16  x  4 16 16 x f '  x   có nghiệm x1,2  2, x3,4  4 Do tổng nghiệm phương trình  Chọn A Ví dụ 3: Tìm m để hàm số y   m  1 x3   m   x   m   x  coù y '  0,  x   A m  B m  C m  D m  Hướng dẫn Ta có : y   m  1 x   m   x   m    Do đó: y    m  1 x   m   x   m    1 Với m = 1  6x    x  1 nên m = (loại) a  m   m  Với m  1 với x       voâ nghieäm     m  m    Vậy khơng có giá trị m thỏa mãn  Chọn D  Ví dụ 4: Cho khai triển sau  x  x  x3  10  a0  a1x   a30 x30 Giá trị tổng S  a1  2a2   30a30 là: A 5.210 C 410 B D 210 Hướng dẫn  Ta có :   x  x  x3   10   a  a x   a x30  30      1  x  x   1  2x  3x   a  2a x   30a  10  x  x  x3  10  x  x  x3 9    x3  a1  2a2 x   30a30 x 29 2 30 x 29 Trang 10 Chọn x   10 1    1  a1  2a2 x   30a30  S   Chọn B Ví dụ 5: Một chất điểm chuyển động thẳng theo phương trình S  t  3t  4t, t tính giây (s) S tính mét (m) Gia tốc chất điểm lúc t = 2s bằng: A 4m / s2 B 6m / s2 C 8m / s2 D 12m / s2 Hướng dẫn    Vận tốc chất điểm lúc t là: v  t   S  t  3t  4t  3t  6t     Gia tốc chất điểm lúc t là: a  t   v  3t  6t   6t  Do a    6.2   6m / s2  Chọn B Bài tập tự luyện Câu Cho hàm số y  x3  3x  9x  Phương trình y '  có nghiệm là: A 1;2 B 1;3 Câu Tìm m để hàm số y  D 1;2 mx3  mx   3m  1 x  có y  0, x   B m  A m  C 0; 4 C m  D m  Đáp án: 1–B 2–C PHẦN 3: BÀI TẬP TỔNG HỢP Câu Đạo hàm hàm số sau y  sin3  2x  1 là: A y  6sin  2x  1 cos  2x  1 B y  3sin  2x  1 cos  2x  1 C y  3cos2  2x  1 cos  2x  1 D y  3sin  2x  1 t  3t , t tính giây (s) S tính mét (m) Vận tốc chuyển động thời điểm t = 4s bằng: Câu 2.Cho chuyển động thẳng xác định phương trình S  A 280m/s Câu Cho hàm số y   A y    6 B 232m/s  C 140m/s  D 116m/s  cosx Giá tri y   bằng:  s inx 6  B y    1 6  C y    6  D y    2 6 Trang 11 Câu Cho hàm số y  f  x   cos2 x với f  x  hàm liên tục  Trong bốn biểu thức đây, biểu thức xác định hàm f  x  thỏa mãn y '  với x   ? A x  cos2x B x  cos2x C x  sin 2x D x  sin 2x Câu Đạo hàm hàm số y  x  2x  1 3x   ax3  bx  cx  d Tính tổng a  b  c  d A 18 B 30 C –30 D –24 Câu Cho chuyển động thẳng xác định phương trình S  t  3t  9t  27, t tính giây (s) S tính mét (m) Gia tốc chuyển động thời điểm vận tốc triệt tiêu là: A m / s2 B m / s2 C 24 m / s2 D 12 m / s2 Câu Tính đạo hàm cấp n hàm số y  cos2x là: n   n A y    1 cos  2x  n  2    n B y   2n cos  2x   2    n C y   2n 1 cos  2x  n  2    n D y   2n cos  2x  n  2  Câu Cho hàm số: y  x3 Tính giá trị biểu thức  y '    y  1 y"? x4 A B  x  4 C D 14  x  4 Đáp án: 1–A 2–D 3–C 4–A 5–A 6–D 7–D 8–A Trang 12 CHƯƠNG DÃY SỐ, GIỚI HẠN, ĐẠO HÀM CHUYÊN ĐỀ PHƯƠNG TRÌNH TIẾP TUYẾN PHẦN 1: LÝ THUYẾT TRỌNG TÂM Phương trình tiếp tuyến  C  : y  f  x  điểm M  x ; y  có dạng: y  f   x  x  x   y Trong k  f   x  gọi hệ số góc phương trình tiếp tuyến f  x   g  x  Điều kiện cần đủ để hai đường  C1  : y  f  x   C2  : y  g  x  tiếp xúc hệ  f   x   g  x  có nghiệm PHẦN 2: CÁC DẠNG BÀI TẬP Dạng 1: Viết phương trình tiếp tuyến biết tọa độ tiếp điểm Phương pháp giải Viết phương trình tiếp tuyến  C  : y  f  x  điểm M  x ; y  Bước 1: Tìm tập xác định Tính f   x  Bước 2: Tính k  f   x  Bước 3: Lập phương trình tiếp tuyến y  f   x  x  x   y Chú ý: Nếu đề cho hoành độ x ta tính y  f  x  Nếu đề cho tung độ y giải phương trình y  f  x  , tìm x Nếu đề yêu cầu viết phương trình tiếp tuyến giao điểm đồ thị với trục tung cho x  Nếu đề yêu cầu viết phương trình tiếp tuyến giao điểm đồ thị với trục hồnh cho y  Ví dụ minh họa Ví dụ 1: Cho đường cong y  A y  2x x2  x   C  Viết phương trình tiếp tuyến  C  điểm M  2;  x 1 B y  x  C y  3x  10 D y   x  Hướng dẫn Tập xác định: D   \ 1  2x  1 x  1   x  x   2x  3x   x  x  x  2x  f x    2  x  1  x  1  x  1 Ta có: x   f   x   f     1 Phương trình tiếp tuyến  C  điểm M  2;  y  1 x     y   x   Chọn D Trang Ví dụ 2: Phương trình tiếp tuyến đồ thị hàm số y   x  1  x   điểm có hồnh độ x  là: A y  8x  B y  9x  18 C y  4x  D y  9x  18 Hướng dẫn Tập xác định: D   Ta có: y   x  1  x    x  3x   y  3x  Gọi M  x ; y  tọa độ tiếp điểm Ta có: x   y  0, f   x   f     Vậy phương trình tiếp tuyến là: y  9(x  2)   y  9x  18  Chọn D Ví dụ 3: Tiếp tuyến đồ thi hàm số y  x  3x  giao điểm đồ thị hàm số với trục tung có 2x  phương trình là: A y  x  C y  x B y  x  D y   x Hướng dẫn  2x  3 2x  1   x  3x  1 2x  2x  1  Tập xác định: D   \   Ta có: y   2 2  2x  1  2x  1 Giao điểm M đồ thị với trục tung có hồnh độ là: x   y  1 Hệ số góc tiếp tuyến M là: k  y    Phương trình tiếp tuyến điểm M là: y  k  x  x   y  y  x   Chọn A Ví dụ 4: Cho hàm số y  2x  có đồ thị  H  Phương trình tiếp tuyến giao điểm  H  với x 3 trục hoành là: A y  2x  B y  3x  C y  2x  D y  2x Hướng dẫn Tập xác định: D   \ 3 Ta có: y  2  x  3 Tung độ giao điểm  H  với trục hoành y   2x    x0  x0  Ta có: f   x   f     2 Phương trình tiếp tuyến cần tìm là: y  2  x   hay y  2x   Chọn C Ví dụ 5: Cho hàm số y  3x  1 Tính diện tích tam giác tạo trục tọa độ tiếp tuyến x 1 đồ thị hàm số 1 điểm M  2;5  ? Trang A 81 B 81 C 81 D 18 Hướng dẫn Tập xác định: D   \ 1 Ta có: y   x  1  y  2   Phương trình tiếp tuyến  d  điểm M  2;5  : y   x     y  2x  9   Gọi A giao điểm  d  trục hoành y A   x A   , nên A   ;0    Gọi B giao điểm  d  trục tung x B   y B  , nên A  0;9  1 9 81 9 Ta có tam giác OAB vng O nên SOAB  OA.OB  2  Chọn A Bài tập tự luyện Câu Cho đường cong  C  : y  f  x   x  3x Viết phương trình tiếp tuyến  C  điểm M  l; 2  A y  3x B y  3x  C y  2x  D y  3x  Câu Cho hàm số  C  : y  l  x  x Tìm phương trình tiếp tuyến với đồ thị  C  điểm có hồnh độ x  A y  2x  B y  x  C y  x  D y  2x  Câu Cho đường cong  C  : y  f  x   x  3x Viết phương trình tiếp tuyến  C  giao điểm C với trục hoành A y  x  1, y  x  B y  x, y  9x  27 C y  0, y  9x  27 D y  0, y  9x  27 Đáp án: 1–B 2–A 3–C Dạng 2: Viết phương trình tiếp tuyến biết hệ số góc k cho trước Phương pháp giải Bước 1: Tìm tập xác định Tính f   x  Gọi x hoành độ tiếp điểm Bước 2: Do phương trình tiếp tuyến có hệ số góc k, giải phương trình k  y  x  tìm x Bước 3: Tính y  f  x  Bước 4: Lập phương trình tiếp tuyến y  f   x  x  x   y Chú ý: Trang Hệ số góc k  y  x  tiếp tuyến thường cho gián tiếp sau: Tiếp tuyến  //d : y  ax  b  k  a Tiếp tuyến   d : y  ax  b  k   a Tiếp tuyến  tạo với trục hồnh góc   k  tan  Tiếp tuyến  cắt trục Ox, Oy A B  k  Tiếp tuyến  tạo với d: y  ax  b góc   OB OA k a  tan   k.a Ví dụ minh họa Ví dụ 1: Cho đường cong y  x2  x   C  Viết phương trình tiếp tuyến  C  biết tiếp tuyến có hệ x 1 số góc k  A y  x  B y  x  10 C Không tồn tiếp tuyến D y  x  Hướng dẫn Tập xác định: D   \ 1 Ta có: f '  x   x  2x   x  1 Gọi x hoành độ tiếp điểm tiếp tuyến với đồ thị Vì tiếp tuyến có hệ sơ góc k  nên f   x    x  2x   x  1   1  (vơ lý) Vậy khơng có tiếp tuyến có hệ số góc  Chọn C Ví dụ 2: Cho đường cong  C  : y  3x  Viết phương trình tiếp tuyến  C  biết tiếp tuyến song 1 x song với đường thẳng  d  : x  4y  21  A y  21 x 4 C y  x  21 B y  21 x ,y  x 4 4 D y  x 4 Hướng dẫn Tập xác định: D   \ 1 Ta có: y  f '  x    d  : x  4y  21   y  1  x  21 x có hệ số góc a  4 Trang Vì tiếp tuyến song song với d nên k  a  Gọi M  x , y  tọa độ tiếp điểm tiếp tuyến Ta có: f '  x   k  1  x  x0    x  1  16   (thỏa mãn điều kiện)  x  3  1 21  x  5   y  x  (loại, trùng với d) 4 Với x   y  4 , phương trình tiếp tuyến là: y  Với x  3  y  2 , phương trình tiếp tuyến là: y  1  x  3   y  x  4  Chọn D Ví dụ 3: Trong tiếp tuyến điểm đồ thị hàm số y  x  3x  , tiếp tuyến có hệ số góc nhỏ bằng: A –3 B C D Hướng dẫn Tập xác định: D   Đạo hàm: y  3x  6x   x  1   3 Vậy tiếp tuyến điểm đồ thị hàm số cho, tiếp tuyến có hệ số góc nhỏ –3  Chọn A Ví dụ 4: Cho đường cong  C  : y  3x  Viết phương trình tiếp tuyến  C  biết tiếp tuyến vng 1 x góc với đường thẳng    : 2x  2y   A y  x  3, y  x  B y  x  8, y  x  C y  x  3, y  x D y  x  8, y  x Hướng dẫn Tập xác định: D   \ 1 Ta có: y  f '  x      : 2x  2y    y   x  1  x   k   1 Vì tiếp tuyến vng góc với  nên k tt k   1  k tt  Gọi N  x , y  tọa độ tiếp điểm tiếp tuyến, ta có f '  x   k tt  1  x     x  1   x   x  1 2 Với x   y  5 , phương trình tiếp tuyến là: y   x  3   y  x  Với x  1  y  1 , phương trình tiếp tuyến là: y   x  1   y  x  Chọn D Trang Ví dụ 5: Cho hàm số y  3x   C  Viết phương trình tiếp tuyến đồ thị  C  biết tiếp tuyến tạo với đường thẳng  d  :  x  3y   góc 300 A y  3x  C y  14 10 , y  3x  3 B y  3x  2, y  14 x  2, y  3x  3 x  x D y  3x  , y  Hướng dẫn Tập xác định: D   Ta có: y  3x d : 3y  x    y  3 x   kd  3 Vì tiếp tuyến tạo với đường thẳng d góc 300 nên thỏa mãn: k tt  k d  tan 300  k tt k d 2    k    1  k   k  3k   k   k   tt   tt  tt tt tt tt    3   1 k tt k tt  Gọi x hoành độ tiếp điểm Với k tt   3x 02   x   y  Phương trình tiếp tuyến điểm  0;  : y  Với k tt   3x 02   x 02  Với x  1  x0   3 13  13 10   y  , phương trình tiếp tuyến y   x    y  3x   3 3  Với x   11  11 14   y  , phương trình tiếp tuyến y   x    y  3x   3 3   Chọn A Ví dụ 6: Viết phương trình tiếp tuyến d đồ thị  C  : y  2x  , biết tiếp tuyến cắt trục Ox, Oy x 1 A B cho AB  82.OB 25 13 A  : y   x  ,  : y   x  9 9 20 11 B  : y   x  ,  : y   x  9 4 19 C  : y   x  ,  : y   x  9 2 3 D  : y   x  ,  : y   x  8 Hướng dẫn Tập xác định: D   \ 1 Ta có: f '  x   1  x  1 Trang Tiếp tuyến  cắt trục Ox, Oy A, B:  OAB vuông O OB tạo với trục Ox góc  với k  tan   OA AB  82.OB OB Ta có:   81.OB2  OA   2 OA OA  OB  AB Hệ số góc tiếp tuyến tính  k  OB k  tan     OA k    Với k  1 : phương trình vơ nghiệm   x  12 x0  1 Với k      x  1    (thỏa mãn điều kiện)  x  1  x  2 25 13 Vậy phương trình tiếp tuyến  : y   x   : y   x  9 9  Chọn A Bài tập tự luyện Câu Cho hàm số y  x  3x  9x   C  Trong tất tiếp tuyến đồ thị  C  , tìm tiếp tuyến có hệ số góc nhỏ A y  14x  B y  18x  C y  2x  D y  12x  x2 1 Viết phương trình tiếp tuyến đồ thị hàm số 1 , biết tiếp tuyến 2x  cắt trục hoành, trục tung hai điểm phân biệt A, B tam giác OAB cân gốc tọa độ O Câu Cho hàm số y  A y  2x B y   x  C y  3x  D y   x, y   x  Câu Viết phương trình tiếp tuyến đồ thị  C  : y   x  x  , biết tiếp tuyến vuông góc với đường thẳng d: y  x  A  : y  6x  10 B  : y  6x  C  : y  6x  12 D  : y  6x  Đáp án: 1–D 2–B 3–A Dạng 3: Viết phương trình tiếp tuyến qua điểm A cho trước Phương pháp giải Bước 1: Gọi M  x ; y  tiếp điểm Tính y  f  x  k  y  x  theo x Bước 2: Viết phương trình tiếp tuyến  M  x ; y   : y  k  x  x   y Bước 3: Do A(x A ; y A )    y A  k(x A  x )  y Giải phương trình x Trang Bước 4: Tính y , k  f   x  Lập phương trình tiếp tuyến y  f   x  x  x   y Ví dụ minh họa Ví dụ 1: Viết phương trình tiếp tuyến đồ thị  C  : y  x  3x biết qua điểm A  1; 4  A y  4, y  x  B y  x  3, y  3x  C y  3x  1, y  9x  D y  4, y  9x  Hướng dẫn Ta có: f   x   3x  6x Gọi  x ; y  tọa độ tiếp điểm phương trình tiếp tuyến d qua điểm A Vì điểm  x ; y    C   y  x 30  3x 02 , f   x   3x 02  6x Phương trình d: y  f   x  x  x   y  y   3x 02  6x   x  x   x 30  3x 02 Vì A  1; 4   d nên: (3x 02  6x )(1  x )  x 30  3x 02  4  2x 30  6x    x   x  1 Với x   y  4, f     , phương trình tiếp tuyến y  4 Với x  1  y  4, f   1  , phương trình tiếp tuyến y   x  1   y  9x   Chọn D Ví dụ 2: Cho hàm số y  x  3mx   m  1 x  , m tham số thực Tìm giá trị m để tiếp tuyến đồ thị hàm số điểm có hồnh độ x  1 qua điểm A 1;  ? A m  C m  B m  D m  2 Hướng dẫn Tập xác định: D   Ta có: f   x   3x  6mx  m  Với x  1  y  2m  1, f   1  5m  Phương trình tiếp tuyến điểm M  1; 2m  1 : y   5m   x  1  2m   d  Ta có A(1; 2)  (d)  (5m  4).2  2m    m   Chọn A Ví dụ 3: Cho hàm số y   x   x  1  C  Tìm điểm M thuộc đường thẳng d: y  2x  19 , biết tiếp tuyến đồ thị  C  qua điểm M vng góc với đường thẳng x  9y    207  A M  3;13 , M  ;   11 11  1  B M  ;18  , M 1;17  2  1  C M  3;13 , M  ;18  2   207  D M 1;17  , M  ;   11 11  Trang Hướng dẫn Vì tiếp tuyến vng góc với đường thẳng x  9y    y   x  9  nên k tt k   1  k tt  Gọi tọa độ tiếp điểm tiếp tuyến I  x ; y  y(x )  k tt  x 02    x   x  2 Với x   y  , phương trình tiếp tuyến d1 : y  y   x     d1 : y  9x  14  y  9x  14 Suy M giao điểm d d1 tọa độ điểm M nghiệm hệ   M  3;13  y  2x  19 Với x  2  y  phương trình tiếp tuyến d : y  9x  18  y  9x  18  207  Suy M giao điểm d d tọa độ điểm M nghiệm hệ   M ;   11 11   y  2x  19  207  Vậy tọa độ điểm M cần tìm M  3;13 M  ;   11 11   Chọn A Bài tập tự luyện Câu Cho đồ thị hàm số y  4x  6x  Viết phương trình tiếp tuyến đồ thị hàm số, biết tiếp tuyến qua điểm M  1; 9  A y  x  y  4x  C y  x  y  B y  24x  15 y  4x  15 21 x 4 D y  24x  15 y  Câu Tìm m để tiếp tuyến đồ thị hàm số y   2m  1 x  m  15 21 x 4 điểm có hồnh độ x  1 vng góc với đường thẳng d : 2x  y   A B C 16 D 16 Đáp án: 1–D 2–D PHẦN 3: BÀI TẬP TỔNG HỢP Câu Cho hàm số y  x  3x  7x  Viết phương trình tiếp tuyến đồ thị hàm số A  0;  A y  7x  B y  7x  C y  7x  D y  7x  Câu Phương trình tiếp tuyến đồ thị hàm số y  x   x  điểm có hồnh độ x  A y  3x  B y  3x  C y  3x  D y  3x  Câu Tìm hệ số góc tiếp tuyến với đồ thị y  tan x điểm có hoành độ x   Trang A k  B k  Câu Cho đồ thị  C  : y  C k  D k  x  2x  Viết phương trình tiếp tuyến  C  giao điểm  C  4 với Ox A y  15x  45, y  15x  45 B y  4x  12, y  4x  12 C y  3x  15, y  3x  15 D y  10x  30, y  10x  30 ax  b có đồ thị cắt trục tung A  0; 1 , tiếp tuyến A có hệ số góc k  3 x 1 Các giá trị a b là: Câu Cho hàm số y  A a  1, b  B a  2, b  C a  1, b  D a  2, b  Câu Cho hàm số y   x  3x  9x   C  Trong tất tiếp tuyến đồ thị  C  , tiếp tuyến có hệ số góc lớn là: A y  12x  B y  10x  Câu Cho đồ thị  Cm  : y   3m  1 x  m xm C y  20x  D y  15x  20 tiếp tuyến giao điểm  Cm  với Ox song song với đường thẳng d: y   x  A m    m  2 B m    m  2 C m    m  3 1 D m    m   2x  có tung độ Tiếp tuyến  C  M cắt trục tọa độ Ox, x 1 Oy A B Tính SOAB Câu Gọi M   C  : y  A 121 B 121 C 121 D 121 Đáp án: 1–C 2–A 3–D 4–A 5–B 6–A 7–D 8–D Trang 10 CHƯƠNG 1: ỨNG DỤNG CỦA ĐẠO HÀM ĐỂ KHẢO SÁT VÀ VẼ ĐỒ THỊ HÀM SỐ CHUYÊN ĐỀ: TÍNH ĐƠN ĐIỆU CỦA HÀM SỐ PHẦN 1: LÝ THUYẾT TRỌNG TÂM Tính đơn điệu hàm số Định nghĩa: Đồng biến Giả sử hàm số y = f ( x ) xác định I, với I khoảng, đoạn nửa khoảng Hàm số y = f ( x ) gọi đồng biến I nếu: ∀x1 , x ∈ I : x1 < x ⇔ f ( x1 ) < f ( x ) Hàm số y = f ( x ) gọi nghịch biến I nếu: ∀x1 , x ∈ I : x1 < x ⇔ f ( x1 ) > f ( x ) Hàm số y = f ( x ) = x có f ′ ( x ) = > , ∀x ∈ ℝ Hàm số đồng biến nghịch biến gọi hàm số f ( x ) đồng biến ℝ chung hàm số đơn điệu I Điều kiện cần để hàm số đơn điệu Nghịch biến Giả sử hàm số y = f ( x ) có đạo hàm I Khi đó: Nếu hàm số y = f ( x ) đồng biến I f ′ ( x ) ≥ , ∀x ∈ I Nếu hàm số y = f ( x ) nghịch biến I f ′ ( x ) ≤ , ∀x ∈ I Hàm số y = f ( x ) = − x có f ′ ( x ) = −1 < , ∀x ∈ ℝ hàm số f ( x ) nghịch biến ℝ Điều kiện đủ để hàm số đơn điệu Giả sử hàm số y = f ( x ) có đạo hàm khoảng I Xét hàm số: y = f ( x ) = x + x Khi đó: y = g ( x ) = −2x + y = h (x) = − Các hàm số có đạo hàm ℝ Trang Nếu f ′ ( x ) > , ∀x ∈ I hàm số f ( x ) đồng biến khoảng I Nếu f ′ ( x ) < , ∀x ∈ I hàm số f ( x ) nghịch biến khoảng I Nếu f ′ ( x ) = , ∀x ∈ I hàm số f ( x ) khơng đổi khoảng I Ta có f ′ ( x ) = 3x + > , ∀x ∈ ℝ nên hàm số f ( x ) đồng biến ℝ Ta có g′ ( x ) = −2 < , ∀x ∈ ℝ nên hàm số g ( x ) nghịch biến ℝ Ta có h′ ( x ) = , ∀x ∈ ℝ nên hàm số h ( x ) khơng đổi ℝ Định lí: Giả sử hàm số y = f ( x ) có đạo hàm khoảng I Hàm số y = f ( x ) = ( m − 1) x + x + xác định ℝ Hàm số có f ′ ( x ) = ( m − 1) x + Hàm số y = f ( x ) đồng biến I f ′ ( x ) ≥ , ∀x ∈ I f ′ ( x ) = xảy hữu hạn điểm t Hàm số y = f ( x ) nghịch biến I f ′ ( x ) ≤ , ∀x ∈ I f ′ ( x ) = xảy hữu hạn điểm t Chú ý: Ta thay khoảng I thành đoạn nửa khoảng, ta cần bổ sung thêm giả thiết: “Hàm số liên tục đoạn nửa khoảng đó” Hàm số y = f ( x ) đồng biến ℝ f ′ ( x ) = ( m − 1) x + ≥ , ∀x ∈ ℝ Hàm số y = f ( x ) nghịch biến ℝ f ′ ( x ) = ( m − 1) x + ≤ , ∀x ∈ ℝ Một số cơng thức tính đạo hàm ( u ± v )′ = u′ ± v′ ( x )′ = nx ( ku )′ = ku′ ( x )′ = x ( uv )′ = u′v + uv′  u ′ u ′v − uv′   = v2 v n n −1 ad − bc  ax + b ′   =  cx + d  ( cx + d ) PHẦN 2: CÁC DẠNG BÀI TẬP Dạng 1: Xét tính đơn điệu hàm số Phương pháp giải Xét hàm số y = f ( x ) = x − 3x + Bước 1: Tìm tập xác định D Bước 2: Tìm f ′ ( x ) Tìm điểm x i mà f ′ ( x i ) = f ′ ( x i ) không xác định Tập xác định D = ℝ Ta có: f ′ ( x ) = 3x − x = f ′ ( x ) = ⇔ 3x − = ⇔   x = −1 Trang Bước 3: Lập bảng biến thiên Bảng biến thiên Thay x = −2 ∈ ( −∞; −1) ⇒ f ′(x) = > nên ⇒ f ′ ( x ) = −3 < f ′ ( x ) có dấu + −∞ x −1 f ′( x) x = ∈ ( −1;1) Thay + − nên f ′ ( x ) có dấu - +∞ + f (x) Dấu +, mũi tên Dấu − , mũi tên lên, hàm số xuống, hàm đồng biến số nghịch biến Bước 4: Kết luận khoảng đồng biến, nghịch biến Kết luận: hàm số Hàm số đồng biến khoảng ( −∞; −1) (1; +∞ ) Hàm số nghịch biến khoảng ( −1;1) Ví dụ minh họa Ví dụ 1: Cho hàm số y = x − 2x + Mệnh đề đúng? A Hàm số đồng biến khoảng ( −∞; −1) B Hàm số nghịch biến khoảng ( −∞; −1) (1; +∞ ) C Hàm số nghịch biến khoảng ( −∞; −1) ( 0;1) D Hàm số đồng biến khoảng ( −1;1) Hướng dẫn Cách 1: Hàm số có tập xác định: D = ℝ x = Ta có y′ = 4x − 4x y′ = ⇔   x = ±1 Bảng biến thiên: x f ′( x) −1 −∞ − + − +∞ + f (x) Trang Vậy: Hàm số đồng biến khoảng ( −1;0 ) (1; +∞ ) Hàm số nghịch biến khoảng ( −∞; −1) ( 0;1) Cách 2: Sử dụng máy tính CASIO fx 570 VNPLUS Nhập MODE 7, nhập f ( X ) = X − 2X + Start ? −5 → End? → Step? Khi ta nhận bảng giá trị: X f (X) X f (X) −5 579 −4 228 -3 −3 67 12 −2 12 67 −1 −3 228 579 Nhìn vào bảng giá trị, ta thấy hàm số nghịch biến khoảng ( −∞; −1) ( 0;1) → Chọn C Ví dụ 2: Cho hàm số y = f ( x ) có bảng xét dấu đạo hàm hình vẽ sau: x −2 f ′( x) + − +∞ − Mệnh đề sau đúng? A Hàm số nghịch biến khoảng (1; +∞ ) B Hàm số nghịch biến khoảng ( 5; +∞ ) C Hàm số đồng biến khoảng ( −2; ) D Hàm số đồng biến khoảng ( −2;5 ) Hướng dẫn Nhìn vào bảng xét dấu đạo hàm, ta thấy hàm số đồng biến khoảng ( −2;1) , nghịch biến khoảng (1;5 ) ( 5; +∞ ) Khi hàm số nghịch biến khoảng ( 5; +∞ ) → Chọn B Ví dụ 3: Hàm số y = − x + 2x − đồng biến trên: x−2 Trang A ( 0; ) B ( 0; ) ( 4; +∞ ) ( 2; ) C ( −∞;0 ) ( 4; +∞ ) D ( −∞; ) ( 2; ) Hướng dẫn Tập xác định D = ℝ \ {2} Ta có y′ = ( −2x + )( x − ) − ( x + 2x − ) − x + 4x = 2 ( x − 2) ( x − 2) x = y′ = ⇔ − x + 4x = ⇔  x = Bảng biến thiên: x −∞ f ′(x) − + + +∞ − f (x) Vậy hàm số đồng biến ( 0; ) ( 2; ) → Chọn A Bài tập tự luyện Câu Cho hàm số y = x +1 Phát biểu sau đúng? 1− x A Hàm số đồng biến khoảng ( −∞;1) B Hàm số đồng biến khoảng ( −∞;1) (1; +∞ ) C Hàm số đồng biến khoảng ( −∞;1) ∩ (1; +∞ ) D Hàm số đồng biến khoảng ℝ Câu Cho hàm số y = x + Kết luận sau đúng? x A Hàm số đồng biến khoảng ( −∞; ) B Hàm số đồng biến khoảng ( 2; +∞ ) C Hàm số đồng biến khoảng ( −2; ) D Hàm số nghịch biến khoảng ( −2; ) Đáp án 1–B 2–B Dạng 2: Điều kiện tham số để hàm số đơn điệu Phương pháp giải Hàm số y = ax + bx + cx + d Tập xác định: D = ℝ y′ = 3ax + 2bx + c Xét hàm số y = mx + x + Tập xác định D = ℝ y′ = 3mx + Để hàm số đồng biến ℝ ⇔ y′ ≥ , ∀x ∈ ℝ Trang + Nếu m = y′ = > (thỏa mãn) Để hàm số đồng biến ℝ thì: y′ ≥ , ∀x ∈ ℝ + Nếu m ≠ : a > Khi đó:  ∆ ≤ 3m > y ′ ≥ , ∀x ∈ ℝ ⇔  ⇔ m > ∆ = −12m ≤ Để hàm số nghịch biến ℝ thì: Vậy m ≥ hàm số đồng biến ℝ y ′ ≤ , ∀x ∈ ℝ Hàm số nghịch biến ℝ y′ ≤ , ∀x ∈ ℝ ⇔ 3mx + ≤ 0, ∀x ∈ ℝ a < Khi đó:  ∆ ≤ Hàm số y = + Nếu m = y′ = > (loại) ax + b cx + d + Nếu m ≠ : 3m < ⇔ y′ ≤ , ∀x ∈ ℝ ⇔  ⇔ m ∈∅ ∆ = −12m ≤  d Tập xác định: D = ℝ \ −   c y′ = Vậy khơng có giá trị m để hàm số nghịch biến ad − bc ( cx + d ) Xét hàm số y = x+m x −1 Hàm số đồng biến khoảng xác định Tập xác định: D = ℝ \ {1} ; khi: −1 − m y′ = y′ > , ∀x ∈ D ⇒ ad − bc > ( x − 1) Hàm số đồng biến khoảng xác định Hàm số nghịch biến khoảng xác định khi y′ > , ∀x ∈ D khi: −1 − m ⇔ > , ∀x ∈ D y′ < , ∀x ∈ D ⇒ ad − bc < ( x − 1) ⇔ −1 − m > ⇔ m < −1 Hàm số nghịch biến khoảng xác định ⇔ y ′ < , ∀x ∈ D ⇔ −1 − m ( x − 1) < , ∀x ∈ D ⇔ −1 − m < ⇔ m > −1 Ví dụ minh họa Ví dụ 1: Tìm điều kiện tham số m cho hàm số y = A −1 < m < B −1 < m ≤ x3 + mx − mx − m đồng biến ℝ ? C −1 ≤ m ≤ D −1 ≤ m < Hướng dẫn Tập xác định: D = ℝ Ta có y′ = x + 2mx − m Hàm số đồng biến ℝ khi: Trang 1 > ⇔ −1 ≤ m ≤ y′ ≥ , ∀x ∈ ℝ ⇔  ∆ = 4m + 4m ≤ → Chọn C Ví dụ 2: Giá trị tham số m để hàm số y = A m < x−m nghịch biến khoảng xác định là: x−2 B m ≥ C m > D m ≤ Hướng dẫn Tập xác định: D = ℝ \ {2} Ta có y′ = −2 + m ( x − 2) Hàm số nghịch biến khoảng xác định khi: y ′ < , ∀x ∈ D ⇔ −2 + m ( x − 2) < , ∀x ∈ D ⇒ −2 + m < ⇔ m < → Chọn A Ví dụ 3: Tìm tất giá trị thực tham số m cho hàm số y = mx + giảm khoảng xác x+m định A −2 < m ≤ B −2 ≤ m ≤ −1 C −2 ≤ m ≤ D −2 < m < Hướng dẫn Tập xác định: D = ℝ \ {− m} Ta có y′ = m −4 ( x + m) Hàm số giảm khoảng xác định hàm số nghịch biến khoảng xác định Khi đó: y ′ < , ∀x ∈ D ⇔ m − < ⇔ − < m < → Chọn D Bài tập tự luyện Câu Tìm tất giá trị thực tham số m cho hàm số y = x−m+2 giảm khoảng mà x +1 xác định A m < −3 B m ≤ −3 C m ≤ D m < 1 Câu Tìm tất giá trị thực tham số m cho hàm số y = − x − mx + ( 2m − 3) x − m + nghịch biến ℝ A −3 ≤ m ≤ B m ≤ C −3 < m < D m ≤ −3; m ≥ Câu Tìm giá trị thực tham số m cho hàm số y = 2x − ( m + ) x + ( m + 1) x − 3m + đồng biến ℝ A B -1 C D Trang Đáp án 1–D 2–A 3–A PHẦN 3: BÀI TẬP TỔNG HỢP Câu Cho hàm số y = − x + 3x − 3x + Khẳng định sau khẳng định đúng? A Hàm số nghịch biến ℝ B Hàm số nghịch biến khoảng ( −∞;1) (1; +∞ ) C Hàm số đồng biến khoảng ( −∞;1) nghịch biến khoảng (1; +∞ ) D Hàm số đồng biến ℝ ( m + 3) x − Câu Tìm số nguyên m nhỏ cho hàm số y = x+m nghịch biến khoảng xác định nó? A m = −1 B m = −2 C m = D Khơng có m Câu Hàm số y = − x + 4x + 20 đồng biến khoảng nào? ( A −∞; − ) ( )( B −∞; − ; − 2;0 ) ( )( C − 2;0 ; 2; +∞ ) ( )( D − 2; ; 0; ) Câu Hỏi hàm số y = x − 3x + 4x − đồng biến khoảng nào? A ( −∞; ) B ℝ C ( 0; ) D ( 2; +∞ ) Câu Tìm điều kiện tham số m để hàm số y = − x + ( − m ) x − 2mx + nghịch biến tập xác định 10 − 19 10 + 19 ≤m≤ 3 A −3 < m < B C −3 < m < D m > Câu Giá trị m để hàm số y = x − ( m + 1) x + ( m + 1) x + đồng biến ℝ là: A −1 ≤ m ≤ B −1 < m <  m < −1 C  m >  m ≤ −1 D  m ≥ Câu Cho hàm số y = 3x − x Khẳng định sau sai? A Hàm số đồng biến khoảng ( 0; ) B Hàm số đồng biến khoảng ( −∞; ) ; ( 2;3) C Hàm số nghịch biến khoảng ( −∞;0 ) ; ( 2;3) D Hàm số nghịch biến khoảng ( 2;3) Câu Hàm số y = A ( −∞;1) ; (1; ) x2 đồng biến khoảng nào? x −1 B ( −∞; ) ; ( 2; +∞ ) C ( −∞; −1) ; (1; +∞ ) D ( 0;1) ; (1; ) Trang Câu Tìm tất giá trị tham số m để hàm số y = A −3 < m < B m < −3 Câu 10 Tìm m để hàm số y = A m ≤ mx + nghịch biến khoảng xác định 3x + m C −3 < m < D m > x−m+2 nghịch biến khoảng xác định x +1 B m < C m ≤ D m < Đáp án: 1-A 2-D 3-A 4-B 5-B 6-A 7-B 8-B 9-A 10 - B Trang CHƯƠNG CHUYÊN ĐỀ 2: CỰC TRỊ CỦA HÀM SỐ PHẦN 1: LÝ THUYẾT TRỌNG TÂM Định nghĩa Cho hàm số y  f  x  xác định liên tục D  D    Nếu tồn  a; b   D x   a; b  cho: • f  x   f  x  , x   a; b  \ x  x gọi điểm cực đại hàm số y  f  x  • f  x   f  x  , x   a; b  \ x  x gọi điểm cực tiểu hàm số y  f  x  Chú ý: Hàm số khơng có cực trị, hay nhiều điểm cực trị Các định lí Trang Hàm số f  x   x  3x có f   x   3x  6x Định lí 1: Giả sử hàm số y  f  x  đạt cực trị điểm x Hàm số đạt cực trị điểm x  Khi đó, y  f  x  có đạo hàm x Khi ta có f     f   0  Định lí 2: Giả sử hàm số y  f  x  liên tục  a; b  chứa x có đạo hàm  a; x   x ; b  Khi đó: • f   x   0, x   a; x  ;f   x   0, x   x ; b  x gọi điểm cực đại hàm số • f   x   0, x   a; x  ;f   x   0, x   x ; b  x gọi điểm cực tiểu hàm số Định lí 3: Giả sử hàm số y  f  x  có đạo hàm cấp hai khoảng  a; b  chứa x Hàm số f  x   x  3x Khi đó: • f   x   f   x   x điểm cực tiểu Ta có f   x   3x  6x f   x   6x  hàm số • f   x   f   x   x điểm cực đại hàm số • Vì f     f      nên x  điểm cực tiểu hàm số • Vì f     f     6  nên x  điểm cực đại hàm số PHẦN 2: CÁC CÔNG THỨC TÍNH NHANH Đường thẳng qua điểm cực đại, điểm cực tiểu    Đưa y dạng y  h  x  y  g  x   : y  g x Hàm số y  ax  bx  cx  d (Phần dư phép chia y cho y )  2c 2b  bc  x d 9a  9a   : y  g x      : y  g  x   9ay  y.y Trang Đường thẳng qua điểm cực đại, điểm cực tiểu     : y  g x  y  Hàm số y  u x vx  : y  y.y 3y u  x  (Đạo hàm tử chia đạo hàm mẫu) v  x  Cực trị hàm số bậc ba: y  ax  bx  cx  d Khơng có cực trị  y  Có cực trị (chỉ có cực trị)  y  A, B điểm cực trị đồ thị AB  Đạt cực trị x b  3ac 4e  16e3 với e  9a a Cực đại  y  x     y  x   Cực tiểu  y  x     y  x   Cùng nằm bên phải (cùng dấu dương)   y   ac  ab   Cùng nằm bên trái (cùng dấu âm)   y   ac  ab   y  x   Hai cực trị nằm hai phía trục tung  Oy  ac  (trái dấu) Hai cực trị nằm phía trục tung  Oy  (cùng dấu)   y   ac  Cực trị hàm số bậc bốn trùng phương: y  ax  bx  c có   b  4ac Có cực trị Có cực trị Cực trị cực đại a   b  Cực trị cực tiểu a   b  điểm cực đại, điểm cực tiểu a   b  ab  ab  Trang Cực trị hàm số bậc bốn trùng phương: y  ax  bx  c có   b  4ac điểm cực tiểu, điểm cực đại a   b   b   b  A  0;c  ; B    ;   ;C   ;   2a 4a   2a 4a   A  Oy ; B C đối xứng qua Oy Ba điểm cực trị đồ thị b4 b b  ; BC   16a 2a 2a AB  AC  Đạt cực trị x Cực đại  y  x     y  x   Cực tiểu  y  x     y  x   y  x     Đặt BAC  b3 cot  8a Công thức thỏa mãn ab  Điều kiện Tam giác vuông ABC cân A b3  8a  Tam giác ABC b3  24a  Tam giác ABC có trọng tâm O b  6ac Tam giác ABC có trực tâm O b3  8a  4ac  Tam giác ABC có O tâm đường tròn nội tiếp b3  8a  4abc  Tam giác ABC có O tâm đường tròn ngoại tiếp b3  8a  4abc  Tam giác ABC có bán kính đường tròn ngoại tiếp R R b3  8a 8a b Trang Tam giác ABC có bán kính đường tròn nội nội tiếp r Tam giác ABC có diện tích S Đồ thị hàm số C : r b2  b3  a  1 1 8a     32a 3S2  b5  y  ax  bx  c cắt trục b2  hoành điểm phân biệt tạo thành cấp số cộng 100 ac PHẦN 3: CÁC DẠNG BÀI TẬP Dạng 1: Tìm cực trị hàm số Phương pháp giải Cách 1: Lập bảng biến thiên hàm số để xác định cực trị hàm số Cách 2: Sử dụng máy tính để xác định cực trị hàm số Ví dụ minh họa Ví dụ 1: Cho hàm số y  x  3x  Khẳng định sau đúng? A Hàm số đạt cực đại x  đạt cực tiểu x  B Hàm số đạt cực tiểu x  đạt cực đại x  C Hàm số đạt cực đại x  2 đạt cực tiểu x  D Hàm số đạt cực đại x  đạt cực tiểu x  2 Hướng dẫn x  Cách 1: Hàm số có tập xác định: D   Ta có y  3x  6x nên y    x  Bảng biến thiên x f x  +  – + f x Nhìn vào bảng biến thiên ta thấy hàm số đạt cực đại x  đạt cực tiểu x  Cách 2: Sử dụng máy tính fx 570 VNPLUS Nhập MODE 7, nhập f  X   X  3X  Start? –5  End?  Step? Khi ta nhận bảng giá trị: X f X X f X –5 –198 –4 –110 –3 –52 –2 –2 –18 Trang X f X X f X –1 –2 18 52 Nhìn vào bảng giá trị, ta thấy hàm số đạt cực đại x  đạt cực tiểu x   Chọn B Ví dụ 2: Cho hàm số y  x  2x  Khẳng định sau đúng? A Hàm số có ba điểm cực trị B Hàm số có hai điểm cực trị C Hàm số khơng có cực trị D Hàm số có điểm cực trị Hướng dẫn x  Cách 1: Ta có y  4x  4x    x   x  1 Ta có y  12x   y    4  0; y 1   0; y  1   Nên hàm số đạt cực đại x  đạt cực tiểu x  x  1 Do hàm số có ba điểm cực trị Cách 2: Ta ý, hàm số bậc bốn có cực trị cực trị, nên loại đáp án B C Mà hàm số có ab   2   2  nên có ba cực trị  Chọn A Ví dụ 3: Cho hàm số y  f  x  có đồ thị hình vẽ Đồ thị hàm số y  f  x  có điểm cực trị? A B C D Hướng dẫn Nhìn vào đồ thị hàm số hình vẽ, ta thấy hàm số đạt cực đại x  cực tiểu x  Vậy hàm số có hai cực trị  Chọn A Ví dụ 3: Cho hàm số y  f  x  Hàm số y  f   x  có đồ thị hình vẽ: Trang Khẳng định sau đúng? A Đồ thị hàm số y  f  x  khơng có điểm cực trị B Đồ thị hàm số y  f  x  có hai điểm cực trị C Đồ thị hàm số y  f  x  có ba điểm cực trị D Đồ thị hàm số y  f  x  có điểm cực trị Hướng dẫn Nhìn vào đồ thị hàm số đạo hàm, ta có bảng biên thiên x  f x – f x +  – 0 + Cực đại Cực tiểu Cực tiểu Nhìn vào bảng biến thiên, ta thấy hàm số có ba điểm cực trị  Chọn C Bài tập tự luyện Câu ho hàm số y  x  17x  24x  Kết luận sau đúng? A x CÑ  B x CÑ  C x CÑ  3 D x CÑ  12 Câu Cho hàm số y  3x  6x  Kết luận sau đúng? A y CÑ  2 B y CÑ  C y CÑ  1 D y CÑ  Câu Cho hàm số y  f  x  có bảng biến thiên: Trang  x f x +  – + f x   –2 Khẳng định sau đúng? A Hàm số đạt cực đại x  B Hàm số đạt cực đại x  C Hàm số đạt cực đại x  D Hàm số đạt cực đại x  2 Câu Trong hàm số sau, hàm số đạt cực đại x  A y  x  x3  x  3x B y  C y  4x  12x  ? x 1 x2 D y  x  3x  Đáp án: 1–D 2–B 3–A 4–D Dạng 2: Phương trình đường thẳng qua hai điểm cực đại cực tiểu Phương pháp giải • Hàm số y  ax  bx  cx  d ; g  x   phần dư phép chia y cho y • Hàm số y  u x ; vx g  x   đạo hàm tử : đạo hàm mẫu Ví dụ minh họa Ví dụ 1: Cho hàm số bậc ba y  x  9x  15x  Phương trình đường thẳng qua hai điểm cực trị đồ thị hàm số là: A x  8y  14  B x  8y  14  C 8x  y  14  D 8x  y  14  Hướng dẫn x   y  Cách 1: Hàm số y  x  9x  15x  có y  3x  18x  15     x   y  26 Suy hàm số có hai điểm cực trị đồ thị A 1;6  ; B  5; 26  Đường thẳng qua điểm cực đại cực tiểu đồ thị hàm số đường thẳng có vectơ phương   AB   4; 32  nên có vectơ pháp tuyến n   8;1 Vậy phương trình đường thẳng qua điểm cực trị  x  1  1 y     8x  y  14  Trang Cách 2: Hàm số có a  1; b  9; c  15; d  1 Theo cơng thức giải nhanh, ta có phương trình đường thẳng qua hai điểm cực đại cực tiểu là:  2.15  9 2   2c 2b   9  15 bc y  gx     y   x 1 x d  9a 9.1  9.1  9a    y  8x  14  8x  y  14   Chọn C 2x  x  Ví dụ 2: Phương trình đường thẳng qua ĐCĐ, ĐCT đồ thị hàm số y  x 1 A x  4y   B 4x  y   C y  x  D y  4x  Hướng dẫn Cách 1: Hàm số có dạng y  u x với u  x   2x  x  v  x   x  vx Nên phương trình đường thẳng qua hai điểm cực đại cực tiểu đồ thị hàm số là: y u   x  4x    4x   4x  y   v  x   Chọn B Cách 2: Ta có y   4x  1 x  1   2x  x  1 2x  4x  2  x  1  x  1  x   y  1 Khi đó: y   2x  4x     x  2  y  9 Hàm số có hai điểm cực trị đồ thị là: A  0; 1 ; B  2; 9  Phương trình đường thẳng qua điểm cực đại, cực tiểu đồ thị hàm số có vectơ phương   AB   2; 8  nên có vectơ pháp tuyến n   4; 1 Vậy PT đường thẳng qua ĐCĐ, ĐCT đồ thị là:  x    1 y  1   4x  y    Chọn B Ví dụ 3: Tìm giá trị thực tham số m để đường thẳng  d  : y   3m  1 x   m vng góc với đường thẳng qua hai điểm cực trị đồ thị hàm số y  x  3x  A m  B m  C m  D m  Hướng dẫn Hàm số y  x  3x  có a  1; b  3; c  0; d  Phương trình đường thẳng qua điểm cực đại, cực tiểu đồ thị là:  2.0  32   2c 2b   3  y  2x  bc y  gx     y   x 1 x d  9a 9.1  9.1  9a    d  Trang Đường thẳng  d  vng góc với đường thẳng  d    3m  1  2   1  m   Chọn A Bài tập tự luyện Câu Biết đồ thị hàm số y  x  3x  có hai điểm cực trị A, B Khi phương trình đường thẳng AB là: A y  x  B y  2x  Câu Cho hàm số y  C y  2x  D y   x  3x  13x  19 Đường thẳng qua hai điểm cực trị đồ thị hàm số có x 3 phương trình là: A y  6x  13 B y  3x  13 C 5x  2y  13  D 2x  4y   Đáp án: 1–C 2–A Dạng 3: Cực trị hàm bậc ba y  ax  bx  cx  d Ví dụ minh họa Ví dụ 1: Tìm tất giá trị tham số m để hàm số y  x  mx   2m  3 x  đạt cực đại x  A m  B m  C m  D m  Hướng dẫn Ta có: y  3x  2mx   2m  3 ; y  6x  2m  y 1  3.12  2m.1  2m   Hàm số đạt cực đại x     m   y 1  6.1  2m   Chọn B Ví dụ 2: Cho hàm số y  mx  3x   m  1 x  Xác định m để đồ thị hàm số có hai điểm cực trị nằm bên phải trục Oy A  13  m  B  13  m  C  m   13 D  m   13 Hướng dẫn Ta có y  3mx  6x   m  1 Đồ thị hàm số có hai điểm cực trị nằm bên phải trục Oy Trang 10 1  13  13 m  62  4.3m  m  1   12m  12m  36    y   m     ac   3m  m  1   m  m  1    ab  3m.6  m  m     m      13 m0  Chọn A Ví dụ 3: Tìm tất giá trị thực tham số m để đồ thị hàm số y  x  3mx  3m3 có hai điểm cực trị A B cho tam giác OAB có diện tích 48 A m  2; m  B m  C m  2 D m  2 Hướng dẫn x  Ta có y  3x  6mx  3x  x  2m  nên y     x  2m Đồ thị hàm số có hai điểm cực trị 2m   m  (1) Khi đó, điểm cực trị đồ thị hàm số A  0;3m3  , B  2m; m3   Ta có: OA  0;3m3   OA  m3 (2) Ta thấy A  Oy  OA  Oy  d  B, OA   d  B, Oy   m (3) Từ (2) (3) suy SOAB  OA.d  B, OA   3m Do đó: SOAB  48  3m  48  m  2 (thỏa mãn (1 ))  Chọn D Bài tập tự luyện Câu Khoảng cách hai điểm cực trị đồ thị hàm số y  x  3x là: A B C D Câu Cho hàm số y  x  mx   2m  1 x  Tìm tất giá trị tham số m để hàm số có cực trị A m  B m C m  D m  Câu Tìm tất giá trị thực tham số m để đồ thị hàm số y   x  3mx  có hai điểm cực trị A, B cho tam giác OAB vuông O (với O gốc tọa độ ) A m  B m   C m  1 D m  Trang 11 Câu Tìm tất giá trị thực tham số m để đồ thị hàm số y  x  mx   3m  1 x  có 3 hai điểm cực tị có hồnh độ x1 , x cho x1.x   x1  x   B m   A m  C m  D m   Đáp án: 1–C 2–A 3–D 4–C Dạng 4: Cực trị hàm bậc bốn trùng phương y  ax  bx  c Ví dụ minh họa Ví dụ 1: Hàm số y  x   m   x  m  2m  có điểm cực trị giá trị m là: A m  B m  C m  D m  Hướng dẫn Hàm trùng phương có điểm cực trị ab   m    m   Chọn A Ví dụ 2: Tìm giá trị tham số m để đồ thị hàm số y  x  2m x  có ba điểm cực trị ba đỉnh tam giác vuông cân A m  1 B m  C m  D m  1 Hướng dẫn Đồ thị hàm số có ba điểm cực trị ba đỉnh tam giác vuông cân khi: b3  8a    2m   8.1   8m    m  1  Chọn D Ví dụ 3: Tìm giá trị tham số m để đồ thị hàm số: y  x  2mx  2m  m có ba điểm cực trị ba đỉnh tam giác A Không tồn m m  B  m  C m  3 D m   Hướng dẫn Đồ thị hàm số có ba điểm cực trị ba đỉnh tam giác   2m   24.1   m3   m  3  Chọn C Ví dụ 4: Tìm giá trị tham số m để đồ thị hàm số y  x  8m x  có ba điểm cực trị, đồng thời ba điểm cực trị ba đỉnh tam giác có diện tích 64 A Không tồn m B m  C m   D m   Hàm số có ba điểm cực trị khi: ab   1.(8m )   m  Đồ thị hàm số có ba điểm cực trị ba đỉnh tam giác có diện tích 64 Trang 12 Ta có: SABC  b2 4a  b 64m 8m 64    m   (thỏa mãn) 2a  Chọn D Bài tập tự luyện Câu Tìm giá trị tham số m để đồ thị hàm số: y  x   m  1 x  m có ba điểm cực trị ba đỉnh tam giác vuông cân B m  A Không tồn m m  C   m  1 D m  1 Câu Cho hàm số y  x  2mx  m  Tìm tất giá trị tham số thực m để đồ thị hàm số có ba điểm cực trị tạo thành tam giác nhận gốc tọa độ O làm trực tâm A m  B m  C m  D m  Câu Tìm tất giá trị thực tham số m để hàm số y   m  1 x  mx  có cực tiểu mà khơng có cực đại A m  1 B 1  m  C m  D 1  m  Câu Cho hàm số y  mx   m   x  10 Tìm tất các giá trị tham số m để hàm số có ba điểm cực trị 0  m  A   m  3 B m  3 0  m  D   m  3 C  m  Đáp án: 1–B 2–D 3–B 4–A PHẦN BÀI TẬP TỒNG HỢP Câu Cho hàm số y  f  x  có đạo hàm f   x    x  1 x    x  3  x   Hỏi hàm số y  f  x  có điểm cực trị? A B C D Câu Tìm tất giá trị thực m để hàm số y  x  2x   m  3 x  khơng có cực trị A m   B m   C m   D m   Câu Cho hàm số y  x  mx   m  1 x  Tìm tất giá trị thực tham số m để hàm số đạt cực đại x  2 A Không tồn m B –1 C D Câu Cho hàm số y  x  3mx   m  l  x  m3  m Gọi x1 , x hai điểm cực trị hàm số Tìm tất giá trị thực tham số m để x12  x 22  x1.x  A m   B m  2 C m  D m  1 Trang 13 Câu Cho hàm số y  f  x  Hàm số y  f   x  có đồ thị hình vẽ: Khẳng định sau khẳng định đúng? A Hàm số y  f  x  đạt cực đại x  B Đồ thị hàm số y  f  x  có điểm cực tiểu C Hàm số y  f  x  đồng biến khoảng  ;1 D Đồ thị hàm số y  f  x  có hai điểm cực trị Câu Tìm giá trị tham số m để đồ thị hàm số y  x  2mx  m  có ba điểm cực trị, đồng thời ba điểm cực trị đồ thị ba đỉnh tam giác có bán kính đường tròn ngoại tiếp m  A   m   1   Câu Hàm số y  m  B   m  1   C m   1  D m  2x  có số cực trị : 3x  A B C D Câu Hàm số y  ax  bx  cx  d  a   có số cực trị là: A B C D hoặc Câu Cho hàm số y  x  3x  mx  m  có đồ thị  Cm  Xác định m để  Cm  có điểm cực đại cực tiểu nằm hai phía Ox A m  B m  C m  D m  2 Câu 10 Cho hàm số y   m   x  3x  mx  Tìm giá trị m để hàm số có điểm cực trị có hồnh độ dương A 3  m  2 B m  2 C 3  m  D m  Câu 11 Cho hàm số y  f  x  liên tục  có đạo hàm f   x    x  1 x    x  3 2017 Khẳng định sau đúng? A Hàm số đồng biến khoảng 1;   3;   B Hàm số có ba điểm cực trị C Hàm số nghịch biến khoảng 1;3 D Hàm số đạt cực đại x  , cực tiểu x  x  Đáp án: 1–A 2–C 3–B 4–B 5–B 6–B 7–D 8–B 9–C 10–A 11–C Trang 14 CHƯƠNG 1: ỨNG DỤNG CỦA ĐẠO HÀM ĐỂ KHẢO SÁT VẼ ĐỒ THỊ HÀM SỐ CHUYÊN ĐỀ 3: GIÁ TRỊ LỚN NHẤT, GIÁ TRỊ NHỎ NHẤT CỦA HÀM SỐ PHẦN 1: LÝ THUYẾT TRỌNG TÂM Định nghĩa: Cho hàm số y = f (x) xác định tập M giá trị Cho hàm số y = f (x) xác định tập D m giá lớn hàm số D trị nhỏ hàm số D x  D, f  x   M M  max f  x    xD x  D, f  x   M x  D, f  x   m m  max f  x    xD x  D, f  x   m Chú ý: Trong sách này, ta viết tắt giá trị lớn GTLN, giá trị nhỏ GTNN GTLN lớn GTNN PHẦN 2: CÁC DẠNG BÀI TẬP Dạng 1: Giá trị lớn nhất, nhỏ hàm số đoạn Phương pháp giải Tìm GTLN, GTNN hàm số y = f (x) đoạn Tìm GTLN, GTNN hàm số [a; b] y  f  x   x  3x đoạn [-1; 3] Cách 1: Ta có f  x   3x  6x Bước 1: Tìm điểm xi thuộc (a; b) mà f’(xi) = f’(xi) không xác định  x1    1;3 Ta thấy f   x     Bước 2: Tính f  a  , f  x i  , f  b   x  2   1;3 Bước 3: Tím số lớn M số nhỏ m Ta có a  1; x  0; b  số f (1)  2; f (0)  0; f (3)  54 Ta có M  max f  x  ; m  f  x  x a;b  x a;b  Ta thấy f (3) = 54 lớn nhất, f (0) = nhỏ Cách 2: Sử dụng máy tính fx 570 VNPLUS Vậy m  max f  x   54; f  x   x 1;3 x 1;3 Nhập Mode 7, nhập f (X) = … Nhập MODE 7, nhập f  X   X  3X Start? a = → End? b = → Step? α = (α ta chọn tùy vào đoạn đề bài) Start? – = End? = Step? 0.5 = Ta nhận bảng giá trị X f (X) Từ bảng giá trị f (x), tìm GTLN, GTNN Bảng giá trị X f (X) -1 - 0.5 0.625 0 Trang 0.5 0.875 1.5 10.125 20 2.5 34.375 54 Từ bảng giá trị, ta thấy f  3  54 GTLN, f    GTNN Vậy max f  x   54; f  x   x 1;3 x 1;3 Ví dụ minh họa Ví dụ 1: Giá trị nhỏ hàm số y  x  3x  đoạn [0; 2] là: A y  B y  x 0;2 C y  x 0;2 x 0;2 D y  x 0;2 Hướng dẫn Xét hàm số y  x  3x  liên tục đoạn [0; 2]  x    0; 2 Có y  3x    x  1 ; y     x  1   0; 2 Ta có: y 1  3; y    5; y    Do y  y 1  x 0;2 → Chọn B Ví dụ 2: Giá trị lớn hàm số f  x   x  2x  đoạn [0; 2] là: A max f  x   64 B max f  x   x 0;2 C max f  x   x 0;2 x 0;2 D max f  x   x 0;2 Hướng dẫn Xét hàm số f  x   x  2x  liên tục đoạn [0; 2]  x    0; 2  Ta có f   x   4x  4x  4x  x  1 ;f   x     x    0; 2   x  1   0; 2 Khi f 1  0;f    1;f    Do max f  x   f    x 0;2 → Chọn D Ví dụ 3: Giá trị lớn nhỏ hàm số y   4x đoạn [-1; 1] là: A max y  y  B max y  y  3 C max y  y  D max y  y   x 1;1 x 1;1 x 1;1 x 1;1 x 1;1 x 1;1 x 1;1 x 1;1 Hướng dẫn Hàm số có điều kiện xác định  4x   x  Suy hàm số xác định với x   1;1 Xét hàm số y   4x liên tục đoạn [-1; 1] Trang Ta có y  2  0, x   1;1 Do max y  y  1  3; y  y 1  x 1;1 x 1;1  4x → Chọn C Bài tập tự luyện Câu Tìm giá trị nhỏ m hàm số y  x  A m = đoạn x C m  B m = 1   ;  17 D m = 10 Câu Giá trị lớn nhất, giá trị nhỏ hàm số y  x    x  là: A 2; B 4; C 4; D 4; 2 Đáp án: 1–B 2-D Dạng 2: Giá trị lớn nhất, nhỏ hàm số khoảng Phương pháp giải Tìm GTLN, GTNN hàm số y = f (x) tập Tìm GTLN, GTNN nửa khoảng [-1; 3) hàm (khoảng, nửa khoảng) số y  f  x   x  3x Cách 1: Tập xác định: D   Ta có: f   x   3x  6x Bước 1: Lập bảng biến thiên hàm số  x1    1;3 Khi f   x      x  2   1;3 Bảng biến thiên (BBT) x - -2 f x -1 - f x - + 54 Bước 2: Từ bảng biến thiên, kết luận GTLN, Từ BBT, ta thấy GTLN hàm số x    1;3 , 54 GTLN hàm số GTNN hàm số x    1;3 Cách 2: Sử dụng máy tính fx 570 VNPLUS Do đó, hàm số khơng có GTLN có GTNN Ta sử dụng máy tính dạng Nhưng cần ý chọn GTLN, GTNN Ví dụ minh họa Ví dụ 1: Giá trị nhỏ hàm số y  x  2x  3x  khoảng (1;5) là: A B 10 C -4 D  10 Hướng dẫn Hàm số có tập xác định: D   Trang Ta có y  x  4x  3; y   x  4x    x  x = Ta có bảng biến thiên x f’ (x) f (x)  - + 8 -4 Từ bảng biến thiên ta thấy giá trị nhỏ hàm số khoảng (1;5) -4 → Chọn C x2  x 1 Ví dụ 1: Giá trị nhỏ hàm số f  x   khoảng 1;   là: x 1 A y  1 B y  x1;   C y  x1;   x1;   D y  x1;   7 Hướng dẫn Hàm số xác định với x  1;   x2  x 1 Xét hàm số f  x   liên tục khoảng 1;   x 1 1 x  2x   f  x  1  ; Ta có f  x   x  2 x 1  x  1  x  1 x  f x    x  Ta lại có lim f  x   ; lim f  x    x  x 1 Bảng biến thiên x f’ (x) f (x)   +  Từ bảng biến thiên ta có: f  x   f    x1;   → Chọn B Bài tập tự luyện Câu 1: Cho hàm số y  x  3x  18x GTNN hàm số nửa khoảng 1;   là: A 10 B 22 C 11  8x Câu 2: Cho hàm số y  GTLN hàm số khoảng  ;1 là: x 1 A – B C Câu Cho hàm số y  x  GTNN hàm số khoảng  0;   là: x A B C Đáp án: D 21 D 10 D Trang 1–B 2–C 3-D Dạng 3: Giá trị lớn nhất, nhỏ hàm số chứa tham số Ví dụ minh họa Ví dụ 1: Cho hàm số y  xm (m tham số thức) thỏa mãn y  Mệnh đề đúng? x 1 x 2;4 A < m   B  m < C m > D m < -1 Hướng dẫn 1  m Hàm số có tập xác định: D   \ 1 Ta có y   x  1 * Nếu m  1  y   Hàm số đồng biến khoảng TXĐ Khi y  y     x 2;4 2m   m  (loại) 1 * Nếu m  1  y   Hàm số nghịch biến khoảng TXĐ Khi y  y     x 2;4 4m   m  (thỏa mãn) Vậy m > 4 1 → Chọn C Ví dụ 2: Cho hàm số y  16 xm (m tham số thực) thỏa mãn y  max y  Mệnh đề x 1 x1;2 x1;2 đúng? A < m  B  m < C m ≤0 D m > Hướng dẫn Hàm số có tập xác định: D   \ 1 Ta có y  1 m  x  1 * Nếu m   y   Hàm số đồng biến khoảng TXĐ Khi y  y 1  x1;2 1 m 1 m 2m 2m  ; max y  y     11 2 1 x1;2 16  m  m 16     m  (loại m < 1) 3 x1;2 x1;2 * Nếu m   y   Hàm số nghịch biến khoảng TXĐ y  max y  Khi y  y    x1;2  y  max y  x1;2 x1;2 2m 2m 1 m 1 m  ; max y  y 1   1 11 x1;2 16  m  m 16     m  (thỏa mãn) Vậy m > 3 → Chọn D Bài tập tự luyện Câu Tìm m để hàm số y  x  mx  5x  đạt giá trị nhỏ lớn A m   B 2  m  C  m   D   m   Đáp án: – D Trang PHẦN 3: BÀI TẬP TỔNG HỢP Câu Cho hàm số y  x  3x  9x  35 Gọi a, b giá trị lớn nhất, nhỏ hàm số đoạn [-4; 4] Tính giá trị a + b A -1 B 71 C -2 D 18 Câu Cho hàm số y  x  3x  18x Giá trị nhỏ hàm số 1;   là: A 10 B 22 C 11 F 21 Câu Cho hàm số y  x  2x  3x  Giá trị nhỏ hàm số [-4; 0] là: A  16 B Câu Giá trị nhỏ hàm số A C -4 x khoảng  0;   là: x B Câu Giá trị nhỏ hàm số y  x  A D C 2 D khoảng  0;   là: x B C D Câu Giá trị nhỏ hàm số y  sin x 1  cos x  [0; π] là: A B C D   Câu Cho hàm số f  x   x  cos x , với x  0;  Giá trị lớn hàm số là:  2 A B Câu Cho hàm số   2 C   D  x 1 Gọi a, b giá trị lớn nhất, giá trị nhỏ hàm số Giá trị a x  x 1 – 3b là: A B C D Đáp án: 1–A 2–B 3–A 4–D 5–B 6–B 7–D 8–B Trang CHƯƠNG CHUYÊN ĐỀ 4: TIỆM CẬN CỦA ĐỒ THỊ HÀM SỐ PHẦN 1: LÝ THUYẾT TRỌNG TÂM Định nghĩa Cho hàm số y = f(x) xác định D  Nếu lim f(x)  y0 x  lim f(x)  y0 đường x  thẳng y = y0 tiệm cận ngang (TCN) đồ thị hàm số  Nếu lim f(x)   lim f(x)   đường x  x0 x  x0 thẳng x = x0 tiệm cận đứng (TCĐ) đồ thị hàm số Một số ý  Ta tính giới hạn cách sử dụng máy tính x x2 X X2 Nhập x 2 x2 X2 lim CALC + 10-9  Đồ thị hàm số đa thức khơng có tiệm cận  Giá trị x0 giá trị mà hàm số khơng xác định  Đồ thị hàm số y  0.75001 ax  b ln có tiệm cận Đồ thị hàm số y  x  có TCĐ: x = 1, TCN: y = cx  d x 1 c  khi:  ad  bc  Trang d a Khi TCĐ x   ; TCN y  c c  Hàm số xác định khoảng, đoạn không chứa Hàm số y   x có TXĐ D = [-2;2] khơng chứa  đồ thị hàm số khơng có tiệm cận ngang  nên đồ thị hàm số khơng có tiệm cận ngang PHẦN 2: CÁC DẠNG BÀI TẬP Dạng 1: Tìm tiệm cận đồ thị hàm số Ví dụ minh họa Ví dụ 1: Cho hàm số y  2x  Đồ thị hàm số có đường tiệm cận đứng tiệm cận ngang x 1 là: A x = y = -3 B x = y = C x = y = D x = -1 y = Hướng dẫn Cách 1: Tập xác định: D   \ 1 Ta có: lim x 1 2x  2x    lim   , nên đồ thị hàm số có TXĐ x = x 1 x  x 1 2x   , nên đồ thị hàm số có TCN y = x  x  Ta lại có: lim Cách 2: Sử dụng máy tính fx 570VNPLUS Nhập biểu thực 2x  x 1 Ấn CALC x   109  kết 999999998 nên lim 2x    x 1 x  Ấn CALC x   109  kết -999999998 nên xlim 1 2x    x 1  Đồ thị hàm số có TCĐ x = Ấn CALC x  1010  kết nên xlim  2x  2 x 1  Đồ thị hàm số có TCN y =  Chọn C Ví dụ 2: Đồ thị hàm số y  2x  có đường tiệm cận đứng tiệm cận ngang là: x  3x  2 A x = 1, x = y = B x = 1, x = y = C x = y = C x = 1, x = y = -3 Hướng dẫn Ta có x  3x   (x  1)(x  2) , nên hàm số không xác định x = x = 2 Sử dụng máy tính, ta tính lim y   lim y   ; lim y   lim y   x 1 x 1 x 2 x 2 Trang Suy đồ thị hàm số có TCĐ x = x = Tương tự, ta tính lim  x  2x   , nên đồ thị hàm số có TCN y = x  3x  2  Chọn A Ví dụ 3: Số đường tiệm cận đồ thị hàm số y  A B C x  x là: x  3x  D Hướng dẫn Ta có x  3x   (x  1)(x  4) , nên hàm số không xác định x = -1 x = Sử dụng máy tính, ta tính lim  y   lim  y   ; lim y   lim y   x ( 1) x 4 x ( 1) x 4 Suy đồ thị hàm số có TCĐ x = -1 x = Tương tự, ta tính lim  x  x  x   , nên đồ thị hàm số khơng có TCN x  3x  Số đường tiệm cận đồ thị hàm số  Chọn C Ví dụ 4: Đồ thị hình vẽ hàm số sau đây: A y  x 1 x 1 B y  3x x 1 C y  x2 x 1 D y  x2 x 1 Hướng dẫn Từ đồ thị ta thấy: Đồ thị hàm số có TCĐ x = 1, nên loại đáp án A Đồ thị hàm số có TCN y = 1, nên loại đáp án B Ta thấy điểm (0;-2) thuộc đồ thị hàm số, nên loại đáp án D  Chọn C Ví dụ 5: Đồ thị hàm số sau khơng có tiệm cận đứng? A y  3x  x2  B y  1 x C y  x3 x2 D y  x  2x  Hướng dẫn Đáp án A: Hàm số xác định  , nên đồ thị hàm số khơng có TCĐ Đáp án B: Hàm số có TCĐ x = Đáp án C: Hàm số có TCĐ x = -2 Đáp án D: Hàm số có TCĐ x =  Chọn A Bài tập tự luyện Trang Câu Đồ thị hàm số y  A x = -2 y = -3  3x có đường tiệm cận đứng tiệm cận ngang là: x2 B x = -2 y = C x = -2 y = D x = y =1 Câu Đồ hàm số sau có ba đường tiệm cận? A y   2x 1 x B y   x2 C y  x3 5x  D y x x x9 Câu Đồ thị hàm số sau khơng có tiệm cận ngang? 2x  A y  x 1 1 B y  x2 C y  x 1 Câu Tìm tất đường tiệm cận đồ thị hàm số y  A y = 1 B x = x  3x  D y  2x  x3 x2  C y = D y = -1 Đáp án: 1-A 2-B 3-D 4-A Dạng 2: Bài toán chứa tham số Ví dụ minh họa Ví dụ 1: Tìm tất giá trị thực m để đồ thị hàm số y  A m = B m = -4 C m  mx  có tiệm cận đứng x2 D m  -4 Hướng dẫn c  1  Đồ thị hàm số có TCĐ     m  4 ad  bc  2m    Chọn C Ví dụ 2: Với giá trị m đồ thị (C): y  A m  2 B m = C m  mx  có tiệm cận đứng qua điểm M(t, 2)? 2x  m D m = Hướng dẫn 2  c  Đồ thị hàm số có đường tiệm cận dứng    m  R ad  bc  m   Khi đó, đồ thị hàm số có đường tiệm cận đứng x   Vậy để tiệm cận đứng qua điểm M(t, 2)  m m  1  m  2  Chọn D Trang x  x   mx có đường tiệm cận đứng khi: x 1 Ví dụ 3: Đồ thị hàm số y  B m  R A m  C m  -1 D m  Hướng dẫn Xét phương trình x  x   mx   Nếu phương trình khơng có nghiệm x = đồ thị hàm số có đường tiệm cận đứng x =  Nếu phương trình có nghiệm x = 1, tức t    m.1   m  1 x2  x   x 1 Khi xét giới hạn: lim  lim  x 1 x  x 1 x  x 1  x Do đó, đồ thị hàm số khơng có đường tiệm cận đứng Vậy đồ thị hàm số có đường tiệm cận đứng m  -1  Chọn C Bài tập tự luyện Câu Giá trị m để đồ thị hàm số y  A m = 0; m =  Câu Cho hàm số y  xm khơng có tiệm cận đứng mx  C m =  B m = -1 D m = mx  có đồ thị (C) Kết luận sau đúng? xm A Khi m = (C) khơng có đường tiệm cận đứng B Khi m = -3 (C) khơng có đường tiệm cận đứng C Khi m  3 (C) có tiệm cận đứng x = -m, tiệm cận ngang y = m D Khi m = (C) khơng có tiệm cận ngang Đáp án: 1–A 2-C PHẦN 3: BÀI TẬP TỔNG HỢP Câu Tiệm cận ngang đồ thị hàm số y  A y = 8x  1999 là: 4x  Câu Tìm tiệm cận đứng đồ thị hàm số y  A x = 0; x = C y  B y = B y = 25 D y = x x  3x C y = D x = Câu Trong hàm số sau, đồ thị hàm số có đường tiệm cận ngang? A y  x  25x  B y  x  8x  99 3x  C y  x 2 2x  D y  x2 Trang Câu Cho hàm số y  x x 9 Khẳng định sau đúng? A Đồ thị hàm số có đường tiệm cận đứng x = 3 đường tiệm cận ngang y =  B Đồ thị hàm số có đường tiệm cận đứng x = 3 đường tiệm cận ngang y = C Đồ thị hàm số có đường tiệm cận đứng x = đường tiệm cận ngang y = D Đồ thị hàm số có đường tiệm cận đứng x = khơng có đường tiệm cận ngang Câu Đồ thị hàm số sau có hai đường tiệm cận ngang? x 1 A y  2x  x 1 B y  x  2x  Câu 6.) Cho hàm số y  A y = A D y  x  3x  x2  Tiệm cận ngang đồ thị hàm số là: x 1 B y = -1 Câu Đồ thị hàm số y  x2  C y  x 3 C y = 1; y = -1 D x = 1; x = -1 x 1 có đường tiệm cận? x  2x  B C D mx  n có đồ thị (C) Biết đường tiệm cận (C) qua điểm A(-1;2) ta có x 1 điểm I(2;1) thuộc (C) Khi giá trị m+n Câu Cho hàm số y  A m + n = -1 B m + n = Câu Giá trị m để đồ thị hàm số y  A m = 0; m =  C m + n = -3 D m + n = xm khơng có tiệm cận đứng mx  C m =  B m = -1 D m = Đáp án: 1–D 2–D 3-C 4–A 5–C 6–C 7–B 8–A 9–A Trang CHƯƠNG CHUYÊN ĐỀ KHẢO SÁT SỰ BIẾN THIÊN VÀ VẼ ĐỒ THỊ HÀM SỐ PHẦN 1: LÝ THUYẾT TRỌNG TÂM Hàm số bậc ba Hàm số bậc ba có dạng: y  f  x   ax3  bx  cx  d  a   * Tập xác định: D   * Đạo hàm: y  f   x   3ax  2bx  c  b * Điểm đối xứng I   ; f  3a  b       3a   * Giao điểm đồ thị hàm số với Oy (0;d) a Đồ thị Trường hợp a>0 a0 a * Luôn đồng biến d  d    ;   ;   ;   c  c   ad – bc < khoảng * Luôn nghịch biến d  d    ;   ;   ;   c  c   khoảng b Nhận dạng đồ thị Dựa vào dấu hệ số; đồng biến, nghịch biến; đường tiệm cận; giao điểm đồ thị với trục tọa đọ suy tính chất PHẦN 2: CÁC DẠNG BÀI TẬP Dạng 1: Nhận dạng đồ thị hàm số Ví dụ minh họa Ví dụ 1: Đường cong hình bên đồ thị hàm số hàm số sau đây? A y  x  x  B y  x  x C y  x  x D y   x  x Hướng dẫn Từ đồ thị ta thấy hàm số bậc bốn trùng phương y  ax  bx  c  a   có a  Vì hàm số có ba cực trị nên ab   b  Do loại đáp án B D Vì đồ thị qua điểm O(0;0) nên c = Do loại đáp án A → Chọn C Ví dụ Hàm số y  A x2 có đồ thị hình vẽ sau đây? x 1 B Trang D C Hướng dẫn Cách 1: Hàm số y  Đồ thị hàm số y  Cách 2: ta có x2 có TCĐ x  TCN y  Do loại đáp án D x 1 x2 qua điểm (0;2) nên chọn đáp án A x 1 x2 d  x2 đồng biến khoảng xác định Do   , suy hàm số y    x 1 dx  x   x 10 81 loại đáp án B D Đồ thị hàm số y  x2 qua điểm (0;2) nên chọn đáp án A x 1 → Chọn A Ví dụ Cho đồ thị hàm số y  f  x  hình bên Khẳng định sau đúng? A Đồ thị hàm số có TCĐ x = -1, TCN y = B Hàm số nghịch biến khoảng (-∞;-1) (-1;+∞) C Hàm số có hai cực trị D Hàm số đồng biến khoảng (-∞;+∞) Hướng dẫn Nhìn vào ta thấy đồ thị có tiệm cận đứng x = -1, tiệm cận ngang y = → Chọn A Ví dụ Bảng biến thiên sau hàm số nào?  x f x f x +  -  CĐ  + CT A y   x3  x  B y  x3  x  C y  x3  x  D y   x3  x  Hướng dẫn Trang Dựa vào bảng biến thiên ta thấy nhánh cuối đồ thị hàm số có hướng lên Suy hệ số a > Do loại đáp án A D Ta có y  có hai nghiệm x = x = nên có đáp án B phù hợp → Chọn B Ví dụ Hàm số y  ax3  bx  cx  d  a   có đồ thị sau, xác định dấu a d A a > 0; d < B a < 0; d > C a > 0; d > D a < 0; d > Hướng dẫn Nhìn vào đồ thị hàm số, ta thấy nhánh cuối có hướng lên suy a > Ta lại thấy đồ thị hàm số cắt trục tung điểm có tung độ dương suy d > Vậy hàm số có a > 0; d > → Chọn C Ví dụ Hàm số y  ax3  bx  cx  d có đồ thị hình vẽ bên Mệnh đề đúng? A a < 0, b > 0, c > 0, d < B a < 0, b < 0, c > 0, d < C a > 0, d < 0, c < 0, d > D a < 0, b > 0, c < 0, d < Hướng dẫn Nhìn vào đồ thị hàm số, ta thấy nhánh cuối đồ thị có hướng xuống  a  Nên loại đáp án C Đồ thị hàm số cắt trục tung điểm có tung độ âm  d  Ta có y  3ax  2bx  c , phương trình y  có nghiệm phân biệt x1 ; x2 hoành độ hai điểm cực trị Từ đồ thị hàm số, ta thấy hai điểm cực trị hàm số có hồnh độ trái dấu  x1.x2   c a 0    c  Nên loại đáp án D 3a Ta lại thấy, điểm đối xứng I đồ thị hàm số có hồnh độ dương  x1    b a 0    b  Nên loại đáp án B 3a → Chọn A Bài tập tự luyện Câu Cho hàm số y  f  x  có bảng biến thiên sau Khẳng định sau đúng? Trang x  f  x f  x  -  -1  -1 A Đồ thị hàm số có tiệm cận đứng x = 1, tiệm cận ngang y = -1 B Đồ thị hàm số có tiệm cận đứng x = -1, tiệm cận ngang y = C Đồ thị hàm số có hai đường tiệm cận đứng D Đồ thị hàm số có hai đường tiệm cận ngang Câu Đồ thị hàm số y  x3  x  hình bốn hình đây? Hình Hình Hình Hình A Hình B Hình Câu Xác định a,b để hàm số y  C Hình D Hình ax  có đồ thị hình vẽ xb Chọn đáp án A a = 1, b = -1 B a = 1, b = C a = -1, b = D a = -1, b = -1 Câu Cho đồ thị hàm số bậc ba y  f  x  hình sau Chọn đáp án A Phương trình f   x   có nghiệm x = B Hàm số đồng biến khoảng (-2;1) (1;2) C Hàm số khơng có cực trị D Hàm số có hệ số a < Trang Đáp án: 1-A 2-A 3-B 4-A Dạng 2: Bài tốn chứa tham số Ví dụ minh họa Ví dụ 1: Đồ thị hàm số y  x3  mx  12 x  13 có hai điểm cực trị cách trục tung khi: A m = -1 B m = C m = -1; m = -2 D m = -2 Hướng dẫn Hàm số có y  x  2mx  12 Để đồ thị hàm số có điểm cực trị    m  72  (luôn đúng) Gọi x1 , x2 nghiệm phương trình y  Khi x1 , x2 hoành độ hai điểm cực trị Đồ thị hàm số có hai điểm cực trị cách trục tung  x1  x2  x1   x2  x1  x2   S   m  Vậy m = → Chọn B Ví dụ 2: tìm giá trị thực tham số m để đồ thị hàm số y  x3  x  m  Cm  có hai điểm phân biệt đối xứng qua gốc tọa độ là: A 1  m  B m  C m  3 D m  Hướng dẫn Gọi điểm M  x0 ; y0  x0     Cm  ; x0  Gọi điểm M   x1 ; y  x1   điểm đối xứng M qua gốc tọa độ O  x0   x1  x0  x1 Vì M  đối xứng với M qua O, nên ta có    y  x0    y  x1   y  x0    y   x0   x03  x02  m     x0     x0   m   x02  m  m    → Chọn D Bài tập tự luyện Câu Cho hàm số y  x3   3m  1 x  2mx  m  Điều kiện tham số m để đồ thị  Cm  hàm số có hai điểm phân biệt đối xứng qua trục Oy là: A m  B m  C m  2 D m  2 Câu Đồ thị hàm số y  ax  bx  c có điểm cực tiểu (0;3) điểm cực đại (1;5) Tìm giá trị biểu thức P  a  2b  c A B C 12 D Trang 1–B Đáp án: 2–D PHẦN 3: BÀI TẬP TỔNG HỢP Câu Cho hàm số y  f  x  có bảng biến thiên đây:  x -1 y y -  +  -1  Khẳng định sau khẳng định đúng? A Đồ thị hàm số có đường tiệm cận B Hàm số nghịch biến khoảng  ;0   0;   C Đồ thị hàm số khơng có tiệm cận D Hàm số có giá trị lớn giá trị nhỏ Câu Đồ thị sau hàm số nào? A y  x3  x  x B y  x3  x  x C y   x3  x  x D y  x3  x  x  Câu cho hàm số y  x3  x  mx  Tìm tất giá trị tham số m để đồ thị hàm số qua điểm A  1;9  A m  B m  2 D m  C m  3 Câu Cho hàm số y   x  x  có giá trị cực đại giá trị cực tiểu y1 y2 Khi khẳng định sau đúng? A y1  y2  15 B y1  y2  D y1  y2  12 C y2  y1  Câu Số giá trị thực tham số m để giá trị nhỏ hàm số y  x  m2  m đoạn  0;1 x 1 -2 là: A B C D Câu Cho hàm số y  f  x  xác định M có đạo hàm f   x    x  1  x  1 x Số điểm cực trị hàm số là: A Câu Cho hàm số y  B C D ax  b có đồ thị hình vẽ sau: cx  d Mệnh đề sau đúng? Trang A bc > 0; ad < C bd < 0; ad > B ac > 0; bd > D ab < 0; cd < Câu Đồ thị hàm số y  x3  x  x  có hai điểm cực trị A B Điểm thuộc đường thẳng AB? A P 1;3 B M  0;1 C Q  3; 29  D N  0;5  Câu Cho hàm số y  f  x  có đạo hàm f   x   x  2, x   Mệnh đề sau đúng? A Hàm số nghịch biến khoảng  3;   B Hàm số nghịch biến khoảng  ;1 C Hàm số đồng biến khoảng  ;   D Hàm số nghịch biến khoảng  ;   Câu 10 Hàm số y  A m  m  1 x3   m  1 x  x  đồng biến    m  1 C  m  B m  D m  Đáp án: -A 2–A 3–C 4–B 5–A 6–A 7–A 8–D 9–C 10 – C Trang CHƯƠNG CHUYÊN ĐỀ MỘT SỐ BÀI TOÁN LIÊN QUAN ĐẾN ĐỒ THỊ HÀM SỐ PHẦN 1: LÝ THUYẾT TRỌNG TÂM Tương giao hai đồ thị hàm số Cho hai hàm số f  x  g  x  Xét phương trình hoành độ giao điểm f  x   g  x  Ta có: • Số giao điểm hai đồ thị = Số nghiệm phương trình • Hồnh độ giao điểm = Nghiệm phương trình Đồ thị có ba giao điểm  phương trình f  x   g  x  có ba nghiệm Hoành độ giao điểm x1 , x , x  x1 , x , x nghiệm f  x   g  x  Phương trình tiếp tuyến đồ thị hàm số điểm Cho hàm số y  f  x  điểm M  x ;f  x   Phương trình tiếp tuyến đồ thị hàm số điểm M là: y  f   x  x  x   f  x  Một Số phép biến đổi đồ thị a Tịnh tiến đồ thị hàm số Cho hàm số y  f  x  có đồ thị  C  ; p, q số dương tùy ý • Tịnh tiến  C  lên q đơn vị đồ thị hàm số y  f  x   q • Tịnh tiến  C  xuống q đơn vị đồ thị hàm số y  f  x   q • Tịnh tiến  C  sang trái p đơn vị đồ thị hàm số y  f  x  p  • Tịnh tiến  C  sang phải p đơn vị đồ thị hàm số y  f  x  p   • Tịnh tiến  C  theo vectơ u   a; b  đồ thị hàm số y  f  x  a   b b Đồ thị hàm số chứa dấu giá trị tuyệt đối Từ đồ thị  C  : y  f  x  suy đồ thị  C  : y  f  x  f  x  x  Ta có y  f  x    y  f  x  hàm chẵn f   x  x  Nên đồ thị  C  nhận Oy làm trục đối xứng Trang Cách vẽ  C  từ  C  : • Giữ nguyên phần đồ thị bên phải trục Oy đồ thị  C  : y  f  x  (bỏ phần bên trái) • Lấy đối xứng phần đồ thị bên phải trục Oy đồ thị  C  : y  f  x  qua Oy Đồ thị  C  Giữ nguyên phần bên phải Lấy đối xứng phần bên phải Từ đồ thị  C  : y  f  x  suy đồ thị  C  : y  f  x  f  x  f  x   Ta có y  f  x    f   x  f  x   Cách vẽ  C  từ  C  : • Giữ nguyên phần đồ thị bên trục Ox đồ thị  C  : y  f  x  (bỏ phần bên dưới) • Lấy đối xứng phần đồ thị bên trục Ox đồ thị  C  : y  f  x  qua Ox Đồ thị  C  Giữ nguyên phần bên Lấy đối xứng phần bên PHẦN 2: CÁC DẠNG BÀI TẬP Dạng 1: Tương giao hai đồ thị Phương pháp giải Tương giao đồ thị hàm số y  ax  bx  cx  d trục Ox   y  • Cắt ba điểm  y CÑ y CT  Trang   y  • Cắt hai điểm  y CÑ y CT    y  • Cắt điểm  y   y y   CÑ CT Tương giao đồ thị hàm số  C  : y  ax  bx  cx  d đường thẳng (d): y  kx  n Xét phương trình ax3  bx  cx  d  kx  n 1 • Nhẩm nghiệm đưa phương trình bậc hai • Cơ lập tham số sau khảo sát hàm số Tương giao đồ thị hàm số  C  : y  ax  bx  c trục Ox ab  • Cắt bốn điểm  y CÑ y CT  ab  • Cắt ba điểm  c  Tương giao đồ thị hàm số  C  : y  ax  bx  c đường thẳng (d): y  k Xét phương trình ax  bx  c  k Đặt t  x  t   ta có phương trình at  bt  c  k  • Cắt bốn điểm 2  3    có bốn nghiệm phân biệt   3 có hai nghiệm dương phân biệt      3 thỏa mãn P  S   • Cắt ba điểm    có ba nghiệm phân biệt   3 có hai nghiệm phân biệt, có nghiệm dương nghiệm t  • Cắt hai điểm    có hai nghiệm phân biệt   3 có nghiệm kép dương có hai nghiệm trái dấu • Cắt điểm    có nghiệm   3 có hai nghiệm phân biệt, t = nghiệm âm  3 có nghiệm kép t  • Khơng cắt    vô nghiệm Trang   3 vơ nghiệm có nghiệm âm Tương giao đồ thị hàm số y  ax  b  C  đường thẳng: y  kx  n bx  c d Ax  Bx  C  ax  b   kx  n   Xét phương trình d cx  d x   c  Cắt hai điểm  4 d    có hai nghiệm phân biệt khác  c Ví dụ minh họa Ví dụ 1: Tìm giao điểm đồ thị  C  : y  x  2x  trục hoành A  1;0  B  0;1 C  1;0  1;0  D M  1;0   0;1 Hướng dẫn  x   thoûa mãn   x   Xét phương trình hoành độ giao điểm: x  2x      x    loaïi   x  1 Vậy đồ thị  C  cắt trục hoành hai điểm A  1;0  , B 1;0   Chọn C Ví dụ 2: Tìm tọa độ giao điểm đường thẳng y   2x với đồ thị hàm số y  x  3x  A M  1;  B M  0;  C M  4; 5  D M  3; 4  Hướng dẫn Xét phương trình hồnh độ giao điểm: x  3x    2x  x  3x  2x  x   y    x   y   x   y  2 Vậy đồ thị hàm số bậc ba cắt đường thẳng ba điểm phân biệt A  0;  , B 1;0  , C  2; 2   Chọn B Ví dụ 3: Tìm tọa độ giao điểm đồ thị  C  : y  A 1;3 2x  đường thẳng d: y  x  2x  3 1 B  ;  2 2 C 1; 3 3 1 D  ;   2 2 Hướng dẫn Xét phương trình hồnh độ giao điểm: 2x   x2 2x  1 Trang Điều kiện xác định: x  Khi 1  2x    2x  1 x    2x  x    x y   2  x   y   1 Vậy tọa độ giao điểm cần tìm   ;  1;3  2  Chọn A Ví dụ 4: Cho hàm số y  mx  x  2x  8m có đồ thị  Cm  Tìm m để đồ thị  Cm  cắt trục hoành ba điểm phân biệt  1 A m  0;   2   B m    ;0     1 C m    ;   2  1 D m   0;   2 Hướng dẫn Xét phương trình hồnh độ giao điểm:  x  2 mx  x  2x  8m    x    mx   2m  1 x  4m      mx   2m  1 x  4m   Cm   2 cắt trục hoành ba điểm phân biệt   có hai nghiệm phân biệt khác –2  m  m  m      Khi đó:   12m  4m      m    1 12m      m    m    1 Vậy m    ;  \ 0 thỏa mãn  2  Chọn D Ví dụ 5: Cho hàm số y  x   3m   x  3m  C  Tìm m để đường thẳng d: y  1 cắt đồ thị  C  bốn điểm phân biệt có hoành độ nhỏ 2? A m   0;1 B m   0;1   C m    ;1     D m    ;1   Hướng dẫn Phương trình hồnh độ giao điểm  C  d: y  1 là: x   3m   x  3m  1  x   3m   x  3m   x2  t  Đặt t  x  t   , ta có phương trình t   3m   t  3m       t  3m   x  3m  2 Theo yêu cầu toán m phải thỏa mãn hệ phương trình sau: Trang 0  3m      m  m   3m   1 Vậy   m  m  thỏa yêu cầu tốn  Chọn B Ví dụ 6: Cho hàm số y  2x  có đồ thị  C  Tìm m để đường thẳng (d): y   x  m cắt đồ thị  C  x 1 hai điểm phân biệt A m   ;1   5;   B m   ;1   5;   C m   ;1  5;   D m   ;1  5;   Hướng dẫn Xét phương trình hồnh độ giao điểm: Điều kiện xác định: x  Khi 1 2x   x  m x 1 1  2x     x  m  x  1 2  x   m  1  m   Đường thẳng  d  cắt  C  hai điểm phân biệt 1 phải có hai nghiệm phân biệt      m  1    m  1        có hai nghiệm phân biệt khác    1   m  1  m  1   m  6m    m   ;1   5;   Vậy giá trị m cần tìm m   ;1   5;    Chọn A Bài tập tự luyện Câu Cho hàm số y  2x  3mx   m  1 x  có đồ thị  C  Tìm tập giá trị m để đường thẳng d: y   x  cắt đồ thị  C  ba điểm phân biệt 8  A  ;0    ;   9  8  B  ;0    ;   9  8  C  ;0   ;   9  8  D  ;0    ;   9  Câu Tìm m để đồ thị hàm số y  x  mx  cắt trục hoành điểm A m  3 Câu Cho hàm số y  B m  3 mx  có đồ thị x2 C m  3 D m  3  Cm  Tìm m để đường thẳng  d  : y  2x  cắt đồ thị  Cm  hai điểm phân biệt A, B cho AB  10 A m  B m  3 C m  3 D m  Câu Cho hàm số: y   x  1  x  mx  m  Tìm m để đồ thị hàm số cắt trục hoành ba điểm phân biệt? Trang B   m  A m  C  m    m0 D   m  Đáp án: 1–A 2–C 3–D 4–D Dạng 2: Phương trình tiếp tuyến Ví dụ minh họa Ví dụ 1: Cho hàm số y  x 1 có đồ thị  H  Tiếp tuyến  H  giao điểm  H  với trục hồnh x2 có phương trình là: A y  3x B y  x  C y  3x  D y   x  1 Hướng dẫn Phương trình hồnh độ giao điểm  H  trục hoành x 1   x   y 1  x2 Phương trình tiếp tuyến  H  điểm 1;0  có dạng: y  y 1  x  1   y   x  1  Chọn D Ví dụ 2: Cho hàm số y  x  3x  2x  Tiếp tuyến đồ thị hàm số song song với đường thẳng d : 2x  y   có phương trình là: A x  2y  19  B 2x  y  19  C 2x  y   D y  2x  Hướng dẫn Hàm số có y  3x  6x  Tiếp tuyến đồ thị hàm số song song với đường thẳng 2x  y    y  2x  x   y  2  3x  6x   2   x  Với x   y  1  Phương trình tiếp tuyến: y  2x  hay 2x  y   Với x   y  15  Phương trình tiếp tuyến: y  2  x    15 hay 2x  y  19   Chọn B Ví dụ 3: Cho hàm số có đồ thị  C  : y  2x  3x  Tìm  C  có điểm M cho tiếp tuyến  C  M cắt trục tung điểm có tung độ A M  0;8  B M  1; 4  C M 1;0  D M  1;8  Hướng dẫn Trang Ta có: M  0;8    C   Loại đáp án A Ta có: M  1;8    C   Loại đáp án D Xét đáp án B: M  1; 4  Hàm số có y  6x  6x  y  1  12 Phương trình tiếp tuyến M  1; 4  ) có dạng y  12  x  1   y  12x  d Có đường thẳng  d  cắt trục tung điểm M  0;8  (thỏa mãn yêu cầu đề bài) Vậy điểm M  1; 4  thỏa mãn  Chọn B Ví dụ 4: : Cho hàm số y  x   m  1 x  m  có đồ thị  C  Gọi  tiếp tuyến đồ thị  C  điểm có hồnh độ Với giá trị tham số m đường thẳng  vng góc với đường thẳng  d  : y   x  2016 ? A m  1 C m  B m  D m  Hướng dẫn Hàm số có y  4x   m  1 x  y 1  4m Vì tiếp tuyến  vng góc với đường thẳng  d  : y   x  2016  Tiếp tuyến  có hệ số góc k  Ta có y 1  k  4m   m  1  Chọn A Bài tập tự luyện 2x   C  Hệ số góc tiếp tuyến đồ thị  C  cho tiếp tuyến cắt x 1 trục Ox, Oy điểm A, B thỏa mãn OA  4OB là: Câu Cho hàm số y  A  B C  1 4 D Câu Viết phương trình tiếp tuyến đồ thị hàm số y   x  6x  điểm cực tiểu A y  B y  5 C y  D y  x  Đáp án: 1–A 2–B Dạng 3: Tịnh tiến đồ thị hàm số Ví dụ minh họa Ví dụ 1: Cho đồ thị hàm số  C  : y  x  3x  4x  Tịnh tiến đồ thị  C  lên đơn vị đồ thị hàm số nào? Trang A y  x  3x  4x  B y  x  3x  4x  C y  x  3x  4x  D y  x  3x  4x  Hướng dẫn Ta có  C  : y  f  x   x  3x  x  Tịnh tiến  C  lên đơn vị ta đồ thị hàm số y  f  x    y  x  3x  4x    y  x  3x  4x   Chọn C Ví dụ 2: Cho hàm số y  f  x   x  4x  Lựa chọn phép tịnh tiến song song với trục Oy đồ thị hàm 1 x số để nhận đồ thị hàm số y  x2 1 x A Tịnh tiến xuống đơn vị B Tịnh tiến lên đơn vị C Tịnh tiến sang phải đơn vị D Tịnh tiến sang trái đơn vị Hướng dẫn Ta có phép tịnh tiến song song với trục Oy phép tịnh tiến lên xuống Suy loại đáp án C D Do đồ thị hàm số nhận có dạng y  f  x   b Ở đó, b  : đồ thị tịnh tiến lên trên; b  : đồ thị tịnh tiến xuống  x2 x2 x  4x   f x  b   b 1 x 1 x 1 x  x  4x   b 1  x  x  x  b  4   b x2 x2    1 x 1 x 1 x 1 x   b    Đồng hệ số, ta   b  4 b   Vậy đồ thị tịnh tiến xuống đơn vị  Chọn A Ví dụ 3: Cho hàm số y  f  x   đồ thị hàm số y  A a  b  x2  x 1  Tịnh tiến đồ thị hàm số theo vectơ u   a; b  để nhận x 1 x2 Biết a  0;  a; b   Giá trị a  b x 1 B a  b  C a  b  D a  b  1 Hướng dẫn  Tịnh tiến đồ thị hàm số y  f  x  theo vectơ u   a; b  nhận đồ thị hàm số y  f  x  a   b Suy x2  f x  a  b x 1 Trang  x  a    x  a  1  b x2   x 1  x  a  1  x  2xa  a  x  a   b  x  a  1 x2  x 1 x  a 1  x  x  2a   b   a  a   ab  b x2  x 1 x  a 1 Đồng hệ số, ta được: 2a   b  a    Suy vectơ u   2; 3 Vậy a  b  1 a  a   ab  b    b  3 a     Chọn D Bài tập tự luyện Câu Tịnh tiến đồ thị hàm số y   x  5x  sang trái đơn vị, ta đồ thị hàm số hàm số sau: A y   x  3x  B y   x  5x  C y   x  5x  D y   x  3x  Câu Cho hàm số y  f  x   x  3x  9x  Lựa chọn phép tịnh tiến song song với trục Ox đồ thị hàm số để nhận đồ thị hàm số y  x  12x  A Tịnh tiến sang phải đơn vị B Tịnh tiến sang trái đơn vị C Tịnh tiến sang phải đơn vị D Tịnh tiến sang trái đơn vị Đáp án: 1–A 2–D Dạng 4: Đồ thị hàm chứa dấu tuyệt đối Ví dụ minh họa Ví dụ 1: Cho hàm số y  x  Chọn khẳng định A Hàm số đạt cực tiểu x  B Hàm số đạt cực tiểu x  2 C Hàm số đạt cực đại x  2 D Hàm số khơng có cực trị Hướng dẫn Ta có y  x   (x  2) Hàm số có đạo hàm y  x2  x  2 nên y   x  2 Ta có y   x   2;   ; y   x   ; 2  , nên hàm số đạt cực tiểu x  2  Chọn B Ví dụ 2: Đồ thị hàm số y  x  2x  đồ thị đồ thị sau? Trang 10 A B C D Hướng dẫn Ta có cách vẽ đồ thị hàm số y  x  2x  sau Bước 1: Vẽ đồ thị y  x  2x  Bước 2: Giữ nguyên phần đồ thị Ox (bỏ phần phía Ox) Bước 3: Lấy đối xứng phần đồ thị Ox qua Ox  Chọn A Ví dụ 3: Cho hàm số y  x  6x  9x có đồ thị Hình Đồ thị Hình hàm số đây? Trang 11 3 A y  x  x  x B y  x  6x  x C y  x  6x  9x D y   x  6x  9x Hướng dẫn Nhìn vào Hình 2, ta thấy đồ thị Hình đối xứng qua trục Oy nên Hình đồ thị hàm số có dạng y  f  x  Do loại đáp án C D Mặt khác, ta thấy đồ thị Hình qua điểm  1;  , 1;  nên chọn đáp án B  Chọn B Ví dụ 4: Hình vẽ sau đồ thị hàm trùng phương Giá trị m để phương trình f  x   m có bốn nghiệm phân biệt là: A m  0; m  B  m  C 3  m  D m  Hướng dẫn Từ đồ thị hàm số y  f  x  , ta có đồ thị hàm số y  f  x  hình bên Trang 12 Ta có, số nghiệm phương trình f  x   m số giao điểm đồ thị hàm số y  f  x  đường thẳng y  m m  Dựa vào đồ thị hàm số, ta thấy phương trình f  x   m có nghiệm phân biệt   m   Chọn A Bài tập tự luyện Câu Cho hàm số y  x  3x  có đồ thị Hình Đồ thị Hình hàm số đây? A y   x  3x  B y  x  x  C y  x  3x  D y  x  3x  Câu Đồ thị sau hàm số sau đây: Trang 13 B y  x  3x  A y  x  3x  C y  x  3x  D x  3x  Đáp án: 1–D 2–D Dạng 5: Biện luận số nghiệm phương trình dựa vào đồ thị hàm số Ví dụ minh họa Ví dụ 1: Tìm m để phương trình x  2x  m   có bốn nghiệm phân biệt A m   2;3 C m   2;3 B m   2;3 D m   2;3 Hướng dẫn 1 Phương trình x  2x  m    x  2x   m 4 Phương trình 1 phương trình hồnh độ giao điểm đồ thị  C  : y  x  2x  đường thẳng d : y  m  Số nghiệm 1 số giao điểm  C   d  Xét hàm số y  x  2x  có tập xác định: D   x  Đạo hàm y  4x  4x nên y   4x  4x    x   x  1 3 Bảng biến thiên: x  y –1 –  +  – +  y 2 Dựa vào bảng biến thiên, ta thấy phương trình 1 có bốn nghiệm phân biệt   m   Chọn B Ví dụ 2: Cho hàm số y  f  x  liên tục  có bảng biến thiên Tìm tất giá trị thực m để phương trình f  x   2m có hai nghiệm phân biệt Trang 14 x  –1 y + 0 –  + 0 – y   –3 m  C  m    m  B   m  3 A m  3 D m   Hướng dẫn Ta có số nghiệm phương trình f  x   2m số giao điểm đồ thị hàm số y  f  x  đường thẳng y  2m song song với trục Oy Do đó, dựa vào bảng biến thiên hàm số f  x   2m , phương trình f  x   2m có hai nghiệm m   2m  phân biệt    m    2m  3   Chọn C Ví dụ 3: Cho phương trình m  x  2x   2x  4x   * Có giá trị nguyên tham số m để phương trình có nghiệm thỏa mãn x  3 ? A B Khơng có giá tị m C Vô số giá trị m D Hướng dẫn Ta có phương trình *  m  x  2x    x  2x    Đặt t  x  2x , ta phương trình: mt  2t   1 Ta có f  x   x  2x; x  3  f  x    t  3;   Ta lại có 1  m  2   f  t  với t  3;   t t3 Khi đó, tốn trở thành: Tìm m để phương trình m  f  t  có nghiệm nửa khoảng 3;   Xét hàm số f  t   2  ; t  3;   có f   t      f   t    t  t t t t  Hàm số f  t  nghịch biến nửa khoảng 3;    Xét nửa khoảng 3;   , ta có f  t   f  3  Suy m  27  Có vơ số giá trị m 27 Trang 15  Chọn C Bài tập tự luyện Câu Tìm giá trị tham số m để phương trình x  3x  2m  có ba nghiệm phân biệt A 3 m 2 B 2  m  C 3 m 2 D 2  m  Câu Tìm giá trị tham số m để phương trình x  3x  m  m có nghiệm phân biệt A 2  m  B 1  m  C m  D m  21 Câu Điều kiện tham số m để phương trình x x   m có nghiệm thực phân biệt là: A  m  B m  C m  D m  Đáp án: 1–A 2–A 3–A PHẦN 3: BÀI TẬP TỔNG HỢP Câu Đường cong hình bên đồ thị hàm số đáp án sau? A y  x  x B y  x  3x C y  x  x D y  x  3x Câu Cho hàm số y  x  mx  m  có đồ thị  Cm  Tọa độ điểm cố định  Cm  là: A  1;0  , 1;0  B 1;0  ,  0;1 C  2;1 ,  2;3 D  2;1 ,  0;1 Câu Cho hàm số  Cm  : y  x  mx  m  Tìm tất giá trị thực tham số m để đồ thị hàm số cắt trục hoành bốn điểm phân biệt m  A  m  B Khơng có m C m  D m  Câu Cho hàm số y  f  x   ax  bx  cx  d với a  Biết đồ thị hàm số có hai điểm cực trị A 1;1 , B  1 ;3 Tính f   A f    14 B f    28 C f    28 D f    14 Câu Tìm số giao điểm đồ thị hai hàm số y  x  y  x  A B C D Trang 16 Câu Đồ thị hàm số y  x  3x  mx  m (m tham số) qua điểm M cố định có tọa độ là: A M  1;  B M  1; 4  C M 1; 2  Câu Biết đồ thị hai hàm số y  x  y  D M 1; 4  2x  cắt hai điểm phân biệt A, B Tính độ dài x 1 đoạn thẳng AB B AB  A AB  Câu Cho đường cong  C  : y  C AB  2 D AB  2x  M điểm nằm  C  Giả sử d1 , d x 1 khoảng cách từ M đến hai tiệm cận  C  Khi d1.d bằng: A B Câu Trên đồ thị  C  hàm số y  A C x2 có điểm tọa độ nguyên? 2x  B Câu 10 Cho hàm số y  D C D x 1  C  Đồ thị hàm số cho cắt đường thẳng y  2x  điểm x 1 A  x1 ; y1  ; B  x ; y  Khi x1  x bằng: A B C D Câu 11 Tìm m để đồ thị hàm số y  x   2m  1 x   m  1 x  m  cắt trục hoành ba điểm phân biệt, có điểm có hồnh độ âm A m  1 B m  Câu 12 Cho hàm số y  C m  1 D m  1 x Tìm tham số m để đồ thị hàm số  C  cắt đường thẳng  d  : y  x  m 2x  hai điểm phân biệt A, B cho AB  A m  B m  1 C m  1 D Không tồn m Câu 13 Tìm tham số m để đường thẳng  d  : y  x  m  cắt đồ thị hàm số  C  : y  2x  hai x 1  4 điểm phân biệt A, B cho OAB có trọng tâm điểm G   ;   3 A m  C m  B m  Câu 14 Cho hàm số y  D m  x 1 Tìm m để đường thẳng  d  : y  m  x cắt đồ thị hàm số  C  điểm x 1 phân biệt A, B cho tiếp tuyến  C  A B song song với A m  1 C m  B m  D m  Đáp án: 1–A 2–A 3–A 4–B 5–C 6–B 7–C 8–C 9–A 10–C 11–A 12–B 13–A 14–B Trang 17 CHƯƠNG 2: HÀM SỐ LŨY THỪA, HÀM SỐ MŨ VÀ HÀM SỐ LÔGARIT CHUYÊN ĐỀ 1: LŨY THỪA VÀ LÔGARIT PHẦN 1: LÝ THUYẾT TRỌNG TÂM Lũy thừa Lũy thừa với số mũ thực số mũ Lũy thừa a  1 Ví dụ: Các lũy thừa 23 ;  4  ;   2 4 số Đọc là: a mũ α Hoặc a lũy thừa α Hoặc Lũy thừa số a số mũ α Số mũ α ĐK số a a Nguyên dương α = n, n  * Không α=0 a0 Nguyên âm    n , n  * a0 Hữu tỉ Vô tỉ Lũy thừa a  a n  a.a .a    n thõa sè a r a>0 m , m , n  , n  n   lim rn , rn   , n  * a>0 a0  1 an a n  m ar  a n  n am a   lim a rn n  n  Chú ý: Chú ý điều kiện số a dạng số mũ α Không tồn lũy thừa 00 Định nghĩa bậc n Ví dụ: Cho b   n   ( n  ) Số gọi bậc số 23  Số a gọi bậc n số b a n  b Số 3 gọi bậc số 81  3  Với n lẻ: Có bậc n b, kí hiệu  81 Ví dụ: n b Số 64 có bậc số 4 Số có bậc số Số -12 có bậc số 12 Ví dụ: Trang Số 16 có hai bậc 2  Với n chẵn: Nếu b > 0: có hai bậc n b hai số đối Số 15 có hai bậc 15  15 nhau, kí hiệu n b   n b  Nếu b < 0: không tồn bậc n b Nếu b = 0: có bậc n b số Lơgarit Ví dụ: Lơgarit log a b Lôgarit số log số Lôgarit số 16 log 16 Đọc là: Lôgarit số a b Ví dụ:  Nếu a = 10, ta có lơgarit thập phân: Lơgarit thập phân log16; log Kí hiệu: log10 b ; logb; lgb Lôga Nê-pe ln16; ln  Nếu a = e, ta có lơgarit tự nhiên Kí hiệu: (Lơga Nê-pe): log e b ; lnb Định nghĩa: Ví dụ: Với a, b  , a  , ta có  log 23  log a b    a   b 4  log Chú ý: Để gọn, ta viết  log a b   log a2 b 1 34   81 81 PHẦN 2: CÁC CÔNG THỨC Các công thức lũy thừa a  1 a   a n   a  0 an a  a  n a m  a  0 a   ab  a a    b b m n r    a   a   Với a,b > 0; ,      a a  a n  a b  ab n a  a   a n n n a na  b b   a n   a b m   n am  a  0, n   , m    * n m a  nm a  a  0, n, m    *  Nếu a > a   a       Nếu < a < a   a      Trang Các công thức lôgarit Với a,b > 0, a  ,    log a  log a  a     log a a  a loga b  b Với a, b, c, b1 , b  , a  ,    log a  b1b   log a b1  log a b log a b log a  log a b1  log a b b2 log a n b    log a b b log a b n log a b    log a b log a b  n    log c b log c a  c  1 * log a  b  log a b     0 log a b  log b a  b  1  Nếu a > log a b  log a c  b  c  Nếu < a < log a b  log a c  b  c PHẦN 3: CÁC DẠNG BÀI TẬP Dạng 1: Rút gọn biểu thức lũy thừa, lôgarit Phương pháp giải Sử dụng công thức lũy thừa cơng thức lơgarit Ví dụ minh họa Ví dụ 1: Tìm tất giá trị x để biểu thức  2x  1 A x  B x  2 có nghĩa: 1  C x   ;  2  D x  Hướng dẫn Cách 1: Biểu thức  2x  1 có nghĩa 2x    x  Cách 2: Sử dụng máy tính CASIO fx570VN PLUS 2   Đáp án A: Chọn x  Nhập   1 , ta thấy máy tính MATH ERROR, tức biểu thức khơng   có nghĩa Loại đáp án A 2 Đáp án C: Chọn x = Nhập  2.2  1 , ta thu kết  , tức biểu thức có nghĩa Loại đáp án C Đáp án B: Chọn x = Nhập  2.0  1 , ta thu kết 1 , tức biểu thức có nghĩa Loại đáp án B 2  Chọn D Ví dụ 2: Viết biểu thức 23 dạng lũy thừa 2m ta giá trị m là: 160,75 Trang A 13 B 13 C D  Hướng dẫn 13 2   2 Cách 1: Ta có 160,75 4 2  Cách 2: Sử dụng máy tính CASIO fx570VN PLUS Nhập vào máy tính biểu thức 23 , ta thu kết xấp xỉ 0,22272 160,75 Thử đáp án: Đáp án A: Nhập  13 , ta thu kết xấp xỉ 0,22272  Chọn A Ví dụ 3: Với giá trị a biểu thức log  2a  a  xác định? A < a < B a > C –1 < a < D a < Hướng dẫn Cách 1: Biểu thức log  2a  a  xác định 2a  a    a  Cách 2: Sử dụng máy tính CASIO fx570VN PLUS Chọn a = Nhập log  2.1  12  , ta thu kết 0, tức biểu thức có nghĩa Nên loại đáp án B đáp án C Chọn a  1 Nhập log   1   1  , ta thấy máy tính MATH ERROR, tức biểu thức khơng   có nghĩa Nên loại đáp án D  Chọn A Ví dụ 4: Cho  a  Rút gọn biểu thức Q  log a A Q  19 B Q  19 a 3.3 a a C Q  19 D Q  19 Hướng dẫn 19  3 23  12  a 3.3 a 19  log a  a Cách 1: Ta có Q  log a   log a a  a   Cách 2: Sử dụng máy tính CASIO fx570VN PLUS 19 23 22 Chọn a = Nhập log , ta thu kết  Chọn D Trang Bài tập tự luyện Bài Với giá trị x để biểu thức  x  1 có nghĩa: A x   ;1  1;   B x   ; 1  1;   C x   1;1 D x   \ 1 Bài Cho biểu thức log 2017   a    2a  3 3  A a   ;3  2  2018 Giá trị a để biểu thức xác định?  3 3  C a   3;    ;3  2 2  B a   3;3 Bài Rút gọn biểu thức P  a a 24 : a ,  a  0 a 1 B P  a A P = a 3 3   D a   3;    ;3  2 2   C P  a D P  a Bài Rút gọn biểu thức P  2log a 12  3log a  log a 15  log a 150 A P  log a Đáp án: B P  log8 a 1-B 2-D C P   log a 3-B D P  log a 4–A Dạng 2: Tính giá trị biểu thức lũy thừa, lôgarit Phương pháp giải Sử dụng công thức lũy thừa công thức lơgarit Ví dụ minh họa 1 Ví dụ 1: Giá trị biểu thức    16  A 12 0,75 1   8  bằng? B 16 C 18 D 24 Hướng dẫn 1 Cách 1:    16  0,75 1   8  3 4   24    23   23  24  24 Cách 2: Sử dụng máy tính CASIO fx570VN PLUS 1 Nhập biểu thức    16  0,75  1    , ta thu kết 24 Nên đáp án D 8  Chọn D Ví dụ 2: Cho P  x   A 0,13 x x2 Khi P 1,3 bằng: x B 1,3 C 0,013 D 13 Hướng dẫn Trang Cách 1: Vì x = 1,3 > nên ta có: P  x   x x x x   x  P 1,3  1,3 x x Cách 2: Sử dụng máy tính CASIO fx570VN PLUS Nhập vào máy tính biểu thức P  x   x x2 , nhập CALC X? 1.3, ta thu kết 1.3 x  Chọn B Giá trị biểu thức log Ví dụ 3: Cho log a  A 25 B 26 a  log a bằng: C 24 D 23 Hướng dẫn Cách 1: Ta có A  log Ta lại có log a  A 1 13 a  log a  log a  log 21 a  log a  log a  log a 22 1 1  log a 23   3log a   log a   log a  2 13 13 log a   26 3 Cách 2: Sử dụng máy tính CASIO fx570VN PLUS Ta có log a  Nhập log 1  a   a  82  64 64  log  64  , ta thu kết 26  Chọn B Ví dụ 4: Tìm n   , biết: 1 1 465 với x >      log x log 22 x log 23 x log 2n x log x x  B n  A n = 31 C n = 30 D n  31 Hướng dẫn Ta có 1 1     log x log 22 x log 23 x log 2n x  log x  log x 22  log x 23   log x 2n  log x  2.22.23 2n   log x 21 23  n Mặt khác 465  465.log x  log x 2465 log x Suy ra:     n  465   n  30 n  n  30  n  1  465  n  n  930     n  31  Chọn C Trang Bài tập tự luyện Bài Cho f  x   x x 12 x Khi f  2,  bằng: A 0,027 B 0,27 C 2,7 Bài Giá trị biểu thức A  log 16  log 27 3  A  17 Đáp án B  1-C 17 C D 27 4log2 bằng: 3log3 17 D 11 2-D Dạng 3: So sánh biểu thức lũy thừa, lôgarit Phương pháp giải Nếu a > a   a      Nếu a > log a b  log a c  b  c Nếu < a < a   a      Nếu < a < log a b  log a c  b  c Ví dụ minh hoa   Ví dụ 1: Nếu  a 2   thì: A a  1 C a  1 B a < D a  1 Hướng dẫn   Cách 1: Ta có  a2    1  1 a2    1 Mà   nên  a + <  a  1 Cách 2: Sử dụng máy tính CASIO fx570VN PLUS Tính giá trị   2.464   Chọn a  1 Tính giá trị    Chọn a = Tính giá trị  1 0  2.464 Nên loại đáp án D    6.071   Nên loại đáp án B C  Chọn A 1 Ví dụ 2: Cho p, q số thực thỏa mãn m    e A p  q B p > q 2p  q ; n  e p  2q , biết m > n So sánh hai giá trị p q C p  q D q > p Hướng dẫn 1 Ta có m    e 2p  q  eq  2p Vì m > n e > nên q – 2p > p – 2q  q > p  Chọn D Trang Ví dụ 3: Cho a 3 a 2 A < a < 1; < b < log b  log b Kết luận sau đúng? B < a < 1; b > C a > 1; < b < D a > 1; b > Hướng dẫn Ta có a 3 a Ta lại có log b 2    a 1 , mà 4  log b , mà   b  5  Chọn B Bài tập tự luyện Bài So sánh số sau a  log 2 b  log 2 A a  b Bài Nếu B a > b  3  A x   x C a  b D a = b C x  1 D x  1   thì: B x < Bài Trong mệnh đề sau, mệnh đề sai? (I): (III): 0,  0,3 (II): 2  4 (IV): A (I) (IV) B (I) (III) 5  3 5  3 C (IV) D (II) (IV) Bài So sánh A  log n  n  1 B  log n 1  n   , với số nguyên n > A A  B Đáp án B A < B 1-B 2-D 3-C C A = B D A > B 4–D Dạng 4: Biểu diễn biểu thức lôgarit Phương pháp giải Sử dụng cơng thức lũy thừa cơng thức lơgarit Ví dụ minh họa Ví dụ 1: Cho log2 = a, log3 = b Khi log15 theo a b bằng: A b – a + B b + a + C 6a + b D a – b + Hướng dẫn Cách 1: Ta có a  log  log 10  log10  log   log  log   a Mà log15  log  3.5   log  log  b   a Vậy đáp án A Cách 2: Sử dụng máy tính CASIO fx570VN PLUS Trang Bước 1: Nhập log2 SHIFT STO A để gán giá trị log2 cho A: log2  A Bước 2: Nhập log3 SHIFT STO B để gán giá trị log3 cho B: log  B Bước 3: Nhập đáp án chọn đáp án có kết Đáp án A, nhập log15 – (B – A +1) ta kết  Chọn A Ví dụ 2: Đặt a  log b  log Hãy biểu diễn log 45 theo a b? A log 45  a  2ab ab B log 45  2a  2ab ab C log 45  a  2ab ab  b D log 45  2a  2ab ab  b Hướng dẫn Cách 1: Ta có log 45  log  log  log   Vì log    log log log 2 2a a a  2ab        log log  log  b  b a  b  a  1 ab  b a a log log b   log log a Cách 2: Sử dụng máy tính CASIO fx570VN PLUS Bước 1: Nhập log SHIFT STO A để gán giá trị log cho A: log  3  A Bước 2: Nhập log SHIFT STO B để gán giá trị log cho B: log  3  B Bước 3: Nhập đáp án chọn đáp án có kết Đáp án A, nhập log 45  A  2AB ta kết khác nên loại đáp án A AB Đáp án B, nhập log 45  2A  2AB ta kết khác nên loại đáp án B AB Đáp án C, nhập log 45  A  2AB ta kết AB  B  Chọn C Bài tập tự luyện Bài Biết a = ln2; b = ln5 ln400 tính theo a b bằng: A 2a + 4b B 4a + 2b D b  a C 8ab Bài Cho a > 0, b > thỏa điều kiện a  b  7ab Khẳng định sau đúng? A 3log  a  b    log a  log b  C  log a  log b   log  7ab  Đáp án 1–B B log  a  b   D log  log a  log b  ab   log a  log b  2-D PHẦN 4: BÀI TẬP TỔNG HỢP Trang Bài Rút gọn biểu thức P  A P  a  b3 a 2  ab  ta được: a1 b 1 C P  B P  a b3 a3 b3 D P  a  b3 Bài Cho log x  Giá trị biểu thức P  log x  log x  log x bằng: A 11 2 2 C  B D Bài Cho hai số thực a b, với < a < b Khẳng định đúng? A log a b   log b a 2017 567 Bài Nếu A m  B  3 C log b a  log a b  D log b a   log a b 2 dạng x biểu thức dạng y Tính x  y Bài Viết biểu thức A B  log a b  log b a  2m  11 C 53 24 D 2017 576   thì: B m  C m  D m  2  4a  9a 1 a   3a 1  Bài Rút gọn biểu thức  với a >  1     2 a a  2a  3a  A 9a Bài Cho a + b = A B 9a C 3a D 3a C D 1 4a 4b  bằng: 4a  b  B Bài Cho a > 0, a  , biểu thức A   ln a  log a e   ln a  log a2 e có giá trị bằng: A ln a  Bài Có giá trị x thỏa mãn A 3  52  x 3x B   52  2x  C Bài 10 Cho a > 0, b > 0, viết log A C ln a  B 4lna + B  ab   D ln a  ? D x y log a  log b x + y bằng: 15 C D Bài 11 Biết x  4 x  23 , tính giá trị biểu thức P  x  2 x A B 27 C 23 D 25 Trang 10 Bài 12 Cho biểu thức P  a b a  ab , với số thực dương a b Rút gọn P kết  a4b 4a4b là: A B b a4b C b – a D a Bài 13 Cho a log  b log  c log  a , với a,b c số hữu tỷ Trong khẳng định sau, khẳng định đúng? A c = a C a = b = c  B a = b Bài 14 Cho a > 0, a  , biểu thức B  ln a  3log a e  A ln a  log a  có giá trị bằng: ln a log a e C 3ln a  B 4lna D b = c log a e D log a e Đáp án 1-B 2-C 3-D 4-D 11 - A 12 - A 13 - B 14 - C 5-C 6-B 7-D 8-A 9-C 10 - D Trang 11 CHƯƠNG 2: HÀM SỐ LŨY THỪA, HÀM SỐ MŨ VÀ HÀM SỐ LÔGARIT CHUYÊN ĐỀ 2: HÀM SỐ LŨY THỪA, HÀM SỐ MŨ, HÀM SỐ LÔGARIT PHẦN 1: LÝ THUYẾT TRỌNG TÂM Hàm số lũy thừa Hàm số lũy thừa có dạng y  x  ,     Tập xác định: Với α nguyên dương D =  Với α nguyên âm D   \ 0 Với α khơng ngun D   0;    Đạo hàm: y   x    x 1  Khảo sát hàm số tập  0;   : Hàm số y  x  (α > 0) Hàm số y  x  (α < 0) Luôn đồng biến Luôn nghịch biến Khơng có tiệm cận Tiệm cận ngang Ox Tiệm cận đứng Oy Luôn qua điểm 1;1 Luôn qua điểm 1;1 Đồ thị: Luôn nằm góc phần tư thứ I Hàm số mũ Hàm số mũ có dạng y  a x , a   Ta có y  a x  , x    Tập xác định: D =   Đạo hàm: y   a x   a x ln a  Khảo sát hàm số với a > 0, a  : Hàm số y  a x (a > 1) Hàm số y  a x (0 < a < 1) Luôn đồng biến Luôn nghịch biến Tiệm cận ngang Ox Tiệm cận ngang Ox Luôn qua điểm  0;1 ; 1;a  Luôn qua điểm  0;1 ; 1;a  Trang Đồ thị: Nằm phía trục Ox Đồ thị: Nằm phía trục Ox Hàm số lơgarit Hàm số lơgarit có dạng y  log a x ; a > 0, a   Tập xác định: D   0;    Đạo hàm: y   log a x   x.ln a  Khảo sát hàm số: Hàm số y  log a x (a > 1) Hàm số y  log a x (0 < a < 1) Luôn đồng biến Luôn nghịch biến Tiệm cận đứng Oy Tiệm cận đứng Oy Luôn qua điểm 1;0  ;  a;1 Luôn qua điểm 1;0  ;  a;1 Đồ thị: Nằm bên phải trục Oy Đồ thị: Nằm bên phải trục Oy PHẦN 2: CÁC CƠNG THỨC ĐẠO HÀM Cơng thức đạo hàm  x   x   e   e x  ln x     e   e u x u ,  x  0 x  log a x    u   x 1 ,  x  0 x.ln a  ln u   u u ,  u  0 u  log a u   u ,  u  0 u.ln a 1 u   a   a ln a  a   a ln a.u  ln x   1x  x    ln u   uu  u   x  log x a x    x  0 x.ln a u u   log a u   u  u  0 u.ln a Trang PHẦN 3: CÁC DẠNG BÀI TẬP Dạng 1: Tập xác định hàm số Ta sử dụng máy tính để tìm tập xác định hàm số tìm điều kiện để biểu thức lũy thừa, lơgarit xác định 1 Ví dụ minh họa Ví dụ 1: Hàm số y   x  2x  3 A x    xác định khi: C x > 1; x  3 B Không tồn x D 3  x  Hướng dẫn Hàm số y   x  2x  3  x  xác định x  2x      x  3  Chọn C Ví dụ 2: Tập xác định hàm số y  32x  x  A  ;1  1;   là: x 1 B  1;1 C 1;   D  0;   Hướng dẫn Hàm số y  32x  x  xác định x    x  x 1 Vậy tập xác định hàm số D   \ 1   ;1  1;    Chọn A Ví dụ 3: Với giá trị x hàm số y  f  x   ln   x  xác định? A x   2;  B x   2; 2 C x   \  2; 2 D x   \  2;  Hướng dẫn Hàm số y  f  x   ln   x  xác định  x   2  x   Chọn A Bài tập tự luyện Bài Tập xác định hàm số y   3x   A D =  2 là: B D   \ 2 Bài Tập xác định hàm số y   x  3x   C D   ;  e A D   ;1   2;   B D   \ 1; 2 Bài Tập xác định hàm số y  log A  0;1 B 1;   D D   ; 2  là: C D   0;   D D  1;  C  \ 0 D  ;0   1;   x 1 là: x Trang 1-B Đáp án 2-A 3-D Dạng 2: Đạo hàm hàm số Phương pháp giải Sử dụng cơng thức đạo hàm để tính tốn Ví dụ minh họa Ví dụ 1: Đạo hàm hàm số y  x  A 2x   52x.ln x B 2x  x  52x là: x  52x.ln 25 C 2x  x  2.52x.ln D 2x  x  52x.ln 25 Hướng dẫn Cách 1: Ta có hàm số y  x  x  52x 1 1  52x.ln 5.2  2x 2.52x.ln  2x   52x.ln 25  y  2x  2 x x x Cách 2: Sử dụng máy tính CASIO fx570VN PLUS Nhập SHIFT    , máy tính  d d   x  Sau nhập  X  X  52X  ta kết dx dx   X 1 82.22 Thử đáp án: Đáp án A: Nhập 2X   52X.ln , CALC X = 1, kết 41.99 Nên loại đáp án A X Đáp án B: Nhập 2X   52X.ln 25 , CALC X = 1, kết 82.22 X  Chọn B ex Ví dụ 2: Cho hàm số f  x    x   f    gần với giá trị giá trị sau: x 1 x A 11,1 B 11,1 C 10,11 D 10,11 Hướng dẫn Cách 1: Ta có hàm số f  x    f   2  e   1  e   1 e x  x  1  e x ex 1  x3    f   x    3x  2 x 1 x x  x  1  3.22   10,11 22 Cách 2: Sử dụng máy tính CASIO fx570VN PLUS Trang Nhập SHIFT    , máy tính   d d  eX  Sau nhập  X    ta kết   x   dx dx  X  X  X2 xấp xỉ 10,11  Chọn D Ví dụ 3: Cho hàm số f  x   log x  x biểu thức P  f   x   4x.f  x   3.f    f  1  Khi x biểu thức P là: A 4xlog x  4x  log  6 x C 4x log x  4x  log  6 x B 4x log x  4x  log  6 x D 4x log x  4x  log  6 x Hướng dẫn Ta có hàm số f  x   log x  x Đạo hàm: f   x    x  0  f    log  1   f  1  1  1 x ln 1.ln ln Khi P  f   x   4x.f  x   3.f    f  1   1     4x  log x  x    log      1  x ln x  ln   1   4x log x  4x  3log    1 x ln x ln x  4x log x  4x  log  6 x  Chọn D Ví dụ 4: Cho hàm số y  x.e A xy  1  x  y  x2 Hệ thức hệ thức sau: B x.y  1  x  y C xy  1  x  y D xy  1  x  y Hướng dẫn  Cách 1: Ta có y   x  e Suy x.y  x 1  x  e  x2 x2  x  x. e   ' x x       e  x.  xe     2 x x  x   2   e  x e  1  x  e   2  1  x  y Cách 2: Sử dụng máy tính CASIO fx570VN PLUS Chọn x = 2: Nhập X.e  X2 , CALC X = 2, kết quả, nhập SHIFT STO  A Trang X  d  Nhập  X.e dx    , kết quả, nhập SHIFT STO  B  X2 Thử đáp án: Đáp án A: Nhập XY  1  X  B , CALC X = 2; Y = A, kết khác 0, nên loại Đáp án B: Nhập X.B  1  X  Y , CALC X = 2; Y = A, kết khác 0, nên loại Đáp án C: Nhập XY  1  X  B , CALC X = 2; Y = A, kết khác 0, nên loại Đáp án D: Nhập X.B  1  X  Y , CALC X = 2; Y = A, kết gần 0, nên chọn  Chọn D Bài tập tự luyện Bài Đạo hàm hàm số y  sin x  log x (x > 0) là: A y  cos x  x ln B y   cos x  x ln C y  cos x  x ln 3 D y   cos x  x ln 3 Bài Cho hàm số y  ex  e  x Nghiệm phương trình y  A x = Bài Cho hàm số y  ln A xy   e y B xy   e y e Đáp án e 2x 2x  1 1-A D x = ln2 Khẳng định sau đúng? x 1 Bài Đạo hàm hàm số y  A y  C x  1 B x = D xy   e y ex  e x là: ex  e x B y  2-C C xy   e y e 4e 2x 2x 3-D  1 C y  e 2e 2x 2x  1 D y  e 3e 2x 2x  1 4-B Dạng 3: Đồ thị hàm số Ví dụ minh họa Ví dụ 1: Hình bên đồ thị ba hàm số y  log a x , y  log b x , y  log c x (  a, b, c  ) vẽ hệ trục tọa độ Khẳng định sau khẳng định đúng? A b > c > a B a > b > c C b > a > c D a > c > b Trang Hướng dẫn Do y  log a x y  log b x hai hàm đồng biến nên a,b > Do y  log c x nghịch biến nên c < Nên c nhỏ a m  x1 log a x1  m Lấy y = m, tồn x1 , x > để   m log b x  m b  x Dễ thấy x1  x  a m  b m  a  b Vậy b > a > c  Chọn C Ví dụ 2: Tìm giá trị lớn hàm số f  x   x e x đoạn  1;1 ? A e B e C 2e D Hướng dẫn  x    1;1 Cách 1: Ta có f   x   2x.e x  x e x  xe x  x    f   x      x  2   1;1 Ta lại có f  1  ; f    ; f 1  e Suy max f  x   e x 1;1 e Cách 2: Sử dụng máy tính CASIO fx570VN PLUS Ta sử dụng máy tính để tìm GTLN hàm số chương  Chọn A Ví dụ 3: Hình bên đồ thị ba hàm số y  a x , y  b x , y  c x (  a, b, c  ) vẽ hệ trục tọa độ Khẳng định sau khẳng định đúng? A a > b > c B b > a > c C a > c > b D c > b > a Hướng dẫn Do y  a y  b hai hàm đồng biến nên a,b > x x Do y  c x nghịch biến nên c < Nên c nhỏ a m  y1 Lấy x = m, tồn y1 , y > để  m Dễ thấy y1  y  a m  b m  a  b b  y Vậy b > a > c  Chọn B Trang Bài tập tự luyện Bài Tìm tất giá trị thực a để hàm số y  log a x (  a  ) có đồ thị hình bên A a  C a  B a   2 D a    Bài Cho hàm số y  x ln x   x   x Khẳng định sau khẳng định sai? A Hàm số xác định khoảng  0;   B Hàm số tăng khoảng  0;   C Hàm số giảm khoảng  0;   D Hàm số có đạo hàm y  ln x   x Bài Trong bốn hàm số y    x 1 ; y  3x ; y  log x ; y  x  x   x , có hàm số mà đồ thị x2 có đường tiệm cận? A B Đáp án 1-A 2-C C D 3–D PHẦN 4: BÀI TẬP TỔNG HỢP Bài Với giá trị x để hàm số y  log x  x  12 có nghĩa? B x   4;3 A x   Bài Tập xác định y  2x  5x   ln A D  1; 2  x  4 C  x  D x   ; 4    3;   C D   1;1 D D   1;  là: x 1 B D  1; 2 Bài Tính đạo hàm hàm số y  log  x   A y   x  1 ln B y  2x  x  2 C y  2x ln  x  2 D y  2x  x   ln Bài Tính đạo hàm hàm số y   x  2x  e  x A y    x   e  x B y   x   e  x C y  xe  x D y   2x   e x Trang Bài Đồ thị sau hàm số nào? A y    x 1 B y    2 x C y    1 D y    3 x x Bài Đồ thị hình bên hàm số nào? A y  log x  Đáp án: 1-D B y  log  x  1 2-A 3-D C y  log x  4-A 5-D D y  log  x  1 6-D Trang CHƯƠNG 2: HÀM SỐ LŨY THỪA, HÀM SỐ MŨ VÀ HÀM SỐ LƠGARIT CHUN ĐỀ 3: PHƯƠNG TRÌNH MŨ PHẦN 1: LÝ THUYẾT TRỌNG TÂM Phương trình mũ có dạng a x  b , ( a  0, a  )  Nếu b  , phương trình vơ nghiệm  Nếu b > 0, phương trình có nghiệm x  log a b Cách sử dụng máy tính CASIO fx570VN PLUS Ví dụ: Giải phương trình mũ f  x   Giải phương trình mũ 32x 1  4.3x   2x 1 x Nhập f  X  , SHIFT SOLVE = , máy tính Nhập  4.3  , SHIFT SOLVE = , máy tính X0 Xa , tức x = nghiệm phương x = a nghiệm LR 0 LR 0 trình f X  , SHIFT SOLVE = , máy tính Nhập 32x 1  4.3x  Nhập , SHIFT SOLVE = , máy tính Xa X0 Xb X  1 x = b nghiệm , tức x  1 nghiệm LR 0 LR 0 f X Nhập , SHIFT SOLVE = , máy phương trình  X  x1  X  x  32x 1  4.3x  Nhập , SHIFT SOLVE = , máy tính Xc X  X  1 tính , x =c nghiệm LR 0 L  R  1,896461.1014  , nên dừng lại phương trình Vậy phương trình có nghiệm x = x  1 Cứ làm máy tính Xm ; n  x=m khơng phải nghiệm LR n phương trình ta dừng lại PHẦN 2: CÁC DẠNG BÀI TẬP Dạng 1: Giải phương trình mũ phương pháp đưa số lơgarit hóa Phương pháp giải Phương trình a f x a  0, a    b  b  f x  log b a    a  0, a  f x g x Phương trình a    a    a   f  x   g  x  Trang Phương trình a f  x   b g x   log a a f  x   log a b g x   f  x   g  x  log a b  Nếu a.b   b  a  0, a   a 1  a f  x   b g x   a f  x   a  g x   a   a f  x   g  x  Ví dụ minh họa Ví dụ 1: Cho phương trình x A 3x   64 , tổng nghiệm thực phương trình là: B C D Hướng dẫn Ta có x 3x   64  x 3x   3  x1   43  x  3x    x  3x      3 x2    x1  x   Chọn B Ví dụ 2: Số nghiệm phương trình mũ  2x  1 A x 7 x B   2x  1 3x  là: C D Hướng dẫn Ta có  2x  1 x 7 x   2x  1 3x  2x     2x   2x    x  7x  3x    1   x  x  x      x  x   x   x  x     x  4x    x      x   lo¹i      Chọn C Ví dụ 3: Cho phương trình: 3x 3  32 x 1 Khẳng định sau đúng? A Tích nghiệm phương trình số âm B Tổng nghiệm phương trình số nguyên C Nghiệm phương trình số vơ tỉ D Phương trình vơ nghiệm Hướng dẫn Ta có 2 3x 3  32 x 1  2 3x 3  25 x 1 5x     3x   5x    3x   5x    3x   5x  Trang x  x     x    3x  5x         x  lo¹i    3x  5x     x    x   x 1   x   lo¹i     Vậy tập nghiệm phương trình là: S  1  Chọn B Bài tập tự luyện Bài Nghiệm phương trình x  x 1  3x  3x 1 A x  log 3 B x  C x  D x  log 3 Bài Cho phương trình e3 2x  e 1 2x   , khẳng định sau đúng? A Phương trình có nghiệm B Phương trình vơ nghiệm C Phương trình có hai nghiệm dương D Phương trình có hai nghiệm âm Đáp án 1–D 2–A Dạng 2: Giải phương trình mũ phương pháp đặt ẩn phụ Phương pháp giải Phương trình có dạng A.a 2x  B.a x  C  Đặt a x  t ,  t   Phương trình có dạng A.a 2x  B.a x b x  C.b 2x  2x x x a a a  A    B    C  Đặt    t ,  t   b b b Phương trình có dạng A.a x  B.b x  C  với a.b = Ta có a.b   b  1 B  b x  x Khi phương trình có dạng A.a x  x  C  a a a Đặt a x  t ,  t   Ví dụ minh họa Ví dụ 1: Phương trình 25x  5x 1   có tổng nghiệm A log B C log D log  log Hướng dẫn Phương trình 25x  5x 1     52   5.5x     5x   5.5x   x (1)  t   tháa m·n  Đặt t  5x  Khi (1)  t  5t      t   tháa m·n  5 x   x  log  x   log  log  log  x  log 5   Chọn C Trang x 1 x Ví dụ 2: Phương trình A       có hai nghiệm x1  x Tính A  x12  6x1x  36  B log D log 62 C log Hướng dẫn x 2    6.6 x   x     x   6.6 x   Ta có 61 x      x    36   6x   t   tháa m·n  Đặt  t , t  Khi phương trình trở thành 5t  6t      t   tháa m·n   x 6x  x    x 1 Vì x1  x nên x1   log ; x  6   x  log   log   Vậy A  x12  6x1x    log     log   log 62  Chọn D  Ví dụ 3: Phương trình  A Lớn   1   x x  có nghiệm thỏa mãn B Nhỏ C Lớn D Nhỏ Hướng dẫn  Ta có   Đặt t     1    x x x    x    1   1    x 6 (1)  t   Khi phương trình (1) trở thành: t  t2  t      x  log 1     t  3  lo¹i   Chọn B Bài tập tự luyện Bài Tập nghiệm phương trình 22x  3.2 x   32  là: A 2;3 B 4;8 C 2;8 D 3; 4 Bài Tập nghiệm phương trình 6.4 x  13.6 x  6.9 x  là: 2 3 B  ;  3 2 A 1;1 C 1;0 D 0;1 Bài Tập nghiệm phương trình e6x  3e3x   là:  ln  B 0;    A 0;ln 2 Đáp án 1–A 2–A  ln  C 1;    D 1;ln 2 3–B Trang Dạng 3: Giải phương trình mũ phương pháp khác Phương pháp giải Một số phương pháp khác để giải phương trình mũ là:  Đưa dạng phương trình tích  Phương pháp hàm số (thường sử dụng gặp phương trình mũ phức tạp) Ví dụ minh họa Ví dụ 1: Tổng nghiệm phương trình 12.3x  3.15x  5x 1  20 là: A log  C log  B log D log  Hướng dẫn Sử dụng phương pháp đưa dạng phương trình tích Ta có 12.3x  3.15x  5x 1  20  12.3x  3.15x  5.5x  20   3.3x   5x    5x      5x   3.3x    5x  phương trình vô nghiệm x 3   5 x    x 3.3    x  log  log  log 3  log   Chọn A x 2 Ví dụ 2: Phương trình A 5 x 12  141 có nghiệm? B C D Hướng dẫn Sử dụng phương pháp hàm số Phương trình có điều kiện xác định x    x  Xét hàm số f  x   Ta có f   x   x 2 x 2 5 x 12 xác định nửa khoảng  2;   x ln 5 x2 12 x ln Với x   f   x   suy hàm số đồng biến nửa khoảng  2;   Mà ta thấy f    141 suy phương trình có nghiệm x =  Chọn C Dạng 4: Phương trình mũ chứa tham số Ví dụ minh họa Ví dụ 1: Phương trình x 4  82x  m có nghiệm khi: Trang A m   13 B m   13 C m   25 12 D m  Hướng dẫn Cách 1: Ta có x 4  82x  m  x 4  26x 3m  x   6x  3m  x  6x   3m  (1) Phương trình ban đầu có nghiệm phương trình (1) có nghiệm Ta có:      3m   m  13 Cách 2: Sử dụng máy tính CASIO fx570VN PLUS Thử đáp án + sử dụng máy tính để tìm nghiệm phương trình Chọn m = Thay vào phương trình, ta thấy có hai nghiệm phân biệt Nên loại đáp án B D Chọn m   13 Thay vào phương trình, ta thấy có nghiệm  Chọn A Ví dụ 2: Phương trình x 1  x   x  m  10 có nghiệm nguyên khi: A m = B m = C m = D m = Hướng dẫn Ta có x 1  x   x  m  10  2.2 x  4.2 x  x.2m  10  x   2m   10  x  10  2m Thử đáp án, ta thấy m = x = nghiệm nguyên phương trình  Chọn C Ví dụ 3: Cho phương trình 25x  2.15x   m   x  Tìm điều kiện tham số m để phương trình có hai nghiệm phân biệt A m   2;3 B m   2;3 D m   2;3 C m   2;3 Hướng dẫn Cách 1: Phương trình 25x  2.15x   m   x   52x  2.5x.3x   m   32x  2x x 5 5        m   3 3 x 5 Đặt t     t   , ta có phương trình t  2t  m   3 (1) Phương trình cho có hai nghiệm phân biệt phương trình (1) có hai nghiệm phân biệt dương  1   m        m   m    2 Ta có: S    0     m  m   m  P    m    Trang Cách 2: Sử dụng máy tính CASIO fx570VN PLUS Chọn m = Thay vào phương trình, ta thấy phương trình khơng có hai nghiệm phân biệt dương Nên loại đáp án A B Chọn m = Thay vào phương trình, ta thấy phương trình khơng có hai nghiệm phân biệt dương  Chọn C Bài tập tự luyện Bài Xác định tất giá trị thực m để phương trình 22x 1  m  m  có nghiệm A m < m  C  m  B < m < D m > Bài Phương trình x  m.2 x 1  2m  có hai nghiệm x1 ; x thỏa mãn x1  x  khi: A m = Đáp án B m = 1–B C m = D m = 2–A PHẦN 3: BÀI TẬP TỔNG HỢP Bài Cho phương trình 28 x 58 x  0, 001 105  A 1 x C 7 B Bài Cho phương trình x A 2  x 1  10.3x  x 2 Bài Phương trình A m  Bài Phương trình 2 x 2 C 3 A D  m  có nghiệm điều kiện m là: C m  B m   D 5   Tổng tất nghiệm phương trình là: B x 1 Tính tổng nghiệm phương trình   x 3    10  x B x D m  có tất nghiệm thực? C D Bài Tọa độ giao điểm đồ thị hàm số y  2 x  đường thẳng y = 11 là: A  3;11 B  3;11 x   Bài Số nghiệm phương trình     3 A 2  k  k   2 x D  4;11 2x  B Bài Phương trình 9sin x  9cos A x  C  4;11   là: C D  có họ nghiệm là: B x   k  k   2 C x   k  k   D x   k  k   Bài Để phương trình  m  116 x   2m  3 x  6m   có hai nghiệm trái dấu m là: A Khơng tồn m B 4  m  1 C 1  m  D 1  m   Bài Với giá trị tham số m phương trình x  m.2 x 1  2m  có hai nghiệm x1 , x thỏa mãn x1  x  ? Trang A m = B m = Bài 10 Phương trình x 3  3x A 3x1  2x  log 5x  C m = D m = có hai nghiệm x1 , x , trrong x1  x , chọn phát biểu đúng? B 2x1  3x  log C 2x1  3x  log 54 D 3x1  2x  log 54 Bài 11 Tổng lập phương nghiệm phương trình x  2.3x  x  là: A 2 B 25 C Bài 12 Tổng nghiệm phương trình 16 A x 10 x 10 B 10  0,125.8 D x 5 x 15 là: C 20 D 25 Đáp án: 1–A 2–A 11 – D 12 – C 3–C 4–A 5–B 6–A 7–A 8–B 9–A 10 – A Trang CHƯƠNG 2: HÀM SỐ LŨY THỪA, HÀM SỐ MŨ VÀ HÀM SỐ LÔGARIT CHUYÊN ĐỀ 4: PHƯƠNG TRÌNH LƠGARIT PHẦN 1: LÝ THUYẾT TRỌNG TÂM Phương trình lôgarit với a > 0; a  có dạng: log a x  b  x  a b (điều kiện: x > 0) Chú ý:  Khi giải phương trình lơgarit, phải đặt điều kiện cho ẩn: log f  x   mò lỴ  a   mò ch½n loga f  x   ®iỊu kiƯn f  x    ®iỊu kiƯn f  x     Kết luận nghiệm, phải so sánh nghiệm với điều kiện  Ta bấm máy tính để giải phương trình lơgarit giải phương trình mũ PHẦN 2: CÁC DẠNG BÀI TẬP Dạng 1: Giải phương trình lơgarit phương pháp đưa số mũ hóa Phương pháp giải f  x   Phương trình log a f  x   b   , với  a  b f  x   a f  x   Phương trình log a f  x   log a g  x    , với  a  f x  g x      f  x   Phương trình log a f  x   g  x    g x  f x  a    Ví dụ minh họa Ví dụ 1: Phương trình log  2x  3  log 5.log  x  1  có số nghiệm là: A B C D Hướng dẫn 2x   Điều kiện:  x x   Ta có log  2x  3  log 5.log  x  1   log  2x  3  log  x  1   log  2x  3  log  x  1   log  2x  3 x  1     2x  3 x  1  31  2x  2x  3x    x   2x  x     x    2 Kết hợp điều kiện, ta nghiệm x =  Chọn A Trang x  3x  Ví dụ 2: Cho phương trình log  có hai nghiệm x1 , x Tích hai nghiệm số x đây: B 2 A C D Hướng dẫn Điều kiện: x  3x  0 x x  3x  x  3x    Ta có log 0   x x 6  x   x  3x    x  4x     x  x      Ta thấy hai nghiệm x1 , x thỏa mãn điều kiện Vậy x1x   2      Chọn C Ví dụ 3: Cho phương trình log x.log x.log 27 x.log81 x  Tổng giá trị tất nghiệm phương trình là: A B 80 C D 82 Hướng dẫn Điều kiện: x > Ta có log x.log x.log 27 x.log81 x  1   log x   x1  x   2  log x.log 32 x.log 33 x.log 34 x  3 x1  32   tháa m·n  log x      log 34 x  16   x  32   tháa m·n  log x  2  82  9  Chọn D Bài tập tự luyện Bài Phương trình log x  log  x  1  có tập nghiệm là: A 1;3 Bài Phương trình log A B 1;3 C 2 D 1 x   có nghiệm? B C D Bài Phương trình log  x    log  log x  log có hai nghiệm x1 , x ,  x1  x  Tỉ số x1 x2 rút gọn là: Trang A B Đáp án: 1–C 2–A C 64 D 64 3–D Dạng 2: Giải phương trình lơgarit phương pháp đặt ẩn phụ Phương pháp giải  A.log a2 x  B.log a x  C  Phương trình có dạng  Đặt log a x  t với x >  A.log a  B.log a x  C.log a x  D  Ví dụ minh họa Ví dụ 1: Tập nghiệm phương trình log 22  x    log x    là: A S  2; 4 B S  0; 2 C S  1; 2 D S  4;6 Hướng dẫn  x   Điều kiện:   x   x   Ta có: log 22  x    log x     log 22  x    3log  x     Đặt t  log  x   , phương trình trở thành: x   tháa m·n  log  x    t  x   t  3t        x  tháa m·n    t  log  x    x     Chọn D Ví dụ 2: Tìm nghiệm lớn phương trình log x  log x  log x  là: A x = 10 B x = 100 C x = D x = 1000 Hướng dẫn Phương trình log x  log x  log x  có điều kiện x > Đặt log x  t Khi phương trình trở thành: t  t2 1  t  2t  t   t  t     t      t  1  t        t  1 t    t  2 x  10  tháa m·n  log x     log x  1  x   tháa m·n   10 log x   x  100  tháa m·n   Chọn B Trang Ví dụ 3: Số nghiệm phương trình  log log  x  1   log log  x  1   là: A B C D Hướng dẫn x   x    Điều kiện log  x  1   log  x  1    log  x  1  log  x  1  Ta có  log log  x  1   log log  x  1     log 22 log  x  1   log log 22  x  1   1    log log  x  1   log  log  x  1   2  Đặt t  log  x  1 , phương trình trở thành: 1 1   log t  log  t     log t  log  log t  2 2   log t    log t   t  28  256  log  x  1  256  x   2256  x  2256  (thỏa mãn)  Chọn A Bài tập tự luyện Bài Số nghiệm phương trình log  log x   log  log x   là: A B C D Bài Phương trình log 22  x  1  log x    có tổng hai nghiệm là: A B C 10 D Bài Nghiệm nhỏ phương trình log 32 x  log 22 x  log x  là: B x  A x = 4 C x = D x  Bài Gọi x1 , x nghiệm phương trình log x  log16 x  Khi tích x1.x bằng: A 1 Đáp án: B 1–A 2–B C 3–D D 2 4–B Dạng 3: Giải phương trình lơgarit phương pháp khác Phương pháp giải Một số phương pháp khác để giải phương trình lơgarit là: Đưa dạng phương trình tích Phương pháp hàm số (thường sử dụng gặp phương trình lơgarit phức tạp) Ví dụ minh họa Trang Ví dụ 1: Gọi nghiệm phương trình log x.log  2x  1  log x x1  x Khi đó, giá trị 2x1  5x  là: A 10 C 20 B 15 D 30 Hướng dẫn Sử dụng phương pháp đưa dạng phương trình tích x  Điều kiện xác định:  x 2x   Ta có log x.log  2x  1  log x  log x.log  2x  1  log x  log x  log x   log x log  2x  1       log  2x  1   log  2x  1   x  20  x  1(tháa m·n)   x  5(tháa m·n)   2x    Vậy 2x1  5x   2.1  5.5   20  Chọn C Ví dụ 2: Phương trình log  x  2x  17   log  5x  1  có nghiệm thỏa mãn: A Lớn B Nhỏ C Là số nguyên tố D Là số âm Hướng dẫn Sử dụng phương pháp hàm số  x  2x  17  Điều kiện:  x 5x   Xét hàm số f  x   log  x  2x  17   log  5x  1 với x  Ta có f   x   2x  Ta thấy f   x   , x   2 ln 4. 5x  1 ln 2. x  2x  17  1  Nên hàm số f  x  đồng biến khoảng  ;   5  Mà ta có f 1  , nên x = nghiệm phương trình ban đầu  Chọn A Dạng 4: Phương trình lơgarit chứa tham số Ví dụ minh họa Ví dụ 1: Tìm m để phương trình log x  m log x   có nghiệm nhỏ A m = B Không tồn m C m  2 D m  2 Hướng dẫn Điều kiện: x > Trang Phương trình log x  m log x   (1) Đặt t  log x , phương trình (1) trở thành t  mt   (2) Phương trình (1) có nghiệm phương trình (2) có nghiệm lớn m  Khi phương trình (2) có nghiệm kép   2   m      m  2 Với m =  t  2t    t   log x   x   (loại) 1 Với m  2  t  2t    t  1  log x  1  x  (thỏa mãn)  Chọn C Ví dụ 2: Tìm m để phương trình log 22 x  log x  m  có hai nghiệm x   0;1 A  m  B m  C m  D m  Hướng dẫn Phương trình log x  log x  m  (x > 0) (1) Đặt t  log x , phương trình (1) trở thành t  t  m  (2) 2 Vì x   0;1  log x   t  Phương trình (1) có hai nghiệm x   0;1 phương trình (2) có hai nghiệm âm khi:  2  1  4m    0m S   1  P  m     Chọn A Ví dụ 3: Cho phương trình log 22 x  log 2x  m  Giá trị tham số m để phương trình có nghiệm là: A m  B m   D m  2 C m < Hướng dẫn Điều kiện: x > Ta có: log 22 x  log 2x  m   log 22 x   log 2  log x    m  log 22 x  log x   m Đặt t  log x , phương trình có dạng: t  2t   m   t  1  m  Nên phương trình có nghiệm m    m  2  Chọn D Ví dụ 4: Tìm điều kiện tham số m để phương trình log  m  4x   log  x    có nghiệm đoạn  2;5 Trang A m   20;69 B m   24;69 D m  10;70 C m  10;70  Hướng dẫn Ta có log  m  4x   log  x      log  m  4x   log  x    2  log  x    log  m  4x    x    m  4x  m  x  8x  2 Xét hàm số f  x   x  8x  đoạn  2;5 Ta có f   x   2x   ; x   2;5 , hàm số ln đồng biến đoạn  2;5 Mặt khác, ta có f    24 ; f    69 nên 24  f  x   69 Vậy với m   24;69 phương trình cho có nghiệm đoạn  2;5  Chọn B Bài tập tự luyện Bài Cho phương trình log  x  1  log  x  3  m Tìm tất giá trị thực tham số m để phương trình có nghiệm kép B m   A m  C m > D m   Bài Tìm tất giá trị thực tham số m để phương trình log 24 x  3log x  2m   có hai nghiệm phân biệt A m  13 B m  13 C m  13 D  m  13 Bài Tìm tất giá trị thực tham số m để phương trình log 32 x  log x  m   có nghiệm B m  A m < C m  D m > Bài Tìm tất giá trị thực tham số m để phương trình log  mx  x   vô nghiệm Đáp án: m  C   m  4 B m  4 A m < 1–A 2–A 3–B D 4  m  4–D PHẦN 3: BÀI TẬP TỔNG HỢP Bài Điều kiện xác định phương trình log 2x 3 16  là: 3  A x   \  ;  2  B x  C x2 D x  Bài Phương trình log  x  3  log  x  1  log có số nghiệm là: A B C D Bài Cho phương trình log  5x  3  log  x  1  có nghiệm x1 , x x1  x Giá trị P  2x1  3x là: Trang A B 14 C D 13   Bài Biết phương trình log log  x   log x  x  1  có nghiệm Nghiệm phương   trình là: A Số nguyên âm B Số phương C Số vô tỉ D Số nguyên tố Bài Điều kiện xác định phương trình log log   x    là: A x   1;1 B x   1;0    0;1 Bài Phương trình ln C x   1;1   2;   D x   1;1 C x = D x = x 1  ln có nghiệm là: x 8 x x  B   x  2 A x  2 Câu Tập nghiệm phương trình A 0 log  x     là: B 0; 4 C 4    D 1;0  Bài Cho phương trình log x  x  log x  x   log x  x  Điều kiện xác định phương trình là: B x  0; x  A x  1 C x  D x  1 x  Bài Phương trình log 2x 3  3x  7x  3   có nghiệm là: A x = 2; x = B x = 1; x = C x = D x =   x2   ta tìm hai nghiệm x1 , x Khi tích x1.x Bài 10 Phương trình log  9x    log 81   là: A 38 B 93 C 93 D 36 Bài 11 Tìm tất giá trị thực tham số m để phương trình log 32 x   m   log x  3m   có hai nghiệm x1 , x thỏa mãn x1.x  27 A m  2 B m  1 C m = D m = Bài 12 Tìm tất giá trị thực tham số m để phương trình log  5x  1 log  2.5x    m có nghiệm x  A m   2;   B m  3;   C m   ; 2 D m   ;3 Đáp án: 1–C 2–A 3–B 4–D 5–A 6–C 7–B 8–C 9–D 10 – B 11 – C 12 – B Trang CHƯƠNG 2: HÀM SỐ LŨY THỪA, HÀM SỐ MŨ, HÀM SỐ LOGARIT CHUYÊN ĐỀ 5: CÁC BÀI TỐN LÃI SUẤT PHẦN 1: CƠNG THỨC TÍNH NHANH Bài tốn Cơng thức Bài tốn lãi đơn S  A 1  r.n  Bài toán lãi kép S  A 1  r  Bài toán tăng trưởng dân số A m  A n e m  n .r Bài toán tăng lương 1  r  S  Ak n k 1 r 1  r  Gửi tiền đầu tháng S  A.1  r  Rút tiền gửi hàng tháng S  A 1  r   X Vay vốn trả góp S  A 1  r   X n n n 1 r 1  r  n 1 n 1 r 1  r  r PHẦN 2: CÁC DẠNG BÀI TẬP Dạng 1: Bài toán lãi đơn Phương pháp giải Số tiền lãi tính số tiền gốc mà khơng tính số tiền lãi số tiền gốc sinh Cơng thức tính lãi đơn S  A 1  r.n  Trong đó: S số tiền vốn lẫn lãi sau n kì hạn A tiền gửi ban đầu n số kì hạn tính lãi r lãi suất định kì tính theo % Ví dụ minh họa Ví dụ: Bà An gửi vào ngân hàng 10 triệu đồng với lãi suất đơn 7% năm sau năm số tiền bà An nhận vốn lẫn lãi bao nhiêu? A.13,5 triệu đồng B 16 triệu đồng C 12 triệu đồng D 12,7 triệu đồng Hướng dẫn Số tiền gốc lẫn lãi bà An nhận sau năm là: S  10 1  5.7%   13,5 (triệu đồng)  Chọn A Dạng 2: Bài toán lãi kép Phương pháp giải Số tiền lãi kỳ hạn trước người gửi không rút tính vào vốn để tính lãi cho kì hạn sau Trang Cơng thức tính lãi kép S  A 1  r  Trong đó: n S số tiền vốn lẫn lãi sau n kì hạn A tiền gửi ban đầu n số kì hạn tính lãi r lãi suất định kì tính theo % Ví dụ minh họa Ví dụ 1: Một người gửi tiết kiệm với lãi suất 7,5% năm lãi hàng năm nhập vào vốn Hỏi sau năm người thu gấp đôi số tiền ban đầu? A năm B 10 năm C năm D năm Hướng dẫn Gọi A số tiền gửi ban đầu, r  7,5% /năm lãi suất, n số năm gửi Ta có cơng thức lãi kép S  A 1  r  số tiền nhận sau n năm n Theo đề bài, ta có S  2A  2A  A(1  r) n  (1  r) n   n  log 1 r   log1,075  9,584 Do kỳ hạn năm nên phải hạn nhận, nên người cần 10 năm  Chọn B Ví dụ 2: Ơng A gửi tiết kiệm 75 triệu đồng vào ngân hàng theo kì hạn tháng lãi xuất 0,58% tháng Nếu ông A không rút lãi tất định kỳ sau năm ông A nhận số tiền bao nhiêu? A 92576000 đồng B 80486000 đồng C 92690000 đồng D 90930000 đồng Hướng dẫn Đây toán lãi kép, chu kỳ quý lãi suất 3.0,59%  1, 77% Sau năm (12 quý), số tiền thu gốc lẫn lãi là: 75 000 000 1  0, 0177   92576000 (đồng) 12  Chọn A Ví dụ 3: Một người gửi 350 triệu đồng vào ngân hàng với lãi xuất 6,7% năm Biết không rút tiền khỏi ngân hàng sau năm số tiền lãi nhập vào gốc để tính lãi cho năm Hỏi sau năm, người nhận số tiền 800 triệu đồng bao gồm gốc lãi? Giả định suốt thời gian gửi, lãi xuất khơng đổi người khơng rút tiền A 13 năm B 14 năm C 11 năm D 10 năm Hướng dẫn Gọi n số năm người gửi để nhận số tiền 800 triệu đồng bao gồm gốc lãi Số tiền người nhận sau n năm là: S  350.1  6,7%   800  n  log 1 6,7%  n 800  n  12,747 350 Vậy sau 13 năm người nhận số tiền 800 triệu đồng bao gồm gốc lãi  Chọn A Dạng 3: Bài toán tăng trưởng dân số Trang Phương pháp giải Công thức tăng trưởng dân số A m  A n e m  n .r Trong đó: A m dân số năm m A n dân số năm n r tỉ lệ tăng dân số từ năm n tới năm m tính theo % Ví dụ minh họa Ví dụ: Sự tăng trưởng dân số ước tính theo cơng thức tăng trưởng mũ Biết tỉ lệ tăng dân số giới hàng năm 1,32%, năm 2003 dân số giới vào khoảng 7095 triệu người Dự đoán dân số năm 2010? A 7781 triệu người B 7782 triệu người C 7783 triệu người D 7784 triệu người Hướng dẫn Theo cơng thức tăng trưởng mũ dự đốn dân số năm 2010 là: A 2010  A 2003 e 2010 2003.1,32%  7781,82 triệu người  Chọn B Dạng 4: Bài toán tăng lương Phương pháp giải Một người nhận lương khởi điểm A đồng tháng Cứ sau n tháng, người tăng thêm r % tháng, số tiền người nhận sau kn tháng 1  r  S  A.k k 1 r Ví dụ minh họa Ví dụ: Một người lãnh lương khởi điểm triệu đồng/tháng Cứ tháng lương người tăng thêm 7% tháng Hỏi sau 36 tháng người nhận lương tất bao nhiêu? A  700 triệu đồng B  623 triệu đồng C  954 triệu đồng D  644 triệu đồng Hướng dẫn Vì tháng, người tăng lương lần, nên số lần tăng lương 12 lần Sau 36 tháng người nhận tiền lương là: 1  7%  S  3.12 12 7% 1  643,98 triệu đồng  Chọn D Dạng 5: Gửi tiền hàng tháng Phương pháp giải Mỗi tháng khách hàng gửi vào ngân hàng số tiền A đồng với lãi kép r % tháng vào thời gian cố định số tiền khách hàng nhận vốn lẫn lãi sau n tháng là: S  A.1  r  1  r  n 1 r Ví dụ minh họa Trang Ví dụ 1: Một người tháng đặn gửi vào ngân hàng khoản tiền A theo hình thức lãi kép với lãi suất 0,6% tháng Biết sau 15 tháng người có số tiền 10 triệu đồng Hỏi số tiền A gần với số tiền số sau? A 535 000 đồng B 635 000 đồng C 613 000 đồng D 643 000 đồng Hướng dẫn Theo công thức gửi tiền hàng tháng, ta có: 10 000 000  A 1  0, 6%  1  0, 6%  15 1 0, 6% Từ ta suy A  635 000 (đồng)  Chọn B Ví dụ 2: Hàng tháng, anh A gửi vào ngân hàng triệu đồng với lãi suất 0,6% tháng Hỏi sau tháng (khi ngân hàng tính lãi) anh A số tiền lãi gốc 100 triệu trở lên? A 30 tháng B 35 tháng C 31 tháng D 40 tháng Hướng dẫn Theo công thức gửi tiền hàng tháng, ta có: 100  1  0, 6%  1  0, 6%   n 1 0, 6%  1  0, 6%   n  1  0, 6%  n 1 0, 6% 100 100.0, 6% n  1  0, 6%    1  0, 6%  1  0, 6%   100.0, 6%  100.0, 6%   n  log 1 0,6%    1  30,3117 1  0, 6%   1  0, 6%   Vậy sau 31 tháng anh A số tiền lãi gốc 100 triệu trở lên  Chọn C Dạng 6: Rút tiền gửi hàng tháng Phương pháp giải Gửi ngân hàng với số tiền A đồng với lãi suất r% tháng Mỗi tháng vào ngày ngân hàng tính lãi, rút số tiền X đồng Số tiền lại sau n tháng S  A 1  r  n 1  r   X n 1 r Ví dụ minh họa Ví dụ: Mẹ Lam gửi ngân hàng 20 tỷ đồng với lãi suất 0,75% tháng Hàng tháng vào ngày ngân hàng tính lãi, mẹ Lam đến ngân hàng rút 300 triệu đồng để chi tiêu Hỏi sau năm số tiền lại ngân hàng bao nhiêu? A 11 tỷ đồng B 15 tỷ đồng C 13 tỷ đồng D 16 tỷ đồng Hướng dẫn Ta có năm 24 tháng Sau năm số tiền lại ngân hàng là: S  20 1  0, 75%  24 1  0, 75%   0,3 0, 75% 24 1  16, 07 (tỷ đồng) Trang  Chọn D Dạng 7: Vay vốn trả góp Phương pháp giải Vay ngân hàng với số tiền A đồng với lãi suất r % tháng Sau tháng kể từ ngày vay bắt đầu hoàn nợ số tiền X đồng, hai lần hoàn nợ cách tháng Số tiền nợ sau n tháng S  A 1  r  n 1  r   X n 1 r Ví dụ 1: Ông Minh vay ngắn hạn ngân hàng 200 triệu đồng, với lãi suất 12% năm Ơng muốn hồn nợ cho ngân hàng theo cách: Sau tháng kề từ ngày vay, ơng bắt đầu hồn nợ; hai lần hoàn nợ liên tiếp cách tháng, số tiền hoàn nợ lần trả hết tiền nợ sau tháng kể từ ngày vay Hỏi theo cách đó, số tiền mà ông Minh phải trả cho ngân hàng lần hoàn nợ bao nhiêu? Biết rằng, lãi suất ngân hàng không thay đổi thời gian ông Minh hoàn nợ A m  67 (triệu đồng) B m  69 (triệu đồng) C m  70 (triệu đồng) D m  68 (triệu đồng) Hướng dẫn Theo đề ta thấy, ông Minh vay tiền ngân hàng theo hình thức “Vay vốn trả góp” Số tiền mà ơng Minh phải trả sau n tháng với lãi suất 12% năm tức 1% tháng là: S  A 1  r  n n  1  r n    0, 01   n  X    200 1  0, 01  X r 0, 01   Vì sau ba tháng ơng Minh trả hết nợ nên:  200.1  0,01 X 1, 01 1, 01 3 1  0,01  X 1 0,01  200.1,01 1,01  X 1 0,01 1  68  Chọn D Ví dụ 2: Anh Ba vay trả góp ngân hàng số tiền 500 triệu đồng với lãi suất 0,9% tháng, tháng trả 15 triệu đồng Sau tháng anh Ba trả hết nợ? A 40 tháng B 50 tháng C 45 tháng D 48 tháng Hướng dẫn Gọi n số tháng anh Ba trả hết nợ Theo cơng thức vay vốn trả góp, ta có số tiền lại sau n tháng là: S  500 1  0,9%  n 1  0,9%   15 n 1 0,9% Vì sau n tháng anh Ba trả hết nợ, nên ta có S  , tức là: S  500 1  0,9%  n 1  0,9%   15 0,9% n 1  Từ suy n  39,809 Vậy sau 40 tháng anh Ba trả hết nợ  Chọn A Trang PHẦN 3: BÀI TẬP TỔNG HỢP Bài Anh B gửi 27 triệu đồng vào ngân hàng theo thể thức lãi kép, kì hạn quý, với lãi suất 1,85% quý Hỏi thời gian nhanh để anh B có 36 triệu đồng tính vốn lẫn lãi? A 19 quý B 15 quý C năm D năm Bài Đầu tháng chị N gửi vào ngân hàng số tiền tỷ đồng Sau năm chị N nhận số tiền gốc lãi 40 tỷ đồng Hỏi lãi suất ngân hàng phần trăm tháng? A 1,51% B 1,52% C 1,71% D 1,61% Bài Bố Lan gửi ngân hàng 20 triệu đồng với lãi suất 0,7% tháng Mỗi tháng vào ngày ngân hàng tính lãi, bố Lan rút số tiền để chi tiêu Hỏi số tiền tháng bố Lan rút để sau năm số tiền vừa hết? A 300 000 đồng B 450 000 đồng C 400 000 đồng D 409 000 đồng Bài Mẹ Lê vay trả góp ngân hàng số tiền 50 triệu đồng với lãi suất 1,15% tháng vòng năm tháng mẹ Lê phải trả tiền hết nợ? A 362 000 đồng B 240 000 đồng C 154 000 đồng D 680 000 đồng Đáp án: 1–C 2-D 3-D 4-A Trang CHƯƠNG 2: HÀM SỐ LŨY THỪA, HÀM SỐ MŨ, HÀM SỐ LOGARIT CHUYÊN ĐỀ: BẤT PHƯƠNG TRÌNH MŨ LƠGARIT PHẦN 1: LÝ THUYẾT TRỌNG TÂM Bất phương trình mũ có dạng a x  b; a x  b ;a x  b; a x  b Bất phương trình lơgarit có dạng log a x  b; log a x  b; log a x  b; log a x  b Chú ý: Ta sử dụng máy tính để giải thử đáp án cho tập giải bất phương trình mũ lơgarit tập phương trình mũ phương trình lơgarit PHẦN 2: CÁC DẠNG BÀI TẬP Dạng 1: Giải bất phương trình mũ lôgarit phương pháp đưa số Phương pháp giải Bất phương trình mũ: • Nếu a  a f  x   a g x   f  x   g  x  (cùng chiều a  ) • Nếu  a  a f  x   a g x   f  x   g  x  (ngược chiều  a  ) f x g x • Nếu a chứa ẩn a    a     a  1 f  x   g  x    (Điều kiện a  ) Bất phương trình lơgarit: g  x   • Nếu a  log a f  x   log a g  x    (cùng chiều a  ) f  x   g  x  f  x   • Nếu  a  log a f  x   log a g  x    (ngược chiều  a  ) f x  g x      log a f  x     a  1 f  x   1   • Nếu a chứa ẩn  log a f  x   log g x   f  x   1 g  x   1   a   Ví dụ minh họa  x   Ví dụ 1: Tập nghiệm bất phương trình     là:  14   14   1 A  0;   5  1 B  0;   5 1  C  ;  5   1 D  0;    0;    5 Hướng dẫn x     Điều kiện: x  Ta có      14   14  Vì 1 1  5x  nên 1       00x x x x 14 Trang  1 Vây tập nghiệm bất phương trình S   0;   5  Chọn B Ví dụ 2: Nghiệm lớn bất phương trình A x  log 3 2.3x  x   là: 3x  x C x  B x  D x  log 3 Hướng dẫn x Ta có 2.3x  x  3x  x 3    x x 2.3  4.2 1    x 1 x x 2 3   1 2 x 3    2 1  x 3   1 2 x 3 x   3 2 3           x  log 3 x 2 3   1 2 1 Vậy nghiệm lớn bất phương trình x  log 3  Chọn A Ví dụ 3: Bất phương trình log  x  x    log 0,5  x  1  có tập nghiệm là:  A ;1     B 1  2;   C ;1   D 1  2;  Hướng dẫn   x  1 x  x    Bất phương trình có điều kiện xác định:     x   x  x   x   Ta có log  x  x    log 0,5  x  1   log  x  x    log 21  x  1   log  x  x    log  x  1   log  x  x    log  x  1    log  x  x    log  x  1  log 2   log x   x    x  1 x  x    x  1 0    x  x    x  1   x  x  x  x  2x   x 2  x  2x  x   x  x  2x  1   x  2x    x    Loaïi    x  1  x    Thỏa mãn   Trang  Chọn D Ví dụ 4: Nghiệm nguyên nhỏ bất phương trình log3  log27 x   log9  log3 x  là: A 19863 B 19683 C 19638 D 19836 Hướng dẫn x  x   Điều kiện xác định: log 27 x     x  Ta có log3  log27 x   log9  log3 x  x  log x   1  log  log x   log  log x   log 33 32 1 1   log3 x   log3  log3 x  3  Đặt t  log3 x Khi bất phương trình trở thành: 1  1 1 log3  t   log3 t  log3  log3 t  log3 t   log3 t    log3 t   t  32  2 3   log x   x  39  19683 Vậy nghiệm nhỏ bất phương trình x  19683  Chọn B Bài tập tự luyện x 2x 1 Bài Tập nghiệm bất phương trình    x 1 là:  16   x  2 A   1  x  B x  2 C 1  x  Bài Nghiệm nguyên dương bất phương trình 11 x 6 D 1  x   11x là: A S  6; 5; 4; 3; 2;0;1; 2;3 B S  0;1; 2;3 C S  1; 2 D S  1; 2;3 Bài Cho bất phương trình log  l  x   log 1  x  Nghiệm nguyên nhỏ bất phương trình là: A x  1 B x  C x  D x  1 2 Bài Bất phương trình log 0,2 x  5log 0,2 x  6 có tập nghiệm là:  1  ;  B S    125 25  A S   2;3   C S   0;   25  D S   0;3 Đáp án: 1–A 2–D 3–C 4–B Dạng 2: Giải bất phương trình mũ lơgarit phương pháp đặt ẩn phụ Ví dụ minh họa Trang Ví dụ 1: Tập nghiệm bất phương trình 16 x  x   là: A x  B x  log C x  log D x  Hướng dẫn Đặt t  x  t   , bất phương trình cho tương đương với: t 0 t  t    2  t    t    x   x  log  Chọn C x 1 Ví dụ 2: Số phương nhỏ nghiệm bất phương trình A x  16 2 2x 1  12  là: C x  B x  x D x  Hướng dẫn x x x Ta có 3x 1  22x 1  12   3.3x  2.22x  12   3.3x  2.4 x  12  x x x x       16     3.9  2.16  12                     3      x x x x  2 Đặt    t; t  , bất phương trình trở thành: 3 t  Kết hợp với điều kiện t  , suy t   2.t  t    t    2 x  2 Từ đó, ta có     x  log  3 Vậy số phương nhỏ nghiệm bất phương trình  Chọn D Ví dụ 3: Tập nghiệm ngun bất phương trình A S  1;0;1 B S  1 x  21 x  là: C S  0;1 D S  0 Hướng dẫn Điều kiện x  Ta có x  21 x 1 x  2 x 1  2 Đặt t  x Do x   t  , ta có:  2  t    t  t    1  t  Kết hợp với điều kiện t  , ta có: t 1 t  1 x    x    x  Vậy tập nghiệm nguyên bất phương trình S  0  Chọn D Bài tập tự luyện Trang Bài Cho bất phương trình x 1 1  Tập nghiệm bất phương trình là:  5x A S   1;0  1;   B S   ;0 C S   1;0  1;   D S   ;0  Bài Tập nghiệm bất phương trình x  3.2 x   là: A  ;0   1;   B  ;1   2;   C  0;1 D 1;  Bài Cho bất phương trình x log2 x   32 Tập nghiệm bất phương trình là: A Một khoảng B Nửa khoảng C Một đoạn D Tập rỗng Bài Có tất số tự nhiên thỏa mãn bất phương trình 31 x  A B C  3 2x  7? D Vô số Đáp án: 1–A 2–A 3–C 4–B Dạng 3: Bất phương trình mũ lơgarit chứa tham số Ví dụ minh họa Ví dụ 1: Tìm tất giá trị thực tham số m để bất phương trình log  mx  x   log vô nghiệm A 4  m  m  B   m  4 C m  D 4  m  Hướng dẫn Vì  nên log  mx  x   log  mx  x   x  mx   5 Bất phương trình ban đầu vơ nghiệm bất phương trình x  mx   vô nghiệm nên x  mx   0, x      m  16   4  m   Chọn D Ví dụ 2: Tìm tất giá trị thực tham số m cho đoạn  2;3 thuộc tập nghiệm bất phương trình log  x  1  log  x  4x  m   A m   12;13 B m  12;13 C m   13;12 D m   13; 12 Hướng dẫn  x  4x  m x   Ta có: log  x  1  log  x  4x  m      x  4x  m   Trang m  4x  4x   f  x  5x   x  4x  m    x  4x  m  m   x  4x  g  x  m  f  x   2 x 3 Hệ thỏa mãn x   2;3   m  max g x    2 x 3  f  x   13 x   2 x 3 Do đó:  g  x   12 x  max 2 x 3 Vậy điều kiện m cần tìm 12  m  13  Chọn A Ví dụ 3: Cho bất phương trình x   m  1 3x  m  Tìm tất giá trị tham số m để bất phương trình nghiệm x  A m   B m   C m   2 D m   2 Hướng dẫn Đặt t  3x Vì x   t  Khi đó: x   m  1 3x  m  1  t   m  1 t  m   2 Bất phương trình 1 nghiệm x    nghiệm t  nên m  t2  t nghiệm t  t 1 Suy m  t 3 t2  t t 1 Xét hàm số g  t   t   2 ; t  Ta có g  t     ; t  t 1  t  1 Nên hàm số g  t  đồng biến nửa khoảng 3;   g  3  Vậy m  t 3 t2  t 3  m   m   t 1 2  Chọn B Bài tập tự luyện Bài Tìm tất giá trị thực tham số m để bất phương trình log  x  4x  m   nghiệm với x   A  m  B m  C m  D m  Bài Tìm tất giá trị thực tham số m để bất phương trình log  5x  l   m có nghiệm x  A m  B m  C m  D m  Đáp án: 1–D 2–A Trang PHẦN 3: BÀI TẬP TỔNG HỢP 1  x 1 là:  1 Bài Tập nghiệm bất phương trình A 1  x  x B x  1 C x  D  x  Bài Điều kiện xác định bất phương trình log log   x    là: A x   1;1 B x   1;0    0;1 C x   1;1   2;   D x   1;1 Bài Tập nghiệm bất phương trình x  4.5x   10 x là: A x  x  C  x  B x  2 D  x  Bài Cho bất phương trình 2sin x  3cos x  m.3sin x Với giá trị thực tham số m bất phương trình có nghiệm? A m  B m  C m  D m  Bài Tập nghiệm bất phương trình log  x  6x    log  x  1  là: A S  1;6 B S   5;6 C S   5;   D S  1;   Bài Cho bất phương trình log 0,2 x  log  x    log 0,2 Nghiệm nguyên nhỏ bất phương trình là: A x  B x  D x  C x  Bài Nghiệm nguyên lớn bất phương trình log  4.3x 1   2x  là: B x  A x  C x   D x  1  Bài Bất phương trình log x log  x  72   có tập nghiệm là:  A S  log 73;  Câu Tập nghiệm bất phương trình log x 125x  log 25 x     A S  1;  C S   ; 2 B S  log 72;    log 52 x là:  B S  1; D S  log 73;    C S   5;1  D S   5; 1 Bài 10 Tập nghiệm bất phương trình  x  x  1  là: x A  0;   Bài 11 Tìm B  ;0  tất giá C  ; 1 trị thực log  7x    log  mx  4x  m  có nghiệm x   số m để bất phương trình A m   2;5 Bài tham D  0;1 12 Tìm B m   2;5 tất giá C m   2;5  trị thực tham  log  x  1  log  mx  4x  m  có nghiệm với x A m   2;3 D m   2;5  số m để bất phương trình B m   2;3 C m   2;3 D m   2;3 Trang Đáp án: 1–A 2–D 3–C 4–A 5–B 6–D 7–C 8–D 9–A 10–C 11–B 12–A Trang CHƯƠNG 3: NGUYÊN HÀM, TÍCH PHÂN CHUYÊN ĐỀ 1: NGUYÊN HÀM HÀM CƠ BẢN PHẦN 1: LÝ THUYẾT TRỌNG TÂM Nguyên hàm Định nghĩa: Cho hàm số y  f  x  xác định tập K (K Hàm số y  2x xác định  Ta có khoảng, đoạn hay nửa khoảng) Hàm số F  x   x   2x gọi nguyên hàm hàm số y  f  x  K F  x   f  x  x  K Định lí Mọi hàm số f  x  liên tục K có nguyên hàm K Định lí Nếu F  x  nguyên hàm hàm số f  x  K với số C, hàm số G  x   F  x   C nguyên hàm f  x  K nên hàm số F  x   x gọi nguyên hàm y  2x  Ta có:  x   2x x    2x  100   2x x  C   2x x (C số) Hàm số F  x   x gọi nguyên hàm y  2x  với số C, hàm số G  x   x  C nguyên hàm y  2x  Định lí Nếu F  x  nguyên hàm hàm số f  x  K nguyên hàm f  x  K có dạng F  x   C với C số Kí hiệu:  f  x  dx  F  x   C Mọi nguyên hàm hàm số y  2x  có dạng x  C với C số Kí hiệu:  2xdx  x  C Trang Tính chất nguyên hàm  f   x  dx  f  x   C với C số   3x dx    x  dx  x  f  x   g  x  dx   f  x  dx   g  x  dx   3x  kf  x  dx  k  f  x  dx với k  F  x  có đạo hàm  d  F  x    F  x   C 3 C  2x  dx   3x dx   2xdx  x3  x  C  8xdx    2x  dx  4 2xdx  4x  dx   x 2 C C PHẦN 2: CÔNG THỨC TÍNH NHANH Nguyên hàm hàm số thường gặp Nguyên hàm mở rộng 1 x a  0dx  C   x  a  x  b  dx  a  b ln x  b  C  dx  x  C x n  x dx  x x n 1  C, n 1 dx   n  1 1 C x  1 x dx  arctan  C a a a x a 2   ax  b    dx  ln x  x  a  C dx  1  C  a   a  ax  b   x dx  ln x  C  ax  b dx  a ln ax  b  C  a    e dx  e e x x  a dx  x C ax  C   a  1 ln a ax  b dx  eax  b  C,  a   a mx  n  a dx  a mx  n  C,   a  1 m.ln a  cos xdx  sin x  C  cos  ax  b  dx  a sin  ax  b   C,  a    sin xdx   cos x  C  sin  ax  b  dx   a cos  ax  b   C,  a    cos x dx  tan x  C 1  cos  ax  b  dx  a tan  ax  b   C  a   Trang Nguyên hàm hàm số thường gặp  sin x Nguyên hàm mở rộng  sin  ax  b  dx  dx   cot x  C 1 cot  ax  b   C  a   a 1 ln cos  ax  b   C a  tan xdx   ln cos x  C  tan  ax  b  dx   cot xdx  ln sin x  C  cot  ax  b  dx  a ln sin  ax  b   C  a   PHẦN 3: CÁC DẠNG BÀI TẬP Dạng 1: Nguyên hàm Phương pháp giải Ví dụ: Tính A  dx x2 2x  2x 3  C 2x C 2x  C x B 2x 3  C x D 2x 3  C x Hướng dẫn: Cách 1: Áp dụng công thức bảng nguyên hàm Cách 1: Áp dụng n  x dx     2x  x dx  1  C x x n 1  C,  n  1 ta có: n 1  dx   2x dx   dx  x  x   x dx  3 2x 3 dx   C x2 x Cách 2: Sử dụng máy tính CASIO fx 570 VNPLUS Cách 2: Sử dụng máy tính CASIO fx 570 VNPLUS Bước 1: Thay x  a (a số bất kì) vào hàm số Thay x  vào hàm số f  x   2x  y  f  x  , ta kết f  a  kết Bước 2: Sử dụng SHIFT Nhập d  x   dx d (các đáp án) x  a dx Đáp án kết với kết bước đáp ta x2 35  8, 75 Đáp án A: Nhập d  2X 3     x  ta dx  2X  kết 7,625, khác kết bước 1, đó loại đáp án A Trang án Đáp án B: Nhập d  2X 3    x  ta kết  dx  X 8,75, kết bước  Chọn B Ví dụ minh họa   Ví dụ 1: Tính nguyên hàm   e3x 1   dx x   A 3x 1 e  C x B 3e3x 1  C x C 3e3x 1  C x D 3x 1 e  C x Hướng dẫn Áp dụng công thức  eax  b dx  eax  b  C,  a   a x dx  1  C ta có: x e3x 1  3x 1  3x 1   e  x  dx   e dx   x dx   x  C  Chọn D Ví dụ 2: Cho f  x    x  3x  2x Một nguyên hàm F  x  f  x  thỏa mãn F 1  A F  x    x4  x3  x  4 B F  x    x4  x3  x  4 C F  x    x4  x3  x  4 D F  x    x4  x3  x  4 Hướng dẫn F  x      x  3x  2x  dx    x 3dx  3 x dx   xdx   x4  x3  x  C 14 F 1       C   C  4 Vậy F  x    x4  x3  x  4  Chọn C  Ví dụ 3: Cho f  x   sinx  cosx Một nguyên hàm F  x  f  x  thỏa mãn F    4 A F  x    cos x  sin x  B F  x    cos x  sin x  C F  x   cos x  sin x  D F  x    cos x  sin x  Hướng dẫn Trang F  x     sin x  cos x  dx   sin xdx   cos xdx   cos x  sin x  C    F      cos    sin    C   C  4 4 4 Vậy F  x    cos x  sin x   Chọn A Ví dụ 4: Tìm m để hàm số F  x   mx   3m   x  10x  2018 nguyên hàm hàm số f  x   3x  10x  10 A m  C m  B m  D m  Hướng dẫn Ta có   3x  10x  10  dx  x3 x2  10  10x  C  x  5x  10x  C Để F  x   mx   3m   x  10x  2018 nguyên hàm hàm số m  f  x   3x  10x  10    m 1 3m    Chọn C Bài tập tự luyện Câu Mệnh đề sai mệnh đề sau? A  kf  x  dx  k  f  x  dx, k   C  f  x   g  x  dx   f  x  dx   g  x  dx B  f  x  g  x  dx   f  x  dx. g  x  dx D  f   x  dx  f  x   C Câu Họ nguyên hàm hàm số f  x   5x  4x  A  x  x  6x  C Câu Hàm số f  x   A ln x   6 x x2 B 20x  8x  C C 20x  8x  C D  x  x  C 4   có nguyên hàm F  x  Biết F 1  Tìm F  x  x x x B ln x   4 x x2 C ln x    12 x x2 D ln x    x x2   Câu Tìm nguyên hàm hàm số f  x   cos  3x   6    A  f  x  dx  sin  3x    C  6   B  f  x  dx  sin  3x    C 6    C  f  x  dx   sin  3x    C  6   D  f  x  dx  sin  3x    C  6 Đáp án: Trang 1–B 2–A 3–C 4–A Dạng 2: Tính nguyên hàm phương pháp đổi biến số Phương pháp giải Nếu  f  u  du  F  u   C u  u  x  có đạo hàm liên tục  f  u  x   u   x  dx  F  u   C Chú ý: Cơng thức tính vi phân: f   x  dx  d  f  x   dx  d  x  1  d  x     d  x  n  1 1 dx  d  ax   d  ax  1  d  ax     d  ax  n  a a a a sin xdx  d   cos x   d  cos x  cos xdx  d  sin x  dx  d  ln x  x e x dx  d  e x  Ví dụ minh họa Ví dụ 1: Nguyên hàm hàm số A C 4036  4x B  2x  2018 1  2x  2018 C C C 4x  4036 D 1  C 2x  2018 Hướng dẫn Cách 1: I    2x  2018 dx Đặt 2x  2018  t  dt  2dx Do đó: I   Trở lại phép đổi biến ta được: I  dt 1  C 2t 2t 1 C  C  2x  2018  4036  4x Cách 2: Sử dụng vi phân I   2x  2018 dx   2018    2x  2018  dx  d   2   2x  2018   2x  2018   1 1 d  2x  2018   C  C   2x  2018  2x  2018 4036  4x Cách 3: Sử dụng cơng thức tính nhanh   ax  b  dx  1 C a  ax  b   Chọn A Trang Ví dụ 2: Nguyên hàm hàm số f  x   A x 5 C B x3 x4  x 5 C C x4  C D x4  C Hướng dẫn Cách 1: I   Đặt x3 x4  dx x   t  t    x   t  4x 3dx  2tdt  x 3dx  Do đó: I   tdt tdt dt t    C 2t 2 x4  C Trở lại phép đổi biến ta được: I  Cách 2: Sử dụng máy tính CASIO fx 570 VNPLUS x3 Bước 1: Thay x  vào hàm số f  x   Bước 2: Sử dụng SHIFT Đáp án A: Nhập x4  ta kết 1,7457 d   x  dx d 1  x   x  ta kết 0,4364, khác kết bước loại đáp  dx   án A Đáp án B: Nhập d 1  x   x  ta kết 0,8728, khác kết bước loại đáp  dx   án B d  x4   Đáp án C: Nhập   x  ta kết 1,7457, kết bước dx    Chọn C Ví dụ 3: Cho nguyên hàm sau I   A I   dt t  t  1 B I  dx x x 1 10 dt  10 t  Khi đặt t  x10  ta C I  dt  10 t  t D I  dt  t2 1 Hướng dẫn I dx x x10   x dx x10 x10  Đặt t  x10   t    t  x10   x10  t   10x dx  2tdt  x dx  tdt , ta có: Trang I tdt dt    t  1 t t   Chọn D Ví dụ 4: Giả sử F  x  nguyên hàm hàm số y  A 10 B 32 C x2 Biết F 10   40 Tính F   x 1 20 D Hướng dẫn Đặt I x   t  t    x   t  x  t   dx  2tdt t    2tdt  Fx  t  x  1 Vì F 10   40   t3  2t    t  3 dt    3t   C   6t  C 3   x 1  C 32   C  40  C   F       3  Chọn B Bài tập tự luyện Câu Tìm nguyên hàm hàm số f  x    3x A  f  x  dx   C  f  x  dx    3x   3x  C   3x   3x Câu Biết nguyên hàm hàm số f  x   B  f  x  dx     3x   3x D  f  x  dx    3x  C  hàm số F  x  thỏa mãn F  1  Khi  3x F  x  hàm số sau đây? A F  x   x   3x  B F  x   x   3x  C F  x   x   3x  D F  x     3x  2x  1  x  C Câu Tìm nguyên hàm hàm số f  x   2x  x 1 A  f  x  dx    2x  1  x  C B  f  x  dx  C  f  x  dx    2x  1  x  C D  f  x  dx  2  x  C 1 x Câu Hàm số f  x   x x  có nguyên hàm F  x  Nếu F    F  3 Trang A 886 105 B 116 15 C 146 15 D 105 886 Câu Biết hàm số F  x    x  2x  2018 nguyên hàm hàm số f  x   ax  b Khi  2x tổng a b A B –2 C D Đáp án: 1–A 2–B 3–A 4–C 5–A Dạng 3: Tính nguyên hàm phương pháp nguyên hàm phần Phương pháp giải Nếu u v hai hàm số có đạo hàm liên tục K thì:  udv  uv   vdu Cách ưu tiên đặt u: log  lnx  , nhì đa  ax  b; ax  bx  c  , tam lượng  sin x;cos x  , tứ mũ  e x a x  Ví dụ: Đặt  ln xdx  x ln xdx  x sin xdx  xe dx  e cos xdx u ln x ln x x x cos x vdv dx xdx sin xdx e x dx e x dx x x Ví dụ minh họa Ví dụ 1: Tính nguyên hàm F  x    x sin xdx A sin x  x cos x  C B x sin x  cos x  C C  x cos x  sin x  C D x sin x  cos x  C Hướng dẫn u  x du  dx Cách 1: Đặt   dv  sin xdx  v   cos x I  uv   vdu   x.cos x   cos xdx   x.cos x  sin x  C Cách 2: Sử dụng vi phân  x sin xdx   xd   cos x   x.cos x    cos xdx  x.cos x  sin x  C  Chọn C Trang Ví dụ 2: Tính nguyên hàm  ln 4xdx A x  ln 4x  1  C B x  ln 4x  1  C C x  ln 4x  1  C D 2x  ln 4x  1  C Hướng dẫn dx  u  ln 4x du  Cách 1: Đặt   x dv  dx  v  x Khi đó:  ln 4xdx  uv   vdu  x.ln 4x   x dx  x.ln 4x   1dx  x  ln 4x  1  C x Cách 2: Sử dụng vi phân  ln 4xdx  x.ln 4x   xd  ln 4x   x.ln 4x   x dx  x.ln 4x   1dx  x ln 4x  x  C  x  ln 4x  1  C 4x Cách 3: Sử dụng máy tính CASIO fx 570 VNPLUS Bước 1: Thay x  vào hàm số f  x   ln 4x ta kết 2,07944 Bước 2: Sử dụng SHIFT Đáp án B: Nhập d   x  dx d X   x  ln  4X   1  x  ta kết 0,519, khác kết bước 1, dx   loại đáp án A Đáp án B: Nhập d X   x  ln  4X   1  x  ta kết 1,039, khác kết bước 1, dx   loại đáp án B Đáp án C: Nhập d X  ln  4X   1 x  ta kết 2,07944, kết bước dx    Chọn C Ví dụ 3: Cho F  x  nguyên hàm hàm số y  x.e 2x , biết F    Tìm F  x  A 2x e  x  2  B 2x  1 e x   2  C 2x  1 e x   2  1  D 2e 2x  x    2  Hướng dẫn du  dx u  x  Cách 1: I   x.e dx Đặt   2x 2x dv  e dx  v  e  2x I  uv   vdu  2x 1 1 1  xe   e 2x dx  xe 2x  e 2x  C  e 2x  x    C 2 2  Trang 10  1 1 1  F    e0     C   C   C  Do F  x   e 2x  x     2 4 2  Cách 2: Sử dụng vi phân  e 2x  x.e 2x e 2x 1 I   x.e dx   xd   dx  xe 2x   e 2x dx  2 2   2x  2x 2x 1  xe  e  C  e 2x  x    C 2   1 1 1  F    e0     C   C   C  Do F  x   e 2x  x     2 4 2   Chọn C Bài tập tự luyện x Câu Tính F  x    xe dx Chọn kết x  x3 e C A F  x   C F  x   x B F  x    x  3 e  C x  x3 e C x D F  x    x  3 e  C Câu Tính F  x    x sin 2xdx Chọn kết A F  x     2x cos 2x  sin 2x   C B F  x    2x cos 2x  sin 2x   C C F  x     2x cos 2x  sin 2x   C D F  x    2x cos 2x  sin 2x   C Câu F  x   x sin x  cos x  2017 nguyên hàm hàm số nào? A f  x   x sin x B f  x   x cos x C f  x    x cos x D f  x    x sin x Đáp án: 1–D 2–A 3–B PHẦN 4: BÀI TẬP TỔNG HỢP Câu Họ nguyên hàm hàm số f  x   e x   e  x  A F  x   3e x  x  C C F  x   3e x  B F  x   3e x  e x ln e x  C C ex D F  x   3e x  x  C  1 Câu Tính     dx  x 2 A x x  C 2 B x  x  C C 1  xC x D x  C x Trang 11 Câu Cho F  x  nguyên hàm hàm số f  x   x  x thỏa mãn F 1  0, F  x   x4 x2   a b c Tính S  a  2b  c A 10 B 12 C 14 D 16 Câu Cho hàm số y  x  x có nguyên hàm f  x  cho f 1  Tính giá trị biểu thức f    f  64  A 2018 B 1792 C 1945 D 1794 x2 x 1 Câu Tìm nguyên hàm hàm số f  x   A  f  x  dx   x  4 x   C B  f  x  dx   x   x   C C  f  x  dx  x C  x  1 x  D  f  x  dx  x   C x 1 Câu Tìm hàm số F  x  biết F  x   3x  2x  đồ thị hàm số y  F  x  cắt trục tung điểm có tung độ A F  x   x  x  x  B F  x   x  x  x  C F  x   6x  D F  x   x  x  x     9x  12 Câu Tính   9x  13 A 13   9x  13  C B 117   9x   13  C C 117   9x  13 C D  C Câu Cho hàm số f  x   2x  sin x  cos x Một nguyên hàm F  x  f  x  thỏa mãn F    A x  cos x  2sin x B x  cos x  2sin x  C  cos x  2sin x D x  cos x  2sin x  1–A 2–B 3–B 4–D 5–A 6–B 7–C 8–B Trang 12 CHƯƠNG 3: NGUYÊN HÀM, TÍCH PHÂN CHUYÊN ĐỀ 3: TÍCH PHÂN PHẦN 1: LÝ THUYẾT TRỌNG TÂM Khái niệm tích phân Cho hàm số y  f  x  liên tục K a, b  K F  x   x nguyên hàm hàm số Nếu F  x  nguyên hàm y  f  x  K y  f  x   2x, tích phân cận từ tới hàm số F  b   F  a  gọi tích phân y  f  x  y  f  x   2x là: b từ a đến b kí hiệu  f  x  dx a b  f  x  dx  F  b   F  a  a Trong đó: a cận trên, b cận f(x) hàm số dấu tích phân dx: gọi vi phân đối số f  x  dx : gọi biểu thức dấu tích phân Chú ý: Tích phân xác định khơng phụ thuộc biến b b a a  f  x  dx   f  t  dt F  b   F  a  2 1  2x dx   2tdt  2udu  Tính chất tích phân 0  f  x  dx  0 b a a b 2  f  x  dx    f  x  dx b b a a b b b a a a  f  x   g  x dx   f  x  dx   g  x  dx Nếu f  x   0, x   a, b  b  f  x  dx  0, x  a, b  2x dx   2x dx  kf  x  dx  k  f  x  dx,  k    2x dx  2 1  2x dx  2 x dx 2 1 2  2x  x  dx   2x dx   x dx Với f  x   2x  0, x  1, 2 , ta có: 2 1  f  x  dx   2x dx   0, x  1; 2 a PHẦN 2: CÁC DẠNG BÀI TẬP Dạng 1: Tính tích phân phương pháp phân tích Trang 1 Phương pháp giải b Tính tích phân  f  x  dx Ví dụ: Tính tích phân I    x  1 dx a A B C D Hướng dẫn Cách 1: Phân tích f  x  thành tổng, hiệu, tích, Cách 1: thương nhiều hàm số khác, mà ta sử dụng trực tiếp bảng nguyên hàm tìm ngun hàm chúng Sau tính tích phân theo cơng thức sau: b  f  x  dx  F  b   F  a  I    x  1 dx    x  2x  1 dx  x3     x2  x     Cách 2: Sử dụng máy tính Casio fx 570 VN PLUS:     13   03      1    02   3  3  a Sử dụng phím  Cách 2: Nhập   X  1 dx, ta kết  Chọn C Ví dụ minh họa Ví dụ 1: Tích giá trị k để   6x  6x   dx  k A  B –1 C D  Hướng dẫn  6x 6x  Ta có   6x  6x   dx     2x    k 0   2x  3x  2x  k  2k  3k  2k k  k  Theo đề ta có   6x  6x   dx  nên 2k  3k  2k    k  k  1 Do tích giá trị k 3  1  2 → Chọn D Ví dụ 2: Tích phân I   x 1 dx  x2 Trang A 1  3ln B 2  3ln D  3ln C ln Hướng dẫn Cách 1: I   4 x 1 x 23   dx   dx   1   dx x2 x  x    3 =  x  3ln  x       3ln     ln1   3ln Cách 2: Sử dụng máy tính Casio fx 570 VN PLUS Bước 1: Sử dụng phím     Nhập X 1  X  dx, ta kết 3,0794415 Bước 2: Tính giá trị bốn đáp án, đáp án có kết kết bước đáp án Ta thấy  3ln2  3,0974415 → Chọn D Ví dụ 3: Biết I   dx  a ln  b ln  c ln với a, b, c số nguyên Tính tổng S  a  b  c x x A S = B S = C S = –2 D S = Hướng dẫn Cách 1: I   4 dx dx x 1 x  1   dx      dx  ln x  ln  x  1  x  x x  x  1 x  x  1 x x 1  3   ln  ln 5   ln  ln   ln  ln  ln Suy ra: a  2, b  1, c  –1  S  a   b  c  Cách 2: Áp dụng cơng thức tính nhanh I 1   x  a  x  b  dx  a  b  ln x  a  ln x  b  4 dx dx   ln x  ln x     ln  ln    ln  ln   ln  ln  ln  x  x x  x  1  Suy ra: a  2, b  1, c  –1  S  a   b  c  → Chọn D Ví dụ 4: Để hàm số f  x   asin x  b thỏa mãn f 1   f  x  dx  a, b nhận giá trị bao nhiêu? A a  ; b  B a  ; b  C a  2; b  D a  2; b  Hướng dẫn Ta có f 1   asin   b   b  suy ra: 1  a  2a f x dx  0   0  asin x   dx     cosx  2x     Trang Mà  f  x  dx   2a    a    Vậy a  , b  → Chọn B Bài tập tự luyện  dx  sin x Câu Tích phân I   A B Câu Biết C D C 27 D 81 C k = D k =  2x  dx  ln a Giá trị a A B k Câu Để   k  4x  dx   5k, giá trị k A k = B k = Câu Biết 2x  dx  a ln  b với a; b   Chọn khẳng định khẳng định sau 2x  A a  B b  C a2  b2  50 D a  b  Đáp án: 1–C 2–B 3–B 4–C Dạng 2: Tích phân phương pháp đổi biến Phương pháp giải b Để tính tích phân I   f  x  dx, Ví dụ: Tính tích phân I    x  1 f  x   g  u  x   u'  x  , ta thực phép A a đổi biến sau: 2000 dx 1 2001 B 22000 2001 C  22000 2001 D 22001 2001 Hướng dẫn Bước 1: Đặt t  u  x   dt  u  x  dx Đặt t  x   dt   x  1 dx  dt  dx Bước 2: Đổi cận: Đổi cận: x a b x t u(a) u(b) t Bước 3: Thay vào ta có Trang I u b   g  t  dt  G  t  b a u a  I  t 2000 t 2001 22001 dt   2001 2001 → Chọn D Ví dụ minh họa Ví dụ 1: Tính tích phân I   x  x dx B I  A I  1 C I  1 D I  Hướng dẫn Cách 1: Đặt t   x  t    t   x  2tdt  2xdx  tdt  xdx Đổi cận: t Khi đó: I   t dt   x t 1 Cách 2: Sử dụng máy tính Casio Fx570 VN Plus Sử dụng phím Nhập hàm số      X  X2 dx ta kết → Chọn D Ví dụ 2: Đổi biến x  2sin t, tích phân I    dx  x2  A  tdt  C  dt t B  dt trở thành:  D  dt Hướng dẫn Cách 1: Đặt x  2sin t  dx  cos tdt Đổi cận:  Khi đó: I   cos tdt  4sin t   x t  cos tdt  sin t   cos tdt cos2 t    dt Trang Cách 2: Sử dụng máy tính Casio fx 570VN PLUS Bước 1: Thiết lập chế độ rađian SHIFT MODE Bước 2: Nhập tích phân I   dx 4X ta kết  Bước 3: Tính tích phân đáp án, đáp án kết bước chọn  Đáp án A, nhập  X dx kết 0,137077, khác với kết bước 1, loại đáp án A  Đáp án B, nhập  dx ta kết  , với kết bước 1, chọn đáp án B → Chọn B Ví dụ 3: Cho I   x3 dx  a  b.ln 2, a; b nguyên Chọn phát biểu x4 1 A a  b  B a  b  C a  b  D a  b Hướng dẫn Cách 1: Đặt u  x  1 u  1  dx  4x3dx Đổi cận x   u  1; x   u  2 du  ln u 4u I Do a  0; b   1 ln  ln1  ln 4 1 nên a  b  4 Cách 2: Sử dụng máy tính CASIO Fx 570VN PLUS X3 Bước 1: Nhập tích phân I   dx, kết 0,1732867951 X 1 Bước 2: Gán giá trị kết cho A: SHIFT STO A Bước 3: Giải hệ phương trình hai ẩn a  b ln  A  Đáp án A: Nhập hệ phương trình  kết nghiệm lẻ, loại a  b   a  b ln  A a    Đáp án B: Nhập hệ phương trình  kết  1 a  b  b    → Chọn B Bài tập tự luyện Câu Tích phân I   x 3x  1dx Trang A 16 135 B 116 135 x Câu Tích phân I   x2  C D 14 135 D dx có giá trị B 2  A 2 114 135 C 2   Câu Cho tích phân I   sinx  cosxdx Đặt u  + cosx kết sau đúng? A I   udu  41001 3003 B C I   udu 1000 Câu Tính tích phân: I   x  3x A B I   udu 29   x D I   udu   dx 31001 3000 C 41000 3000 D 31001 3003 Đáp án: 1–B 2–B 3–D 4–A Dạng 3: Tích phân phương pháp tích phân phần Phương pháp giải Cho hai hàm số u v liên tục a; b  có đạo hàm liên tục a; b  Khi đó: b b b  udv  uv   vdu a a a Ví dụ minh họa  Ví dụ 1: Tính tích phân  xcos xdx A   B   C  1 D  1 Hướng dẫn  ux du  dx Cách 1: Đặt   dv  cos xdx v  sin x Do I   x s inx       sin xdx    cosx      Cách 2: Sử dụng máy tính CASIO Fx 570VN PLUS Bước 1: Thiết lập chế độ rađian SHIFT MODE Trang    Bước 2: Sử dụng phím   Nhập  xcos xdx, ta kết 0,5707963 Bước 3: Tính giá trị bốn đáp án, đáp án có kết kết bước đáp án Ta thấy    0,5707963 → Chọn B Ví dụ 2: Tích phân  x.e2x dx có giá trị A –3 e2  a Tính ab b B –4 C D Hướng dẫn du  dx  u  x   Đặt  e2x 2x dv  e dx v   xe2x Do I  1 e2x e2 e2x dx   2   e2  e2 e  e2 e2         4 4 Do a  1, b   ab  → Chọn D Ví dụ 3: Cho tích phân I    4x  3 ln xdx = aln2 + b Tính giá tri a  2b A B –1 C D Hướng dẫn   u  ln x du  dx  Cách 1: Đặt  x dv   4x  3 dx v  2x  3x   2   14 ln   x  3x  2x  3x dx  2.22  3.2 ln  2.12  3.1 ln1    2x  3 dx x 1  Do I  2x  3x ln x           14 ln    22  3.2  12  3.1   14 ln  10    14 ln    Do a  14; b  6  a  2b  14   6   Cách 2: Sử dụng máy tính CASIO Fx 570VN PLUS Bước 1: Nhập tích phân   4X  3 ln  X  dx, kết 3,704060528 Bước 2: Gán giá trị kết cho A: SHIFT STO A Bước 3: Giải hệ phương trình hai ẩn Trang a ln  b  A Đáp án A: Nhập hệ phương trình  kết nghiệm lẻ, loại a  2b  a ln  b  A Đáp án B: Nhập hệ phương trình  kết nghiệm lẻ, loại a  2b  1 a ln  b  A Đáp án C: Nhập hệ phương trình  kết a  2b  a  14   b  6 → Chọn C Bài tập tự luyện ln x dx Khẳng định sau đúng? x Câu Cho tích phân I   2 1 A I   ln x   dx x x 1 B I   ln x   dx x x 2 C I   ln x   dx x D I   ln x   dx x Câu Tính tích phân: I    x   e2x 1dx A 5e  e5 B 5e  e5 C 5e  e5 D 5e  e5 e Câu Tính tích phân  x ln xdx A I  e2  B I  e2  2 C I  D I  e2  Đáp án: 1–B 2–A 3–D PHẦN 3: BÀI TẬP TỔNG HỢP x 1 dx x  2x  Câu Tích phân I   A ln B ln C ln D 2 ln C –1 D e Câu Tích phân I   dx x A e B 1 e Trang Câu Tính I  10  x   2x  dx 99 10100 2030  A I  100 10100 1970  B I  100 10100 1970  C I  100 10100 2030  D I  100 x4 dx = aln2 + bln3 Tính P  a.b x  3x  Câu Biết I   A –10 B Câu Cho I   C –15 D  x x3 cosx dx, phát biểu sau đúng? 3s inx  12 dx J   A I  J C J  ln B I  2 Câu Cho tích phân  x   2x  x  1 x 1 khẳng định D I  2J dx = a + bln3 + cln2  a; b; c    Chọn khẳng định sau A a  B c  a C b  D a  b  c  x Câu Tìm a  cho  x.e dx  A B C D a Câu Có số a   0;20  cho  sin x.sin 2xdx  A 20 B 19  C D 10  Câu Tích phân I   ln x  x dx  a.ln  b Tính a A B C –2 D –3 C D e Câu 10 Giá trị  ln xdx A B Đáp án: 1–B 2–B 3–B 4–A 5–A 6–D 7–D 8–D 9–A 10 – A Trang 10 CHƯƠNG CHUYÊN ĐỀ TÍCH PHÂN HÀM ẨN PHẦN 1: LÝ THUYẾT TRỌNG TÂM Tính chất tích phân  f  x  dx  b a a b  f  x  dx   f  x  dx b b a a b b b a a a  kf  x  dx  k  f  x  dx  f  x   g  x  dx   f  x  dx   g  x  dx b b c b  f  x  dx   f  x  dx   f  x  dx a a  f   x  dx  f  x  c a b a  f b  f a  Tích phân xác định khơng phụ thuộc biến b b b a a a  f  x  dx   f  t  dt  f  u  du PHẦN 2: CÁC DẠNG BÀI TẬP Dạng 1: Phép tính tích phân Ví dụ minh họa 8 2 Ví dụ 1: Cho  f  x dx  18,  f  x dx  14 Biểu thức  f  x  dx A 44 B C -4 D -44 Hướng dẫn 8 8 2 4 2 Ta có  f  x  dx   f  x dx   f  x dx   f  x dx   f  x  dx   f  x dx  14  18  4 → Chọn C Ví dụ 2: Cho  f  x  dx  10 Khi A 32 B 46  2  4f  x  dx C 36 D 43 Hướng dẫn 5 2  2  4f  x  dx   2dx  4 f  x  dx 5 2  2x   f  x  dx       f  x  dx   4.10  46 → Chọn B Ví dụ 3: Cho hàm số f  x  có đạo hàm đoạn 1; 4 f    2, f 1  Tính I   f   x  dx A I  B I  C I =  D I  10 Trang Hướng dẫn I   f   x  dx  f  x   f    f 1    3 → Chọn C Ví dụ 4: Cho 5 1  f  x   3g  x  dx  10,  2f  x   g  x  dx  Tính A B  f  x   g  x  dx C D Hướng dẫn 5 1  f  x   3g  x  dx  10   f  x  dx  3 g  x  dx  10 5 1  2f  x   g  x  dx   2 f  x  dx   g  x  dx  I  3J  10 I  Đặt I   f  x  dx, J   g  x  dx , theo đề ta có   2I  J  J  1 Ta có 5 5 1  f  x   g  x  dx  f  x  dx   g  x  dx  I  J    → Chọn C Bài tập tự luyện 3 0 Câu Cho tích phân  f  x  dx  1,  f  x  dx  Tính giá trị biểu thức sau: I   f  x  dx A B C 4 1 D Câu Cho biết  f  x  dx  2,  f  x  dx  3,  g  x  dx  Khẳng định sau sai? 4 A B  f  x  dx   f  x   g  x  dx  10 3 C  f  x  dx  5 D  4f  x   2g  x  dx  2 Câu Cho hàm f  x  liên tục  thỏa mãn d d a b  f  x  dx  10,  f  x  dx  8, c  f  x  dx  Tính tích a c phân I   f  x  dx b A I  5 B I  Câu Cho biết  3f  x  dx  2g  x dx  A B C I  D I  7 2 1  2f  x  dx  g  x dx  3 Giá trị  f  x  dx bằng: C  D Trang Đáp án: 1–B 2–B 3–C 4–C Dạng 2: Phương pháp đổi biến số Ví dụ minh họa Ví dụ 1: Cho I   f  x xdx  Tính giá trị  f  x  dx 0 A I  B I  C I  D I  Hướng dẫn Đặt t  x  t   dt  2xdx Đổi cận: x t 4 1 I Khi tích phân  f  x xdx trở thành   f  t  dt   f  t  dt   I  20 0 → Chọn A Ví dụ 2: Cho f  x  hàm số chẵn  f  x  dx  a Chọn mệnh đề 3 3 A  f  x  dx  a B  f  x  dx  2a C 3  f  x  dx  a 3 D  f  x  dx  a Hướng dẫn Đặt t   x  dt  dx Đổi cận: x -3 t 0 3 3 0 Khi đó:  f  x  dx    f   t  dt   f   t  dt   f   x  dx 3 3 0 Vì f  x  hàm số chẵn nên f   x   f  x    f  x  dx   f   x  dx   f  x  dx  a Do 3 3 3  f  x  dx   f  x  dx  f  x  dx  a  a  2a → Chọn B Trang Ví dụ 3: Cho  0  xf  x  dx  Tính I   f  cos2x  sin 4xdx A I  B I  D I  C I  3 Hướng dẫn Đặt t  cos2x  dt  2.sin 2xdx Đổi cận: x  t    0 I   f  cos2x  sin 4xdx   f  cos2x  2cos2x.sin 2xdx    f  cos2x  cos2x  2  sin 2xdx 1 0    f  t  tdt   f  t  tdt   xf  x  dx 3 → Chọn B Ví dụ 4: Cho f     f   x   f    x  dx  Tính f  3 A f  3  C f  3  B f  3  D f  3  3 Hướng dẫn Đặt t   x  dt  dx Đổi cận: 3 x t 3  f    x  dx   f   t  dt   f   t  dt  f   x  dx 0 3 3 0 0 Do   f   x   f    x   dx   f   x  dx   f   x  dx   f   x  dx Suy  f   x  dx  5 5  f  x    f  3  f     f  3    f  3  2 2 → Chọn A Bài tập tự luyện 2020 Câu Cho  f  x  dx  Khi giá trị tích phân 1010 A 32  f  2x  dx bằng: B C D Trang Câu Cho hàm số f  x  hàm chẵn, liên tục  A B C 2  f  x  dx  Tính I   f  3x  1 dx D Câu Cho hàm số f  x  liên tục  thỏa mãn f   x   2.f  x   cos x Tính tích phân I   f  x  dx   A I  B I  C I  D I  Đáp án: 1–D 2–C 3–A Dạng 3: Phép tính tích phân Phương pháp giải Áp dụng cơng thức tính tích phân phần: b b  udv  uv  vdu b a a a Ví dụ minh họa Ví dụ 1: Cho   x   f   x dx  7, f    Tính A I  B I  5 I   f  x  dx C I  7 D I  Hướng dẫn u  x  du  dx Đặt   , áp dụng cơng thức tích phân phần ta có: dv  f   x  dx  v  f  x  2    x   f   x dx   x   f  x    f  x dx  2f    I   I  I  5 0 → Chọn C  Ví dụ 2: Cho F  x  là nguyên hàm f  x  Biết F    3   xF  x  dx  Tính giá trị  I   x f  x  dx A I  2 2 B I  2  C I  2 2 D I  2  Trang Hướng dẫn u  x  u   du  2xdx Đặt  , áp dụng cơng thức tích phân phần ta có:  v  F x   dv  f x dx       I   x f  x  dx  x F  x  2   2   2xF  x  dx  2  F    2 3 → Chọn A Ví dụ 3: Cho F  x  là nguyên hàm f  x  Biết F  3  ,  F  x  1 dx  Tính giá trị 1 I   xf  x  dx B I  11 A I  10 C I  D I  Hướng dẫn Đặt t  x   dt  dx Đổi cận: x -1 t 3 1 0 Suy  F  x  1 dx  F  t  dt    F  x  dx  u  x du  dx Đặt   , áp dụng cơng thức tích phân phần ta có: dv  f  x  dx  v  F  x  3 I   xf  x  dx  x F  x    F  x  dx  3F  3   0 → Chọn D Bài tập tự luyện 1 0 Câu Cho f 1  2f     f  x  dx  15 Tính I     x  f   x  dx A 13 B 17 C -13 1 0 D 15 Câu Cho   2x   f   x  dx  f    Tính  f  x  dx A -3 B -9 C D Đáp án: 1–B 2–C Phần BÀI TẬP TỔNG HỢP b b c a c a Câu Giả sử  f  x  dx   f  x  dx  a  b  c  f  x  dx Trang A B C -1 1 2 D -5 Câu Nếu  f  x  dx   f  x  dx   f  x  dx A Câu B C D -3 Cho f  x  , g  x  hàm số liên tục đoạn  2;6 thỏa mãn  f  x  dx  ; 6 3  f  x  dx  7;  g  x  dx  Hãy tìm mệnh đề khơng  3g  x   f  x  dx  A B ln e6  C  3f  x   4 dx  ln e6  2f  x   1 dx  16 D   4f  x   2g  x   dx  16 Câu Cho biết 1  f  x  dx  15 Tính giá trị P   f   3x    dx A 15 B 37 C 27 D 19 5 0 Câu Giả sử  f  x  dx   f  z  dz  Tổng  f  t  dt   f  t  dt A 12 B C D Câu Cho y  f  x  hàm số chẵn, có đạo hàm đoạn  6;6 Biết  f  x  dx  1 1  f  2x  dx  Tính  f  x  dx A I  11 Câu B I  C I  Cho f, g hai hàm liên tục đoạn 1;3 D I  14 thỏa mãn:  f  x   3g  x  dx  14 3 1  2f  x   g  x  dx  Tính  f  x   g  x  dx A B b b a a C Câu Cho  f  x  dx   g  x  dx  3 Tích phân A -4 B D b   f  x   2g  x   dx C Câu Cho hàm số f  x  liên tục khoảng  0;19  thỏa mãn D 19  f  x  dx  10 ; 19 a  f  x  dx  Khi P   f  x  dx   f  x  dx có giá trị A B C D 13 Trang Câu 10 Cho f  x  hàm số liên tục đoạn  0;3 f  x  f   x   với x   0;3 Tính dx 1 f  x K A K  B K  C K  D K  Đáp án: 1-C 2-C 3-D 4-D 5-C 6-D 7-C 8-D 9-C 10 - C Trang CHƯƠNG CHUYÊN ĐỀ ỨNG DỤNG CỦA NGUYÊN HÀM, TÍCH PHÂN PHẦN 1: CÁC DẠNG BÀI TẬP Dạng 1: Diện tích hình phẳng Phương pháp giải a Diện tích hình phẳng giới hạn bởi:  Đồ thị  C  hàm số y  f  x  liên tục đoạn  a; b   Trục hoành y   Hai đường thẳng x  a, x  b b S   f  x  dx a b Diện tích hình phẳng giới hạn bởi:  Đồ thị  C  hàm số y  f  x  , y  g  x  liên tục đoạn  a; b   Hai đường thẳng x  a, x  b b S   f  x   g  x  dx a c Diện tích hình phẳng giới hạn bởi:  Đồ thị  C  hàm số y  f  x  , y  g  x  liên tục đoạn  a; b  Trường hợp 1: Giải phương trình: x  a f x  gx   , a  b x  b b S   f  x   g  x  dx a Trường hợp 2: Giải phương trình: x  a f  x   g  x    x  b ,  a  b  c   x  c b c a b S   f  x   g  x  dx   f  x   g  x  dx d Hình phẳng giới hạn nhiều hai đường cong Diện tích S hình phẳng giới hạn đồ thị chia thành nhiều phần diện tích, mà phần Trang ta tích theo công thức: c b a c S   f  x   h  x  dx   g  x   h  x  dx Chú ý: Bằng cách coi x hàm biến y, diện tích S hình phẳng giới hạn đồ thị hàm số x  f  y  , x  g  y  liên tục đoạn  a; b  hai đường thẳng y  a , y  b tính theo công thức: b S   f  y   g  y  dy a Ví dụ minh họa Ví dụ 1: Diện tích hình phẳng giới hạn đồ thị hàm số y   x  1 , trục hoành, đường thẳng x  đường thẳng x  là: A S   B S  C S  25 D S  25 Hướng dẫn Diện tích hình phẳng cần tính là: S   2dx  x  1 2        x 1  → Chọn B Ví dụ 2: Gọi S diện tích hình phẳng giới hạn đồ thị hàm số y  x  y  4x Xác định mệnh đề A S   x  4x  dx   C S   x   4x dx B S    x  4x  3 dx D S   x  4x  dx Hướng dẫn x  Phương trình hồnh độ giao điểm x   4x   x  3 Do ta có S   x  4x  dx → Chọn A Ví dụ 3: Diện tích hình phẳng giới hạn đồ thị  C  hàm số y  2x  x  x  đồ thị Trang  C  hàm số y  x  x  A B C D Hướng dẫn x  Ta có: 2x  x  x   x  x    x   x  1 S  2x  2x dx  1  2x  2x dx  1  2x 2x         2x  2x  dx 0  2x 2x     1   1 → Chọn B Ví dụ 4: Diện tích hình phẳng giới hạn đường y  x  2x  1, y  x  1, x  0, x  m,   m  3 A m3 3m  B  m3 3m  C m3 m   2m D m3 m   2m Hướng dẫn Ta có: x  3x  0, x   0; m   m  m Do đó: S   m  x 3x  3m m3 x  3x dx     x  3x dx        0  m 2 → Chọn B Ví dụ 5: Cho  H  hình phẳng giới hạn parabol y  3x , cung tròn có phương trình y   x (với  x  ) trục hồnh (phần tơ đen hình vẽ) Diện tích  H  A 4  12 B 4  C 4   D  2 Hướng dẫn Ta có  x   thỏa mãn   4 3x   x  3x  x    x   x      3  x  1  loại    Do đó: S   3x dx    x dx   2 31 x    x dx     x dx 3 1 Trang Ta tính: I    x dx 1    x   sin t   t  Đặt x  2sin t  dx  cos tdt Đổi cận:   x   sin t   t    2       I    x dx    4sin t.2 cos tdt   cos tdt    cos 2t  1 dt   6  sin 2t 2  2t 2  Suy S  2  3 2 4     3 Chú ý: Ta tính tích phân S máy tính CASIO fx 570 VN PLUS, sau đối chiếu với bốn đáp án → Chọn B Ví dụ 6: Một vật chuyển động với vận tốc v (km/h) phụ thuộc thời gian t(h) có đồ thị phần đường parabol có đỉnh I (1;1) trục đối xứng song song hình bên Tính quãng đường S mà vật di chuyển kể từ lúc xuất phát (tính theo km) A S  6km C S  B S  8km 46 km D S  40 km Hướng dẫn Gọi parabol  P  có dạng y  at  bt  c,  a   a  a  b  c    Đồ thị  P  qua điểm M  0;  đỉnh I 1;1 suy  b  b  2  2a  1; c  c   Suy  P  :y  t  2t  Vậy quãng đường S cần tính t  2t  dt  40 km → Chọn D Ví dụ 7: Cho hàm số y  f  x   ax  bx  cx  d  a, b, c  , a   có đồ thị  C  Biết đồ thị  C  tiếp xúc với đường thẳng y  điểm có hồnh độ âm đồ thị hàm số y  f   x  cho hình vẽ bên: Trang Tính diện tích S hình phẳng giới hạn đồ thị  C  trục hoành A S  C S  21 B S  27 D S  Hướng dẫn Từ đồ thị suy f   x   3x  f  x    f   x  dx    3x  3dx  x  3x  C Do  C  tiếp xúc với đường thẳng y  điểm có hoành độ x âm nên f   x    3x 02    x  1 Suy f  1   C    C  : y  x  3x   x  2 Xét phương trình x  3x     x  1 Diện tích hình phẳng cần tìm là:  x  3x   dx  2 27 → Chọn B Ví dụ 8: Cho hàm số y  f  x  Đồ thị hàm số y  f   x  hình bên Đặt y  f   x  hình bên Đặt h  x   2f  x   x Mệnh đề đúng? A h    h  2   h   B h    h  2   h   C h    h    h  2  D h    h  2   h   Hướng dẫn Gọi S1 ,S2 diện tích hình phẳng hình vẽ bên Ta có 2S1   f   x   x dx   2f  x   x  2 2  h  x  2  h    h  2    h    h  2  (1) Tương tự ta có 2S2    x  f   x  dx   2f  x   x  2 2   h x Trang  h    h     h    h   (2) Nhìn đồ thị ta có S1  S2  2S1  2S2  h    h  2   h    h    h    h  2  (3) Từ (1), (2), (3) suy h    h    h  2  → Chọn C Bài tập tự luyện Câu Diện tích hình phẳng giới hạn đồ thị hàm số y  e x  , trục hoành hai đường thẳng x  ln 3, x  ln nhận giá trị sau đây? A S   ln 3 B S   ln C S   ln D S   ln Câu Diện tích hình phẳng giới hạn đồ thị hàm số y  x  x , trục tung đường thẳng x  A S  B S  2 1 C S  2 1 D S    1 Câu Diện tích hình phẳng giới hạn đồ thị hàm số y  e x  x, y  x  x  ln A S   ln B S   ln C S   ln D S   ln Câu Tính diện tích hình phẳng giới hạn đồ thị hàm số y  x ln x trục hoành đường thẳng xe A S  e2  B S  e2  C S  e2  D S  e2  Đáp án: 1–B 2–B 3–D 4–A Dạng 2: Thể tích vật thể thể tích khối tròn xoay Phương pháp giải a Thể tích vật thể: Gọi B phần vật thể giới hạn hai mặt phẳng vng góc với trục Ox điểm a b S  x  diện tích thiết diện vật thể bị cắt mặt phẳng vng góc với trục Ox điểm có hồnh độ x a  x  b S x  liên tục đoạn  a; b  Cơng thức tính thể tích B b V   S  x  dx a b Thể tích khối tròn xoay giới hạn đường  C  :y  f  x  , trục hoành y  , hai đường thẳng x  a, x  b  a  b  sinh quay quanh Ox là: Trang b V   f  x  dx a c Thể tích khối tròn xoay giới hạn hai đường y  f x, y  gx , hai đường thẳng x  a, x  b  a  b  sinh quay quanh Ox là: b V   f  x   g  x  dx a Ví dụ minh họa Ví dụ 1: Tính thể tích vật thể nằm hai mặt phẳng có phương trình x  x  , biết thiết diện vật thể bị cắt phần mặt phẳng vng góc với trục Ox điểm có hồnh độ x   0; 2 phần tư đường tròn bán kính A V  32 2x , ta kết nào? C V  B V  64 16  D V  8 Hướng dẫn 1 Diện tích thiết diện là: S  x   S r    4  2x   x 2   x5  x  25 16 Khi đó, thể tích cần tìm là: V   dx      2 5   0 → Chọn C Ví dụ 2: Thể tích phần vật thể giới hạn hai mặt phẳng có phương trình x  x  , có thiết diện bị cắt phần mặt phẳng vng góc với trục Ox điểm có hồnh độ x   x  3 hình chữ nhật có hai kích thước x  x A V  B V  18 C V  20 D V  22 Hướng dẫn Diện tích hình chữ nhật có hai cạnh x  x 2x  x b a Khi đó, thể tích vật thể xác định công thức: V   S  x  dx   2x  x dx Đặt t   x  t    t   x  xdx   tdt x   t  Đổi cận:  x   t  Trang 2t Suy V  2  t dt   18 3 → Chọn B Ví dụ 3: Khối tròn xoay tạo nên ta quay quanh trục Ox hình phẳng D giới hạn đồ thị  P  : y  2x  x A V  trục Ox tích 16  15 B V  11  15 C V  12  15 D V   15 Hướng dẫn x  Phương trình hồnh độ giao điểm  P  Ox 2x  x    x  Thể tích khối tròn xoay: V    2x  x  2  x5 4x  16 dx    x  4x  4x dx     x     15  → Chọn A Ví dụ 4: Thể tích khối tròn xoay giới hạn đồ thị hàm số y   x , y  0, x  x  quay quanh trục Ox A 8 B 2 C 46  15 D 5 Hướng dẫn Thể tích khối tròn xoay giới hạn đồ thị hàm số y   x , y  0, x  x  quay quanh trục Ox là: V   1  x  2  2x x  46 dx   1  2x  x  dx    x      15  2 → Chọn C Ví dụ 5: Thể tích vật thể tròn xoay quay quanh hình phẳng giới hạn đường y   x , y  quanh trục Ox có kết dạng A 11 a a  a; b    ; phân số tối giản Khi a  b có kết b b B 17 C 31 D 25 Hướng dẫn Ta có  x   x  1 Vậy V    1  x  dx  1 16  a  16, b  15  a  b  31 15 → Chọn C Ví dụ 6: Quay hình phẳng  H  hình tơ đậm hình vẽ bên quay quanh trục Ox ta khối tròn xoay tích Trang A V  3 B V  3 C V  3 D V  3 Hướng dẫn x  y2   x   x   Xét hệ phương trình:    y  y   x  Do  H  đối xứng qua Oy nên: 3 V  2    x   12 dx  2    x dx 0  x3   2  3x    3 0  → Chọn A Ví dụ 7: Một chng có dạng hình vẽ Giả sử cắt chuông mặt phẳng qua trục chng, thiết diện có đường viền phần parabol (hình vẽ) Biết chng cao 4m, bán kính miệng chng 2 Tính thể tích chng A 6 B 12 C 23 D 16 Hướng dẫn Xét hệ trục hình vẽ, dễ thấy parabol qua ba   điểm  0;0  , 4; 2 , 4; 2 x  nên có phương trình y2 Thể tích chng thể tích khối tròn xoay tạo hình phẳng y  2x, x  0, x  quay quanh trục Ox Do V   2xdx   x   16 → Chọn D Bài tập tự luyện   Câu Cho hình phẳng  H  giới hạn đồ thị hàm số y  cosx ,   x   hai trục tọa độ Ox, 2  Oy Thể tích khối tròn xoay tạo thành quay  H  xung quanh trục Ox A  B C  D  Trang Câu Hình phẳng giới hạn đồ thị hàm số y  2x  x y  x quay quanh trục Ox tạo thành tích khối tròn xoay tích A V   B V   C V   D V   Câu Tính thể tích vật thể nằm hai mặt phẳng x  0; x   , biết thiết diện vật thể với mặt phẳng vng góc với trục Ox điểm có hồnh độ x   x    tam giác có cạnh s inx A B  C D 2 Câu Kí hiệu V1 , V2 thể tích hình cầu bán kính đơn vị thể tích khối tròn xoay sinh quay hình phẳng giới hạn đường thẳng y  2x  đường cong y   x xung quanh trục Ox Hãy so sánh V1 , V2 A V1  V2 B V1  V2 C V1  V2 D V1  2V2 Câu Hình phẳng S1 giới hạn y  f  x  , y  0, x  a, x  b  a  b  quay quanh Ox tích V1 Hình phẳng S2 giới hạn y  2f  x  , y  0, x  a, x  b  a  b  quay quanh Ox tích V2 Lựa chọn phương án A V1  4V2 B V2  8V1 C 2V1  V2 D 4V1  V2 Đáp án: 1–A 2–C 3–C 4–B 5–D Dạng 3: Ứng dụng nguyên hàm tích phân toán thực tế Phương pháp giải  Giả sử v  t  vận tốc vật thời điểm t s  t  quãng đường vật sau khoảng thời gian t tính từ lúc bắt đầu chuyển động Mối liên hệ s  t  v  t  sau: Đạo hàm quãng đường vận tốc: s  t   v  t  Chú ý: Khi vật dừng hẳn  v  t   Nguyên hàm vận tốc quãng đường s  t    v  t  dt Quãng đường vật khoảng thời gian t   a; b  b  v  t  dt  s  b   s  a  a  Nếu gọi a  t  gia tốc vật ta có mối liên hệ v  t  a  t  sau: Đạo hàm vận tốc gia tốc: v  t   a  t  Nguyên hàm gia tốc vận tốc v  t    a  t  dt Trang 10 Ví dụ minh họa Ví dụ 1: Một vật chuyển động chậm dần với vận tốc v  t   160  10t  m / s  Quãng đường mà vật chuyển động từ thời điểm t   s  đến thời điểm mà vật dừng lại A 1028 m B 1280 m C 1308 m D 1380 m Hướng dẫn Khi vật dừng lại v  t   160  10t  t  16 Quãng đường mà vật chuyển động từ thời điểm t   s  đến thời điểm mà vật dừng lại 16 16 S   v  t  dt   160  10t  dt  160t  5t  0 16  160.16  5.162  1280 m → Chọn B Ví dụ 2: Một tơ chuyển động với vận tốc v  t   m / s  , có gia tốc a  t   v  t    m / s2  2t  Vận tốc tơ sau 10 giây (làm tròn đến hàng đơn vị) A 4,6 m/s B 7,2 m/s C 1,5 m/s D 2,2 m/s Hướng dẫn Vận tốc ô tô sau 10 giây là: 10 10 10 3 v   a  t  dt   dt  ln 2t   ln 21  4,  m / s  2t  2 0 → Chọn A Ví dụ 3: Một vật chuyển động với vận tốc10 m/s tăng tốc với gia tốc a  t   3t  t Tính quãng đường vật khoảng thời gian 10 giây kể từ lúc bắt đầu tăng tốc A 4300 m B 4300 m C 430 m D 430 m Hướng dẫn Hàm vận tốc v  t    a  t  dt    3t  t dt  3t t  C Lấy mốc thời gia lúc tăng tốc  v    10  C  10 Ta v  t   3t t   10 Sau 10 giây, quãng đường vật là: 10  3t t   t3 t  4300 S    10 dt     10t   m 3   12 0 0 10 → Chọn A Ví dụ 4: Gọi h  t   cm  mức nước bồn chứa sau bơm t giây Biết h   t   13 t 8 Trang 11 lúc đầu bồn khơng có nước Tìm mức nước bồn sau bơm nước giây (chính xác đến 0,01 cm) A 2,67 cm B 2,66 cm C 2,65 cm D 2,68 cm Hướng dẫn Hàm h  t    13 t    t  8 t   C 20 Lúc t = 0, bồn không chứa nước Suy h     12 12 12  C   C   Vậy, hàm h  t    t   t   5 20 Mức nước bồn sau giây h    2, 66 cm → Chọn B Ví dụ 5: Một đám vi trùng ngày thứ t có số lượng N  t  Biết N  t   4000 lúc đầu đám  0,5t vi trùng có 250000 Hỏi sau 10 ngày số lượng vi trùng gần với số sau nhất? A 251000 B 264334 C 261000 D 274334 Hướng dẫn Nt   4000 dt  8000.ln  0,5t  C  0,5t Lúc đầu có 250000 con, suy N    250000  C  250000 Vậy N  t   8000.ln  0,5t  250000  N 10   264334, 0758 → Chọn B Bài tập tự luyện Một hạt prôtôn di chuyển điện trường có biểu thức gia tốc ( theo cm / s ) 20 a t  ( với t tính giây) Tìm hàm vận tốc v theo t, biết t  v  30  cm / s  1  2t  Câu A v  10  2t B v  10  20  2t C v  1  2t   30 3 D v  20 1  2t   30 Câu Một tia lửa bắn thẳng đứng từ mặt đất với vận tốc 15 m/s Hỏi sau 2,5 giây, tia lửa cách mặt đất mét, biết gia tốc 9,8m / s A 30,625 m B 37,5 m C 68,125 m D 6,875 m Câu Một vật chuyển động với vận tốc v  t    2sin 2t  m / s  Quãng đường mà vật chuyển động khoảng thời gian t   s  đến thời điểm t  A 3 1 B 3  3  s  C 3  D 3 1 Trang 12 t2   m / s  Quãng đường mà vật t2 giây bao nhiêu? (làm tròn kết đến hàng phần trăm) Câu Một vật chuyển động với vận tốc v  t   1,5  A 12,60 m B 12,59 m C 0,83 m D 6,59 m Đáp án: 1–B 2–D 3–A 4–B Phần BÀI TẬP TỔNG HỢP Câu Tính diện tích hình phẳng giới hạn đồ thị hàm số y  e x , y  e  x , x  A e  2e  e B e  2e  e C e  2e  e D e  2e  e Câu Thể tích khối tròn xoay giới hạn đồ thị hàm số y  ln x, y  0, x  1, x  quay quanh trục Ox có kết A 2  ln  1 B 2  ln  1 2 C   ln  1 Câu Diện tích hình phẳng giới hạn đồ thị hàm số y  A S  ln  B S  ln D   ln  1 2 3x  , Ox, Oy x 1 C S  ln  D S  ln  Câu Cho hàm số y  f  x  liên tục đoạn  a; b  Gọi D diện tích hình phẳng giới hạn đồ thị  C  : y  f  x  , trục hoành, hai đường thẳng x  a, x  b (như hình vẽ đây) Giả sử SD diện tích hình phẳng D Chọn cơng thức b a 0 b a A SD   f  x  dx   f  x  dx C SD   f  x  dx   f  x  dx b a 0 b a B SD    f  x  dx   f  x  dx D SD    f  x  dx   f  x  dx Câu Một ô tô chạy với vận tốc 18m/s người lái hãm phanh Sau hãm phanh, ô tô chuyển động chậm dần với vận tốc v  t   36t  18  m / s  , t khoảng thời gian tính giây kể từ lúc bắt đầu hãm phanh Quãng đường ô tô di chuyển kể từ lúc hãm phanh đến dừng mét? A 5,5 m B 3,5 m C 6,5 m D 4,5 m Trang 13 Câu Cho hình phẳng giới hạn đồ thị hàm số y  , y  0, x  0, x  quay xung quanh   3x trục Ox Thể tích khối tròn xoay tạo thành A    ln  1 6  B    ln  1 4  C    ln  1 6  D    ln  1 9  Câu Một ô tô chạy với vận tốc 20m / s người lái đạp phanh Sau đạp phanh, ôtô chuyển động chậm dần với vận tốc v  t   40t  20  m / s  , t khoảng thời gian tính giây kể từ lúc bắt đầu đạp phanh Hỏi từ lúc đạp phanh đến dừng hẳn, tơ di chuyển mét? A 10m B 7m C 5m D 3m Câu Diện tích hình phẳng giới hạn đồ thị hàm số y  e  x , trục hoành, trục tung đường x thẳng x  A S  e  B S  e  C S  e  D S  e  Câu Cho đồ thị hàm số y  f  x  Diện tích hình phẳng (phần gạch hình) 3 A I   f  x  dx   f  x  dx 0 3 3 0 B I   f  x  dx   f  x  dx C I   f  x  dx D I   f  x  dx   f  x  dx 3 Câu 10 Cho hình thang cong  H  giới hạn đường y  e x , y  0, x  0, x  ln Đường thẳng x  k   x  ln  chia  H  thành hai phần có diện tích S1 S2 hình vẽ bên Tìm k để S1  2S2 A k  ln B ln C ln D ln Câu 11 Trong hệ tọa độ Oxy, parabol y  x2 chia đường tròn tâm O (O gốc tọa độ) bán kính r  2 thành phần, diện tích phần nhỏ A 2  B C 2  3 D 2  Trang 14 Đáp án: 1–B 2–A 3–C 4–B 5–D 6–D 7–C 8–B 9–B 10 – D 11 - A Trang 15 CHƯƠNG 4: SỐ PHỨC CHUYÊN ĐỀ 1: CÁC PHÉP TOÁN TRÊN TẬP SỐ PHỨC PHẦN 1: LÝ THUYẾT TRỌNG TÂM Định nghĩa Số phức có dạng Xét số phức sau: z  a  bi(a, b  ) Phần thực Phần ảo Đơn vị ảo i  1 Nếu a = 0, số phức z số ảo z  2i Phần thực Số ảo z  3i   3i Phần thực Nếu b = 0, số phức z số thực Phần ảo – Phần ảo Số thực: z    0i Phần thực Phần ảo Tập hợp số phức  Ta có: N       Chú ý: Số vừa số thực, vừa số ảo Số đối số phức z = – i – z = - + i Số đối số phức z  a  bi z  a  bi Số đối số phức z = – z = -3 Số đối số phức z = 3i – z = -3i Tính chất đơn vị ảo i Nhan i Nhan i i  1   i3  i i  i   i  i3 i  i.i   i x  ? Để tính i x ta thực phép chia x cho Tính giá trị i 2018 Nếu số dư 0, ta kết Ta chia 2018 cho 4, được: Nếu số dư 1, ta kết i 2018  504.4  2, dư Nếu số dư 2, ta kết -1 Do i 2018  1 Nếu số dư 3, ta kết -i Số phức liên hợp Số phức liên hợp số phức z  a  bi Số phức liên hợp z  a  bi Số phức z = – i z   i Chú ý: Số phức z = 3i z  3i  z z số thực  z  z Số phức z = z   z z số ảo  z  z Trang Môđun số phức Môđun số phức z  a  bi z  a  b Môđun số phức Chú ý: z   i z  22  (1)  z  0, a   z  3i z  02  32  z 0z0 z = z  32  02  Hai số phức Cho hai số phức z1  a1  b1i, z  a  b 2i Hai số Cho hai số phức z1  a  2i, z  3  bi Hai số phức khi: phức khi: a  a  b1  b a  3  b  Các phép toán tập số phức Cho hai số phức z1  a1  b1i, z  a  b 2i Cho hai số phức z1   2i, z  3  5i Tổng, hiệu hai số phức Tổng, hiệu hai số phức z1  z   a1  a    b1  b  i z1  z  (1  2i)  (3  5i)   2i   5i  (1  3)  (2  5)i  2  3i z1  z z1  z   a1  a    b1  b  i  (1  2i)  (3  5i)   2i   5i  (1  3)  (2  5)i   7i Phép nhân hai số phức Phép nhân hai số phức z1 z  (a1  b1i)(a  b i)  a1 a  a1 b i  a b1i  b1 b i z1 z  (1  2i)(3  5i)  a1 a  a1 b i  a b1i  b1 b   a1 a  b1 b    a1 b  a b1  i Phép chia hai số phức  3  5i  6i  10i  3  11i  10   11i Phép chia hai số phức Muốn chia hai số phức, ta nhân tử mẫu với liên hợp mẫu Trang z1  2i  z 3  5i z1 a1  b1i  z a  b 2i    (1  2i)(3  5i) (3  5i)(3  5i) 3  5i  6i  10 13  i   (3)  52 34 13   i 34 34 (a1  b1i)(a  b 2i) (a  b 2i)(a  b 2i)   a1a  b1b2   (b1a  a1b2 )i a 22  b 22 a1a  b1b  b1a  a1b  i  a 22  b 22 a 22  b 22 PHẦN 2: CÁC DẠNG BÀI TẬP Dạng 1: Các phép toán tập số phức Phương pháp giải Các phép tính số phức: Sử dụng công thức cộng, trừ, nhân, chia lũy thừa số phức Tìm phần thực phần ảo, số phức liên hợp, môđun số phức: số phức z = a + bi có phần thực a, phần ảo b, số phức liên hợp z  a  bi môđun z  a  b 2 Ví dụ minh họa Ví dụ 1: Cho số phức z  (2  7i)(1  3i) Số phức liên hợp z là: A z   7i B z  2  7i C z  2  7i D z  23  i Hướng dẫn Cách 1: z  (2  7i)(1  3i)  2  6i  7i  21i  2  21  i(6  7)  23  i Do số phức liên hợp z z  23  i Cách 2: Sử dụng máy tính FX750VNPLUS Bước 1: Thiết lập chế độ số phức: MODE Bước 2: Nhập (2+7i)(-1+3i) ta kết -23 – i Do số phức liên hợp z z  23  i  Chọn D Ví dụ 2: Cho số phức z thỏa mãn (3  2i)z  (2  i)  20  3i Hiệu phần thực phần ảo số phức z là: A B C D Hướng dẫn (3  2i)z  (2  i)  20  3i  (3  2i)z   4i  i  20  3i Cách 1: 17  7i (17  7i)(3  2i) 65  13i  (3  2i)z  17  7i  z   z  5i 2  2i 2 13 Có phần thực 5, phần ảo – Vậy hiệu phần thực phẩn ảo z – (-1) = Cách 2: Sử dụng máy tính fx 570 VNPLUS Bước 1: Thiết lập chế độ sử dụng số phức: MODE Trang Bước 2: Nhập 20  3i  (2  i) ta kết – i  2i Vậy hiệu phần thực phẩn ảo z – (-1) =  Chọn D Ví dụ 3: Cho số phức z = – i Tìm số phức w  A w   i 5 zi z i B w    i 5 C w   i 5 D w    i 5 Hướng dẫn Cách 1: Ta có w  z i 3i i 3(2  i)  3i       i z   i   i  (1) 5 Cách 2: Cách sử dụng máy tính fx 570 VNPLUS Bước 1: Thiết lập chế độ sử dụng số phức: MODE Bước 2: Nhập 3i i ta kết w   i  i 1 5  Chọn A Ví dụ 4: Cho số thực x, y thỏa mãn: 3x + y – 3xi = 2y – + (x – y)i Tính tổng x + y A -3 B C -1 D Hướng dẫn 3x  y  2y  3x  y  1  x  1 3x  y  3xi  2y   (x  y)i     3x  x  y 2x  y   y  2 Do x + y = -3  Chọn A Bài tập tự luyện Câu Cho hai số phức z1   i z   2i Phần ảo số phức w  2z1  3z là: A B 4i C -6 D -6i Câu Tìm môđun số phức z thỏa mãn (1  i)z  (2  i)(3  i) A B C Câu Cho số phức z  (4  2i)(2  3i) Tìm phần ảo số phức w  A 10 B C D z2 zz D Câu Khẳng định sau sai? A Môđun số phức z số thực không âm B Môđun số phức z số thực C Môđun số phức z = a + bi z  a  b D Môđun số phức z luôn số thực dương Trang Đáp án: 1–A 2–A 3–B 4–D Dạng 2: Tìm số phức thỏa mãn điều kiện cho trước Phương pháp giải Để tìm số phức thỏa mãn điều kiện cho trước, ta Ví dụ: Tìm phần thực số phức z biết z thỏa mãn làm theo bước sau: z  2z   i A B C D -1 Hướng dẫn Bước 1: Gọi số phức cần tìm có dạng Gọi z  x  yi(x, y  ) Ta có: z  x  yi(x, y  ) z  2z   i  (x  yi)  2(x  yi)   i Bước 2: Thay số phức vào phương trình, khai triển  x  yi  2x  2yi   i Bước 3: Chuyển vế, rút gọn đưa dạng  x  yi  2x  2yi   i  A + Bi =  (x  2x  3)  i(y  2y  1)   (3x  3)  i( y  1)  Bước 4: Cho phần thực A 0, phần ảo B Ta có hệ: Thiết lập hệ phương trình 3x   x  A      y    y  1 B  Vậy z = – i có phần thực Bước 5: Giải hệ phương trình, tìm số phức z  Chọn B Ví dụ minh họa Ví dụ 1: Cho số phức z thỏa mãn (3  2i)z   11i  (2  2i)z Môđun số phức z là: A 10 B C D Hướng dẫn Cách 1: Gọi z  x  yi(x, y  ) Ta có: (3  2i)z   11i  (2  2i)z  (3  2i)(x  yi)   11i  (2  2i)(x  yi)  3x  3yi  2xi  2yi   11i  2x  2yi  2xi  2yi  3x  3yi  2xi  2y   11i  2x  2yi  2xi  2y   (3x  2y   2x  2y)  (3x  2x  11  2y  2x)i   (x  1)  (4x  5y  11) i  x   x  Ta có hệ:   4x  5y  11  y  3 Vậy z = – 3i nên z  32  (1)  10 Cách 2: Cách sử dụng máy tính fx 570 VNPLUS Trang Bước 1: Thiết lập chế độ sử dụng số phức: MODE Bước 2: Nhập (3  2i)(X  Yi)   11i  (2  2i)(X  Yi) Bước 3: Gán giá trị X = 0, Y = 0: CACL X? = Y ? 0, ta kết – – 11i, điền vào giá trị cột c bảng bước Bước 4: Nhập (3  2i)(X  Yi)   11i  (2  2i)(X  Yi)  (1  11i) Bước 5: Gán giá trị X = 0, Y = : CACL X? = Y ? =, ta kết -5i, điền vào giá trị cột b bảng bước Bước 6: Gán giá trị X = 1, Y = : CACL X? = Y ? =, ta kết - 4i, điền vào giá trị cột b bảng bước Bước 7: Ta có bảng a b C 1 -4 -5 -11 1x  0y   x  Bước 8: Giải hệ phương trình   4x  5y  11   y  3 Do số phức z thỏa mãn yêu cầu đề z = – 3i Vậy z  10  Chọn A Chú ý: Ta tổng quát cách bấm máy tính dạng tập theo bước sau: Bước 1: Thiết lập chế độ sử dụng số phức: MODE Bước 2: Nhập biểu thực đề cho, ý chuyển tất sang vế trái Bước 3: Gán giá trị X = 0, Y = : CACL X? = Y ? =, ta kết c1  c 2i Bước 4: Nhập biểu thức bước 1, trừ kết bước Bước 5: Gán giá trị X = 0, Y = : CACL X? = Y ? =, ta kết b1  b 2i Bước 6: Gán giá trị X = 1, Y = : CACL X? = Y ? =, ta kết a1  a 2i Bước 7: Ta có bảng a b c a1 b1 c1 a2 b2 c2 a x  b1 y  c1   x  x1 Bước 8: Giải hệ phương trình   a x  b y  c   y  y1 Ta số phức z z  x1  y1i Ví dụ 2: Cho số phức z thỏa mãn: z  z   4i Số phức liên hợp z là: A – 4i B + 4i C -3 + 4i D -3 – 4i Hướng dẫn Trang Cách 1: Đặt z  x  yi(x, y  )  z  x  y Khi đó: z  z   4i  x  yi  x  y   4i  (x  x  y  8)  (y  4) i   x  x  y    x  x  y     y    y  4  x  16   x  x  16  64  16x  x 16x  48  x      y   y     y  4  y  4  Vậy z = – 4i  z   4i Cách 2: Thử trực tiếp đáp án Đáp án A: z   4i  z   4i , z  z   4i  32  42   4i, loại Đáp án B: z   4i  z   4i , z  z   4i  32  (4)   4i, thỏa mãn  Chọn B Ví dụ 3: Cho số phức z thỏa mãn z.z  z   Xác định phần thực số phức z A B C -1 D Hướng dẫn Đặt: z  x  yi(x, y  ) Suy z  x  yi Ta có: z.z   (x  yi)(x  yi)   x  (yi)   x  y  z   (x  1)  i( y)  z    (x  1)  ( y)   (x  1)  y  Ta có hệ phương trình:  x  y   y   x  y2   x  x  1        2 2 (x  1)  y  (x  1)   x  y  2x   Vậy z = -1, có phần thực -1, phần ảo  Chọn C Ví dụ 4: Cho số phức z thỏa mãn (1+2i)z số ảo 2z  z  13 Môđun số phức z A B C D 13 Hướng dẫn Giả sử z  x  yi(x, y  ) , (1  2i)z  (1  2i)(x  yi)  (x  2y)  (2x  y)i Vì (1  2i)z số ảo x  2y   x  2y 2z  z  x  3yi  2y  3yi  13y  13  y  1 Với y = 1, ta có x = 2, số phức z   i  z  22  12  Với y = -1, ta có x = -2, số phức z  2  i  z  (2)  (1)   Chọn C Trang Bài tập tự luyện Câu Cho số phức z thỏa mãn z  z   8i Tìm số phức liên hợp z A -15 – 8i B -15 + 6i C -15 +2i D -15+ 7i Câu Cho số phức z thỏa mãn hệ thức: z  2z  (1  5i) Tính modun z A 45 B 41 C 40 D 41 Câu Tìm số phức z biết: (z  1)  z   10i  z  Tìm phần ảo số phức z, biết z có phần thực số dương A B C  D Đáp án: 1–A 2–D 3-A PHẦN 3: BÀI TẬP TỔNG HỢP Câu Modun số phức z   3i A B C D C D -8 Câu Phần thực số phức z  (2  3i)(1  2i) A B -1 Câu Cho số phức z1   3i, z  3  2i Tính modun số phức z1  2z A 24 B C 74 D 74 Câu Cho số phức z   3i Tìm số phức liên hợp w biết w  iz  z A w   3i B w  1  i C w   i Câu Cho số phức z thỏa mãn điều kiện (2  i)z  D w  1  i 1 i   i Modun số phức w   2z  z có 1 i giá trị A 10 B -10 C 100 D -100 Câu Cho số phức z thỏa mãn điều kiện (2  i)z   3i  Phần ảo số phức w   iz  z A B C -2 D -1 Câu Cho số phức z thỏa mãn: 3z  2z  (4  i) Tổng phần thực phần ảo số phức z A -11 B C 11 D -5 Câu Số phức z thỏa mãn: z  (2  3i)z   9i Modun z A B C D Câu Tìm modun số phức z thỏa mãn hệ thức z  (2  i)  10 z.z  25 A B -5 C 10 D 10 Câu 10 Tìm hai số x, y Biết x, y số thực thỏa mãn đẳng thức x(3  5i)  y(1  2i)  35  23i Trang A (x;y)  (3;4) B (x;y)  (3;4) C (x;y)  (3; 4) D (x;y)  (3; 4) C D -4 Câu 11 Giá trị i105  i 23  i 20  i 38 A B -2 Đáp án: 1–C 2–A –C 4–D 5–A 6–C 7–D 8–D 9–A 10 - B 11 - A Trang CHƯƠNG 4: SỐ PHỨC CHUYÊN ĐỀ 2: TẬP HỢP ĐIỂM BIỂU DIỄN SỐ PHỨC PHẦN 1: LÝ THUYẾT TRỌNG TÂM Kiếm thức hình học giải tích mặt phẳng Tọa độ điểm: Trong mặt phẳng tọa độ Oxy, cho hai điểm Trong mặt phẳng tọa độ Oxy, cho hai điểm A(1;2),B(3; 4)  A(x A ;y A ),B(x B , y B ) AB  (3  (1); 4  2)  (4; 6)  AB  (x B  x A ;y B  y A ) Độ dài AB   x B  x A    yB  yA  2 Phương trình đường thẳng: Dạng tổng quát ax + by + c =  Trong n  (a;b) vectơ pháp tuyến (VTPT) Độ dài AB   (6)2  13 Phương trình 3x – y + = phương trình đường  thẳng có vectơ pháp tuyến n  (3; 1) đường thẳng Phương trình đường tròn tâm I(a;b), bán kính R: (x  a)  (y  b)  R 2 Phương trình x  y  2ax  2by  c  với điều kiện a  b  c  phương trình đường tròn có 2 Phương trình (x  1)2  (y  3)2  phương trình đường tròn tâm I(-1;3), bán kính R = Phương trình x  y  2x  6y   có tâm I(a,b) bán kính R  a  b  c a  1;b  3;c  1;a  b  c   phương trình đường tròn tâm I(-1;3), bán kính R = Phương trình elip: Phương x y2  1 a b2 Với hai tiêu cự F1 (c;0), F2 (c;0);F1F2  2c Độ dài trục lớn 2a, độ dài trục bé 2b a  b  c2 trình đường elip x y2   có 25 a  5;b  3;c  a  b  Với hai tiêu cự F1 (4;0), F2 (4;0), F1 F2  Độ dài trục lớn 10, độ dài trục bé Biểu diễn hình học số phức Trong mặt phẳng phức Oxy, số phức Số phức z   i biểu diễn điểm A(3;1) z  a  bi(a, b  ) biểu diễn điểm M(a;b) Số phức liên hợp z z   i biểu diễn (Oy trục ảo, Ox trục thực) điểm B(3;-1) Số đối z – z = - – i biểu diễn điểm C(-3;-1) Trang Chú ý: Hai điểm biểu diễn số phức z z đối xứng với Hai điểm A B đối xứng với qua Ox Hai điểm A C đối xứng với qua tâm O qua trục Ox Hai điểm biểu diễn số phức z – z đối xứng với qua tâm O Ý nghĩa hình học mođun:  Đồ dài vecto OM mođun số phức z  z  OM  OM Độ dài OA  10  z Tập hợp điểm biểu diễn số phức Quỹ tích điểm M biểu diễn số phức z  x  yi đường thẳng điểm M(x;y) thỏa mãn phương trình đường thẳng Ax + By + C = Quỹ tích điểm M biểu diễn số phức z  x  yi đường tròn điểm M (x;y) thỏa mãn phương trình đường tròn (x  a)2  (y  b)2  R Trong I(a;b) tâm đường tròn R bán kính đường tròn Quỹ tích điểm M biểu diễn số phức z = x + yi đường elip điểm M(x;y) thỏa mãn phương trình đường elip (E) : x y2   1, a, b bán kính trục lớn, trục nhỏ elip a b2 PHẦN 2: CÁC DẠNG BÀI TẬP Dạng 1: Phương pháp giải Số phức z = a + bi biểu diễn điểm M(a;b) mặt phẳng tọa độ Oxy Ví dụ minh họa Ví dụ 1: Cho số phức z  1  2i Điểm biểu diễn số phức liên hợp z mặt phẳng phức A M(-1;-2) B M(-1;2) C M(-2;1) D M(2;-1) Hướng dẫn Số phức liên hợp z z  1  2i nên có điểm biểu diễn M(-1;2)  Chọn B Ví dụ 2: Cho số phức z = -1 +3i Điểm biểu diễn số phức 3  A M   ;    10 10   3 B M   ;   10 10  mặt phẳng phức z 3  C M  ;    10 10   3 D M  ;   10 10  Hướng dẫn Ta có 3 1 1  3i       i có điểm biểu diễn M   ;   2 z 1  3i (1)  10 10  10 10   Chọn A Trang Ví dụ 3: Điểm hình vẽ điểm biểu diễn số phức z = (1 + i)(3 – i)? A P B M C N D Q Hướng dẫn Ta có z  (1  i)(3  i)   i  3i  i   2i    2i có điểm biểu diễn Q(4;2)  Chọn D Bài tập tự luyện Câu Cho số phức z thỏa (1  2i) z   3i Tìm tọa độ điểm M biểu diễn số phức z mặt phẳng phức A M(2; 1) B M(2;1) C M(2; 1) D M(2;1) Câu Gọi A điểm biểu diễn số phức z1   i B điểm biểu diễn z   i Trong khẳng định sau, khẳng định đúng? A Hai điểm A B đối xứng qua trục tung B Hai điểm A B đối xứng qua gốc tọa độ O C Hai điểm A B đối xứng qua đường thẳng y = x D Hai điểm A B đối xứng qua trục hoành Câu Cho số phức z thỏa mãn (2  i)z   3i Điểm biểu diễn z điểm hình bên? A Điểm M B Điểm N C Điểm P D Điểm Q Đáp án: 1–D 2–D 3-C Dạng 2: Tập hợp điểm biểu diễn số phức Phương pháp giải Giả sử số phức z =x + yi biểu diễn điểm M(x;y) Tìm tập hợp điểm M tìm hệ thức x y thỏa mãn yêu cầu đề Chú ý: Tập hợp điểm M thỏa mãn điều kiện z  (a  bi)  R,(R  0) đường tròn có tâm I(a;b) bán kính R Tập hợp điểm M thỏa mãn điều kiện z  (a  bi )  R, ( R  0)  z  (a  bi )  R đường tròn có tâm I (a; b) có bán kính R Tập hợp điểm M thỏa mãn điều kiện z  (a1  b1i)  z  (a  b i) đường trung trục đoạn thẳng AB với A(a1 , b1 );B(a , b ) Trang Ví dụ minh họa Ví dụ 1: Trong mặt phẳng với hệ tọa độ Oxy, tập hợp điểm biểu diễn số phức z thỏa mãn điều kiện z  i(2  i)  Phát biểu sau sai? A Tập hợp điểm biểu diễn số phức z đường tròn tâm I(-1;-2) B Tập hợp điểm biểu diễn số phức z đường tròn có bán kính R = C Tập hợp điểm biểu diễn số phức z đường tròn có đường kính 10 D Tập hợp điểm biểu diễn số phức z đường tròn có tâm I(1;2) Hướng dẫn Cách 1: Gọi z  x  yi,(x;y  ) Theo giả thiết, ta có: z  i(2  i)   x  yi  2i  i   x  y  2i    (x  1)  i(y  2)    x  1   y   2   (x  1)2  (y  2)2  25 Vậy tập hợp điểm biểu diễn số phức z đường tròn tâm I(-1;-2), bán kính R = Cách 2: z  i(2  i)   x  yi  2i  i   z  2i    z  (1  2i)  Do áp dụng “tập hợp điểm M thỏa mãn điều kiện z  (a  bi)  R,(R  0) đường tròn có tâm I(a;b) bán kính R” ta tập hợp điểm biểu diễn số phức z đường tròn tâm I(-1;-2), bán kính R =  Chọn D Ví dụ 2: Tập hợp điểm biểu diễn số phức z thỏa mãn z   i  A (x  2)2  (y  1)2  B (x  2)2  (y  1)2  C (x  2)2  (y  1)2  D (x  2)2  (y  1)2  Hướng dẫn Cách 1: Gọi z  x  yi,(x;y  ) , z  x  yi Theo ta có: x  yi   i   x   (y  1)   (x  2)2  (y  1)2   (x  2)2  (y  1)2  Cách 2: Áp dụng ý phần phương pháp giải ta có: z   i   z  (2  i)   z  (2  i)  có tập hợp điểm biểu diễn số phức z đường tròn tâm I(2;-1), bán kính R=3 Phương trình đường tròn tâm I(2;-1), bán kính R = có dạng (x  2)2  (y  1)2   Chọn A Ví dụ 3: Tập hợp điểm biểu diễn số phức z thỏa mãn z   i  z  2i đường thẳng có phương trình A 3x  y   B 3x  y   C 3x  y   D 3x  y   Hướng dẫn Cách 1: Gọi z  x  yi,(x;y  ) , z  x  yi Theo ta có: Trang x  yi   i  x  yi  2i  x   (y  1)i  x  (2  y)i  (x  3)2  (y  1)2  x  (2  y)2  x  6x   y  2y   x  y  4y   6x  2y  10  4y   6x  2y   Do tập hợp biểu diễn số phức z đường thẳng 6x  2y    3x  y   Cách 2: Sử dụng máy tính Casio fx 579 VNPLUS Bước 1: Thiết lập chế độ sử dụng số phức: MODE Bước 2: Sử dụng SHIFT (CMPLX) (Conjg) để nhập số phức liên hợp Lấy điểm thuộc đáp án, thửu vào xem có thỏa mãn z   i  z  2i chọn Đáp án A: Chọn x   y  6 ta z = – 6i, nhập  6i   i  Conjg(1  6i)  2i kết số khác nên loại Đáp án B: Chọn x   y  66 ta z = + 6i, nhập  6i   i  Conjg(166i)  2i kết số khác nên loại Đáp án C: Chọn x   y  3 ta z = - 3i, nhập  3i   i  Conjg(2  3i)  2i kết số khác nên loại Đáp án D: Chọn x   y  ta z = + 3i, nhập  3i   i  Conjg(2  3i)  2i kết  Chọn D Ví dụ 4: Cho số phức z thỏa z  Biết tập hợp số phức w  z  2i đường tròn Tâm đường tròn A I(0;2) B I(0;-2) C I(-2;0) D I(2;0) Hướng dẫn Cách 1: Đặt w  x  yi (x, y  ), ta có: z  w  2i  z  x  yi  z  x  (y  2)  z  x  (y  2)i Theo đề suy z   x  (y  2) i   x  (y  2)2  Vậy tập hợp số phức cần tìm nằm đường tròn có tâm I(0;2) Cách 2: w  z  2i  w  2i  z  w  2i  z Mà z  z  nên w    w  (0  2i)  Do điểm biểu diễn số phức w đường tròn tâm I(0;2), bán kính R =  Chọn A Ví dụ 5: Cho số phưc z thỏa mãn z  Biết tập hợp điểm biểu diễn số phức w   2i  (2  i)z đường tròn Bán kính R đường tròn là: A B C D 13 Hướng dẫn w   2i  (2  i)z  w   2i  (2  i)z  w   2i  (2  i)z Trang Áp dụng công thức z.z '  z z ' ta có: w   2i   i.z  2  (1)2  Do điểm biểu diễn số phức w đường tròn tâm I(3;-2), bán kính R   Chọn B Bài tập tự luyện Câu Trên mặt phẳng tọa độ Oxy, tập hợp điểm biểu diễn số phức z thỏa mãn điều kiện z   z  i đường thẳng có phương trình là: A y = x B x + y = C y = 2x +1 D y – x + = Câu Trong mặt phẳng tọa độ Oxy, tập hợp điểm biểu diễn số phức z thỏa mãn điều kiện z   4i  A Đường thẳng qua gốc tọa độ B Đường tròn bán kính C Đường tròn tâm I(3;-4), bán kính D Đường tròn tâm I(3;-4), bán kính Câu Tập hợp điểm biểu diễn số phức z mặt phẳng tọa độ thỏa mãn điều kiện z  5z  5z  A Đường thẳng qua gốc tọa độ B Đường thẳng x – y = C Đường tròn tâm I(5;0), bán kính D Đường tròn tâm I(-5;0), bán kính Câu Cho số phức z thỏa mãn z  số phức w thỏa mãn w  (4  3i)z  Biết tập hợp điểm biểu diễn số phức w đường tròn Tính bán kính r đường tròn A r = B r = 10 C r = 14 D r = 20 Đáp án: 1–B 2–C 3–C 4–B Dạng 3: Cực trị số phức Phương pháp giải Áp dụng bất đẳng thức z1  z  z1  z  z1  z 2 Ví dụ minh họa Ví dụ 1: Số phức z thỏa mãn điều kiện z   3i  Giá trị lớn z A B C D Hướng dẫn Áp dụng công thức z1  z  z1  z ta có: z  z   3i   3i  (z   3i)  (4  3i)  z   3i   3i    Do giá trị lớn z  Chọn D Ví dụ 2: Cho số phức z thỏa mãn z   3i  Giá trị lớn z   i A 13  B C D 13  Hướng dẫn Trang Áp dụng công thức z1  z  z1  z ta có: z   3i   z  (2  3i)   z  (2  3i)   z  (2  3i)   z   3i  Áp dụng công thức z1  z  z1  z ta có: z   i  (z   3i)  (3  2i)  z   3i   2i   32  (2)2   13 Do giá trí lớn z   i  13  Chọn A Ví dụ 3: Cho số phức z thỏa mãn z   2i  Gọi M, m giá trị lớn nhất, giá trị nhỏ z   i Tính S = M2 + m2 A S = 34 B S = 83 C S = 68 D S = 36 Hướng dẫn z   i  z   2i   3i  (z   i)  (3  3i) Áp dụng z1  z  z1  z  z1  z : z   2i   3i  (z   2i)  (3  3i)  z   2i   3i    (z   2i)  (3  3i)   Hay m  4   z   i   Vậy S = M2 + m2 = 68  Chọn C Bài tập tự luyện Câu Cho số phức z thỏa mãn z   2i  Giá trị nhỏ z  i A 1 B 1 C 2 D 52 Câu Cho số phức z thỏa mãn z   4i  Giá trị lớn z A B C D Câu Cho số phức z thỏa mãn z  i  Giá trị lớn z   i A B 1 C D 1 Đáp án: 1–A 2–D 3-A PHẦN 2: BÀI TẬP TỔNG HỢP Câu Điểm biểu diễn số phức z  A (1;-4) B (-1;4) (2  3i)(4  i) có tọa độ  2i C (1;4) D (-1;-4) Câu Tập hợp điểm biểu diễn số phức z thỏa mãn zi  (2  i)  Trang A (x  1)2  (y  2)2  B (x  1)2  (y  2)2  C x + 2y – = D 3x + 4y – = Câu Cho số phức z thỏa mãn z   i  z   2i Tập hợp điểm biểu diễn số phức z mặt phẳng tọa độ đường thẳng Đường thẳng có phương trình A 4x + 6y – = B 4x – 6y – = C 4x + 6y + = D 4x – 6y + = Câu Cho điểm A biểu diễn số phức – 2i, điểm B biểu diễn số phức -1 + 6i Gọi M trung điểm AB Khi điểm M biểu diễn số phức số phức sau: A – 2i B – 4i C + 4i D + 2i Câu Tập hợp biểu diễn số phức z thỏa mãn z  3z  (2  3i) z A Là đường thẳng y   3x B Là đường thẳng y  3x C Là đường thẳng y = -3x D Là đường thẳng y = 3x Câu Tập hợp điểm biểu diễn số phức z thỏa mãn điều kiện z   2i  nằm đường tròn có tâm A I(1;2) B I(-1;2) C I(1;-2) D I(-1;-2) Câu Tập hợp điểm biểu diễn số phức z đường tròn tâm (a;b), cho u  z   3i zi số ảo đường tròn tâm I(a;b) Tổng a + b A B C -2 D Câu Cho số phức z  thỏa mãn z  Tổng giá trị lớn giá trị nhỏ biểu thức P zi z A B C D Đáp án: 1–D 2–A 3–B 4–D 5–A 6–B 7–C 8-B Trang CHƯƠNG SỐ PHỨC CHUYÊN ĐỀ 3: PHƯƠNG TRÌNH TRÊN TẬP SỐ PHỨC PHẦN 1: LÝ THUYẾT TRỌNG TÂM Căn bậc hai số phức Số phức z = x + yi bậc hai số phức Cho z   2i, z  (1  2i)2  3  4i  w w  a  bi z  w Ta nói số phức z = + 2i bậc hai số phức w = có bậc hai z = w  3  4i w  có hai bậc hai Phương trình bậc hai Phương trình bậc hai az  bz  c  với a, b, c Phương trình bậc hai z  z   có a = 1; b = -1; số phức cho trước c =   b  4ac có bậc hai  , đó:   b  4ac  3  3i  (i 3)2   , phương trình có nghiệm phân biệt  có bậc hai   i b   z1,2  Phương trình có hai nghiệm phức phân biệt 2a 1 i b z1,2    , phương trình có nghiệm kép z1  z  2a Hệ thức Viet phương trình bậc hai với hệ số thực Phương trình az  bz  c  0(a  0) có hai nghiệm Phương trình bậc hai z  z   có a = 1; b = -1; c = phân biệt z , z (thực phức) b  S  z1  z   a  Ta có hệ thức Viet  P  z z  c  1  a b  S  z1  z   a Ta có hệ thức Viet  P  z z  c  a PHẦN 2: CÁC DẠNG BÀI TẬP Dạng 1: Tìm bậc hai số phức Phương pháp giải Tìm bậc hai số phức w: Trường hợp w số thực: Nếu a số thực Số có hai bậc hai   3 a < 0, a có bậc hai i a Số -9 có hai bậc hai 3i a = 0, a có bậc hai a > 0, a có hai bậc hai  a Trường hợp w w  a  bi(a, b  , b  0) số phức có dạng Ví dụ: Số phức w = – 6i có hai bậc hai Tìm phần thực bậc hai có phần ảo số dương A -2 B -3 C D Trang Hướng dẫn Cách 1: Gọi z  x  yi(x, y  ) bậc hai Cách 1: Gọi z  x  yi(x, y  ) bậc hai w số phức w = – 6i Ta có: Ta có: z  w  (x  yi)  a  bi z  w  ( x  yi )   6i  x  2xyi  (yi)  a  bi  x  xyi  ( yi )   6i  x  y  2xyi  a  bi  x  y  xyi   6i x  y2  a  2xy  b  x2  y   x2  y     3 2 xy  6 y   x    x4  8x2    x  x     3  y  3 y  x   x 2 Giải hệ phương trình nghiệm (x;y) Mỗi cặp số thực (x;y) nghiệm hệ phương trình cho ta bậc hai z  x  yi số phức w = a + bi  x    x  (tm)     x  1(loai )   y  1     x  3 3    y  x   y  Vậy w = – 6i z1   i, z  3  i bậc hai là: Cách 2: Sử dụng casio fx-570 VNPLUS Cách 2: Sử dụng casio fx-570 VNPLUS Mode (COMP) Bước 1: Mode (COMP) Bước 2: Nhấn SHIFT + (pol), ta nhập Pol(a,b), ấn = Bước3: Nhấn Shift – (Rec), t nhập Re c có   X, y : , ta thu kết X = x, Y = y Nhấn SHIFT + (pol), ta nhập Pol(8,-6), ấn = Nhấn Shift – (Rec), ta nhập Re c   X, y : ta thu kết X = 3, Y = -1 Vậy hai bậc hai cần tìm – i -3 + i Căn bậc hai cần tìm x + yi –x – yi  Chọn B Ví dụ minh họa Ví dụ 1: Một bậc hai số phức w = + 4i có dạng z = x + yi Trong x, y số nguyên dương, tổng x + y A -3 B C D Hướng dẫn Cách 1: Vì z  x  yi bậc hai số phức w   4i nên z  w x   x  y2   y  2 (x  yi)   4i  x  y  2xyi   4i      x  2 2xy     y  1 Trang Vì x, y số nguyên dương nên x = 2, y =  x + y = Cách 2: w   4i   4i   22  2.2i  i  (2  i) Do bậc hai w = +4i có phần thực, phần ảo số nguyên dương z = + i Cách 3: Sử dụng Casio fx-570VNPLUS Bước 1: Mode (COMP) Bước 2: Nhấn Shift + (Pol), ta nhập Pol (3,4), ấn = Nhấn Shift – (Rec), ta nhập Re c   X, Y : , ấn =, ta thu kết X = 2, Y = Vậy hai số phức cần tìm + i – – i  Chọn C Ví dụ 2: z bậc hai có phần ảo âm số phức 24 – 10i Phần thực z A -1 B C D -5 Hướng dẫn 24  10i  25  2.5i   52  2.5i  i  (5  i) Vì z có phần ảo âm nên z = – i Vậy phần thực z  Chọn B Bài tập tự luyện Câu Căn bậc hai  3i A 2  3i B  3i C (2  3i) D (2  3i) Câu z bậc hai có phần thực âm số phức 35 – 12i Phần ảo z A -1 B i C D -i Đáp án: 1–C 2–C Dạng 2: Phương trình tập số phức Ví dụ minh họa Ví dụ 1: Gọi z1 z2 hai nghiệm phức phương trình z  2z  10  Giá trị A  z12  z 22 A 30 B 10 C 20 D 50 Hướng dẫn Cách 1: Phương trình z  2z  10  có  '  (1)  10  9  (3i) nên phương trình có hai nghiệm phức z1  1  3i, z  1  3i A  (1  3i)  (1  3i)  8  6i  8  6i  (8)  62  (8)  62  20 Cách 2: Sử dụng Casio fx-570VNPLUS Bước 1: Sử dụng chức giải phương trình bậc MODE Bước 2: Nhập hệ số a = 1, b = 2, c = 10 Trang Ta hai nghiệm z1  1  3i, z  1  3i Bước 3: Sử dụng SHIFT hyp (abs) để bấm dấu môđun Nhập A  (1  3i)  (1  3i)  20  Chọn C Ví dụ 2: Kí hiệu z1, z2, z3, z4 bốn nghiệm phương trình z  z  12  Tổng T  z1  z  z3  z A B 26 C  D 10 Hướng dẫn t  Đặt z2 = t, phương trình trở thành t  t  12     t  3 z  Với t = 4, z2 =    z  2 z  i Với t = 3, z2 = -3 = 3i2    z  i Vậy P  z1  z  z3  z   2  i  i    Chọn C Ví dụ 3: Phương trình z  az  b  có nghiệm phức z   2i Tổng a + b A B -3 C D Hướng dẫn Vì z = + 2i nghiệm phương trình z  az  b  nên ta có: (3  2i)  a(3  2i)  b    12i  3a  2ai  b   (3a  b  5)  (12  2a)i  3a  b   a  6   12  2a  b  13 Do đó: a + b = -6 + 13 =  Chọn D Ví dụ 4: Cho phương trình z  mz  2m   m tham số phức Để phương trình có hai nghiệm z1, z2 thỏa mãn z12  z 22  1 giá trị m m  A  m   m  1 B   m  3  m  1 C  m  m  D   m  3 Hướng dẫn Phương trình z  mz  2m   có a = 1, b = -m, c = 2m – z12  z 22  1  (z1  z )  2z1z  1 Trang b  z1  z   a  m Theo định lí Viet, ta có:  , thay vào ta được: z z  c  2m   a m  m  2(2m  1)  1  m  4m     m   Chọn A Ví dụ 5: Gọi z1, z2 hai nghiệm phứ ccủa phương trình z  z   Phần thực số phức (i  z1 )(i  z ) 2017 A 22016 B 21008 C 21008 D 22016 Hướng dẫn z  z  Ta có z1, z2 hai nghiệm phương trình: z  z   nên  z1.z  Ta có  (i  z1 )(i  z )  2017   z1z  i(z1  z )  i  1008  (1  i) 2016 (1  i)  (1  i)  2017  (2  i  1) 2017  (1  i) 2017 (1  i)  (2i)1008 (1  i)  21008  21008 i Vậy phần thực  (i  z1 )(i  z )  2017 -21008  Chọn B Bài tập tự luyện Câu Phương trình z  bz  c  có nghiệm phức z = – 2i Tích hai số b c A B -2 C -10 D Câu Trên tập hợp số phức, phương trình z  7z  15  có hai nghiệm z1, z2 Giá trị biểu thức z1  z  z1z A -7 B C 15 D 22 Câu Kí hiệu z1, z2, z3, z4 bốn nghiệm phương trình z  z   Tổng P  z1  z  z3  z A 2(  3) B (  3) C 3(  3) D 4(  3) Đáp án: 1–C 2–B 3-A Bài tập tổng hợp Câu Tập hợp nghiệm phương trình z  A 0;1  i B 0 z zi C 1  i D 0;1 Câu Gọi z1, z2 nghiệm phức phương trình z  2z   Biết A, B điểm biểu diễn số phức z1, z2 Độ dài đoạn AB Trang A B C D Câu Trên tập số phức C cho phương trình (z  2z)  5(z  2z)   Các nghiệm phương trình  z  1  i A   z  1  i  z  1  i B  z   i z   i C   z  1  i  z  2  i D   z  1  i Câu Phương trình z2 = có nghiệm phức với phần ảo âm? A B C D Câu Phương trình (2  i)z  az  b  0(a, b  ) có hai nghiệm + I – 2i Giá trị a A -9 – 2i B 15 + 5i C + 2i D 15 – 5i Câu Tìm số nguyên x, y cho số phức z = x + yi thỏa mãn z3 = 18 + 26i x  A   y  1 x  B   y  1 x  C  y   x  3 D   y  1 Câu Trên tập số phức, cho phương trình sau: (z  i)  4z  Có nhận xét số nhận xét sau? Phương trình vơ nghiệm trường số thực  Phương trình vơ nghiệm trường số phức  Phương trình khơng có nghiệm thuộc tập số thức Phương trình có nghiệm thuộc tập số phức Phương trình có nghiệm số phức Phương trình có nghiệm số thực A B C D Câu Phương trình z  9z3   có nghiệm tập số phức? A B C D Câu Giả sử z1, z2 nghiệm phương trình z  2z   A, B điểm biểu diễn z1, z2 Tọa độ trung điểm I đoạn thẳng AB A I(1;1) B I(-1;0) C I(0;1) D I(1;0) Câu 10 Cho số phức z1   2i, z   2i Hỏi z1, z2 nghiệm phương trình phức sau đây? A z  2z   B z  2z   C z  2z   D z  2z   Câu 11 Gọi z1, z2 nghiệm phương trình z  (1  3i)z  2(1  i)  Khi w  z12  z 22  3z1z số phức có mơđun A B 13 C 13 D 20 Đáp án: 1–A 2–C 3–A 4–A 5–A 6–C –D 8–D 9–D 10 –C 11 - D Trang ... đúng? A Tổng hai số tự nhiên số chẵn hai số số chẵn B Tích hai số tự nhiên số chẵn hai số số chẵn C Tổng hai số tự nhiên số lẻ hai số số lẻ D Tích hai số tự nhiên số lẻ hai số số lẻ Câu Trong... chữ số chữ số bên phải số Nếu chữ số sau hàng qui tròn lớn hay ta thay chữ số chữ số bên phải số cộng thêm đơn vị vào chữ số hàng qui tròn Chữ số Cho số gần a số a với độ xác d Trong số a, chữ số. .. đúng? A Số tập X 16 B Số tập X gồm có phần tử C Số tập X chứa số D Số tập X gồm có phần tử Hướng dẫn Số tập tập hợp X là: 24  16 Số tập có phần tử tập hợp X là: C24  Số tập tập hợp X chứa số là:

Ngày đăng: 01/07/2019, 15:42

Từ khóa liên quan

Tài liệu cùng người dùng

  • Đang cập nhật ...

Tài liệu liên quan